Sie sind auf Seite 1von 270

Iranian University Students Mathematics

Competitions, 1973-2007

Bamdad R. Yahaghi

School of Mathematics, Institute for Studies in Theoretical Physics


and Mathematics (IPM), P.O. Box: 19395-5746 Tehran, Iran
E-mail address: bamdad5@ipm.ir, bbaammddaadd55@gmail.com
Contents

Preface ix

Chapter 1. Problems 1
1.1. First Competition, University of Tehran, March 1973 1
1.2. Second Competition, Shiraz (former Pahlavi) University, March 1974 3
1.3. Third Competition, (former Jondi Shapour) University of Ahwaz,
March 1975 6
1.4. Fourth Competition, University of Tabriz, March 1976 8
1.5. Fifth Competition, Sharif (former Aryamehr) University of Technology,
March 1977 9
1.6. Sixth Competition, The University of Isfahan, March 1978 10
1.7. Seventh Competition, Ferdowsi University of Mashhad, March 1980 12
1.8. Eighth Competition, Shiraz University, March 1984 13
1.9. Ninth Competition, Tehran Teacher Training (Tarbiat Moallem)
University, September 1985 14
1.10. Tenth Competition, University of Sistan and Baluchestan, March
1986 16
1.11. Eleventh Competition, The University of Birjand, March 1987 17
1.12. Twelfth Competition, Guilan University , March 1988 18
1.13. Thirteenth Competition, University of Tehran, March 1989 20
1.14. Fourteenth Competition, The University of Isfahan, March 1990 21
1.15. Fifteenth Competition, Ferdowsi University of Mashhad, March 1991 23
1.16. Sixteenth Competition, Razi (Rhazes or Rasis) University of
Kermanshah, March 1992 24
1.17. Seventeenth Competition, Shahid Beheshti (former National)
University, March 1993 25
1.18. Eighteenth Competition, Sharif University of Technology, March
1994 26
1.19. Nineteenth Competition, University of Kerman, March 1995 27
1.20. Twentieth Competition, Sharif University of Technology, February
1996 28
1.21. Twenty First Competition, University of Tehran, March 1997 28
1.22. Twenty Second Competition, University of Ahwaz, March 1998 29
1.23. Twenty Third Competition, Sharif University of Technology, March
1999 36
1.24. Twenty Fourth Competition, Khajeh Nasir Toosi University of
Technology, May 2000 37
1.25. Twenty Fifth Competition, Imam Khomeini International University
of Qazvin, May 2001 39
v
vi CONTENTS

1.26. Twenty Sixth Competition, Institute for Advanced Studies in Basic


Sciences (IASBS) of Zanjan, May 2002 40
1.27. Twenty Seventh Competition, Bu-Ali Sina (Avecina) University of
Hamedan, May 2003 41
1.28. Twenty Eighth Competition, Sharif University of Technology, May
2004 43
1.29. Twenty Ninth Competition, University of Mazandaran in Babolsar,
May 2005 45
1.30. Thirtieth Competition, Tafresh University, May 2006 46
1.31. Thirty First Competition, Ferdowsi University of Mashhad, May
2007 48

Chapter 2. Solutions 51
2.1. First Competition 51
2.2. Second Competition 56
2.3. Third Competition 64
2.4. Fourth Competition 71
2.5. Fifth Competition 74
2.6. Sixth Competition 83
2.7. Seventh Competition 86
2.8. Eighth Competition 91
2.9. Ninth Competition 96
2.10. Tenth Competition 109
2.11. Eleventh Competition 115
2.12. Twelfth Competition 119
2.13. Thirteenth Competition 125
2.14. Fourteenth Competition 134
2.15. Fifteenth Competition 143
2.16. Sixteenth Competition 149
2.17. Seventeenth Competition 152
2.18. Eighteenth Competition 155
2.19. Nineteenth Competition 160
2.20. Twentieth Competition 165
2.21. Twenty First Competition 172
2.22. Twenty Second Competition 176
2.23. Twenty Third Competition 179
2.24. Twenty Fourth Competition 184
2.25. Twenty Fifth Competition 196
2.26. Twenty Sixth Competition 202
2.27. Twenty Seventh Competition 205
2.28. Twenty Eighth Competition 210
2.29. Twenty Ninth Competition 215
2.30. Thirtieth Competition 220
2.31. Thirty First Competition 226

Chapter 3. Problem Index 235


First Competition 235
Second Competition 235
Third Competition 236
CONTENTS vii

Fourth Competition 236


Fifth Competition 236
Sixth Competition 237
Seventh Competition 237
Eighth Competition 237
Ninth Competition 238
Tenth Competition 238
Eleventh Competition 239
Twelfth Competition 239
Thirteenth Competition 240
Fourteenth Competition 240
Fifteenth Competition 240
Sixteenth Competition 241
Seventeenth Competition 241
Eighteenth Competition 241
Nineteenth Competition 241
Twentieth Competition 242
Twenty First Competition 242
Twenty Second Competition 242
Twenty Third Competition 242
Twenty Fourth Competition 243
Twenty Fifth Competition 243
Twenty Sixth Competition 243
Twenty Seventh Competition 244
Twenty Eighth Competition 244
Twenty Ninth Competition 244
Thirtieth Competition 245
Thirty First Competition 245
Classification of problems 246
Index 247
Preface

taa khod raa be tchizi nadaadi be kol-liyat,


aan tchiz sa’b-o doshvaar minomaayad.
tchoon khod raa be kol-liyat be tchizi daadi,
digar doshvaari namaanad.
[Until you do not devote yourself to a task totally,
that task looks hard and unreachable.
As you devote yourself to a task totally,
there remains no difficulty.]
–Shamsoddin Tabrizi

International mathematical competitions have gained great popularity in recent


years. They provide the first test for a young person’s mathematical prowess, and
success in these competitions often translates into gaining admittance to the best
research institutions for graduate work. For this reason some colleges and even
high schools have organized special classes for training mathematically talented
students for these competitions. The purpose of this text is to provide a selection
of mathematical problems that are not only suitable for special college level (and
occasionally even at high school level) courses designed for these competitions, but
can significantly improve the level of mathematical sophistication of students who
are intrigued by and envision a career in mathematics in general.
I was asked by the Iranian Mathematical Society (IMS) to prepare a problem
book on the basis of college level competitions in Iran. Since Iranian students
have done quite well in international competitions and their success to some extent
reflects the training that they received in special courses, I decided to make the book
available to a wider audience. The problems (and their solutions) that are presented
in this book are from national mathematical competitions in Iran at college level
from 1973 to 2007. I provided my own solutions to most of the problems and also
utilized solutions from the files of the IMS, which contained approximately about
40% of all the solutions.
I should point out that I have not edited the problems of the competitions and
they are direct translations from Persian into English. Unfortunately, there were
some typos and mistakes in the original version. The errors have been corrected and
except for trivial typographical ones, the corrections are so indicated as footnotes
throughout Chapter One. Some comments on the problems also appear in footnotes.
I am indebted, and hence express my deepest gratitude, to the colleagues who
contributed in different ways to these competitions and to this book — either as
members of the Scientific Committee for the competitions, or proposed problems
or assisted in finding elegant solutions. Unfortunately, some of those colleagues
have left us; may they rest in peace. I am thankful to my dear friend Dr. Hossein
ix
x PREFACE

Hajiabolhassan who constantly encouraged me and helped me with the prepara-


tion of the book. I would like to thank Professor Alireza Jamali, Professor Ebad
Mahmoodian, the outgoing President of the IMS, Dr. Rashid Zaare-Nahandi, and
Messrs. M. Shokoohi and M. Abdi-Zadeh and the administrative assistants at the
main office of the IMS for their assistance in gaining access to the existing solution
files of the IMS. I am grateful to Ms. Anahita Samie for drawing the figures for
this book...

Bamdad R. Yahaghi
PREFACE xi

Acknowledgement

A number of friends and colleagues took interest in this work and their contri-
butions are implicit in some elegant solutions to the problems. While inadvertently
I may have left out the names of some, the contributions of
Saeed Akbari (the second solution of Problem 7 of 1.9.3),
Kasra Alishahi (the third solution of Problem 1.b of 1.24.2),
Rajendra Bhatia (the first proof of the lemma presented in Solution 3 of 2.3.1),
Hossein Hajiabolhassan (Problems: 2 of 1.21.2, 2 of 1.23.2, and 6 of 1.28.1),
Hossein Hajiabolhassan and the late Mojtaba Mehrabadi (Problem 2 of 1.14.2),
Ramin Mohammadalikhani (the second solution of Problem 2 of 1.13.1),
Ali Mohammadian (Problems: 6 of 1.27.2 and 5 of 1.25.1),
Omid Naghshineh Arjmand (Problem 3.b of 1.3.1),
are duly acknowledged. Needless to say I am responsible for any shortcomings or
errors.
xii PREFACE

Iranian University Students Mathematics Competitions – a historical


introduction

The history of the Iranian Mathematics Competitions for university students is


an integral part of the contemporary history of mathematics in Iran. On March 31st,
1972, the first general assembly of the Iranian Mathematical Society (IMS) was held
at the former National University (now Shahid Beheshti University). Upon a mo-
tion proposed by Mehdi Behzad, then president of the society –which at that time
was called the Society’s secretary– the need for a mathematics competition aimed at
university students was approved. Herewith a committee, consisting of Houshang
Attarchi, Mehdi Behzad, Fereidoun Ghahramani, the late Mohammad-Ali Gheyni,
and Bahman Vahidi, was first appointed to prepare a guideline for the competitions
and to make preparations for holding theses competitions, which were intended for
university students. This committee developed a guideline and required that the
competitions must be held annually and simultaneously with the Annual Iranian
Mathematical Society Conference. The primary goals of the competitions were to
discover, encourage, and nurture mathematical talents throughout the country and
to create a friendly and scientific rivalry among students and the Iranian universi-
ties. In those years, the mathematics courses in most of the universities across the
country were heavily influenced by the traditional and old curriculum as opposed to
the modern one. Only a handful of universities, such as the University of Tehran,
Sharif (then Aryamehr) University of Technology, (former Pahlavi) University of
Shiraz, and Mossaheb Institute of Mathematics offered courses such as Set Theory,
Modern Algebra, Modern Analysis, and Combinatorics. The secondary goal of the
competitions was to motivate the students to view mathematics in new ways to
maintain and sustain a more diverse perspective of mathematics, thereby putting
pressure on universities to change their educational systems.
The following year, on March 30th, 1973, simultaneous with the Fourth Annual
IMS Conference held at the University of Tehran, the First National Mathematics
Competition was held with the participation of 22 students from five universities
across the country. Half of the questions were in the areas of Mathematical Analysis
and Algebra and the rest were in other areas categorized as General, i.e., innovative
questions, Probability and Statistics, Differential Equations, and Topology. The
second competition took place in March 1974 at the, former Pahlavi, University of
Shiraz with the participation of 51 students from eleven universities. The students’
participation grew steadily in the following years so that in the thirtieth competition
more than 180 students from 40 universities took part.
Ever since 1973, the competitions were held annually except for 1979 with clo-
sure of universities because of the revolution and 1981-1983 when universities were
closed due to the cultural revolution. Students’ and universities’ reception of the
competitions caused a great increase in the number of participating students. On
the other hand, when the competitions were held on the same date and location
as the annual IMS conferences, the organizational matters of the event, in particu-
lar grading the student’s papers, had become so overwhelming that it would take
a number of months to announce the results. This would sometimes lead to dis-
content and objections, which occasionally resulted in changes to the final scores.
However, it was only for the tireless and diligent efforts of the questions commit-
tees in those years that the competitions survived and were kept alive despite all
difficulties and shortcomings. In view of all this, the IMS was persuaded to hold
PREFACE xiii

the competitions in a date and location different from those of the Annual IMS
conferences. Consequently, from 1996-2003 the competitions took place during a
three day period. Since 2004 they have been taking place in a four day period at
one of the Iranian universities during the Spring of every year.
In June 1999, the IMS’ executive committee and board of trustees gathered
at Tafresh University to reconsider the bylaws and regulations of the IMS. At
that time, a committee was established to substantially change the regulations
concerning the National Mathematics Competitions. From 2000, the competitions
are being held according to the new regulations. It must, however, be said that some
minor changes and improvements were made at some stages due to the experiences
obtained from previous competitions.
According to the current guideline of the competitions, each university or higher
educational institution, can send only one team consisting of at most five students
and a leader, possibly along with a deputy leader, to the competitions. Each
university is responsible for selecting its own team, presumably through holding
preliminary tests among its students. The contests take place in two sessions, with
each session lasting three and a half hours. The students would answer twelve
questions of which four are in the area of analysis, four in the area of algebra,
and another four in other areas such as discrete mathematics, probability, number
theory, etc, categorized as innovative questions. Attempts are made to propose
problems whose solutions require not only mathematical knowledge but innovation.
The questions of the competitions are now designed and selected by a questions
committee consisting of a chairperson, a supervisor, and three other people as the
heads of the following sections: analysis, algebra, and innovative questions. The
heads of the three sections can each choose a vice head to assist them. The questions
committee is appointed by the Executive Committee of the IMS for a period of
two up to three years. The committee is responsible for designing questions and
holding the competitions. It is also responsible for appointing a grading team,
whose members are mostly chosen from the winners of the previous competitions.
In the course of a number of months, the committee selects 24 questions which are
divided into two sets of questions, each of which has 12 questions. The contests are
held in two sessions. For each sitting, 6 questions are selected out of the 12 questions
by a jury consisting of the leaders of the participating teams. The participants are
ranked based on their individual scores. The top 5 individual scorers win gold
medals, the next 10 top contestants are awarded silver medals, and up to the next
top 15 receive bronze medals. It is worth mentioning that since April 2004, i.e.,
from the 28th competition on, the problems together with their proposed solutions
are posted online, on the IMS website currently only in Persian. This is done for two
reasons. Firstly, the universities can simultaneously hold the competitions for their
interested students. Secondly, this way the problems and their proposed solutions
are preserved on the Internet permanently.
Now after thirty one competitions, it seems that most of the goals of the com-
petitions have been achieved. A glance at the list of the winners of competitions
reveals that many of them have gone on to become distinguished mathematicians
and are now working at prominent universities and institutions throughout the
world. Also, a glance at the questions of these thirty one competitions shows that
the competitions and the Iranian mathematical community, on the whole, have
xiv PREFACE

evolved to a greater level of sophistication in terms of content, quantity and qual-


ity.
In 1995, the executive committee of the IMS approved the preparation of a
book containing the problems and solutions to the problems of the first twenty
competitions. It took the IMS several months to collect the problems of the first
twenty competitions. About one third of the proposed solutions of all problems
were available in the IMS files. Rashid Zaare-Nahandi informed Bamdad Yahaghi
about the decision made by the IMS and asked whether Yahaghi was interested
in preparing such book. Yahaghi showed interest and readiness and it took him
a year of hard work to prepare a preliminary copy of the manuscript. Under the
recommendation of the IMS, the analysis and the algebra part of the manuscript
was then refereed by the late Karim Seddighi and Mohammad Reza Darafsheh,
respectively. This was simultaneous with Yahaghi’s departure for Canada to con-
tinue his PhD studies at Dalhousie University. Unfortunately, the original plan by
the IMS to publish this book within a year could not be realized. Two years ago,
shortly after the author’s return to Iran, his interest in the project on which he
had spent such considerable time and energy was rekindled. He began to revise,
update, and rewrite the book in both Persian and English. It must be said that
the IMS did not have the proposed solutions to the problems of Competitions 21,
23, 24, and 27. It took Yahaghi almost two years to prepare both the English and
Persian versions of the book. We must point out that this book could not have
come into existence without the efforts of the colleagues who have contributed to
these competitions. This book perhaps sheds light on the time and effort that have
been put in organizing these competitions. It is hoped that this book succeeds in
introducing these competitions to an international audience.
Following this introduction, the names of the members of the questions com-
mittees1 and the date, location, and the winners of the competitions, together with
their educational and employment affiliations, are quoted.

Bamdad R. Yahaghi and Rashid Zaare-Nahandi

1Up until 1998, the executive committee of the IMS used to appoint only one person as the
chairperson of the competition. This person was charged with the task of inviting other colleagues
to submit questions and to mark the student’s papers. As outlined before, as of 1999, the questions
committee is appointed by the Executive Committee of the IMS. And it is this committee that is
responsible for designing questions and holding the competitions.
PREFACE xv

The First Competition, University of Tehran, March 1973


Questions Committee: Mehdi Behzad.

1. Elizabeth Ebrahimzadeh, Sharif (former Aryamehr) University of Technology.


PhD (University of California, Berkeley, 1984), Professor (California State Uni-
versity, Sacramento).
2. Mohammdad Reza Darafsheh, University of Tehran.
PhD (University of Birmingham, UK, 1978), Professor (University of Tehran,
Iran).
3. Hashem Madadi-Almousavi, Ferdowsi University of Mashhad.
Deceased.
4. Mirebrahim Hashemi Aghdam, Sharif (former Aryamehr) University of Technol-
ogy.
5. Yousef Bahrampour, (former Pahlavi) University of Shiraz.
PhD (University of Oregon, USA, 1983), Professor (University of Kerman,
Iran).

The Second Competition, Shiraz (former Pahlavi) University, March


1974
Questions Committee: Mehdi Behzad.

1. Mohammad Ali Najafi, Sharif (former Aryamehr) University of Technology.


2. Firouz Khosroyani, University of Tehran.
3. Mehdi Zekavat, (former Pahlavi) University of Shiraz.
4. Hamid Hamed Akbari-Tousi, University of Tehran.
5. Shahram Arshad-Riyazi, former National University.

The Third Competition, (former Jondi Shapour) University of Ahwaz,


March 1975
Questions Committee: Mohammad Ali Gheyni.

1. Pirouz Vakili, Sharif (former Aryamehr) University of Technology.


2. Moslem Nikfar, University of Tehran.
3. Ali Asghar Babadi Margha, University of Tehran.
4. Saeed Ghahramani, Sharif (former Aryamehr) University of Technology.
5. Homayoun Moeen, Sharif (former Aryamehr) University of Technology.

The Fourth Competition, University of Tabriz, March 1976


Questions Committee: Vahab Davarpanah.

1. Pirouz Vakili, Sharif (former Aryamehr) University of Technology.


2. Homayoun Moeen, Sharif (former Aryamehr) University of Technology.
3. Nasser Hosseini, (former Pahlavi) University of Shiraz.
4. Shahla Marvizi, Sharif (former Aryamehr) University of Technology.
5. Ali Karimi, Tarbiat Moallem (Teacher Training) University.

The Fifth Competition, Sharif (former Aryamehr) University of


Technology, March 1977
xvi PREFACE

Questions Committee: Mohammad Reza Nouri-Moghadam.

1. Hamid Kazemi, Sharif (former Aryamehr) University of Technology.


2. Hossein Masoumi Fakhar, Sharif (former Aryamehr) University of Technology.
3. Masoud Khalkhali, Sharif (former Aryamehr) University of Technology.
4. Ebrahim Saátchi, (former Azarabadegan) University of Tabriz.
5. Safa Nourbakhash, former National University.

The Sixth Competition, The University of Isfahan, March 1978


Questions Committee: Magerdich Toumanian.

1. Nasser Boroujerdian, University of Tehran.


2. Hamid Kazemi, Sharif (former Aryamehr) University of Technology.
3. Ali Asghar Alikhani Kouhpaee, University of Isfahan.
4. Mehdi Salehi Nejad, Ferdowsi University of Mashhad.
5. Ali Rejali, University of Isfahan.

The Seventh Competition, Ferdowsi University of Mashhad, March


1980
Questions Committee: Akbar Hassani.

1. Fereydoun Rezakhanlou, University of Tehran.


2. Mehdi Alavi Shoushtari, University of Ahwaz.
3. Seyed Esmail Seyedabadi, Sharif University of Technology.
4. Rajabali Kamyabigol, Ferdowsi University of Mashahd.
5. Ali Asghar Jodeyri Akbarfam, University of Tabriz.

The Eighth Competition, Shiraz University, March 1984


Questions Committee: Asadollah Niknam.

1. Mohammad Hassan Jahanbakht, University of Isfahan.


2. Mojtaba Moniri, University of Tehran.
3. Mohammad Taghi Jahandideh, Shiraz University.
4. Samad Ahmadi, Tarbiat Moallem (Teacher Training) University.
5. Kamal Atigh, University of Tabriz.

The Ninth Competition, Tehran Teacher Training (Tarbiat Moallem)


University, September 1985
Questions Committee: Rahim Zaare-Nahandi.

1. Nasser Boroujerdian, University of Tehran.


2. Mojtaba Moniri, University of Tehran.
3. Majid Ashrafi, Shiraz University.
4. Jamal Rouin, Tarbiat Moallem (Teacher Training) University.
5. Ali Parsian, University of Tehran.

The Tenth Competition, University of Sistan and Baluchestan, March


1986
PREFACE xvii

Questions Committee: Karim Seddighi.

1. Reza Jahani Nejad, University of Kashan.


2. Amir Akbary Majdabadno, University of Tehran.
3. Shaahin Ajoodani Namini, University of Tehran.
4. Hamid Reza Farhadi, Tarbiat Moallem (Teacher Training) University.
5. Masoud Amini, Ferdowsi University of Mashhad.

The Eleventh Competition, The University of Birjand, March 1987


Questions Committee: Karim Seddighi.

1. Vahid Tarokh, Sharif University of Technology.


2. Masoud Amini, Ferdowsi University of Mashhad.
3. Shaahin Ajoodani Namini, University of Tehran.
4. Reza Karami, Isfahan University of Technology.

The Twelfth Competition, Guilan University , March 1988


Questions Committee: Mohammad Ali Shahabi.

1. Shaahin Ajoodani Namini, University of Tehran.


2. Shaahin Amiri Sharifi, Sharif University of Technology.
3. Gholam Hossein Eslamzadeh, Shiraz University.
4. Bamdad Yahaghi, Sharif University of Technology.
5. Ali Iranmanesh, Shiraz University.

The Thirteenth Competition, University of Tehran, March 1989


Questions Committee: Mohammad Ali Shahabi.

1. Kambiz Mahmoodian, University of Tehran.


2. Saeed Zakeri, University of Tehran.
3. Mohammad Sal Moslehian, Ferdowsi University of Mashhad.
4. Bamdad Yahaghi, Sharif University of Technology.
5. Behrooz Mashayekhifard, Ferdowsi University of Mashhad.

The Fourteenth Competition, The University of Isfahan, March 1990


Questions Committee: Heydar Zahed Zahedani.

1. Shaahin Amiri Sharifi, Sharif University of Technology.


2. Hessam Hamidi Tehrani, Sharif University of Technology.
3. Saeed Zakeri, University of Tehran.
4. Hamid Mousavi, Tarbiat Moallem (Teacher Training) University.
5. Shahab Shahabi, University of Tehran.

The Fifteenth Competition, Ferdowsi University of Mashhad, March


1991
xviii PREFACE

Questions Committee: Mohammad Reza Darafsheh.

1. Hessam Hamidi Tehrani, Sharif University of Technology.


2. Ali Rajai, Sharif University of Technology.
3. Shahriar Mokhtari Sharghi, Sharif University of Technology.
4. Pedram Safari, Sharif University of Technology.
5. Ataollah Togha, University of Kerman.

The Sixteenth Competition, Razi (Rhazes or Rasis) University of


Kermanshah, March 1992
Questions Committee: Mohammad Reza Darafsheh.

1. Shahriar Mokhtari Sharghi, Sharif University of Technology.


2. Ataollah Togha, University of Kerman.
3. Pedram Safari, Sharif University of Technology.
4. Ali Sabetian, Shiraz University.
5. Ali Rajai, Sharif University of Technology.
6. Mehdi Najafikhah, Iran University of Science and Technology.

The Seventeenth Competition, Shahid Beheshti (former National)


University, March 1993
Questions Committee: Mohammad Reza Darafsheh.

1. Hossein Hajiabolhassan, Sharif University of Technology.


2. Payman L. Kassaei, Sharif University of Technology.
3. Behrang Noohi, Sharif University of Technology.
4. Arash Rastegar, Sharif University of Technology.
5. Ali Dadban, University of Tehran.

The Eighteenth Competition, Sharif University of Technology, March


1994
Questions Committee: Jafar Zafarani.

1. Ramin Takloo-Bighash, Sharif University of Technology.


2. Kasra Rafi, Sharif University of Technology.
3. Behrang Noohi, Sharif University of Technology.
4. Payman L. Kassaei, Sharif University of Technology.
5. Aminollah Zargarian, University of Tehran.

The Nineteenth Competition, University of Kerman, March 1995


Questions Committee: Ahmad Haghani.

1. Amir Jafari, Sharif University of Technology.


2. Ali Lashgari Faghani, Isfahan University of Technology.
3. Fatemeh Ayatollahzadeh Shirazi, University of Tehran.
4. Mohammad Reza Raoofi, Isfahan University of Technology.
5. Reza Naserasr, Sharif University of Technology.

The Twentieth Competition, Sharif University of Technology, February


1996
PREFACE xix

Questions Committee: Omid Ali Karamzadeh and Yahya Tabesh.

1. Keivan Mallahi-Karai, Sharif University of Technology.


2. Hossein Movasati, Sharif University of Technology.
3. Omid Naghshineh Arjmand, Sharif University of Technology.
4. Ali Reza Amini Harandi, Isfahan University of Technology.
5. Ebrahim Samei, Shahid Beheshti (former National) University.

The Twenty First Competition, University of Tehran, March 1997


Questions Committee: Omid Ali Karamzadeh.

1. Kia Dalili, Sharif University of Technology.


2. Maryam Mirzakhani, Sharif University of Technology.
3. Ebrahim Samei, Shahid Beheshti (former National) University.
4. Hadi Jorati, Sharif University of Technology.
5. Hossein Abedi Andani, Isfahan University of Technology.

The Twenty Second Competition, University of Ahwaz, March 1998


Questions Committee: Omid Ali Karamzadeh.

1. Maryam Mirzakhhani, Sharif University of Technology.


2. Eaman Eftekhari, Sharif University of Technology.
3. Payam Nasser Tayoub, University of Tehran.
4. Mohammad Ahmadvand, Bu-Ali Sina (Avecina) University of Hamedan.
5. Abolghassem Karimi, Shahid Beheshti (former National) University.

The Twenty Third Competition, Sharif University of Technology,


March 1999
Questions Committee: Omid Ali Karamzadeh and Yahya Tabesh.

1. Hadi Salmasian, Sharif University of Technology.


2. Mohsen Bahramgiri, Sharif University of Technology.
3. Mohammad Javaheri, Sharif University of Technology.
4. Bijan Ahmadi, Shahid Beheshti (former National) University.
5. Kamal Azizi, University of Tabriz.

The Twenty Fourth Competition, Khajeh Nasir Toosi University of


Technology, May 2000
Questions Committee: Saeed Azam, Rouzbeh Tusserkani, Ali Reza Jamali (Chair),
and Hossein Mohebi.

1. Omid Amini, Sharif University of Technology.


2. Kasra Alishahi, Sharif University of Technology.
3. Maziar Mirrahimi, Sharif University of Technology.
4. Seyed Reza Moghaddasi, Sharif University of Technology.
5. Masoud Aryapour, Sharif University of Technology.

The Twenty Fifth Competition, Imam Khomeini International


University of Qazvin, May 2001
xx PREFACE

Questions Committee: Saeed Azam, Rouzbeh Tusserkani, Ali Reza Jamali (Chair),
and Hossein Mohebi.

1. Amir Mohammadi, Sharif University of Technology.


2. Salman Abolfath Beigi, Sharif University of Technology.
3. Hamid Reza Darbidi, Sharif University of Technology.
4. Babak Amini, Shiraz University.
5. Afshin Amini, Shiraz University.

The Twenty Sixth Competition, Institute for Advanced Studies in


Basic Sciences (IASBS) of Zanjan, May 2002
Questions Committee: Saeed Azam, Rouzbeh Tusserkani, Ali Reza Jamali (Chair),
and Hossein Mohebi.

1. Salman Abolfath Beigi, Sharif University of Technology.


2. Ali Shourideh, Sharif University of Technology.
3. Javad Ebrahimi Boroujeni, Sharif University of Technology.
4. Amin Aminzadeh Gohari, Sharif University of Technology.
5. Majid Hadian, Sharif University of Technology.

The Twenty Seventh Competition, Bu-Ali Sina (Avecina) University of


Hamedan, May 2003
Questions Committee: Gholam-Hossein Eslamzadeh, Hossein Hajiabolhassan,
Mohammad Reza Pournaki, and Mohammad Taghi Dibaei (Chair).

1. Mohsen Sharifi Tabar, Sharif University of Technology.


2. Ali Shourideh, Sharif University of Technology.
3. Mohammad Farajzadeh Tehrani, Sharif University of Technology.
4. Payam Valadkhan, Sharif University of Technology.
5. Hamid Hassanzadeh, Tarbiat Moallem (Teacher Training) University.
6. Rahbar Rasooli, University of Tehran.
7. Maryam Khosravi, Tarbiat Moallem (Teacher Training) University.
8. Amir Moradifam, Iran University of Science and Technology.

The Twenty Eighth Competition, Sharif University of Technology, May


2004
Questions Committee: Gholam Hossein Eslamzadeh, Hossein Hajiabolhassan,
Majid Mirza-Vaziri, Abdolrasool Pourabbas, Mohammad Reza Pournaki, and Rashid
Zaare-Nahandi (Chair).

1. Iman Setayesh, Sharif University of Technology.


2. Omid Haji Mirsadeghi, Sharif University of Technology.
3. Armin Morabi, Sharif University of Technology.
4. Sajad Lakzian, Amir Kabir University of Technology.
5. Mohammad Kazem Anvari, Ferdowsi University of Mashhad.

The Twenty Ninth Competition, University of Mazandaran in


Babolsar, May 2005
PREFACE xxi

Questions Committee: Hossein Hajiabolhassan, Majid Mirza-Vaziri, Mojtaba


Moniri, Mohammad Reza Pournaki, Mehdi Radjabalipour (Chair), Bamdad R. Ya-
haghi, Rashid Zaare-Nahandi (Supervisor), and Manouchehr Zaker.

1. Iman Setayesh, Sharif University of Technology.


2. Mohammad Farajzadeh Tehrani, Sharif University of Technology.
3. Mohammad Abbas Rezai, Sharif University of Technology.
4. Mohammad Hossein Mousavi, Sharif University of Technology.
5. Fatemeh Doroodian, Amir Kabir University of Technology.
6. Mahmoud Hassanzadeh, University of Tehran.

The Thirtieth Competition, Tafresh University, May 2006


Questions Committee: Hossein Hajiabolhassan, Majid Mirza-Vaziri, Mojtaba
Moniri, Mohammad Reza Pournaki, Mehdi Radjabalipour (Chair), Bamdad R. Ya-
haghi, Rashid Zaare-Nahandi (Supervisor), and Manouchehr Zaker.

1. Ali Akbar Daemi, Sharif University of Technology.


2. Mohammad Gharakhani, Sharif University of Technology.
3. Omid Haji Mirsadeghi, Sharif University of Technology.
4. Mostafa Einollahzadeh Samadi, Sharif University of Technology.
5. Behzad Mehrdad, Sharif University of Technology.

The Thirty First Competition, Ferdowsi University of Mashhad, May


2007
Questions Committee: Mojtaba Gheerati, Hassan Shirdareh Haghighi, Omid
Naghshineh Arjmand, and Fariborz Azarpanah (Chair).

1. Ali Akbar Daemi, Sharif University of Technology.


2. Nasser Talebizadeh, Sharif University of Technology.
3. Erfan Salavati, Sharif University of Technology.
4. Nima Ahmadipour Anari, Sharif University of Technology.
5. Mohammad Gharakhani, Sharif University of Technology.
CHAPTER 1

Problems

1.1. First Competition, University of Tehran, March 1973

1.1.1. Analysis. 1. A function φ that is a pointwise limit of continuous real


functions is called a Baire function.
(a) If f is a real function of the real variable x whose derivative exists every-
where, prove that the function f 0 is a Baire function.
(b) By giving an example show that every Baire function is not necessarily the
derivative of a function.

2. Let’s call the set of all n × n real matrices M. The set M is a metric space if
we view the elements of it as n2 -dimensional vectors equipped with the Euclidean
norm . That is, for every matrix A = (aij ), we define the norm of A as follows
X 1
||A|| = a2ij 2 .
1≤i,j≤n

Prove that the set of invertible matrices is


(a) open.
(b) disconnected.
3. Suppose that the function f is defined on the half-line (0, +∞) by

0 x∈/ Q,
f (x) = 1 p
p+q x = q , p, q ∈ N, gcd(p, q) = 1.
(a) Show that the limit of this function exists at any point of (0, +∞).
(b) At what points of (0, +∞) is the function f continuous? Prove your claim.

1.1.2. Algebra. 1. Let R be commutative ring with identity element.


(a) Prove that an ideal P is prime if and only if
∀a ∈ R, ∀b ∈ R : ab ∈ P =⇒ a ∈ P or b ∈ P.
(b) Prove that every maximal ideal is a prime ideal.
• Hint. In a ring R, the product of two ideals T1 and T2 is defined as
X
xy : x ∈ T1 , y ∈ T2 .
finite
An ideal M 6= R is called maximal if for any ideal T , we have
M ⊆ T =⇒ T = R.
An ideal P is prime if and only if for any two ideals T1 and T2 , from T1 T2 ⊆ P , it
follows that T1 ⊆ P or T2 ⊆ P .
1
2 1. PROBLEMS

2. Suppose that every element of a group G satisfies the equality x2 = e. Prove


that the group G is abelian (i.e., commutative).

3. Let E and F be two isomorphic sets (that is, there is a one-to-one correspon-
dence between them), and that f is a function from P(E) into P(F ) satisfying the
following three conditions, where P stands for the power set operation.
(a) f (∅) = ∅.
(b) ∀X ∈ P(E), ∀Y ∈ P(E) : f (X ∪ Y ) = f (X) ∪ f (Y ).
(c) ∀X ∈ P(E) : cardX ≤ cardf (X).
(cardA ≤ cardB means that there exists a one-to-one mapping from A into B.)
3.1. Prove that E and f (E) are isomorphic. Moreover, if X and Y are two
finite subsets of E such that X is isomorphic to f (X) and Y is isomorphic to f (Y ),
show that X ∪ Y and f (X ∪ Y ), and X ∩ Y and f (X ∩ Y ) are isomorphic sets,
respectively, and that for any two such sets we have f (X ∩ Y ) = f (X) ∩ f (Y ).
3.2. Prove that if E is finite and normal, then there exists a normal subset
X0 6= ∅ of E such that for any other normal subset X of E, we have
X ∩ X0 = ∅ or X0 ⊆ X.
(A subset A of E which is isomorphic to f (A) is called a normal subset of E.)

1.1.3. General. 1. A triangle with sides a, b, c such that a < b < c is given.
Set
a b c a b c
S = max , , min , , .
b c a b c a
Show that S ≥ 1.

2. Show that the decimal fraction 0.123456789101112131415 . . ., which is formed


by putting consecutive positive integers next to one another, is not periodic.

3. Find two distinct real or complex numbers in such a way that each of which is
the cube of the other.

4. Let x, y be two positive real numbers with x + y = 1. Prove that



xx + y y ≥ 2.

1.1.4. Differential Equations. Find the general solution of the following


differential equation
d2 x
+ a2 x = f (t),
dt2
where a is a constant and f is a continuous real function.
1.2. SECOND COMPETITION 3

1.1.5. Probability and Statistics. Let X1 , . . . , X10 be ten independent ran-


e−µi µX
i
i

dom variables with the probability density functions fi (Xi ) = , where


Xi
µi = i (i = 1, . . . , 10) and that the codomain of the variable Xi is {0, 1, 2, . . .}.
Set
10
1 X
X= Xi .
10 i=1
What is the probability that X equals one?

1.1.6. Topology. Let Q be the set of all rational numbers. For each q ∈ Q,
the set ]q, +∞[ is denoted by Aq . If T is the set consisting of R and ∅ and all Aq ’s,
show that T is not a topology on R. √
• Hint. Consider the sets of the form Aq with q > 2 and q ∈ Q.

1.2. Second Competition, Shiraz (former Pahlavi) University, March


1974

1.2.1. Analysis. 1. Let f be a real continuous function with nonnegative


values on the closed interval [0, 1]. Set
Z 1  n1
n
un = (f (x)) dx , M = sup f (x).
0 0≤x≤1

(a) Assuming that 0 < ε < 1, prove that there exist numbers α, β subject to
0 ≤ α < β ≤ 1 such that M (1 − ε) ≤ f (x) ≤ M for all x belonging to the open
interval ]α, β[.
(b) Prove that limn→+∞ un = M .

2. Evaluate 

Z 1

D = inf sup 1 − f (t) + 1 − f (t) dt ,
f ∈F 0≤t≤1 0

where F is the vector space of all continuous functions from [0, 1] into R whose
elements take the value zero at zero. Describe the geometrical interpretation of the
number D.

3. Assuming that C is the Cantor set, show that


C + C = [0, 2].
• Hint. 1. If A and B are two subsets of real numbers, by definition,

A + B := a + b : a ∈ A, b ∈ B .
2. The Cantor set consists of all real numbers between zero and one whose
ternary expansions do not have any one.
4 1. PROBLEMS

1.2.2. Algebra. 1. Let F be a field with n elements. Prove that for every
integer m with m ≥ 1, we have

X n−1 n − 1 | m,
xm =
0 n − 1 - m.
x∈F

• Hint. F = F \ {0} is a cyclic group.

2. Let A be a ring.
• Definition 1. An element z ∈ A is called right quasi-regular if there is a z 0 ∈ A
such that z + z 0 − zz 0 = 0.
• Definition 2. An element z ∈ A is called left quasi-regular if there is a z 0 ∈ A
such that z + z 0 − z 0 z = 0.
• Definition 3. An element z ∈ A is called quasi-regular if there is a z 0 ∈ A
such that z + z 0 − zz 0 = z + z 0 − z 0 z = 0.
(a) An element z ∈ A is quasi regular if an only if it is right and left quasi-
regular.
(b) If A has an identity element, then the identity element is not right quasi-
regular. And if x is a right quasi-regular element, then 1 − x is invertible1 in A.

3. Let E be a finite-dimensional vector space over a field K and φ a homomorphism


from the ring K[x] into the ring L(E) with the hypothesis that φ(1) is the identity
element of L(E).
(a) If P and D belong to K[x] and D is a divisor of P , prove that
 
ker φ(P ) ⊃ ker φ(D) .
(b) If R is another element of K[x] and D is the greatest common divisor of P
and R, prove that
  
ker φ(D) = ker φ(P ) ∩ ker φ(R) .
• Hint. By K[x], we mean the ring of all polynomials in one indeterminate x
with coefficients coming from K and L(E) is the ring of all endomorphisms of E.
For any u belonging to L(E), ker u means the kernel of the linear transformation
u.

1.2.3. General. 1. Find the minimum and the maximum number of “Wednes-
days occurring on the 13th of the month” which might happen in a solar year,
according to the Persian calendar that is.
Remark. Recall that there are 12 months in the Persian calendar, each of the
first six months has 31 days, each of the sixth-eleventh months has 30 days, and
the last month has 29 days except that in a leap year it has 30 days.

2. Prove that  x + y + z + t n xn + y n + z n + tn
≤ ,
4 4
where x, y, z, t are positive real numbers and n is a natural number.

3. A particle, moving along a straight line, goes one unit of distance in one unit of
time in such a way that the speed of it at the initial point as well as the final point
1“invertible” must read “right invertible”!
1.2. SECOND COMPETITION 5

is zero. Prove that [the absolute value of] the particle acceleration at some point
on its path is greater than or equal to four.

4. We know that in Iran the plate number of every car is a five-digit number
none of whose digits is zero. In big cities, like Tehran, in addition to a number a
[Persian] letter is also used to characterize the plate, e.g., “alef”, “be”, “pe”, etc.
By explaining your reasoning, find the total number of the car plates that contain
the letter “dal”.
Remark. The Persian alphabet has 32 letters.

1.2.4. Probability. Let X and Y be two random variables subject to the


following conditions.

var(X) = var(Y ) = σ 2 , cov(X, Y ) = λ,

where σ and λ are given real numbers. Define the random variables U and V by
U = X + Y and V = X − Y .
(a) Find the covariance of U and V .
(b) Are U and V independent? If so, prove it; if not, by an example justify
your answer.

1.2.5. Topology. Consider a nonempty topological space (E, T ) and two non-
empty subsets A and K of E with K ⊃ A. Suppose that B and C are two closed
subsets of the topological space (K, TK ) such that

A ⊂ B ∪ C, A ∩ C 6= ∅, A ∩ B 6= ∅.

Prove that if A is connected, then A ∩ B ∩ C 6= ∅.


• Hint. By TK we mean the induced topology which is induced by T on K.
The problem can be proved by contradiction.

1.2.6. Differential Equations. Find the general solution of the following


differential equation subject to the two cases |x| > 1 and |x| < 1.

3(x2 − 1)y 2 y 0 + xy 3 = x3 + x2 − x − 1.
6 1. PROBLEMS

1.3. Third Competition, (former Jondi Shapour) University of Ahwaz,


March 1975

1.3.1. Analysis. 1. Let f be a real function whose domain is [a, b]. If f 00


exists on [a, b] and is positive, then for all ξ ∈ [a, b],2 there exists a point x0 ∈ [a, b]
such that
f (a) − f (x0 ) f (b) − f (x0 )
f 0 (ξ) = or f 0 (ξ) = .
a − x0 b − x0

2. A sequence (an )+∞n=1 of numbers in the closed interval [0, 1] is given such that the
elements of the sequence are all distinct. The goal is to find a continuous function
f from [0, 1] into [0, 1] such that f (an ) = an+1 . Prove that
(a) in general, the problem has no solution.
(b) if the sequence is increasing or decreasing, the problem has a solution.

3. In each part of this problem, you can use the preceding parts.
(a) If the complex number z is a root of the following equation with complex
coefficients
xp + c1 xp−1 + · · · + cp−1 x + cp = 0,
then p
∃ k ∈ {1, . . . , p} : |z| ≤ 2 k |ck |.
(b) The following two equations with complex coefficients are given.
xp + c1 xp−1 + · · · + cp−1 x + cp = 0,
xp + c01 xp−1 + · · · + c0p−1 x + c0p = 0.
If the positive numbers K and δ satisfy the following
∀i = 1, . . . , p : |ci | < K i , |ci − c0i | < K i δ ,


then for any root zj of the first equation there exists a root zk0 of the second equation
such that √
zj − zk0 < 2K p δ.

(c) Suppose that the real numbers K and α are such that K > 0 and 0 < α ≤
1. Also suppose that the complex valued functions b1 , . . . , bp of a real variable t
belonging to the compact interval [t1 , t2 ] are such that
|bi (t)| < K i , |bi (t) − bi (t0 )| < K i |t − t0 |α ,
for all t, t0 ∈ [t1 , t2 ] and each i = 1, . . . , p. Prove that if the complex valued function
f is continuous on the closed interval [t1 , t2 ] and that it satisfies the equation
f p + b1 f p−1 + · · · + bp−1 f + bp = 0,
then p
f (t2 ) − f (t1 ) < 4pK p (t2 − t1 )α .
• Hint. If F : I → U is a function from an interval into an open subset of the
complex plane, the image of F lies in one of the connected components of U .

2 “ξ ∈ [a, b]” must read “ξ ∈ (a, b)”!


1.3. THIRD COMPETITION 7

1.3.2. Algebra. 1. Let M be a module on a ring K and M1 and M2 two


submodules of M . Set
M3 = M1 + M2 , M4 = M1 ∩ M2 .
M3 M2
(a) Show that the quotient modules and are isomorphic.
M1 M4
(b) If K is a commutative field and M1 and M2 are finite-dimensional, show
that
dim M1 + dim M2 = dim M3 + dim M4 .

2. Prove that a necessary and sufficient condition for a group to be commutative


is that the following function is a homomorphism of groups.
f : G → G, f (x) = x2 .

3. Suppose that for any element y of a semigroup S with a right identity element
(that is, xe = x for all x ∈ S), there exists a y with the property that yy = e.
(a) By giving an example, show that S might not be a group.
(b) If the right identity element is unique, prove that S is a group.

1.3.3. General. 1. The real function F (x, y) where x and y are two real
variables is considered. Suppose that this function has the following two properties.
3

∀x, y ∈ R : F (x, y) = 0 ⇐⇒ x = y,
∀x, y, z ∈ R : F (y, x) ≤ F (x, z) + F (z, y).
(a) Prove that F (x, y) ≥ 0 for all x, y ∈ R.
(b) Prove that F (x, y) = F (y, x) for all x, y ∈ R.

2. Without using mathematical induction, prove that


n  2
X n! (2n)!
= ,
m=0
m!(n − m)! (n!)2

for all n ∈ N.

3. Two flat mirrors ∆ and ∆0 which are perpendicular to a plane P and two points
M, N ∈ P are given. From the point M , shine a beam of light on the mirror ∆, in
the plane P , in such a way that the reflected light beam after hitting the mirror ∆0
passes through the point N .

3 “∀x, y ∈ R : F (x, y) = 0 =⇒ x = y” is redundant!


8 1. PROBLEMS

M
N

Figure 1

1.4. Fourth Competition, University of Tabriz, March 1976

1.4.1. Analysis. 1. In this problem R denotes the real line and R2 the Euclid-
ean plane. Prove that if f is a continuous and one-to-one function from R into R2 ,
its inverse is not necessarily continuous.

2. Let (αn )+∞


n=1 be a sequence of nonnegative real numbers such that

lim αn = 0.
n→+∞

Prove that there are infinitely many number of indices n such that αm ≤ αn for all
m greater than or equal to n.

1.4.2. Algebra. 1. Let A be a commutative ring and B an ideal of A. Set


R(B) = x ∈ A : ∃r ∈ N  xr ∈ B .


We call R(B) the radical of B. We call the ideal B of a commutative ring A a


prime ideal if
∀a, b ∈ A, ab ∈ B =⇒ a ∈ B or b ∈ B.
(a) Prove that R(B) is an ideal of A.
(b) Prove that if the ideal B is prime, then so is the ideal R(B).
(c) If we set A = Z, and if a is an element of Z and4


B = a = aZ = aA,
prove that there are prime numbers p1 , . . . , pr such that


R(B) = p1 p2 · · · pr .

2. Prove that in a vector space E, whose dimension is at least 2, for any two vectors
x and y which are not linearly dependent, one can find an automorphism u : E → E
such that u(x) = x and u(y) = x + y. From this, conclude that if an endomorphism
f : E → E commutes with any automorphism v (i.e., f ◦ v = v ◦ f ), then for every
vector x ∈ E, the two vectors x and f (x) are linearly dependent whence f is just
the scalar product of the identity automorphism by a fixed scalar (i.e., f = λidE ).

4 Obviously, we must have a 6= 0, ±1.


1.5. FIFTH COMPETITION 9

3. Let R be an integral domain with identity. Suppose that every descending chain
of the ideals of R terminates. (that is, for every chain of ideals of R as follows
I1 ⊇ I2 ⊇ · · · ⊇ Ik ⊇ · · · ,
there exists a positive integer n such that Im = In for all m ≥ n.) Prove that R is
a field.
• Hint. Assume that x is a nonzero element in R, investigate the ideals xi R
(i = 1, 2, . . .).

1.4.3. General. 1. Evaluate the following integral.


Z π2
sinn x
I= dx, n ∈ N.
0 cosn x + sinn x

2. We have three boxes A , B, and C, of which two are empty and one has a prize
in it. We choose one of these boxes at random (say, for example B); it is plain that
the probability that the chosen box contains the prize is 13 because of the three
boxes only one contains the prize. Now, suppose that of the two not-chosen boxes
A and C, we open the one that does not contain the prize (say, for example A).
Therefore, the prize is in one of the boxes B and C.
(a) What is the probability that the chosen box contains the prize?
(b) What is the probability that the other box, i.e., C, contains the prize?5

1.5. Fifth Competition, Sharif (former Aryamehr) University of


Technology, March 1977

1.5.1. Analysis. 1. If two functions are integrable on the interval (0, 1) in


the Riemann sense, is it true that the composition of the two functions is integrable
on the interval (0, 1) in the Riemann sense? Explain your reasoning.

2. Let R be the set of all real numbers and f a continuous function from R into R
that does not assume any value more than twice. Prove that f assumes at least a
value exactly once.

3. Let A be a subset of the real numbers R. A point p of A is called a congestion


point of A if every neighborhood of p of the form (p − ε, p + ε) contains uncountably
many points of A. Prove that all but countably many points of A are congestion
points of A (A is uncountable).

5 This problem is also known as the Monty Hall problem.


10 1. PROBLEMS

1.5.2. Algebra. 1. Let V be a finite-dimensional vector space over C (the


complex numbers). If A : V → V is a linear transformation such that A 6= I,
A2 6= I, A3 = I, find the eigenvalues of A. Extend this to the case where Ak = I.
Do the eigenvalues of A form a group under multiplication? Find a necessary and
sufficient condition for this set to form a group.

2. Let G be group and a and b two elements of it satisfying the following relations.
a 6= 1, b 6= 1, aba−1 = b2 , a7 = 1,
where 1 is the identity element of the group. Find the orders of a and b.

3. Let R be a ring with identity element such that every x ∈ R satisfies the
following relation: x3 + 2x2 + x = 0.
(a) Prove that 2x = 0 for all x ∈ R.
(b) Prove that R is a commutative ring.

(mn)!
1.5.3. General. 1. Let m, n ∈ N. Prove that the number is an
m!(n!)m
integer.

2. If S = {z1 , . . . , zk } is a subset of the complex numbers, define the set C(S) by



C(S) := z = α1 z1 + · · · + αk zk |αi ≥ 0, α1 + · · · + αk = 1 .
If f is a polynomial of degree greater than or equal to two and A and A0 denote
the sets of the roots of the equations f (z) = 0 and f 0 (z) = 0, respectively, where
f 0 denotes the derivative of f , prove that
C(A) ⊃ C(A0 ).

3. Consider the trajectory of a billiard ball B which moves on an ellipse-shaped


billiard table ξ. Suppose that the angle of incidence is equal to the angle of reflec-
tion. Assuming that the initial ball trajectory does not intersect the line segment
joining the the two foci, show that there exists another ellipse η which is confocal
with the ellipse ξ such that the ball trajectory is always tangent to η.
• Hint. Recall that if P L and P L0 are two tangents from a point P to an
ellipse, then the angles F P L and F 0 P L0 are equal. (F and F 0 are the foci of the
ellipse.)

1.6. Sixth Competition, The University of Isfahan, March 1978

1.6.1. Analysis. 1. In the xy-plane a point is called rational if both of its


coordinates are rational. Prove that if the center of a given circle in the plane is
not rational, then there are at most two rational points on the circle.

2. Suppose that in a metric space M a sequence (fn )+∞


n=1 of continuous functions
is uniformly convergent to a function f . Prove that for every sequence (xn )+∞
n=1
converging to a point x ∈ M , we have
lim fn (xn ) = f (x).
n→+∞
1.6. SIXTH COMPETITION 11

R +∞
3. If f is a continuous function on R and moreover 0 f (x) dx < +∞, prove
that
Z +∞
f (x + 1 ) − f (x) dx = 0.

lim
n→+∞ 0 n

1.6.2. Algebra. 1. Let G be a group with |G| = pn a, where p is a prime and


p and a are relatively prime. If G has subgroups A and B satisfying the following
conditions
|A| = pn , |B| = pm , 0 < m ≤ n, B * A,
prove that AB cannot be a subgroup of G.

2. Let A1 , . . . , An be mutually commuting m × m matrices such that A2i = 0 for


all 1 ≤ i ≤ n. If m < 2n , prove that A1 A2 · · · An = 0.

3. If R is a ring with identity (1 6= 0) and e and f are two commuting elements of


R such that e2 = e and f 2 = f , prove that

(e − f )n = 0 =⇒ e = f,

for all positive integers n.

1.6.3. General. 1. Let n1 , n2 , . . . , nk be k integers of which m1 , m2 , . . . , mk


is a permutation. Prove that

|n1 − m1 | + |n2 − m2 | + · · · + |nk − mk |

is an even integer.

2. An n × n matrix whose elements are nonnegative integers is given. If the sum


of the elements on any row and any column corresponding to any nonzero element
of the matrix is at least n, prove that the sum of all elements of this matrix is no
2
less than n2 .

3. Suppose 1000000 points inside a circle are given. Can one find a straight line
not passing through any of the points that divides the circle into two sections each
of which containing 500000 of the points?
12 1. PROBLEMS

1.7. Seventh Competition, Ferdowsi University of Mashhad, March


1980

1.7.1. Analysis. 1. Is there a closed set S $ R2 such that for each x ∈ R2 \ S


there are exactly two points in S as the closest point of S to x?

2. For each positive rational number r (r ∈ Q+ ), suppose Ir S


is an open interval
such that r < s =⇒ I r ⊂ Is . The function f on the set A = r∈Q+ Ir is defined
by

f (x) = inf r : x ∈ Ir .
Show that f is continuous.

3. Suppose that the sequence (xn )+∞


n=1 is defined by

n
n + 1 X 2k
xn = n+1 , n = 1, 2, . . . .
2 k
k=1

Prove that limn→+∞ xn exists and find its value.

1.7.2. Algebra. 1. (a) If in a ring R the element e is a left identity element


(i.e., ex = x, ∀x ∈ R) that is unique, show that e is the identity element of this ring
(i.e., ex = xe = x, ∀x ∈ R).
(b) If zero is the only nilpotent element of the ring R, show that for every
idempotent element a of R we have

ax = xa, ∀x ∈ R.

• Note. The element b of a ring R is called nilpotent if there exists a positive


integer n such that bn = 0; b is called idempotent if b2 = b.

2. Let G be a group and G1 a normal subgroup of G such that the groups GG1
and G1 are commutative. Prove that for any arbitrary subgroup H of G, there is
a commutative subgroup H1 of H such that H1 is a normal subgroup of H and HH1
is commutative.

3. An n-dimensional vector space V and a linear transformation θ : V → V are


given. Consider the powers of θ, i.e., θ0 = 1, θ, θ2 , . . .. Prove that there is a nonzero
integer s such that
V = im(θs ) ⊕ ker(θs ).
1.8. EIGHTH COMPETITION 13

1.8. Eighth Competition, Shiraz University, March 1984

1.8.1. Analysis. 1. Let F and G be two closed subsets of Rn , where R


denotes the real numbers endowed with its ordinary topology. Suppose that f :
F → R and g : G → R are two continuous functions which coincide on F ∩ G.
(a) Prove that there is a continuous function h : F ∪ G → R such that h is an
extension of both of the functions f and g.
(b) By giving an example, show that if F and G are not closed, the assertion
in (a) is not correct.
P+∞
2. Suppose that (an )+∞n=1 isP
a decreasing sequence of real numbers and that n=1 an
+∞
is convergent. Prove that n=1 n(an − an+1 ) is convergent.6

3. Let the function g be continuously differentiable on [0, 1]. Prove that


Z 1
lim xn dg(x) = 0.
n→+∞ 0

1.8.2. Algebra. 1. In the multiplicative group of nonsingular 2 × 2 matrices


with real entries, let    
2 0 1 1
a= , b= .
0 1 0 1
If H is the subgroup generated by b, show that aHa−1 is a proper subgroup of H.

2. Let R be an infinite integral domain (i.e., a commutative ring with identity


without divisors of zero). If R has finitely many units, prove that R has infinitely
many maximal ideals. (A unit is an element which has a multiplicative inverse.)

3. Let A =P(aij ) be an n × n matrix over the field of real numbers such that for all
n
i we have j=1 aij = a. If A2 = I (I is the identity matrix), find a.

1.8.3. General. 1. Without using derivative, find the minimum of the three
variable real function
f (x, y, z) = x2 + 4y 2 + z 2 − 6x + 4y.

2. Five people have 719 rials altogether. If each person has an integer amount of
money, that the money of no two people is equal, and that the ratio of a person’s
money with respect to that of any other person with less amount of money is an
integer, determine how much money each person has.

3. A bus which must pass through the city A passes through the crossroad B of the
city with the probability of 31 . If the traffic-light of the crossroad is, consecutively,
30 seconds red and 30 seconds green, find the average of the stop time of the bus
at the crossroad.

6 For a counterpart of this problem for sequences of real functions, see Problem 2 of 1.12.1.
14 1. PROBLEMS

1.9. Ninth Competition, Tehran Teacher Training (Tarbiat Moallem)


University, September 1985

1.9.1. Analysis. 1. On the set of real numbers R, define the equivalence


relation ∼ as follows
a ∼ b ⇐⇒ a − b ∈ Q,
where Q is the set of rational numbers. Prove that every equivalence class of ∼ is
dense in R.

2. Prove that if f is a continuous and one-to-one function from R into R, then the
inverse of f (from f (R) onto R) is continuous as well.

3. Let f be a continuous
 and increasing function
from [a, b] into [a, b] and f (a) = a.
Prove that if E = x|a ≤ x ≤ b, f (x) ≥ x , then f (E) = E.

4. Let the function f be defined on [0, 1] by



0 x∈ / Q,
f (x) = 1
q x ∈ Q, x = pq , p, q ∈ N, gcd(p, q) = 1.
R1
Prove that 0 f (x)dx exists.

5. Let f : [0, 1] → [0, 1] × [0, 1] be a function. Prove that the function f can have
any two properties of the following three properties but cannot have more than two
properties.
Continuity, Injectivity, Surjectivity.

1.9.2. Algebra. 1. Let G be a finite group and p the smallest prime dividing
the order of G. Prove that every subgroup of index p in G is a normal subgroup of
G.

2. Let Z3 denote the field of integers mod 3. Define, explicitly, an isomorphism


Z3 [x] Z3 [x]
from the field 2 onto the field 2 .
(x + 1) (x + x + 2)

3. Find the greatest common divisor of the two polynomials 4x4 − 2x2 + 1 and
−3x3 + 4x2 + x + 1 in Z7 [x].

4. A quadratic extension K ⊇ F such that ch(F ) 6= 2 is given (here, ch(F ) denotes


the characteristic of the field F ). Prove that there exists an element y ∈ K such
that y 2 ∈ F and that {1, y} is a basis for K over F .

5. Let F be a field with characteristic 2, V an n-dimensional vectorspace over F ,


2
T : V → V a linear transformation such that T = I. Set W = v ∈ V : T v =
and
v . Prove that dim(W ) ≥ n/2. (Here, I denotes the identity transformation on
V .)
1.9. NINTH COMPETITION 15

1.9.3. General. 1. Evaluate the determinant of an n × n matrix whose di-


agonal entries are all equal to r and whose off-diagonal entries are all equal to
λ.

2. A husband and a wife, who are working at one of the poultry farms of the
country, have posed a problem on the number of their chickens as follows. If they
sell 75 of their chickens, their chicken food will finish twenty days after they run out
of it. But if they buy another one hundred chicken, their chicken food will finish
fifteen days before they run out of chicken food. Find the number of chickens in
the chicken farm.

3. Let α be a real number. Prove that there is no positive continuous function


f : [0, 1] → R such that
Z 1 Z 1 Z 1
f (x)dx = 1, xf (x)dx = α, x2 f (x)dx = α2 .
0 0 0

4. Find the number of the solutions of the following equation in the set of positive
integers.
x1 + x2 + · · · + xm = n,
where m and n are natural numbers with m < n.

5. On a square grid paper the following figure is drawn. Can you cut the paper
along the lines and divide it into two pieces in such a way that putting the pieces
next to one another forms a chessboard?

6. The start hour in a factory is 8 o’clock in the morning. A worker has estimated
that, by car, s/he goes the distance from home to work within 10 to 20 minutes
(with uniform distribution).
(a) If this worker leaves his/her home at 7 34 , what is the probability that s/he
does not get to work on time?
(b) If 15 minutes is needed to have breakfast at the work place, determine the
latest time that this worker can leave home to get to work on time and with the
probability of 75% to have time to eat breakfast.

7. Let c1 , . . . , cn be n real numbers. Consider the matrix (ci cj )n×n . Evaluate


det I + (ci cj ) , where I is the identity matrix of size n.
16 1. PROBLEMS

1.10. Tenth Competition, University of Sistan and Baluchestan, March


1986

1.10.1. Analysis. 1. The real function f satisfying the following conditions


is given.
(a) f (1) = 1.
1
(b) f 0 (x) = 2 for all x ≥ 1.
x2 + f (x)
Prove that limx→+∞ f (x) exists and is less than 1 + π4 .

2. The continuous function f : [a, b] → R satisfying the following conditions is


given.
(a) f (a) = f (b) = 0.
(b) The second derivative of f on the interval (a, b) exists [and is bounded].
Show that
Z b 3
f (x) dx ≤ M (b − a) ,

a 12
00
where M = sup{ f (x) : x ∈ (a, b)}.
• Hint. Use the auxiliary function g(t) = (t − a)(t − b)f (x) − (x − a)(x − b)f (t).

3. Prove that
3
x2 (1 − x)xn
Z
lim dx = 0.
n→+∞ 0 1 + x2n

1.10.2. Algebra. 1. Prove that the additive group of R has no maximal


subgroup.

2. Let A, B, C, D be ideals in a unital ring R such that

A + B = A + C = A + D = R. (∗)

If M = B ∩ C ∩ D, prove that A + M = R.
Give an example of a unital ring R having ideals A, B, C, D which satisfy (∗).

3. Let E be an extension field of F and α ∈ E algebraic on F of degree n. If m < n


and gcd(n, m!) = 1, then show that F (α) = F (αm ).

4. Prove that no set of nilpotent matrices can span Mn (F ), where Mn (F ) is the


vector space of all n × n matrices over the field F .
1.11. ELEVENTH COMPETITION 17

1.10.3. General. 1. Consider a test in which the probability of success (S)


is equal to p and the probability of failure (F) is equal to q = 1 − p. Perform this
test two times. If the results turn out to be F S or SF , set the random variable X
to be 0 or 1, respectively. If not, do the test another two more times. Again if the
results of the two tests are F S or SF , we take the random variable X to be 0 or 1,
respectively; and if not, we perform the test another two more times... Prove that
P {X = 0} = P {X = 1} = 12 .

2. Show that every closed set A in the plane with the property that A◦ = ∅ is the
boundary of an open set in the plane. (Here, A◦ denotes the interior of the set A.)

3. Prove that every solution of the differential equation (∗) is a solution of the
integral equation (∗∗) and vice versa.
x00 = f (t, x), x(0) = x0 , x0 (0) = y0 , (∗)
where f (t, x) is a continuous function in a region D which contains the point (0, x0 ).
Z t
x(t) = x0 + y0 t + (t − s)f (s, x(s))ds. (∗∗)
0

4. Prove that the product of r consecutive natural numbers is divisible by r! and


n!
from there conclude that is always an integer.
r!(n − r)!

1.11. Eleventh Competition, The University of Birjand, March 1987

1.11.1. Analysis. 1. The function f : [0, 1] → R is differentiable and f and


f 0 have no common zero. Prove that the set of zeros of f in [0, 1] is finite. (Recall
that a solution of the equation f (x) = 0 is called a zero of the function f .)

·x
··
2. (a) Show that the sequence of functions fn (x) = xx is pointwise convergent
1
to a function f on [1, e ] which satisfies the functional equation f (x) = xf (x) .
e
1 +∞
• Hint. Prove that for all a ∈ [1, e e ] the sequence fn (a) n=1 is increasing and
bounded. Then, in view of the fact that the function g(x) = ax is continuous on
R, show that f (a) = af (a) .
(b) Show that the function f is one-to-one. Then using the inverse of f , con-
clude that f is continuous. And then, prove that the sequence (fn )+∞ n=1 converges
1
uniformly on [1, e ].
e

0
 00n ∈ N, ϕn : [−1, 1] → R has second order derivative
3. Suppose that for every
P+∞
and
that ϕn (0) = 1. If sup ϕn (x) : n ∈ N, x ∈ [−1, 1] < +∞, prove that n=1 an is

R1
convergent, where an = n1 −1 ϕn (t) cos nπtdt .
18 1. PROBLEMS

1.11.2. Algebra. 1. Suppose that the ring R has exactly two two-sided ideals.
Prove that if there is an element u in R for which ux = x for all x ∈ R, then R is
a ring with identity and u = 1R .

2. Prove that A, the field of algebraic numbers, is not a finite extension of Q.

3. Let G be a finite group with the following property: for every two elements x
and y of G with x 6= e and y 6= e, where e is the identity element of G, there exists
an automorphism θ ∈ Aut(G) such that y = θ(x). Prove that there is a prime
number p such that
G∼= Zp ⊕ Zp ⊕ · · · ⊕ Zp .

1.11.3. General. 1. The matrix A is chosen at random from the set of all
2 × 2 matrices with entries from Z. What is the probability that the determinant
of A is an even number?

2. A real number c is given. Show that if one of the roots of the equation
3
x3 − x + c = 0
4
is in the closed interval [−1, 1], then all of the roots of this equation are in the
interval [−1, 1].

3. Let X be a set with n elements and C a family of the subsets of X which satisfies
the following conditions.
(i) If A, B ∈ C, then A ∪ B ∈ C.
(ii) If A ∈ C, then Ac := X \ A ∈ C.
(iii) ∅ ∈ C.
Show that the number of the elements of C is equal to 2k where k ≤ n.

1.12. Twelfth Competition, Guilan University , March 1988

1.12.1. Analysis. 1. Let f : R → R be a continuous function. Moreover,


suppose that there is an M > 0 such that for all x, y ∈ R

f (x) − f (y) ≥ M x − y .
Show that the function f is one-to-one and surjective.

2. Let (fn )+∞


n=1 be a decreasing sequence of nonnegative functions on a nonvoid set
S. That is,
∀n ∈ N : fn : S −→ R, fn ≥ fn+1 , fn ≥ 0.
P
Prove that the series n≥1 fn converges uniformly on S if and only if the series
7
P
n≥1 n(fn − fn+1 ) does.

7 This problem, as stated, is not correct! More precisely, for the “if part” of the problem, we
need to assume further that the sequence (fn )+∞
n=1 converges uniformly to zero on S.
1.12. TWELFTH COMPETITION 19

3. Let g : [0, 1] → R be a Riemann integrable function. Prove that


Z 1 Z 1  Z 1
g(t)dt dx = tg(t)dt.
0 x 0

1.12.2. Algebra. 1. Let G be a group of order 2p, where p is an odd prime.


Prove that G has one and only one subgroup of order p. Also prove that G has p
subgroups of order p, or one subgroup of order p. In the latter case, show that G
is a cyclic group.

2. Let G be an abelian group of odd order and φ a homomorphism of G of order 2.


Show that every element g ∈ G can, uniquely, be written as g = xy where φ(x) = x
and φ(y) = y −1 .

3. Let R be a unital ring with characteristic 2 such that for every x 6= 1 and y 6= 1,
we have xy 2 = xy. Show that R is commutative.

4. Suppose that R is a unital ring, that every ideal of R is principal, and that
f : R → R is a surjective homomorphism. Prove that f is an isomorphism.

5. Let A be a 3 × 3 invertible matrix over a field F such that det A = 1 and


tr(A) = tr(A−1 ) = 0. Prove that A3 = I.

6. If α and β are two distinct roots of the equation


x2 xp
1+x+ + ··· + = 0,
2! p!
where p is a prime with p > 2, show that α − β and α + β and αβ are irrational
numbers.

1.12.3. General. 1. A circle C centered at O is given and the center of a


square is on the circle. If the area of the square is not greater than half of the area
of the circle, prove that one of the vertices of the square is inside the circle.

2. 1700 people have participated in a true-false test. Knowing that 15 questions


have been given in the test, that none of the participants has answered two con-
secutive questions correctly, and that all of them have answered all questions, do
there exist two equal answer sheets?

3. Evaluate
1 − Cn2 + Cn4 − Cn6 + · · · ,
n!
where n ∈ N, Cnk := if k = 0, . . . , n, and Cnk := 0 otherwise.
k!(n − k)!
20 1. PROBLEMS

1.13. Thirteenth Competition, University of Tehran, March 1989

1.13.1. Analysis. 1. Let f : R → R be (Riemann) integrable and f (x + y) =


f (x) + f (y) for all x, y ∈ R. Show that there exists a number c such that f (x) = cx
for all x ∈ R.

2. Show that if an , bn are in R for all n ∈ N, (an + bn )bn 6= 0, and that both series
+∞ +∞ +∞
X an X an 2 X an
and are convergent, then so is the series .
b
n=1 n n=1
b n n=1 n
a + bn

3. Let (fn )+∞


n=1 , with fn : [0, 1] −→ R, be a sequence of differentiable functions
such that ||fn0 ||∞ ≤ 1 for all n ∈ N. Show that if for any continuous function
g : [0, 1] −→ R, we have
Z 1
lim fn g = 0,
n→+∞ 0
then the sequence (fn )+∞
n=1 converges uniformly to zero on [0, 1].

1.13.2. Algebra. 1. Let R be a unital, commutative, and uncountable ring


such that for every ideal 0 6= I C R, the quotient ring R/I is countable. Prove that
R is an integral domain.

2. Let R be a subring of Mn (Q). If


 
α 0
∈ R for all α ∈ Q, (∗)
0 α
prove that every left (or right) ideal of R is finitely generated. By finding a left
ideal in the ring   
z q1
T = |z ∈ Z, q1 , q2 ∈ Q ,
0 q2
which is not finitely generated, show that the condition (∗) cannot be dropped.

3. Give an example of a group G which contains two elements a and b such that a
and b are of order two but ab is of order infinity.

4. Let G = GL2 (Z3 ) (the multiplicative group of all 2 × 2 invertible matrices on


Z3 ), K = Z(G) denote the center of G, and
  
a b
H= : a, b, c ∈ Z3 , ac 6= 0 .
0 c
(a) Prove K ≤ H ≤ G and that the order of H is 12.
(b) Prove
\
K= x−1 Hx.
x∈G
(c) Prove
G ∼
= S4 ,
K
where S4 denotes the group of all permutations on four letters.
1.14. FOURTHEENTH COMPETITION 21

5. A = (aij ) is an n × n matrix, where



δi,n j = 1,
aij =
δi,j−1 j > 1.

Suppose that ξ is an nth root of unity in C. Set


 
1
 ξ 
2
 
ν(ξ) =  ξ .
 
 .. 
 . 
ξ n−1

Prove that ν(ξ) is an eigenvector of A and find its corresponding eigenvalue.

1.13.3. General. 1. A regular n-gon is inscribed in a circle with radius one.


Choose an arbitrary point on the circle and find the squares of the distances from
the point M to the vertices of the regular n-gon. Prove that the sum of these values
is 2n.

2. Find all polynomials f (x) with rational coefficients with the property that f (x)
is irrational whenever x is irrational.

3. Let n parallel lines in the space that do not lie in a plane be given. Show that
there are at least n distinct planes each of which passes through at least two lines
of these given lines (n > 2).

1.14. Fourteenth Competition, The University of Isfahan, March 1990

1.14.1. Analysis. 1. Let f : R −→ R be twice differentiable on R \ {x0 } for


some x0 ∈ R. If f 0 (x) < 0 < f 00 (x) on x < x0 , and f 0 (x) > 0 > f 00 (x) on x > x0 ,
then f is not differentiable at x0 .

2. Let g : R −→ R be a continuous function with the property that g(x) > 0 for
x 6= 0 and g(0) = 0. Let f : R −→ R be a uniformly continuous and bounded
function such that g ◦ f is integrable on R. Prove that limx→∞ f (x) = 0.

3. Let f : R −→ R be a continuous
 function and [a, b] and [c, d] two closed intervals
such that [a, b] ⊂ f [c, d] . Prove that there exists a subinterval [r, s] of [c, d] such
that f [r, s] = [a, b].
22 1. PROBLEMS

1.14.2. Algebra. 1. Prove that if G is a finite p-group (p a prime number),


then G0 6= G, where G0 is the commutator (a.k.a. derived) subgroup of G.

2. Let G be a finite group and K a normal subgroup of G of order p, where p is


the smallest prime dividing |G|. Prove that K is a subgroup of Z(G), where Z(G)
denotes the center of the group.

3. Give an example of a ring R having two elements which have the greatest
common divisor in R but do not have the least common multiple in R.

4. Prove that if F is a field and n an integer greater than one, then xn y + xn−1 + 1
is irreducible in F [x, y]. What can be said in case n = 1?

5. Let V be a finite-dimensional vector space over a field F and V1 and V2 two


subspaces of V with dim V1 = dim V2 . Prove that there exists a subspace U of V
such that
U ⊕ V1 = U ⊕ V2 .

1.14.3. General. 1. Let S be a set with n elements, and



A = A1 , . . . , An

a family containing n distinct subsets of S. Show that there exists an element x ∈ S


such that the sets
A1 ∪ {x}, . . . , An ∪ {x}
are distinct.

2. A publisher is to exhibit 1369 titles of its published books. The books are to
be exhibited as follows. Each day 100 titles are to be placed on the exhibition
table and that no two titles are placed on the table more than once. Determine the
maximum number of the days of the publisher exhibition.

3. Find all nonzero real numbers a1 , . . . , an with the following property:


n
X n
X
am
i = ai , m = 1, . . . , n + 1.
i=1 i=1
1.15. FIFTEENTH COMPETITION 23

1.15. Fifteenth Competition, Ferdowsi University of Mashhad, March


1991

1.15.1. Analysis. 1. The real function f is defined on [0, +∞) and is increas-
ing. The function ϕ is defined on [0, +∞) by
Z x
ϕ(x) = f (t)dt.
0
(a) Prove that for all x, y ≥ 0
x + y  1 
ϕ ≤ ϕ(x) + ϕ(y) .
2 2
(b) Conclude that ϕ is convex.

2. The real function g is continuous on [0, 1] and g(0) = 0. Define the sequence
(fn )+∞
n=1 of functions on [0, 1] by
g(x)(sin x)n
fn (x) = .
1 + nx
Prove that the sequence (fn )+∞
n=1 is uniformly convergent on [0, 1].

3. A function f from (0, +∞) into (0, +∞) is defined; it is a one-to-one correspon-
dence and for all x, y ∈ (0, +∞), we have
2xy ≤ xf (x) + yf −1 (y).
(f −1 is the inverse of f .)
(a) Show that for all x, y ∈ (0, +∞),
y−x y−x
f (x) ≤ f (y) − f (x) ≤ f (y).
y x
(b) Conclude that there exists a c ∈ R such that f (x) = cx for all x ∈ (0, +∞).

1.15.2. Algebra. 1. Let G be a group of order pα m, where p is an odd prime,


a ∈ N, p - m, and that G has exactly (1 + p) Sylow p-subgroups. Show that
p+1
\
Pi = pα−1 ,
i=1
where Pi ’s are the Sylow p-subgroups of G.
• Hint. [G : A ∩ B] ≤ [G : A][G : B].

2. Let R be a unital ring such that every left ideal of it is also a right ideal of
R. Prove that the intersection of all prime ideals of R is equal to the set of the
nilpotent elements of R.

3. Show that for every n × n matrix A, there is an n × n matrix B such that AB


is an idempotent. 8

8 This problem, as stated, is trivial! To make the problem nontrivial, prove that for every
n × n matrix A with entries from a field F , there is a matrix B ∈ Mn (F ) such that AB is an
idempotent whose rank is equal to that of A (see Solution 3 of 2.15.2).
24 1. PROBLEMS

1.15.3. General. 1. Find a necessary and sufficient condition for the follow-
ing property: the product of two integers is divisible by the sum of them.

R1
2. Let f be nonnegative on [0, 1] and 0
f (x)dx = 1. Prove that
Z 1  Z 1 2
1
x− uf (u)du f (x)dx ≤ .
0 0 4

3. A train has n wagons. Any passenger who is boarding the train (independent
of others), chooses a wagon at random to get on.
(a) What is the probability that at least one passenger gets on every wagon?
(b) Using (a), evaluate the following sum.
       
n p n p n p n−1 n
1 − 2 + 3 − · · · + (−1) np ,
1 2 3 n
where 1 ≤ p ≤ n.

1.16. Sixteenth Competition, Razi (Rhazes or Rasis) University of


Kermanshah, March 1992

1.16.1. Analysis. 1. Let g : [0, 1] −→ R be a continuous function with


g(1) = 0. If fn (x) = xn g(x), show that the sequence (fn )+∞
n=1 converges uniformly
on [0, 1].

2. The function f : R −→ {0, 1, ..., 9} is defined by



a2 x 6= [x],
f (x) =
9 x = [x],

where [x] denotes the integral part of x and a2 is the second digit of the decimal
expansion of x − [x].
(a) Prove that f is periodic and determine its period.
(b) If c is the period of f , evaluate the following integral
Z c
xdf (x).
0

(If the decimal expansion of a number ends in zero, reduce the last nonzero digit
by one and change all other digits on the right of it to 9.)

3. Let f : R −→ R be a uniformly continuous


function. Prove that there are
positive numbers a and b such that f (x) ≤ a x + b for all x ∈ R.
1.17. SEVENTEENTH COMPETITION 25

1.16.2. Algebra. 1. Let G be a finite nonabelian group, and A, B two


distinct abelian subgroups of G such that

[G : A] = [G : B] = p,

where [G : A] and [G : B] denote the indexes of A and B in G, respectively, and


p is the smallest prime dividing |G|. Prove that Inn(G) = Zp × Zp , where Inn(G)
stands for the set of all inner isomorphisms of G.

2. Let R be a ring and r ∈ R be such that r − r2 is nilpotent. Prove that R has a


nonzero idempotent element whenever r is not nilpotent.

3. Let A = (aij ) ∈ Mn (Q), where aij = gcd(i, j). Is A invertible? Why?

1.17. Seventeenth Competition, Shahid Beheshti (former National)


University, March 1993

1.17.1. Analysis. 1. Suppose that the function f is continuous on the inter-


val [a, b] and that it is differentiable on (a, b). Also suppose that the graph of f is
not a line segment. Prove that there exists a c ∈ (a, b) such that
0 f (b) − f (a)
f (c) >
b−a
.

2. If p1 (x), p2 (x), p3 (x), p4 (x) are real polynomials in the indeterminate x, prove
that
Z x Z x Z x Z x
p1 (t)p3 (t)dt p2 (t)p4 (t)dt − p1 (t)p4 (t)dt p2 (t)p3 (t)dt
−1 −1 −1 −1

is divisible by (x + 1)4 .

3. Let X be a metric space and f : X −→ R a continuous function. Show that


the set {x ∈ X : f (x) = 0} is an open subset of X if and only if there exists a
continuous function g : X −→ R such that f = gf 2 .

4. Let f : [0, 1] −→ R be a continuous function such that f (0) = f (1). Prove that
there are two points a, b satisfying the following: 0 < a ≤ b ≤ 1, b − a = 12 , and
f (b) = f (a).
26 1. PROBLEMS

1.17.2. Algebra. 1. Let G be a finite group and H ≤ G be such that


/ H =⇒ H ∩ (x−1 Hx) = {eG } .

∀ x ∈ G if x ∈
Prove that [G : H] and |H| are relatively prime.

2. Let R be the ring of all n × n matrices over a field F , and R[x, y] the ring of
all polynomials in two indeterminates x, y with coefficients from R. (note: ∀a ∈
R : ax = xa, ay = ya, xy = yx.) Suppose f, g ∈ R[x, y] are such that f g = 1.
Determine gf .

3. Let V be a finite-dimensional vector space over a field F and T : V −→ V a


linear transformation. Determine the dimension of ker T ∩ T (V ) in terms of the
ranks of the powers of T .

1.18. Eighteenth Competition, Sharif University of Technology, March


1994

1.18.1. Analysis. 1. Let (an )+∞


n=0 be a sequence of positive reals and An =
an
a0 + · · · + an . If limn→+∞ An = +∞ and lim = 0, show that the radius of
n→+∞ An
P+∞
convergence of the power series n=0 an xn is one.

2. Let f, g be continuous and periodic functions whose periods are one. Show that
Z 1 Z 1  Z 1 
lim f (x)g(nx)dx = f (x)dx g(x)dx .
n 0 0 0

3. For any real number x, write the binary expansion of x − [x] as follows

X ak
x − [x] = .
2k
k=1
Pn
j=1 aj
Show that the function g : R −→ R defined by g(x) = lim sup maps every
n→∞ n
nonempty interval onto [0, 1] (i.e., for every nonempty interval I, we have g(I) =
[0, 1]). Using this, construct an open map from R into R that is not continuous.

1.18.2. Algebra. 1. Let G be a finite group and K a normal subgroup of G


such that K as a group is simple and the square of its order does not divide the
order of G. Prove that if H is a subgroup of G such that H ∼
= K, then H = K.

2. The following ring is given.


    
Z Q a b
R= = : a ∈ Z, b, c ∈ Q
0 Q 0 c
Prove that every ascending chain of right ideals of R is necessarily stable; that is,
if
I1 ⊆ I2 ⊆ I3 ⊆ · · · , ∀j ∈ N : Ij Cr R,
then there is an n ∈ N such that In = In+1 = In+2 = · · · .
1.19. NINETEENTH COMPETITION 27

3. Let V be a finite-dimensional vector space over a field F . If {Vi }i∈I is a set of


proper subspaces of V such that dim Vi = dim Vj for all i, j ∈ I and if |I| < |F |,
prove that there is a proper subspace U of V such that
V = Vi ⊕ U,
for all i ∈ I.

1.19. Nineteenth Competition, University of Kerman, March 1995

1.19.1. Analysis. 1. If f : [0, 1] −→ [0, 1] is a continuous function, show that


the equation Z x
2x − f (t)dt = 1
0
has only one root in [0, 1].

2. Let f : (0, +∞) −→ R be a continuously differentiable function such that


limx→+∞ f (x) + hf 0 (x) = 0 for some h ∈ R+ . Show that limx→+∞ f (x) =
limx→+∞ f 0 (x) = 0.

3. Let A be the set of all continuous real functions on [0, 1] 9 and, for each n ∈ N,
En denote the set consisting of all f ∈ A for which there is an a ∈ [0, 1] such that
|f (x) − f (a)| ≤ n|x − a|,
for all x ∈ [0, 1].
(a) Show that En ’s are closed sets with empty interior.
(b) Conclude that there is a continuous function on [0, 1] which is nowhere
differentiable.

1.19.2. Algebra. 1. Let a group G be given. Suppose that the set


 G ∼
A= N CG: =G
N
is ordered by inclusion and that A has a maximal element. Prove that the trivial
group {e} is the only maximal element of A.
P
2. Let R be a ring with identity. Set S = J, where J can be any minimal left
ideal of R (in case such ideals are non-existent, set S=0).
(a) Prove that S is a two-sided ideal in R (i.e., S C R).
(b) If S 6= 0 and the product of anyT two nonzero two-sided ideals of R is a
nonzero two-sided ideal of R, then S = I, where I can be any nonzero two-sided
ideal of R.

3. A square matrix A = (aij )n×n over a field F is given. Suppose that there are
scalars xi , yi ∈ F such that aij = xi + yj (1 ≤ i, j ≤ n). Prove that rank(A) ≤ 2.

4. Let F be a field, n ∈ N, and A ∈ Mn (F ) with rank(A) = 1. Prove


det(I + A) = 1 + tr(A).
9 The set, in fact the algebra, A is assumed to be endowed with the uniform metric of A,
which is induced by the uniform norm of A, usually denoted by ||.||∞
28 1. PROBLEMS

1.20. Twentieth Competition, Sharif University of Technology,


February 1996

1. Let C be the field of complex numbers and


(x, y), (z, t), (x0 , y 0 ), (z 0 , t0 ) ⊆ C2 .


Prove that there are scalars α, β ∈ C which are not simultaneously zero such that
the vectors θ = α(x, y) + β(z, t) and θ0 = α(x0 , y 0 ) + β(z 0 , t0 ) are linearly dependent.

2. The sequence (an )+∞


n=1 of nonnegative real numbers satisfies the following prop-
erty:
1 + am+n ≤ (1 + am )(1 + an ),

for all m, n ∈ N. Prove that the sequence (xn )+∞
n=1 defined by xn =
n
1 + an is
convergent.

3. Let G be a group such that for all σ ∈ Aut(G) and for all x ∈ G we have
σ(x) = x or σ(x) = x−1 . Prove that G is solvable.

4. The function f : ( 41 , 1) −→ R for all x ∈ ( 41 , 1) satisfies the following


xf (x) = f (x).
Prove that f is uniformly continuous on ( 14 , 1).

5. Let R be a commutative ring with identity and with the following properties:
(a) The intersection of all of its nonzero ideals is nontrivial.
(b) If x and y are zero divisors in R, then xy = 0.
Prove that R has exactly one nontrivial ideal.

6. Let f , g : [0, +∞) −→ R be two functions. Suppose that Rf is decreasing and


p
limx→+∞ f (x) = 0 and that g is a periodic function such that 0 g = 0, where p is
R +∞
the period of g. Show that 0 f g is convergent.

1.21. Twenty First Competition, University of Tehran, March 1997

1.21.1. Analysis. 1. A set S ⊂ R and a function f : S −→ R are given.


 (a) Assume that α is a limit point of the set S and that for every ε > 0 the set
x ∈ S : |f (x)| ≥ ε is finite. Show that limx→α f (x) = 0.
(b) Assume that S is compact and  that for every limit
point α of S, we have
limx→α f (x) = 0. Show that the set x ∈ S : |f (x)| ≥ ε is finite for all ε > 0.

2. Let g : R −→ R be a continuous function satisfying the following



lim g(x + t) − g(x) = 0,
x→+∞

for all t ∈ R.
(a) Suppose that K is a compact subset of R. Show that

∀ε > 0 ∃M > 0  x > M ⇒ g(x + t) − g(x) < ε (∀ t ∈ K).
1.22. TWENTY SECOND COMPETITION 29

(b) Use (a) to show that


Z x+1 
g(x)
lim g(u)du − g(x) = 0, lim = 0.
x→+∞ x x→+∞ x

3. Let f : [0, 1] −→ R be a bounded function whose limit from the left exists at
any point in [0, 1]. Prove that f is Riemann integrable on [0, 1].

1.21.2. Algebra. 1. Let R be a ring and A and B ideals of R with A ⊆ B.


s
By definition, A is said to be small in B and we write A ⊆ B if for every ideal C
s s
of R for which A + C = B, then C = B. Show that if A ⊆ B and C ⊆ D and
s
A ∩ C = B ∩ D = {0}, then A + C ⊆ B + D.10

2. Let G be a group and H a subgroup of G such that H ≤ Z(G), where Z(G)


denotes the center of G. Prove that if [G : H] = p2 , where p is prime, then the
derived subgroup of G is cyclic.

3. Let A be a nonzero real n × n matrix such that


A = (aij )1≤i,j≤n , aik ajk = akk aij , ∀i, j, k.
Prove that
(a) tr(A) 6= 0.
(b) The matrix A is symmetric. 
(c) The characteristic polynomial of A is equal to xn−1 x − tr(A) .

1.22. Twenty Second Competition, University of Ahwaz, March 1998

1.22.1. Analysis. 1. Let f : [0, 1] −→ R be a differentiable function with


f (0) = f (1) − 1 = 0. Show that for each n ∈ N, there are x1 , . . . , xn ∈ [0, 1] such
n
X 1
that 0 (x )
= n.
i=1
f i

2. Let X be a metric space, f : X −→ R a continuous function, and (fn )+∞ n=1 a


sequence ofPcontinuous nonnegative functions on X.
+∞
(a) If n=1 fn = f 2 , then there exists a sequence (gn )+∞
n=1 of continuous real
functions on X suchPthat fn = gn f for all n ∈ N.P
+∞ +∞
(b) Prove that n=1 gn = f provided that n=1 gn is uniformly convergent
on X and that the interior of f −1 ({0}) is the empty set.

3. Let (fn )+∞


n=1 be a sequence of analytic functions from the region D ⊂ C into C
such that fn uniformly converges to f on D. Prove that if γ is a simple closed curve
inside D, then f is analytic inside and on the curve γ and that for all z0 inside γ,
we have Z Z
fn (z) f (z)
lim dz = dz.
n γ z − z 0 γ z − z0
10 The hypothesis “A ∩ C = B ∩ D = {0}” is redundant!
30 1. PROBLEMS

1.22.2. Algebra. 1. Let G be a group, H ≤ K ≤ G, [G : K] an odd number,


and [K : H] = 2. Also suppose that there is a k ∈ K of order 2 which is not
conjugate to any element of H in G. Prove that G has a subgroup of index 2.

2. In a commutative ring R (which is not necessarily unital), the ideal M is called


maximal if R2 * M and M is not contained in any ideal other than itself and R .
Now, we define a subring J of R as follows. If R has no maximal ideal, set J = R.
If not, set J to be the intersection of all maximal ideals of R. Prove that J has no
maximal ideal.

3. Let V be an n-dimensional vector space over a field F (n ∈ N) and S, T : V −→ V


two linear transformations such that the characteristic polynomial of one of them
is irreducible over F . If L = T S − ST 6= 0, then the rank of L is greater than one.

 b + x x
1.22.3. General. 1. If lim = 9, then b is equal to
x→+∞ b−x
(a) 3. (b) 3e . (c) ln 3. (d) ln 9.
Z x
1 t2
2. If lim √ dt exists and is nonzero, then which of the following
x→0 2bx − sin x 0 t−a
is admissible?

(a) a < 0, b = 12 .
(b) a = 0, b = 21 .
(c) a > 0, b = 12 .
(d) a = −1, b = −1 2 .

3. Which of the following is the general solution of the differential equation ex (1 +


x)dx − (xex − yey )dy = 0?

(a) x2 ex−y + y 3 = c.
(b) y 2 + 2xex−y = c.
(c) y 2 ex−y + x2 = c.
(d) xex−y + 12 y 2 = 0.

4. If A and B are two ends of the diagonal of a cube with side length 3, and an
ant wants to travel from A to B, what is the smallest distance in meters that the
ant needs to travel?
√ √ √ √
(a) 3 3. (b) 3 + 3 3. (c) 3 6. (d) 3 5.
+∞
X (x + 2)n
5. If f (x) = , then which one is the domain of f ?
n=1
2n (n + 1)

(a) [−4, 0).


(b) [−4, 0].
(c) (−4, 0].
(d) None of (a), (b), and (c).
1.22. TWENTY SECOND COMPETITION 31

1cm
A

Figure 2

6. If x3 + y 3 = xy + 1 and in a neighborhood of (1, 1), y is a function of x, which


one is y 0 (1)?
−2
(a) −1. (b) 3 . (c) 1. (d) 32 .

7. A particle, initially located in the origin, is discretely moving along the x-axis
one unit to the right or one unit to the left with probability p and q = 1 − p,
respectively. The probability that after 2k moves (k ≥ 5), the particle is 10 units
far away from the origin is
k+5 k k 10
(a) C2k p q (p + q 10 ).
k+5 k+5 k−5
(b) C2k p q .
k+5 k−5 k+5
(c) C2k p q .
k+5 k−5 k−5 10
(d) C2k p q (p + q 10 ).

8. An arrow, as shown in the figure, is moving according to the following law. If


the arrow is put at a point B, then it jumps as much as the length of the arc AB in
the trigonometric direction. At how many point(s) one can put the arrow so that
after 36 seconds it hits the point A?

(a) Exactly at one point.


(b) Exactly at 18 points.
(c) Exactly at 35 points.
(d) At no point.

x2

x irrational
9. Consider the function f (x) = . The point(s) at which f
0 x rational
is differentiable is/are

(a) the two points x = 0, 1.


(b) exactly one point.
(c) rational points.
(d) no point.
+∞
X cos(xn )
10. Consider the series .
n=0
1 + n2

(a) The series converges only at zero.


32 1. PROBLEMS

(b) The series converges everywhere.


(c) The series converges at nonzero points.
(d) The series converges absolutely at one point.

11. If we consider the sequence (xn )+∞


n=1 (x1 6= 0) of real numbers satisfying the
xn
following recursive relation xn+1 = , then
1 + xn
(a) xn → 1 as n → +∞.
(b) (xn )+∞
n=1 has a subsequence converging to one.
(c) (xn )+∞
n=1 has a subsequence converging to zero.
(d) (xn )+∞
n=1 is divergent.

12. Which of the following propositions is correct?


R n dx
(a) limn 0 1+x exists and is positive.
R n dx 2
(b) limn 0 1+x exists and is negative.
R n dx 2
(c) limn 0 1+x2 is divergent.
1
(d) f (x) = 1+x 2 is not integrable on the interval [0, n].

13. If the function f : R −→ R is differentiable and its derivative is bounded, then

(a) the function f is bounded.


(b) the function f is increasing.
(c) the function f is uniformly continuous.
(d) the function f is decreasing.

14. If the function f : [0, 1] −→ [0, 12 ] is continuous, then

(a) the graph of f intersects the line y = 2x.


(b) there exists only one point x0 for which f (x0 ) = x0 .
(c) f (x) 6= 0 for all x ∈ [0, 1].
(d) the equation f (x) = x has at most two solutions.
|x|
15. If f (x) = for all x ∈ R, then
1 + |x|
(a) the function f is increasing.
(b) f (x + y) =f (x) + f (y).
 
(c) f f (x + y) ≤ f f (x) + f f (y) .
(d) the function f is continuous only at zero and f (x + y) ≤ f (x) + f (y).

16. Below is drawn the graph of the polar curve r = θ. Which one is the area of
the shaded region?

π
2
R
(a) π
3
θ dθ.
6
−π
1
θ2 dθ.
R
(b) 6
−π 2
3
5π π
6 1 1 2
θ2 dθ −
R R
(c) 2π 2 π
3
2 θ dθ.
3 6
11π π
1 1 2
θ2 dθ −
R R
(d) 5π
6
2 π
3
2 θ dθ.
3 6
1.22. TWENTY SECOND COMPETITION 33

Figure 3

R1
17. The value of 0
x5 ex dx is

(a) −44e. (b) 120 + 44e. (c) 44e. (d) e.


Z x2
dy
18. If y = (x + t)dt, then at x = 0 is equal to
sin x dx
(a) 5. (b) 0. (c) 2. (d) −1.
Z 2
dx
19. For what values of p, does the integral converge?
1 x(ln x)p
(a) p = 1. (b) p > 1. (c) p < 1. (d) p = 2.
Z +∞ −ax
e − e−bx
20. dx (0 < a < b) is equal to
0 x

(a) ln( ab ). (b) ln( ab ). (c) ln(ab). (d) eab .

21. Which of the following statements is not correct?

(a) A power series might converge at both of the end points of its interval of
convergence.
(b) A power series can converge conditionally or diverge at any of the end points
of its interval of convergence.
(c) A power series can converge absolutely at one of the end points of its interval
of convergence and converge conditionally at the other end point.
(d) A power series can converge conditionally at one of the end points of its
interval of convergence and diverge at the other end point.

22. Using the spherical coordinates to compute the volume of the ellipsoid, the
Jacobian of the transformation is equal to

(a) ρ2 sin(ϕ). (b) ρ sin(ϕ). (c) abcρ2 sin(ϕ). (d) abc.



− →
− →

23. Letting F (x, y, z) = (x − y) i + (y − z) j + (x − y) k (−1 ≤ x, y, z, ≤ 1) be
a vector field, how much is the flux of the field F through the surface of a cube of
side length two centimeters?

(a) 8 cm3 . (b) 16 cm3 . (c) 1 cm3 . (d) 14 cm3 .


34 1. PROBLEMS


− →
− →

24. For the vector field
H F (x, y, z) = f (x, y, z) 2i + 2g(x, y, z) j + h(x, y, z) k , we
have Curl(F ) = 0 but C F.dR 6= 0, where C : x + y = 1. Then

(a) div(F ) < 0.


(b) the domain of F is connected.
(c) the domain of F is simply connected.
(d) the domain of F is not connected.

25. The family of orthogonal curves to the curves r = c(1 + cos θ) is

(a) r = c(1 − sin θ).


(b) r = c sin θ.
(c) r = c(1 + sin θ).
(d) r = c(1 − cos θ).

26. The general solution of the differential equation y 00 − 2y 0 − 3y = 64xe−x is

(a) y = c1 e3x + c2 e−x − e−x (8x2 + 4x + 1).


(b) y = c1 e3x + c2 e−x .
(c) y = c1 e3x + c2 e−x − 8x2 e−x .
(d) y = c1 e3x + c2 e−x − e−x (8x2 + 1).

27. Using the Gamma function for the definition of the factorial of numbers, ( 21 )!
is
√ √
(b) 2π . (c) √1π . (d) π2 .
p
a) π.

28. The equation x4 + x2 − 1 = 0 has roots as follows.

(a) Two positive roots, two negative roots.


(b) One positive root, one negative root, two nonreal roots.
(c) No real root.
(d) Four real roots.

29. The equation f (x) = e−x − sin x = 0 has a real root in the interval [0.4, 0.8].
The appropriate fixed point iteration function to find this root is

(a) g(x) = x + e−x − sin  x.


(b) g(x) = − ln sin(x) .
(c) g(x) = sin−1 (e−x ).
(a) g(x) = 12 2x + e−x − sin x .


30. In the bisection method, the number of times needed in order to obtain accuracy
within ε is equal to

log b−a
ε
(a) 1 + .
log 2
(b) log(b − a) − log(ε).
(c) cannot be determined.
log(b − a)
(d) 1 + .
log 2
1.22. TWENTY SECOND COMPETITION 35

f (x)
31. In the iteration method, Pn+1 = g(Pn ) (n ≥ 0) where g(x) = x − , as the
f 0 (x)
multiplicity of the root x = P increases,

(a) g 0 (P ) approaches zero.


(b) the speed of the convergence does not change.
(c) g 0 (P ) gets very close to one.
(d) the method is not convergent.

32. Which of the following propositions is correct?


P+∞ an P+∞
(a) If n=1 an+1 converges, then so does the series n=1 an .
P+∞
(b) If n=1 an converges, then limnPnan = 0.
+∞
(c) If limn nan = 0, then the series n=1 an is convergent.
P+∞ an+1 P+∞
(d) If n=1 an converges, then so does the series n=1 an .

 of the plane which includes the line 2y − 2 = 2x = z


33. Which one is the equation
x−y =0
and is parallel to the line ?
z=0

(a) y − x = 1.
(b) y = 2x.
(c) z − 2y − 2x = 2.
(d) y − x = −1.

34. Which of the following statements is a more complete definition of “algorithm”?

(a) “Algorithm” is a method for solving a problem.


(b) “Algorithm” is a logical method together with a terminating condition for
solving a problem.
(c) “Algorithm” is a logical procedure without ambiguity, which includes a
finite set of stages that are related to one another, together with a terminating
condition.
(d) “Algorithm” just means method.

35. Every digit in octal expansion is equivalent to

(a) one digit in decimal expansion.


(b) five digits in binary expansion.
(c) two digits in hexadecimal expansion.
(d) three digits in binary expansion.

36. A is infinite if and only if

(a) A is countable.
(b) card(A \ B) = card(A) for all B ⊆ A.
(c) A is equivalent to any of its infinite subsets.
(d) for all natural numbers n, A includes a subset which is equivalent to an
n-set.
36 1. PROBLEMS

37. Let A be an ordered set and B ⊆ C ⊆ A. Which of the following claims is


true?

(a) If sup(A) and sup(C) exist, then sup(B) ≤ sup(C).


(b) If sup(B) and sup(C) exist, then sup(C) ≤ sup(B).
(c) If sup(B) and sup(C) exist, then inf(B) = inf(C).
(d) There is no relation between sup(B) and sup(C).

38. Which of the following is equivalent to Zorn’s Lemma?

(a) Every subset of an ordered set having an upper bound has sup.
(b) There is a choice function for every nonempty family of nonempty subsets
of an arbitrary set X.
(c) If card(A) ≤ card(B) ≤ card(P (A)), then card(A) = card(B) or card(B) =
card(P (A)).
(d) Every bounded subset of an ordered set has a maximal element.
Z a
sin xdx
39. The integral x + e−x
is equal to
−a e

(a) 2.
(b) 0.R
a
(c) 2 0 esin xdx
x +e−x .

(d) It does not exist.


Rb
40. If f (a + b − x) = f (x) for all x ∈ [a, b], then a
xf (x)dx is equal to
Rb
(a) a−b
2 f (x)dx.
Rb a
(b) a f (x)dx.
Rb
(c) a+b
2 f (x)dx.
a R
b
(c) (a + b) a f (x)dx.

1.23. Twenty Third Competition, Sharif University of Technology,


March 1999

1.23.1. Analysis. 1. Let X = Cb (R) be the space of all continuous bounded



functions g : R −→ R which is endowed with the norm ||g|| = supt∈R g(t) . For a
given function f ∈ X , define fα : R −→ R by fα (t) = f (t + α), where α ∈ R. Prove
or disprove the following
propositions.
(a) If fα : α ∈ R is compact, then f is uniformly continuous on R.
(b) If f is uniformly continuous on R, then the set fα : α ∈ R is compact in
X.
+∞ inα
X e
2. Let α ∈ R and α 6= 2kπ for all k ∈ Z. Find the sum of the series and
n=1
n
+∞ +∞
X sin(nα) X cos(nα)
justify your answer. Ditto for and .
n=1
n n=1
n
1.24. TWENTY FOURTH COMPETITION 37

3. Let f, f1 , f2 , . . . be real continuous functions on the interval [a, b] whose deriva-


tives are also continuous on [a, b]. If the sequence (fn )+∞n=1 converges to f pointwise
and the sequence (fn0 )+∞ n=1 converges pointwise to a continuous function g, show that
f 0 (x) = g(x) for all x ∈ [a, b]. (Note: at the end points a and b, only the right and
left derivatives are to be considered.)

1.23.2. Algebra. 1. Let G be a nonabelian group. Prove that the group of


the inner automorphisms of G cannot be nonabelian of order 8.11

2. Let R be a ring and H the intersection of all nonzero right ideals of R. Show
that if H 6= 0, then H is a two-sided ideal in R and that we have H 2 = 0, or else
R is a division ring.

3. Let A be an n × n matrix with entries from a field F . Prove that if for every
matrix B with trace zero, we have tr(AB) = 0, then A = λI for some scalar λ in
F.

1.24. Twenty Fourth Competition, Khajeh Nasir Toosi University of


Technology, May 2000

0
1.24.1. First Day. 1. Let f,g : [a, b] −→ R  be continuous and that g exists
0 0
on [a, b]. Show that if f (a) − g (a) g (b) − f (b) > 0, then there exists a c ∈ (a, b)
such that f (c) = g 0 (c).

2. Let D be a domain in the complex plane and u a real valued harmonic function
on D. Show that if the set

A = (x0 , y0 ) ∈ D : ∃r > 0  ∀(x, y) ∈ Br (x0 , y0 ) ∩ D ⇒ u(x, y) ≤ u(x0 , y0 )
is nonempty, then u is constant on D. Recall that a domain is a connected open
set and that
p
Br (x0 , y0 ) = (x, y) ∈ R2 : (x − x0 )2 + (y − y0 )2 < r .


3. Let t(n) be the smallest prime factor of n for all 1 < n ∈ N. Prove that
t(n) < t(3n − 2n ).

4. Let G be an infinite group with the property that for every two infinite subsets
X and Y of G, there exist x ∈ X and y ∈ Y such that xy = yx. Prove that if the
center of G has finite index in G, then G is abelian.

5. Let M (n, d) denote the maximal number of n-tuples from {0, 1} such that every
two of which differ at least in d components. (For instance, M (4, 3) = 2.) Prove
that M (2d − 1, d) ≤ 2d.

6. A ruthless governor has made three mathematicians, who have been sentenced
to death, play the following game.
11 This problem is wrong!
38 1. PROBLEMS

Two lookalike boxes, one of which contains two black marbles and a white
marble, and the other contains two white marbles and a black marble, are provided.
Each of the convicts picks a marble, which is not to be replaced, from one of the
boxes at random. If the picked marble is black, the convict will be executed; if not,
s/he will be set free. Assuming that every convict witnesses the choice(s) made by
the convict(s) prior to oneself and that the convicts will do the most logical effort,
what is the probability that the second person survives? The probability of survival
for the third person is higher or for the second person?

1.24.2. Second Day. 1. Let B be a nonempty bounded set in Rn with the


property that for each pair of points x, y in B, there exists an open ball U such
that U ⊆ B and x, y ∈ U .
(a) Prove that B is an open ball.
(b) Show that if we replace Rn by a complete metric space, then the conclusion
of (a) does not necessarily hold.
Rb
2. Let f be a continuous function on the interval [a, b] such that a
f (t)dt 6= 0.
Show that for each k ∈ (0, 1), there exists a c ∈ (a, b) such that
Z c Z b
f (t)dt = k f (t)dt.
a a

3. Let matrices A, B, C be such that AB, BC, and ABC make sense. Denoting
the rank of any matrix P by r(P ), prove that
(a) r(BC) + r(AB) ≤ r(ABC) + r(B).
(b) r(A) + r(B) ≤ r(AB) + n,
where n is the number of columns of the matrix A.

4. Let R be a ring. Prove that if for any left ideal I of R, we have I 2 = I, then
for any two-sided ideal K of R and any left ideal I of R, we have I ∩ K = KI.
(Remark: Note that R is not necessarily unital.)

5. In an athletic tournament, n teams have participated and any two teams have
played once against one another. Assuming that the game result is to be either win
or lose, prove that at the end of the tournament there are two teams whose wins
are equal if and only if there are three teams A, B, C such that A has won B, B
has won C, and C has won A.

6. Two persons are playing the following game.


First, the first person chooses a triple from {0, 1} (for instance, (0, 1, 0)). The
second person, who knows the triple chosen by the first person, chooses a different
triple. Then a machine which at random generates 0 with probability 21 and 1 with
probability 12 is turned on. We write the numbers generated by the machine from
left to right and the player whose chosen triple comes first is to win the game. Show
that no matter what choice is made by the first person, there is a choice for the
second person so that the probability of winning the game by the second person is
greater than 12 .
1.25. TWENTY FIFTH COMPETITION 39

1.25. Twenty Fifth Competition, Imam Khomeini International


University of Qazvin, May 2001

1.25.1. First Day. 1. Let G be a finite group of order n such that [G :


Z(G)] = 4. Prove that 8|n. For any given natural number n such that 8|n, construct
a group with the aforementioned property. (Here, Z(G) denotes the center of G.)

2. Let S, T be two linear transformations on a vector space V . Prove that if S 2 = S,


T 2 = T , ker T ⊆ imS, and imT ⊆ ker S, then T + S is the identity transformation.

3. Let f be a twice differentiable function such that f 00 (t) < 0 for all t ∈ R. Prove
that if for two real numbers x and y we have f 0 (y) + x < f (y + 1), then f (y) > x.

4. Let f : (a, b) −→ R be continuously differentiable and


lim f 2 (x) = 0, lim− f 2 (x) = e − 1.
x→a+ x→b

Prove that if 2f (x)f 0 (x) − f 2 (x) ≥ 1 for all x ∈ (a, b), then 0 < b − a ≤ 1. Give an
2
example in which b − a = 1. (Note: f 2 (x) = f (x) .)

5. Seven boxes, on each of which a number from 1 to 7 is written, are at our


disposal. Seven marbles numbered 1, seven marbles numbered 2, ..., and seven
marbles numbered 7 are provided. We place the marbles in the boxes at random
in such a way that each box has exactly seven marbles. Play the following game.
In stage one, a marble is drawn at random from box number one (without
replacement). In stage i (i ≥ 2), a marble is drawn at random from the box
whose number is identical to that of the marble drawn in stage (i − 1) (without
replacement). The game is stopped when it is needed to draw a marble from an
empty box. Find the probability that all the marbles are drawn from all boxes.

6. Prove that the number of triangles, with integer sides, each of which having
perimeter n is equal to the number of partitions of the number n into the summands
2, 3, and 4 in which the summand 3 appears at least once.

1.25.2. Second Day. 1. Let R be an integral domain and U (R) the mul-
tiplicative group of the units of R. Prove that every finite subgroup of U (R) is
cyclic.

2. Let p be an odd prime. Prove that every prime factor of 2p + 1 which is different
from 3 is of the form 2kp + 1.

3. Let (X, d) be a compact metric space and f : X −→ X a surjective function.


Show that if 
d f (x), f (y) ≥ d(x, y),
for all x, y ∈ X, then f is continuous.

4. Let f : C −→ C be a function with f (0) = 0 and such that for all z ∈ C and
w ∈ {0, 1, i}, we have
f (z) − f (w) = z − w .
40 1. PROBLEMS

Find the function f explicitly. (i2 = −1.)

5. Let n be a an odd number and A an n × n matrix whose entries are from the set
{−1, 1}. If the product of the entries of the ith row is shown by ai and the product
of the entries of the jth column is shown by bj , prove that
n
X n
X
ai + bj 6= 0.
i=1 j=1

Does the above assertion hold for an even n?

6. A society is called ideal if for every two distinct members a and b of the society,
there exists a member c of it such that c is acquainted with one and only one of
a or b (We assume that acquaintance is a symmetric and nonreflexive relation.)
Prove that every ideal society of n people (n ≥ 2) contains an ideal society of n − 1
people.

1.26. Twenty Sixth Competition, Institute for Advanced Studies in


Basic Sciences (IASBS) of Zanjan, May 2002

1.26.1. First Day. 1. Let f : [a, b] → [a, b] be a continuous function which is


differentiable on (a, b) and f (a) = a , f (b) = b. Prove that there exist two distinct
points x1 and x2 in (a, b) such that f 0 (x1 )f 0 (x2 ) = 1.

2. Suppose that U = z ∈ C : |z| < 1 and D is an open set in C such that U ⊆ D,
and that the function f is analytic on D. Also let f (0) = 1 and that |f (z)| > 2 for
all z for which |z| = 1. Prove that there exists z0 ∈ U such that f (z0 ) = 0.

3. Letting p be an odd prime, prove that


pp−1 + (2p − 2)p−1 ≡ 1

mod p(2p − 2) .

4. Let R be a unital commutative ring such that every ideal of it is principal. Prove
that if R has only one maximal ideal, then Rx ⊆ Ry or Ry ⊆ Rx for all x, y ∈ R.

5. A k-element cover for a set S is a collection of k nonempty subsets of S such


that the union of it is S. An n-element cover of the set S is called minimal if no
n − 1 elements of it cover S. If the number of the minimal k-element covers of a
set with n elements is denoted by M (n, k), prove that
n n 
M (n, n − 1) = 2 −n−1 .
2

6. Let X be a set with n elements and positive integers m, k be such that m ≤ k


and m + k ≤ n. Denote the set of all subsets of X with m elements by X {m} .
Suppose that f : X {m} → R is a function where RXis the set of real numbers. Prove
{k}
that if for every member S of X we have f (T ) = 0, then f ≡ 0.
T ∈X {m} ,T ⊆S
1.27. TWENTY SEVENTH COMPETITION 41


1.26.2. Second Day. 1. Let r1 ,r2 , . . . be the set
of rational numbers in
[0, 1]. For each x ∈ [0, 1], we let Ax = n ∈ N : rn ≤ x and define the function
f : [0, 1] → R by
X 1
f (x) = .
2n
n∈Ax
Prove that f is continuous at any irrational point of the interval [0, 1].

2. Let f : R → R be a continuous function and that f ◦ f ◦ f (x) = x for all x ∈ R.
Prove that f (x) = x for all x ∈ R.

3. Let G be a group which has two distinct maximal subgroups, say, H and K.
Prove that if H and K are abelian and Z(G) = {e}, then H ∩ K = {e}.

4. Let A be an n × n matrix with entries in a field F . Let λ ∈ F be an eigenvalue


of An corresponding n ∈ F , 12
to an eigenvector x such that for all a0 , a1 , . . . , an−1 if
Pn−1 i
 n
i=0 ai A x = 0, then a0 = a1 = · · · = an−1 = 0. Prove that A − λI = 0.

5. An extended die is a homogeneous cube on each of whose sides an arbitrary


positive integer is written. (These numbers are not necessarily distinct.) Can one
design two extended dice (which are not necessarily identical) in such a way that
the probability that the sum of the numbers on the rolled dice is the same as those
1
of two ordinary dice? (That is, the probability of getting a sum of 2 is 36 , the
2
probability of getting a sum of 3 is 36 , etc.) In case, you answer in the affirmative,
find all possible answers; and if you answer in the negative, prove your claim.

6. A matrix Mm×n = (mij ) with real entries is said to be balanced, if whenever


for all i, i0 , j, j 0 we have 1 ≤ i < i0 ≤ m and 1 ≤ j < j 0 ≤ n, then mij + mi0 j 0 ≤
mij 0 + mi0 j . Suppose that there are two rows i1 i2 such that by interchanging the
rows, the matrix remains balanced. Prove that for all i01 and i02 , if i1 < i01 < i02 < i2 ,
then by interchanging the rows i01 and i02 the matrix remains balanced.

1.27. Twenty Seventh Competition, Bu-Ali Sina (Avecina) University


of Hamedan, May 2003

1.27.1. First Day. 1. Let n be a natural number greater than 1. Denote the
set of all n × n matrices with real entries by Mn (R). Define the following metric
on Mn (R). For A = (Aij ) and B = (Bij ),

d(A, B) = max Aij − Bij : i, j = 1, . . . , n .
Prove that GLn (R), the set of nonsingular matrices, is an open and disconnected
subset of Mn (R).

2. Let f be a function that is analytic on C. Let L and M be two orthogonal lines


intersecting at a point A such that f (L) = L and f (M ) = M . Prove that if z1 and
z2 are two complex numbers that are symmetric with respect to A, then f (z1 ) is
the image of f (z2 ) with respect to A.
12 Prove that An − λI = 0!
42 1. PROBLEMS

3. Let (an )+∞


n=1 be a sequence of real numbers defined as follows
q p
a0 = 0, a1 = b, an+1 = an 1 + a2n−1 + an−1 1 + a2n , n ≥ 1.
Determine an in terms of b.

4. Let K be a nonempty set and ∅ 6= I ⊆ K. Let Ai : i ∈ I be a family of the
subsets of K. Prove that if
  
i:i∈
/ Ai ∈ Ai : i ∈ I ∪ ∅ ,
then i ∈ Ai for all i ∈ I.

5. Let A be a 3 × 2 matrix and B a 2 × 3 matrix with complex entries for which


 
8 2 −2
AB =  2 5 4 .
−2 4 5
Determine the matrix BA. (Hint: Compute (AB)2 .)

6. Let G be a group and H a subgroup of it such that for all x ∈ G \ H and all
y ∈ G, there is a u ∈ H such that y −1 xy = u−1 xu. Prove that H is normal in G
and G/H is abelian.

1.27.2. Second Day. 1. Let f : R → R be a differentiable function, a, b ∈ R


and a < b. If f (a) = f (b) = 0, f 0 (a) > 0, and f 0 (b) > 0, prove that f 0 has at least
two roots in the interval (a, b).

2. Prove that the interval [0, 1] cannot be written as a union of pairwise disjoint
closed intervals each of which having a positive length less than one.

3. Let D be a countable subset of the Euclidean plane, i.e., R × R. Prove that


there is a partition for D into two subsets X and Y such that any line parallel to
the x-axis intersects X in finitely many points and any line parallel to the y-axis
intersects Y in finitely many points.
 Sn
4. Let A1 , . . . , An be a family of finite sets and S = i=1 Ai . Suppose that a
fixed number k, 1 ≤ k ≤ n, satisfies the following conditions.

(a) The union of any k elements of the family A1, . . . , An is equal to S.
(b) The union of any k − 1 elements of the family A1 , . . . , An is not equal to
S.
n

(c) |S| = k−1
Find the number of the elements of all of Ai ’s.

5. Let a, b be two natural numbers such that gcd(a, b) = 1. Prove that


ordab (a + b) = lcm[ordb (a), orda (b)]
(Here, by ordn (m), we mean the order of m modulo n.)

6. Let R and R0 be two rings whose elements are all idempotents and consider a
function f : R → R0 that is one-to-one and onto such that f (xy) = f (x)f (y) for all
x, y ∈ R. Prove that R ∼
= R0 .
1.28. TWENTY EIGHTH COMPETITION 43

1.28. Twenty Eighth Competition, Sharif University of Technology,


May 2004

1.28.1. First Day. 1. Prove that any one-to-one and entire function is of the
form f (z) = az + b, where a and b are fixed complex numbers and a 6= 0.

 Let d1 and d2 be the Euclidean metrics on R and R × R, respectively, A =


2.
(x, y) ∈ R × R : x = 0 or y = 0 , and d = d2 |A the metric induced by the
Euclidean metric of R × R on A. Prove that if f : (R, d1 ) → (A, d) is continuous
and surjective, then f −1 {(0, 0)} has at least three elements.13

3. P
Let f be an arithmetic
P function with the property that for every natural number
n, d|n f (d) = n2 . ( d|n means sum over all positive divisors of n.) If φ is Euler’s
totient function, prove that
f (n) Y  1
=n 1+ ,
φ(n) p
p|n,p prime

for all natural numbers n > 1.

4. Let G be a subgroup of Sn with the property that for all 1 ≤ i ≤ n and


1 ≤ j ≤ n, there exists σ ∈ G such that σ(i) =
j. Prove that for all 1 ≤ k ≤ n,
Gk ∩ Z(G) = {e} where Gk = τ ∈ G : τ (k) = k , Z(G) denotes the center of G,
and e is the identity element of G.

5. Let A be the set of all vectors of n-tuples whose entries are zero or one in such
a way that in each vector the numbers of one entries is odd. By explaining your
reasoning, determine, the maximal number of vectors from the set A such that
every two of which share an even number of one entries.

6. Let (P, ≤) be a poset (reflexive, antisymmetric,


 and
transitive)
 with n elements.

For every element i ∈ P , set Ui = j : j ∈ P, j > i and Li = j : j ∈ P, j < i .
Suppose that relative to every element i of P , there corresponds a real number Xi
subject to the following
( 1− P
j∈Li Xj
n−|Ui | if Li 6= ∅,
Xi = 1
n−|Ui | if Li = ∅.
Prove that 0 ≤ Xi ≤ 1 for all i ∈ P .

1.28.2. Second Day. 1. Let, in Figure 4, the function f be one-to-one and


continuous and that for every point P on the curve y = 2x2 , the areas of the regions
A and B are equal to one another. Determine the function f explicitly.

2. Let f : [a, b] → (a, b) be a continuous function. Prove that for every natural
number n, there exist a positive number α and c ∈ (a, b) such that
n
f (c) + f (c + α) + · · · + f (c + nα) = (n + 1)(c + α).
2
13 Prove that f −1 {(0, 0)} has infinitely many elements!
44 1. PROBLEMS

P
B
A

1.5

Figure 4

3. A matrix O is called orthogonal if OOt = Ot O = I, where Ot denotes the


transpose of the matrix O . Let A and B be two orthogonal square matrices with
entries in C and det(A) + det(B) = 0. Can one conclude that det(A + B) = 0?
Why?

4. Suppose that R is a unital ring and that there is a natural number n with the
property that for all k ∈ {n, n + 1, n + 2} and for all x, y ∈ R, (xy)k = xk y k . Prove
that R is commutative.

5. Of three people who are suspected to have committed a murder, one is the
murderer. The three people are to be interrogated by taking a test which has
five questions. If the person being tested is innocent, the probability that the
person responds positively to any question is 0.4; and if guilty, the probability of
responding positively is 0.8. Of these three people, one who is chosen at random is
being interrogated. This person responds positively to four questions and negatively
to one question. What is the probability that the person is the murderer?

6. Let X be a set and r a natural number. Let Xr be the set of all subsets of X
which have r elements. Suppose that F is a subset of Xr with the property that
the intersection of every k elements of F is nonempty (k ≥ 2 is a fixed number). If
we set

I(F ) = min |T | : T ⊆ X, T ∩ A 6= ∅, ∀A ∈ F ,
r−1
prove that I(F ) ≤ k−1 + 1.
1.29. TWENTY NINTH COMPETITION 45

1.29. Twenty Ninth Competition, University of Mazandaran in


Babolsar, May 2005

1.29.1. First Day. 1. Let f : [0, a] → R be a continuous and positive func-


tion. Prove that
Z a  Z a 
dx
f (x)dx ≥ a2 .
0 0 f (x)

2. Let (ni )+∞


i=1 be an increasing sequence (not necessarily strictly) of natural num-
+∞
X 1
bers with n1 ≥ 2 such that the series converges to a real number x.
n
i=1 1
· · · ni
Prove that x is rational if and only if there exists a natural number ` such that
ni = n` for all i ≥ `.

3. Consider the sequence of all natural numbers whose digits consist of 1.


1, 11, 111, . . .
Prove that if the natural number m is relatively prime with respect to 30, then
infinitely many terms of the above sequence are divisible by m.

4. Let R be an arbitrary ring (not necessarily unital) that has no nonzero nilpotent
two-sided ideal. Prove that every nonzero right ideal in R has an element whose
square is not zero.

5. Let Z, E, and O be the set of integers, even integers, and odd integers, respec-
tively. Set

X := A ∈ P(Z) : both A ∩ E, A ∩ O are infinite ,

Y := A ∈ P(Z) : A is infinite .
We know that there exists a one-to-one correspondence from X into Y . Exhibit a
law for a surjective function f : X → Y .

6. Let S be the k-dimensional vector space of all binary (zero and one) sequences
of length n over the field Z2 . The distance between two elements X, Y of S are
to be defined as the number of entries of X and Y that are different from one
another. (More precisely, if X = (x1 , . . . , xn ) and Y = (y1 , . . . , yn ), then the
distance between X and Y is equal to the number of i’s for which xi 6= yi .) Suppose
that the minimum distance between two distinct elements of S is equal to d. Prove
that
n2k−1
d≤ k .
2 −1
46 1. PROBLEMS


1.29.2. Second Day. 1. Let D = z ∈ C : |z| < 1 and f : D → C be an
analytic function such that f ( n1 ) ∈ R for all natural numbers n ≥ 2. Prove that for
all natural numbers n, f (n) (0) ∈ R, where f (n) denotes the nth derivative of the
function f .

2. (i) Prove that if X is a connected metric space, then for all ε > 0 and every two
points x, y ∈ X, there exist an n ∈ N and points x1 , . . . , xn ∈ X such that x1 = x ,
xn = y and that d(xi , xi+1 ) < ε for all i < n.
(ii) Give an example showing that the converse of the assertion of (i) does not
hold.
(iii) Prove that the converse of the assertion of (i) holds provided that X is
compact.

3. Let G be a group and K a subgroup of it.


NG (K)
(i) Prove that is isomorphic to a subgroup of Aut(K).
CG (K)
(ii) Prove that if K is cyclic and K E G = G0 , then K ≤ Z(G).

4. Let F be a field, Mn (F ) the set of all n × n matrices with entries in F ,


A ∈ Mn (F ), and the invertible matrix P ∈ Mn (F ) be such that P −1 AP is up-
per triangular. Prove that every two invariant subspaces of A are comparable with
respect to inclusion if and only if there exist a λ ∈ F and a nilpotent matrix
N ∈ Mn (F ) with N n−1 6= 0 such that A = λI + N .

5. Two people, named A and B, are playing a coin-flipping game as follows. They
throw their coins, if the result of flipping the two coins turns out to be the same,
A is going to win both coins; if not, B is going to take both coins. Suppose that A
has m coins and B has n coins. On average, how many times should the game be
played till one of the players runs out of the coins?

6. Let C be the Cantor  C − C = [−1, 1]. (Hint: it is worth


set. Prove that
mentioning that C − C = x − y : x, y ∈ C and that C is equal to the set of all
numbers in [0, 1] whose ternary expansions have only 0 or 2.)

1.30. Thirtieth Competition, Tafresh University, May 2006

1.30.1. First Day. 1. Let f : [0, +∞) −→ R be a continuous function and


limx→+∞ f (x) = 1. Evaluate the following limit.
Z 2006
lim f (nx)dx.
n→+∞ 1385

2. Let m ∈ N, c ∈ C, aj ∈ C, and |aj | = 1 for all 1 ≤ j ≤ m. If


m
X
lim anj = c,
n→+∞
j=1

then c = m and aj = 1 for all 1 ≤ j ≤ m.


1.30. THIRTIETH COMPETITION 47

3. Let R be a commutative ring with identity and a an element of R with a3 −a−1 =


0. Prove that if J is an ideal of R such that the quotient ring R/J has at most four
elements, then J = R.

4. Let p, q be prime numbers such that p = 2q + 1 and q ≡ 1 (mod 4). Prove that
2 is a primitive root modulo p.

5. Prove that
X  2m 
≥ 22m−1 ,
√ m+k
|k|< m
for all m ∈ N with m ≥ 1
Hint. By the Chebyshev Inequality if X is a random variable whose mean and
variance are µ and σ 2 , respectively, then P (|X − µ| ≥ λσ) ≤ λ12 for all λ > 0.

6. A financial group has n members each of whom has a number of coins. Let
k ∈ N be a fixed positive integer. A business among four people, say, p1 , p2 , p3 ,
and p4 , of the n people, who are chosen arbitrarily, can be done subject to the
following conditions: 
(a) sum of the numberof coins owned by p3 and p4 − sum of the number
of coins owned by p1 and p2 + 2k > 0,
(b) each of p1 and p2 has at least k coins.
If the conditions (a) and (b) are met, then a business is done as follows. Each
of p3 and p4 earns exactly k coins which are lost by each of p1 and p2 . Prove that
after a finite number of doing business, the condition (a) or (b) in the above does
not hold for any four people of the n people.

1.30.2. Second Day. 1. Let X be a separable metric space, that is, X has
a countable dense subset. Let f : X −→ R be a function for which limx→a f (x)
exists for all a ∈ X. Prove that the set of points at which f is discontinuous is at
most countable.
P+∞
2. Suppose that f (z) = n=0 an z n defines a nonconstant analytic function, where
the radius of the convergence of the series is equal to R > 0. Prove that the closest
0|
distance from a zero of f to the origin is at least MR|a +|a0 | , where M = M (R) =
sup|z|=R |f (z)|.

3. Let G be a group with the property that the order of any element of G0 , the
derived subgroup of G, is finite. Prove that the set of all elements of G whose orders
are finite is a subgroup of G.

4. Let K be a field, F a subfield of K, n ∈ N, and A an F -algebraic n×n matrix with


entries in K such that rank(A) = rank(A2 ). Prove, firstly, that K n = imA ⊕ ker A
and, secondly, that there is a polynomial f ∈ F [x] such that E := f (A) is an
idempotent matrix satisfying E(x + y) = x for all x ∈ imA and y ∈ ker A.

5. For an arbitrary subset C of the natural numbers, set C ⊕ C := x + y|x, y ∈
C, x 6= y . Prove that there is a unique partition of the set of natural numbers into
two subsets A and B such that none of A ⊕ A and B ⊕ B contains a prime.
48 1. PROBLEMS

Hint. Recall that by Bertrand’s conjecture (a.k.a. Bertrand’s principle) for


every natural number n there exists at least a prime number p such that n < p ≤ 2n.

6. Let C be a circle whose perimeter is equal to one and 0 < α < 21 . The distance
between two points of the circle is defined to be the length of the shortest arc joining
the two points. Let T ⊆ C and T = I1 ∪· · ·∪Im , where m ∈ N and Ij ’s are disjoints
arcs of the circle (1 ≤ j ≤ m). Prove that if the distance between every two points
of T is less than or equal to α, then
m
X
`(Ij ) ≤ α,
j=1

where `(Ij ) denotes the length of the arc Ij .

1.31. Thirty First Competition, Ferdowsi University of Mashhad, May


2007

1.31.1. First Day. 1. We have colored a circle by two colors. Does there
necessarily exist a monocolored triangle inscribed in the circle which is
(a) an equilateral triangle?
(b) a right triangle?
(c) an isosceles triangle?

2. The Minkowski sum of two sets A, B ⊆ Rd is defined as follows


a + b ∈ Rd |a ∈ A, b ∈ B .

A+B =
Prove that if A is bounded and B is closed, then
(A + B)0 = (A0 + B) ∪ (A + B 0 ),
where A0 stands for the set of the limit points of the set A.

3. Suppose that there is a group G which has exactly n subgroups of index 2 (n


is a natural number.). Prove that there exists a finite abelian group which has
exactly n subgroups of index 2.

4. Can one find two biased dice in such a way that the probability of getting a sum
of j, for all 2 ≤j ≤ 12, for the two simultaneously thrown dice is a number in the
2 4
interval 33 , 33 ?

5. Show that R2 has a dense subset whose no three points are collinear.

6. Let A be an n × n matrix with real entries. Prove that


 
tr(A) 1 0 ··· ··· 0
 tr(A2 ) tr(A) 2 0 ··· 0 
..
 
 3 2 .. 
1  tr(A ) tr(A ) tr(A) 3 . . 
det(A) = det  . . ..
.
n! 
.. .. .. .. 

 . . . 0  
 tr(An−1 ) tr(An−2 ) · · · ··· tr(A) n − 1 
tr(An ) tr(An−1 ) · · · ··· tr(A2 ) tr(A)
1.31. THIRTY FIRST COMPETITION 49

1.31.2. Second Day. 1. Let n be an odd natural number. Prove that there
are n consecutive natural numbers whose sum is a complete square. In this spirit,
do there exist twelve natural numbers whose sum is a complete square?

2. For what values of the nonzero reals α and β the following limit exists (and is
finite)?
x2α y 2β
lim + 3α
x,y→0 x + y 3β

3. Let A be a nonvoid set and An be the set of all ordered n-tuples of the elements
of A. For α = (α1 , . . . , αn ) and β = (β1 , . . . , βn ) in An , define
d(α, β) = the number of corresponding components of α and β that differ with
one another.
For two arbitrary elements x and y in An , prove that there is a one-to-one corre-
spondence between the following two sets
z ∈ An : d(x, z) < d(y, z) ,

C =
z ∈ An : d(y, z) < d(x, z) .

D =

4. Let R be a commutative ring with identity. Prove that the ring R[x] has infinitely
many maximal ideals.

5. Let x1 , . . . , x2n be real numbers such that removing any of them the remaining
ones can be partitioned into two sets with equal sums (n ≥ 2). Prove that xi ’s are
all zero.

6. Let T be the union of all line segments with one end at M = (0, 1) and the other
end at a point on the x-axis having rational coordinates. In other words,
(tq, 1 − t) ∈ R2 |t ∈ [0, 1], q ∈ Q .

T =
(a) Assume that A, B ∈ T such that A, B, and M are not collinear. Show
that any continuous path from the point A to the point B in the set T has to pass
through the point M (by the aforementioned path, we mean a continuous function
γ : [0, 1] → T with γ(0) = A and γ(1) = B.).
(b) Prove that every continuous function from T into T has at least a fixed
point.
CHAPTER 2

Solutions

2.1. First Competition

2.1.1. Analysis. 1. (a) For the given function f , let fn : R → R be defined


by fn (x) = n f (x + n1 ) − f (x) , where n ∈ N. It is plain that the sequence (fn )+∞
n=1
converges pointwise to f 0 on R, finishing the proof of this part.

(b) First, we recall the following which is known as Darboux’s Theorem.

Darboux’s Theorem. The derivative of any differentiable function on a


closed interval has the intermediate value property.

Proof. Suppose that f : [a, b] −→ R is a differentiable function, f 0 (a) 6= f 0 (b),


and λ ∈ R is between f 0 (a) and f 0 (b). We need to show that there exists a c ∈ (a, b)
such that f 0 (c) = λ. To this end, if necessary replacing f by −f , without loss of
generality, we may assume that f 0 (a) < f 0 (b). There are two cases to consider.
(i) f 0 (a) < λ ≤ f (b)−f
b−a
(a)
.
Define the function g : [a, b] −→ R by

f 0 (a)

x = a,
g(x) = f (x)−f (a)
x−a a < x ≤ b.

It is readily seen that g is continuous on [a, b]. We have g(a) < λ ≤ g(b). It thus
follows from the Intermediate Value Theorem that there exists an x0 ∈ (a, b] such
that g(x0 ) = f (xx00)−f
−a
(a)
= λ. Therefore, by the Mean Value Theorem, λ = f 0 (c) for
some c ∈ (a, b), which is what we want.
(i) f (b)−f
b−a
(a)
< λ < f 0 (b).
In this case, defining the function g : [a, b] −→ R by
 f (b)−f (x)
b−x a ≤ x < b,
g(x) =
f 0 (b) x = b,

the assertion follows in a similar fashion. 

Define the function f : R → R by



0 x ≤ 0,
f (x) =
1 x > 0.
51
52 2. SOLUTIONS

The function f is a Baire function because the sequence (fn )+∞


n=1 of continuous
functions, where fn : R → R is defined by

 0 x ≤ 0,
fn (x) = nx 0 < x < n1 ,
1
1 n ≤ x,

converges to the function f pointwise. Yet, in view of Darboux’s Theorem , the


function f is not the derivative of any function on R because f does not have the
intermediate value property (a.k.a. Darboux’s property ). 

2. (a) For X = (xij ) ∈ M, by definition, we have


X
det(X) = sgn(σ)x1σ1 · · · xnσn ,
σ∈Sn

where Sn denotes the symmetric group on {1, . . . , n} and sgn(σ) stands for the sign
of the permutation σ ∈ Sn . Hence, the function det : M → R is continuous, for f
is a polynomial function in n2 variables xij where 1 ≤ i, j ≤ n. We have
U (M) = det−1 R \ {0} ,


where U (M) denotes the set of invertible matrices in M. It follows that U (M) is
an open subset of M because R \ {0} is an open subset of R.

(b) To see that M is disconnected, just note that


U (M) = det−1 R− ∪ det−1 R+
 

where R− = (−∞, 0) +
 and R −1= (0,
+∞). Thus, in view of the proof of (a), the
−1 −
open sets det R and det R+ form a disconnectedness for U (M), finishing
the proof. 

3. (a) We need the following lemma.

Lemma. Let N ∈ R+ and I a bounded interval of R. Then, in the interval I


there are finitely many r ∈ Q whose denominators are less than or equal to N .

Proof. Without loss of generality assume that N is a natural number. Just


note that, by the Archimedean property of real numbers and the well-ordering
SN
principle of natural numbers, the set b=1 { ab ∈ I : a ∈ Z} is a finite set, proving
the assertion. 

We claim that limx→a f (x) = 0 for all a ∈ R+ . To see this, for a given ε > 0,
first pick N ∈ N such that N1 < ε. It follows from the lemma above that there
exists δ > 0 such that x = pq ∈ Q with gcd(p, q) = 1 and 0 < |x − a| < δ imply that
q > N . So for this given ε > 0, find δ > 0 as in the above. If 0 < |x − a| < δ, there
are two cases to consider:
(i) x = pq ∈ Q with gcd(p, q) = 1. Noting that q > N , we can write
1 1 1
|f (x) − 0| = < < < ε.
p+q q N
(ii) x ∈
/ Q. In this case, we have
|f (x) − 0| = 0 < ε.
From these two cases, we see that limx→a f (x) = 0, which is what we want.
2.1. FIRST COMPETITION 53

(b) In view of the claim we made in (a), it follows that f is continuous at the
irrational points of (0, +∞) and is discontinuous at the rational points of (0, +∞).


2.1.2. Algebra. 1. (a) “if”: Suppose that I1 , I2 C R with I1 I2 ⊆ R and


I1 * P . We show that I2 ⊆ P . Choosing an x0 ∈ I1 \ P and assuming that y ∈ I2
is arbitrary, we have x0 y ∈ I1 I2 ⊆ P . Hence, y ∈ P , for x0 ∈ I1 \ P . As y was
arbitrary, we see that I2 ⊆ P , which is what we want.
“only if”: Let hai and hbi denote the ideals generated by a and b in R, respec-
tively. The ring R is commutative with identity element. So we have hai = aR,
hbi = bR, and haihbi = habi. Now from ab ∈ P , it follows that haihbi ⊆ P . But P
is prime. Therefore, hai ⊆ P or hbi ⊆ P , implying that a ∈ P or b ∈ P , which is
what we want.

(b) Let M be a maximal ideal in R. In view of (a), it suffices to show that M


is prime. To this end, suppose that ab ∈ M , where a, b ∈ R. We need to show that
a ∈ M or b ∈ M . If a ∈/ M , then M + hai = R because M is maximal and a ∈ / M.
In particular, we see that there are m ∈ M and x ∈ R such that 1 = m + ax,
implying that b = bm + abx ∈ M , for M is an ideal and ab ∈ M . 

2. It follows from the hypothesis that g −1 = g for all g ∈ G. We can write


ab = (ab)−1 = b−1 a−1 = ba,
for all a, b ∈ G. That is, G is abelian, which is what we want. 

3. (3.1) In view of (c), we can write


cardE ≤ cardf (E) ≤ cardF = cardE.
This, together with the Schroder-Bernstein Theorem, implies that cardE = cardf (E).
Now, suppose that X and Y are two finite subsets of E such that cardX = cardf (X)
and cardY = cardf (Y ). We can write

cardf (X ∪ Y ) = card f (X) ∪ f (Y )

= cardf (X) + cardf (Y ) − card f (X) ∩ f (Y ) . (∗)
On the other hand,
card(X ∪ Y ) = cardX + cardY − card(X ∩ Y ). (∗∗)

But card(X ∪ Y ) ≤ cardf (X ∪ Y ) = card f (X) ∪ f (Y ) . So it follows from (∗) and
(∗∗) that card f (X) ∩ f (Y ) ≤ card(X ∩ Y ). As f (X ∩ Y ) ⊆ f (X) ∩ f (Y ), we can
write

card(X ∩ Y ) ≤ cardf (X ∩ Y ) ≤ card f (X) ∩ f (Y ) ≤ card(X ∩ Y ).
Therefore, 
cardf (X ∩ Y ) = card f (X) ∩ f (Y ) = card(X ∩ Y ).
That is, X ∩ Y and f (X) ∩ f (Y ) are isomorphic sets. Now, since f (X ∩ Y ) ⊆
f (X) ∩ f (Y ) and these two sets are finite, it follows that f (X ∩ Y ) = f (X) ∩ f (Y ).
This equality together with (∗) and (∗∗) implies that card(X ∪ Y ) = cardf (X ∪ Y ),
finishing the proof of assertion in this part.
54 2. SOLUTIONS

(3.2) Set

T = card(A) : ∅ 6= A ⊆ E and card(A) = cardf (A) .
It is obvious that the set T is a nonempty subset of N. It thus follows from the
well-ordering principle of N that the set T has an initial element, say, cardX0 ,
where X0 is a nonempty normal subset of E. We claim that for any other normal
subset X of E, we have
X ∩ X0 = ∅ or X0 ⊆ X.
Note that X0 ∩ X ⊆ X0 . If X0 ∩ X = X0 , we obtain X0 ⊆ X; if not, we show that
X0 ∩ X = ∅, completing the proof. Suppose that X0 ∩ X $ X0 . Since X0 is finite,
it follows that
card(X0 ∩ X) < card(X0 ).
On the other hand, by (3.1), card(X0 ∩ X) = cardf (X0 ∩ X). We claim that
X0 ∩ X = ∅. Otherwise, we see that card(X0 ∩ X) ∈ T , yielding card(X0 ) ≤
card(X0 ∩ X), for card(X0 ) is the initial element of T . This is a contradiction.
Hence, X0 ∩ X = ∅ and we are done. 

a b c
2.1.3. General. 1. From a < b < c, we see that < 1 , < 1 and 1 < .
b c a
Thus,
a b c c a b c c
max , , = , min , , 6= .
b c a a b c a a
There are two cases to consider.
a b c a a b c b
(i) min , , = and (ii) min , , = .
b c a b b c a c
In both cases, in view of the above inequalities, it is easily seen that S > 1. 

2. We show that the decimal fraction a = 0.123456789101112131415 . . . cannot be


periodic after any digit. To prove this by contradiction, assume that the decimal
fraction a is periodic after its first k digits and that its period is l. Obviously, the
number 10k+l occurs somewhere after the kth digit of the decimal fraction a. As
10k+l has k + l zeros in its decimal expansion and that the period of a after the
kth digit is assumed to be l, it follows that the digits of a after the kth digit are all
zero, which is obviously impossible, settling the proof. 

3. We need to solve the following system of equations in C


x = y3

, x 6= y.
y = x3
It is plain that x, y 6= −1, 0, 1, for otherwise x = y ∈ {−1, 0, 1}. By substitution,
we obtain
x9 = x, x 6= 0 =⇒ x8 = 1 =⇒ x = ω j , 0 < j < 7, j 6= 4,
where ω = cos π4 + i sin π4 . It is now easily seen that
x = ωj

, 0 < j < 7, j 6= 4,
y = ω 3j
is the general solution of the above system of equations. 
2.1. FIRST COMPETITION 55

4. Using the AM-GM Inequality, we can write



xx + y y ≥ 2 xx y y = 2ef (x) ,
where f : (0, 1) → R is defined by f (x) = x ln x + (1 − x) ln(1 − x). We have f 0 (x) =
− ln( x1 − 1). Thus, f 0 (x) < 0 on (0, 12 ) and f 0 (x) > 0 on ( 21 , 1), implying that f
attains its only local minimum and hence its absolute minimum at 12 . Consequently,
we have
1 √
xx + y y ≥ 2ef (x) ≥ 2ef ( 2 ) = 2,
for all x, y ∈ R+ with x + y = 1 and that the equality happens if and only if
x = y = 12 , which is what we want. 

2.1.4. Differential Equations. We know that the general solution of a non-


homogeneous linear equation is y = yc + yp , where yc is the general solution of the
corresponding homogeneous equation and yp is a particular solution of the nonho-
mogeneous linear equation x00 + a2 x = f (t). In view of this, first of all, it is easily
seen that yc (t) = c1 cos at + c2 sin at (t ∈ R), where c1 , c2 ∈ R, is the general solu-
tion of the homogeneous equation. Using the method of variation of parameters, a
straightforward calculation shows that the function yp defined by

1 t
Z
yp (t) = f (x) sin a(t − x)dx (t ∈ R),
a t0
where t0 ∈ R is a fixed arbitrary real number, is a particular solution of the non-
homogeneous linear equation x00 + a2 x = f (t). Therefore,
1 t
Z
y(t) = c1 cos at + c2 sin at + f (x) sin a(t − x)dx (t ∈ R),
a t0

is the general solution of the equation x00 + a2 x = f (t). 

P10 
2.1.5. Probability and Statistics. We need to calculate P i=1 Xi = 10 .
To do this, let, first of all, X be a Poisson random variable with parameter λ defined
by its probability density function as follows
 −λ i
e λ
i ∈ N ∪ {0},
fX (i) = p(X = i) = i!
0 otherwise.
Next, we find the moment generating function of X.
+∞
 X tXi X
mX (t) = E etX = e fX (Xi ) = eti fX (i)
Xi i=0
+∞
X (λet )i t
= e−λ = eλ(e −1)
.
i=0
i!
56 2. SOLUTIONS

Now, since each Xi is a Poisson random variable


Pwith parameter µi and that Xi ’s
10
are independent random variables, letting Y = i=1 Xi , we can write
P 10
10
Y  Y10
= E et Xi
etXi = E etXi
 
mY (t) i=1 =E
i=1 i=1
10
Y
(
P 10
µi )(et −1)
= mXi (t) = e i=1 .
i=1
P10 P10
Therefore, Y = i=1 Xi is a Poisson random variable with parameter i=1 µi ,
whence
10 P 10 P10
X  e− i=1 µi
( i=1 µi )10
P Xi = 10 = fY (10) = .
i=1
10!
But µi = i. So, we have
10
 X  e−55 5510
P X=1 = P Xi = 10 = .
i=1
10!


2.1.6. Topology. To prove the assertion by contradiction, suppose that T is


a topology on R. We must have
[ √
Aq = ( 2, +∞) ∈ T,

q> 2
q∈Q

implying that 2 ∈ Q, which is a contradiction, settling the proof. 

2.2. Second Competition

2.2.1. Analysis. 1. (a) Since f is continuous and the interval [0, 1] is com-
pact, there exists an x0 ∈ [0, 1] such that
M = sup f (x) = max f (x) = f (x0 ).
0≤x≤1 0≤x≤1

If M = 0, there is nothing to prove. If not, as f is continuous at x0 , we see that for


given M ε > 0, there exists a neighborhood of x0 , say, (α, β), where 0 ≤ α < β ≤ 1,
such that
|f (x) − f (x0 )| < M ε
whenever α < x < β. Noting that f (x0 ) = M ≥ f (x) for all x ∈ [0, 1], in view of
the above inequality, we obtain
M (1 − ε) < f (x) ≤ M,
for all x ∈ (α, β), which is what we want.

(b) For given 0 < ε < 1, find 0 ≤ α < β ≤ 1 from (a). We have 0 < M (1 − ε) ≤
f (x) ≤ M on [α, β], for f is continuous on [0, 1]. We can write
Z β Z 1
n n n
M (1 − ε) (β − α) ≤ f ≤ f n ≤ M n.
α 0
2.2. SECOND COMPETITION 57

Taking nth root, we obtain


Z 1  n1
1
n
M (1 − ε)(β − α) n ≤ f ≤ M. (∗)
0

Now taking limn and limn of both sides of the left and right inequalities above, we
obtain
Z 1  n1 Z 1  n1
n
M (1 − ε) ≤ limn f ≤ limn fn ≤ M.
0 0
R  n1 R  n1
1 1
Thus, 0 ≤ limn fn 0
− limn fn ≤ M − M (1 − ε) = M ε. As 0 < ε < 1
0
is arbitrary, we conclude that
Z 1  n1 Z 1  n1
n
limn f = limn fn .
0 0
R  n1
1
That is, limn 0
fn exists. Letting n → +∞ in (∗), we obtain
Z 1  n1
n
M (1 − ε) ≤ lim f ≤ M.
n 0

Again, 0 < ε < 1 being arbitrary, we see that


Z 1  n1
lim fn = M = f (x0 ) = sup f (x),
n 0 0≤x≤1

finishing the proof. 

2. Note first that



Z 1
sup 1 − f (t) ≥ 1 − f (0) = 1, 1 − f (t) dt ≥ 0,
0≤t≤1 0

and hence

Z 1
sup 1 − f (t) + 1 − f (t) dt ≥ 1,
0≤t≤1 0

for all f ∈ F. This implies that




Z 1

D = inf sup 1 − f (t) +
1 − f (t) dt ≥ 1.
f ∈F 0≤t≤1 0

We claim that D = 1. To see this, just note that for the sequence (fn )+∞
n=1 , where
fn : [0, 1] → R (n ∈ N) is defined by
0 ≤ t ≤ n1 ,

nt
fn (t) = 1
1 n < t ≤ 1,

we have
 Z 1   
1
lim sup 1 − fn (t) + 1 − fn (t) dt = lim 1 + = 1.
n 0≤t≤1 0 n 2n
58 2. SOLUTIONS

As for the geometrical interpretation of D, we view F as a subspace of the


normed space all continuous functions on [0, 1], denoted by C[0, 1], which is equipped
with the following norm
Z 1

||f || := sup f (t) + f (t) dt.
0≤t≤1 0

If 1 : [0, 1] → R is defined by 1(x) = 1 for all x ∈ [0, 1], we would then have
d({1}, F) = inf ||1 − f ||
f ∈F


Z 1

= inf sup 1 − f (t) +
1 − f (t) dt = D = 1.
f ∈F 0≤t≤1 0

That is, the distance from the vector 1 ∈ C[0, 1] to the subspace F of C[0, 1] is equal
to D = 1. 

3. It suffices to show that [0, 2] ⊆ C + C. To this end, let x ∈ [0, 2] be arbitrary. If


x
= 0.x1 x2 . . .
2
denotes the ternary expansion of x2 where xi ∈ {0, 1, 2}, then we can write
x
= 0.a1 a2 . . . + 0.b1 b2 . . . ,
2
where ai , bi ∈ {0, 1} and xi = ai + bi for all i ∈ N. So we have
x = 2(0.a1 a2 . . .) + 2(0.b1 b2 . . .) = 0.a01 a02 . . . + 0.b01 b02 . . . := a + b,
where a0i = 2ai and b0i = 2bi , and hence a0i , b0i ∈ {0, 2} for all i ∈ N. Therefore,
a, b ∈ C because there is no one in their ternary expansions. This finishes the proof.


2.2.2. Algebra. 1. To prove the assertion we need the following general


lemma.

Lemma. Let G be a finite group with the property that the equation xn = e
has at most n solutions in G for all n ∈ N, where e is the identity element of G.
Then the group G is cyclic.

Remark. In case the group G is abelian, the lemma is a quick consequence of


the Fundamental Theorem of finite abelian groups .

First proof. Let |G| = n and C be a cyclic group with n elements so that
C = hai, where ord(a) = n. Suppose that for a divisor d of n, there is an element
gd ∈ G such that ord(gd ) = d. It follows that gd , gd2 , . . . , gdd−1 , gdd = e are d solutions
of the equation xd = e. As d n = |G| = |C| and C is cyclic, there is an ad ∈ C such
that ord(ad ) = d. Consequently, hgd i ∼ = had i, and hence C has at least as many
elements of order d as G has. But G and C both have n elements. Thus, G must
have the same number of elements of order d as C, for all d dividing n. It follows
that G is cyclic because C has an element of order n, namely the element a. So the
proof is complete.
2.2. SECOND COMPETITION 59

Second proof. Define the relation ∼ on G as follows


a ∼ b ⇐⇒ ord(a) = ord(b).
It is obvious that ∼ is an equivalence relation on G. Hence, the relation ∼ partitions
G into its equivalence classes. In other words, we can write
k
[
G= [gi ],
i=1

where k ∈ N is less than |G|, {g1 , . . . , gk } is a maximal set of nonequivalent elements


of G, and [gi ] denotes the equivalence class containing the element gi .
Note that if G happens to be a cyclic group of order n so that G = hai with
ord(a) = n, we can write
k
X
n = G = [gi ] ,
i=1
where gi ’s (1 ≤ i ≤ k) are as in the above and gi = aji for some 1 ≤ ji ≤ n. Let
di = gcd(ji , n). It is easily checked that
ord(a) n
ord(ak ) = = (∗)
gcd k, ord(a) gcd(k, n)
for all 1 ≤ k ≤ n. In view of this, we see that di 6= di0 whenever i 6= i0 and
n
that ak ∈ [gi ] if and only if gcd(k,n) = nd if and only if gcd(k, n) = d if and only

if gcd( kd , nd ) = 1. Thus, [gi ] = φ( nd ), where φ denotes Euler’s totient function .
Consequently,
k
X X n X
n= [gi ] = φ = φ(d).
i=1
d
d|n d|n

Now back to a general finite group G, letting di = ord(gi ), we claim that


[gi ] = φ(di ). That [gi ] ≥ φ(di ) follows from the hypothesis that di = ord(gi ) and
of (∗), the cyclic group hgi i has exactly φ(di ) generators. On the other
that, in view
hand, [gi ] ≤ φ(di ), for otherwise there must exist a g ∈ G \ hgi i with ord(g) = di ,
which, in turn, implies that the equation xdi = e has at least di + 1 solutions,
namely, g, gi , gi2 , . . . , gidi = e, which is a contradiction. Thus, [gi ] = φ(di ), and
hence
k
X X
n = G = φ(di ) = φ(d).
i=1 d|n
This, in particular, implies that G has an element of order n. Therefore, G is a
cyclic group of order n, which is what we want. 

Corollary. Let D be a division ring and G a finite abelian subgroup of the


multiplicative group of D, i.e., D∗ = D \ {0}. Then, G is a cyclic group. In
particular, the multiplicative group of any finite field is cyclic. 

By the corollary above, there exists an a ∈ F ∗ such that F ∗ = hai. Letting


b = am , we can write
X n−2
X n−2
X
m i m
S := x = (a ) = bi .
x∈F i=0 i=0
60 2. SOLUTIONS

If n − 1|m, then, as P an−1 = 1, we have b = 1. This together with the above


equality implies that x∈F xm = n − 1. If n − 1 - m, then bm 6= 1. So we have
S = bS = 1 + b + · · · bn−1 , yielding (b − 1)S = 0 which, in turn, implies that S = 0,
proving the assertion. 

2. (a) “⇐=” This is obvious.


“=⇒” Assume that z is right and left quasi-regular. Thus, there exist z 0 , z 00 ∈ A
such that
z + z 0 − z 0 z = z + z 00 − zz 00 = 0. (∗)
We can write

(z + z 0 − z 0 z)z 00 = 0 =⇒ zz 00 + z 0 z 00 − z 0 (z + z 00 ) = 0 =⇒ zz 00 = z 0 z.
This together with (∗) implies that z 0 = z 00 . That is, z is quasi-regular, which is
what we want.

(b) Proceed by contradiction. The identity element of A being right quasi-


regular means there exists 10 ∈ A such that 1 + 10 − 10 = 0, yielding 1 = 0, which
is impossible. Likewise, the identity element is not left quasi-regular either.
As pointed out in the footnote of this problem, from the element x being right
quasi-regular one can only conclude that 1 − x is right invertible. First, let x be
right quasi-regular. So there exists x0 ∈ A such that x + x0 − xx0 = 0. From this, we
obtain (1−x)(1−x0 ) = 1. That is, 1−x is right invertible. In fact, as this argument
is reversible, in a similar fashion, one can prove that the element x is right (resp.
left) quasi-regular if and only if 1 − x is right (resp. left) invertible. We now show
that in general from x being right quasi-regular one cannot conclude that 1 − x is
invertible. To this end, in view of the aforementioned comment, it suffices to show
that in general from x being right quasi-regular one cannot conclude that x is left
quasi-regular as well. To see this, let F be a field of characteristic zero and F [x]
the ring of all polynomials with coefficients from the field F . View F [x] as a vector
space over F and use A to denote L(F [x]), the ring of all linear transformations
from F [x] into F [x]. Let I denote the identity transformation, D the differentiation
linear transformation, i.e. D(f0 + f1 x + · · · + fn xn ) = f1 + 2f2 x + · · · + nfn xn−1 ,
and I1 and I2 denote the linear transformations defined by
f1 fn n+1
I1 (f0 + f1 x + · · · + fn xn ) = 1 + f0 x + x2 + · · · + x ,
2 n+1
f1 fn n+1
I2 (f0 + f1 x + · · · + fn xn ) = 2 + f0 x + x2 + · · · + x .
2 n+1
Clearly, DI1 = DI2 = I. That is, D has two right inverses, and hence D is
not invertible. This implies that I − D is right quasi-regular but it is not left
quasi-regular. Because otherwise I − (I − D) = D will become invertible, which is
impossible. 

3. (a) Since D divides P , there exists Q ∈ K[x] such that P = QD. We conclude
that φ(P ) = φ(Q)φ(D) because φ is a ring homomorphism. It is now obvious that
ker φ(D) ⊆ ker φ(P ), which is what we want.

(b) Suppose that D = gcd(P, R), where P, R ∈ K[x]. Since K[x] is a Euclidean
ring, it follows that there exist M, N ∈ K[x] such that D = M P + N R. As φ
2.2. SECOND COMPETITION 61

is a ring homomorphism, we conclude that φ(D) = φ(M )φ(P ) + φ(N )φ(R). This
implies ker φ(R) ∩ ker φ(P ) ⊆ ker φ(D). On the other hand, since D = gcd(P, R),
in view of (a), we see that ker φ(D) ⊆ ker φ(P ) and ker φ(D) ⊆ ker φ(R), yielding
ker φ(D) ⊆ ker φ(R)∩ker φ(P ). Therefore, ker φ(D) = ker φ(R)∩ker φ(P ), finishing
the proof. 

2.2.3. General. 1. If the first day up to the seventh day of the first month
of the Persian calendar, called “Farvardin”, corresponds to 1 up to 7, respectively,
then the thirteenth day of the first month corresponds to 6. From this convention,
we will obtain the 12-tuple
(6, 2, 5, 1, 4, 7, 3, 5, 7, 2, 4, 6)
whose ith component is corresponded to the thirteenth day of the month i of the
Persian calendar (1 ≤ i ≤ 12). For instance, the number 1 being the fourth
component of the above 12-tuple means that the thirteenth day of the fourth month
of the Persian calendar occurs on the same day of the week as does the first day of
the first month of the calendar. For 1 ≤ i ≤ 7, let n(i) denote the number of times
that the number i occurs in the above 12-tuple. Obviously,

1 i ∈ {1, 3},
n(i) =
2 i∈/ {1, 3}.
As the first day of “Farvardin” can occur on any day of the week, we see
that “Wednesday” can correspond to any of the numbers 1, 2, . . . , 7. Therefore,
in the Persian calendar, the minimum and the maximum number of “Wednesdays
occurring on the 13th of the month” is 1 and 2, respectively. 

2. Recall that a real function f on an interval I is called convex if



f λx + (1 − λ)y ≤ λf (x) + (1 − λ)f (y),
for all x, y ∈ I and 0 ≤ λ ≤ 1. It is well-known that a function f : I → R which is
twice differentiable is convex if and only if f 00 ≥ 0 on I. From this, it follows that
the function gn : (0, +∞) → R defined by gn (x) = xn , where n ∈ R+ , is convex
if and only n ≥ 1. In particular, gn is convex whenever n ∈ N. This yields
 gifn (x)+g
x+y n (y)
gn 2 ≤ 2 for all n ∈ N and x, y ∈ R+ . So, we can write
 x + y + z + t n x + y + z + t 1 x + y 1 z + t
= gn ≤ gn + gn
4 4 2 2 2 2
gn (x) + gn (y) + gn (z) + gn (t) xn + y n + z n + tn
≤ = ,
4 4
for all n ∈ N and x, y, z, t ∈ R+ , which is what we want. 

3. We prove the following proposition of which the assertion is a quick consequence.


Note that if the particle were moving along a straight line, then the tangential
acceleration of the motion would be equal to the acceleration of the motion along
the straight line.

A particle moving along a curve goes s0 unit(s) of distance in t0 unit(s) of


time in such a way that the instantaneous speed of it at the initial point as well
62 2. SOLUTIONS

as the final point is zero. Prove that the absolute value of the particle tangential
acceleration at some point on its path is greater than or equal to 4s 0 unit of distance
t2 (unit of time)2
.
0

Assume that the initial and final moments of the motion are 0 and t0 , respec-
tively. If the equation of the motion is given by a vector function x : [0, t0 ] → R3 ,
it follows that x is twice differentiable, x0 (0) = x0 (t0 ) = 0, and s(0) = 0, s(t0 ) = s0 ,
Rt 0
where x0 = dx dt denotes the instantaneous velocity vector and s(t) = 0 ||x (τ )||dτ
the length of the path at time t. It is plain that s0 (0) = s0 (t0 ) = 0. Also, recall
that the tangential acceleration of the motion, denoted by at , is, by definition, the
d2 s 00
time derivative of the instantaneous speed, i.e., at = dv dt = dt2 = s (t). As s is
continuous, we see from the Intermediate Value Theorem that there is a moment
0 < c < t0 at which we have s(c) = s(t0 )+s(0) 2 = s(t20 ) . There are two cases to
consider.
(i) 0 < c < t20 .
In this case, using the extension of the Mean Value Theorem for second order
derivatives, we obtain a moment t1 with 0 < t1 < c such that
(c − 0)2 c2
s(c) = s(0) + s0 (0)(c − 0) + s00 (t1 ) = s00 (t1 ) ,
2! 2
2
from which, we get s(t20 ) = s00 (t1 ) c2 , and hence s00 (t1 ) = s(t 0)
c2 . This together
with the hypothesis that 0 < c < 2 implies that s00 (t1 ) ≥ t2 0 ) > 0, yielding
t0 4s(t
0
4s(t0 ) 4s0
|s00 (t1 )| ≥ t20
= t20
, which is what we want.
(ii) t20 ≤ c < t0 .
Again using the extension of the Mean Value Theorem for second derivatives,
we obtain a moment t1 with t20 < t1 < t0 such that
(c − t0 )2
s(c) = s(t0 ) + s0 (t0 )(c − t0 ) + s00 (t1 ) ,
2!
2
s(t0 ) −s(t0 )
from which, we obtain 2 = s(t0 ) + s00 (t1 ) (c−t2 0 ) , and hence s00 (t1 ) = (c−t0 )2 < 0.
t0 s(t0 ) 4s(t0 )
This together with 2 ≤ c < t0 implies that |s00 (t1 )| = (c−t0 )2 ≥ t20
= 4s
t20
0
, which
is what we want. 

4. Since the digits can be selected independent of the letter, using the product rule
of combinatorics, it is obvious that the total number of the car plates that contain
the letter “dal” is equal to
9 × 9 × 9 × 9 × 9 × 1 = 59049.


2.2.4. Probability. (a) We can write


cov(U, V ) = E(U V ) − E(U )E(V )

= E (X + Y )(X − Y ) − E(X + Y )E(X − Y )
= E(X 2 − Y 2 ) − (E(X))2 − (E(Y ))2


= E(X 2 ) − (E(X))2 − E(Y 2 ) − (E(Y ))2




= var(X) − var(Y ) = σ 2 − σ 2 = 0.
2.2. SECOND COMPETITION 63

Therefore, cov(U, V ) = 0.
Note: As is obvious now the condition cov(X, Y ) = λ was redundant.

(b) Not necessarily. Suppose that U, V are two random variables with cov(U, V ) =
0, which are not independent. If we let X = 12 (U + V ) and Y = 12 (U − V ), we
would have
1  1 
var(X) = var(U ) + var(V ) + 2cov(U, V ) = var(U ) + var(V ) ,
4 4
1  1 
var(Y ) = var(U ) + var(V ) − 2cov(U, V ) = var(U ) + var(V ) .
4 4
1

Therefore, var(X) = var(Y ) = 4 var(U ) + var(V ) = σ 2 . Note that the random
variables X + Y = U and X − Y = V are not independent by our hypothesis. To
give a concrete example, let U and V be two random variables defined by their
joint distribution table as follows.

U V 1 2 3
1 0.1 0.1 0.1 0.3
2 0.1 0.1 0.1 0.4
3 0.1 0.1 0.1 0.3
0.3 0.3 0.4

It is easily verified that

UV 1 2 3 4 6 9
0.1 0.2 0.2 0.1 0.3 0.1

From the above table, we obtain E(U V ) = 4.2. On the other hand, we have
E(U ) = 2 and E(V ) = 2.1, implying that E(U V ) = E(U )E(V ), and hence
cov(U, V ) = 0. But P (U = 1, V = 1) = 0.1 and P (U = 1) = P (V = 1) = 0.3,
yielding P (U = 1, V = 1) 6= P (U = 1).P (V = 1). In other words, U and V are not
independent, which is what we wanted. 

2.2.5. Topology. Proceed by contradiction. As A ⊆ B ∪C, we have A = (A∩


B)∪(A∩C). Also, by the contradiction hypothesis, (A∩B)∩(A∩C) = A∩B∩C = ∅.
But A ∩ B 6= ∅ and A ∩ C 6= ∅. Hence, it suffices to show that A ∩ B and A ∩ C
are closed sets in the induced topology of A, for A would then be disconnected, a
contradiction. To this end, note that B and C are closed in (K, TK ). Thus, there
exist closed sets B1 and C1 in (E, T ) such that B = B1 ∩ K and C = C1 ∩ K, from
which, we obtain

A ∩ B = A ∩ B1 , A ∩ C = A ∩ C1 .
Now as B1 and C1 are closed in E, it follows that A ∩ B and A ∩ C are closed in
A, which is what we want. 
64 2. SOLUTIONS

2.2.6. Differential Equations. Two cases to consider.


(i) |x| > 1.
Rewrite the equation as

x (x2 − 1)(x + 1)
(y 3 )0 + y3 = = x + 1.
−1 x2 x2 − 1
Multiplying both sides of the equation by
Z 
xdx p
2 − 1| =
p
µ(x) = exp = |x x2 − 1,
(x2 − 1)
we obtain
d  3p 2  p
y x − 1 = (x + 1) x2 − 1,
dx
from which, we get
p Z p Z p
y3 x2 − 1 = x x2 − 1dx + x2 − 1dx.

Using integration by parts and dividing both sides of the above equality by x2 − 1,
we see that the general solution of the equation is
1 x 1 p c
y 3 = (x2 − 1) + + √ ln |x + x2 − 1| + √ ,
3 2 2 x −12 2
x −1
where c is an arbitrary fixed constant.
(ii) |x| < 1. √
In this case, note that |x2 − 1| = 1 − x2 yields µ(x) = 1 − x2 . From this, a
similar argument as in (i) shows that the general solution of the equation is
−1 2 x 1 c
y3 = (x − 1) + + √ arcsin x + √ ,
3 2 2 1 − x2 1 − x2
where c is an arbitrary fixed constant. 

2.3. Third Competition

2.3.1. Analysis. 1. Remark. It is worth mentioning that the assertion holds


under the weaker hypothesis that f 00 is nonnegative on (a, b) or f 0 is monotonic on
[a, b]. Also in the statement of the problem “ξ ∈ [a, b]” must read “ξ ∈ (a, b)” and
that “ξ ∈ (a, b)” cannot be weakened, because for the function f : [0, 1] → R with
f (x) = x2 and ξ ∈ {0, 1}, there is no x0 ∈ (0, 1) such that
f (0) − f (x0 ) f (1) − f (x0 )
f 0 (ξ) = or f 0 (ξ) = .
0 − x0 1 − x0

Since f 00 > 0 on [a, b], it follows that f 0 is strictly increasing on [a, b]. By the
Mean Value Theorem, there exists a < ξ0 < b such that
f (b) − f (a)
f 0 (ξ0 ) = .
b−a
Now, if f 0 (ξ) = f 0 (ξ0 ), there is nothing to prove. If not, there are two cases to
consider.
2.3. THIRD COMPETITION 65

(i)
f (b) − f (a)
f 0 (ξ) < f 0 (ξ0 ) = .
b−a
Define the function g : [a, b] → R by
f 0 (a)

x = a,
g(x) = f (a)−f (x)
a−x a < x ≤ b.
It is plain that
f (a) − f (x)
lim g(x) = lim+ = f 0 (a) = g(a).
x→a+ x→a a−x
Hence, g is continuous at a from the right, and therefore it is continuous on [a, b].
As f 0 is strictly increasing on [a, b], we obtain f 0 (a) < f 0 (ξ). So we can write
g(a) = f 0 (a) < f 0 (ξ) < f 0 (ξ0 ) = g(b).
Thus, it follows from the Intermediate Value Theorem that there exists an a <
x0 < b such that
f (a) − f (x0 )
f 0 (ξ) = g(x0 ) = ,
a − x0
which is what we want.
(ii)
f (b) − f (a)
f 0 (ξ) > f 0 (ξ0 ) = .
b−a
The proof, which is omitted, is almost identical to that of (i) except that we
need to define the function g : [a, b] → R by
f (b)−f (x)

b−x a ≤ x < b,
g(x) = 0
f (b) x = b.

 1
2. (a) Set B = Q ∩ [0, 1] \ n : n ∈ N . Plainly, we can write B = {bn : n ∈ N}.
Define the sequence (an )+∞
n=1 by
 1
n n odd,
an =
bn n even.
We claim that for this sequence (an )+∞
n=1 , there is no continuous function f satisfying
f (an ) = an+1 for all n ∈ N. Suppose to the contrary that f : [0, 1] → [0, 1] is a
continuous function such that f (an ) = an+1 for all n ∈ N. By proving that f = 0
on (0, 1], and hence on [0, 1], we obtain a contradiction, proving the claim. For a
given x ∈ (0, 1], pick a subsequence (ani )+∞
i=1 such that limi→+∞ ani =  x. As x 6= 0,
/ n1 : n ∈ N

if necessary by passing to a subsequence, we may assume that ani ∈
for all i ∈ N. This implies that ni ’s are all even numbers, from which, we see that
(ni + 1)’s are all odd numbers and hence
1
f (x) = lim f (ani ) = lim ani +1 = lim = 0,
i→+∞ i→+∞ i→+∞ ni + 1

as desired.

(b) We sketch the proof for the case in which the sequence (an )+∞
n=1 is increasing.
In case the sequence is decreasing, the proof can be carried out in a similar fashion.
66 2. SOLUTIONS

Without loss of generality, if necessary by redefining the sequence (an )+∞


n=1 , we may
assume that a1 = 0. Let a∞ = limn→+∞ an . Define the function f on [0, 1] by
 an+2 −an+1
an+1 −an (x − an ) + an+1 an ≤ x < an+1 , n ∈ N,
f (x) =
x a∞ ≤ x ≤ 1.
It is not difficult to see that f : [0, 1] → [0, 1] is continuous and that f (an ) = an+1
for all n ∈ N, as desired. 

3. (a) We need the following proposition.

Proposition. Let the complex number z be a root of the following equation


with complex coefficients
xp + c1 xp−1 + · · · + cp−1 x + cp = 0,
Pp
where p ∈ N. If λk > 0 for all 1 ≤ k ≤ p and k=1 λ1k ≤ 1, then
p
|z| ≤ max k λk |ck |.
1≤k≤p

Under the hypothesis of the proposition, taking λk = 2k for all 1 ≤ k ≤ p, we


obtain
p
k
|z| ≤ 2 max |ck |,
1≤k≤p

proving the assertion.

In order to prove the proposition, we need the following lemma.

Lemma. Let the complex numbers z1 , . . . , zp (p ∈ N) be the roots of the fol-


lowing equation with complex coefficients
xp + c1 xp−1 + · · · + cp−1 x + cp = 0.
Set r0 = max1≤i≤p |zi |. If r > 0 and
rp > |c1 |rp−1 + · · · + |cp |,
then r0 < r.

First proof. Let z ∈ C with |z| ≥ r be arbitrary. We can write



f (z) c1 cp
z = 1 + + ··· + p

z z
c c
1 p
≥ 1 − − ··· − p
z z
|c1 | |cp |
≥ 1− − · · · − p > 0.
r r
Thus, f (z) 6= 0, and hence r0 < r, as desired.
2.3. THIRD COMPETITION 67

Second proof. Let k(x) := xp − |c1 |xp−1 − · · · − |cp |. By the hypothesis,


Pp |ci |
k(r) > 0, which is equivalent to k(r)
rp = 1 − i=1 r i > 0. Thus, for all x ≥ r, we
have
p p
X |ci | X |ci |
1− i
≥ 1 − > 0.
i=1
x i=1
ri
Consequently,
k(x)  k(x) 0
k 0 (x) = (xp )0
p
+ xp
x xp
p
X |ci |  p
X i|ci |
= pxp−1 1 − i
+ xp
i+1
> 0,
i=1
x i=1
x

for all x ≥ r. Thus, k is strictly increasing on the interval [r, +∞). Now, let z ∈ C
be such that |z| ≥ r. It follows that k(|z|) ≥ k(r) > 0. So, we can write
|z p + c1 z p−1 + · · · + cp | ≥ |z|p − |c1 ||z|p−1 − · · · − |cp |
= k(|z|) ≥ k(r) > 0.
That is, z p +c1 z p−1 +· · ·+cp 6= 0 whenever |z| ≥ r. This yields r0 = max1≤i≤p |zi | <
r, which is what we want.

Third proof. Define f (z) = z p + c1 z p−1 + · · · + cp−1 z + cp , g(z) = z p , and


h(z) = c1 z p−1 + · · · + cp = f (z) − g(z) on C. Note that f (z) 6= 0 for all z ∈ C with
|z| = r, for otherwise
p−1
rp = z p = − c1 z p−1 − · · · − cp ≤ c1 z

+ · · · + cp ,
from which, we obtain rp ≤ |c1 |rp−1 + · · · + |cp |, which is a contradiction. For all z
on the circle |z| = r, we have
h(z) ≤ c1 rp−1 + · · · + cp < rp = g(z) .

So it follows from Rouché’s Theorem that the entire functions g(z) = z p and f (z) =
h(z) + g(z) have the same number of zeros inside the circle |z| = r. Consequently,
f has p zeros inside the circle |z| = r, implying that r0 < r, which is what we want.

p
ProofPof Proposition. Set r1 = max1≤k≤p k λk |ck |, where λk ’s in R+ are
p
such that k=1 λ1k ≤ 1. For all r > r1 and for each k = 1, . . . , p, we obviously have
1 |ck |
λk > rk
. Thus,
p p
X 1 X |ck |
1≥ > ,
λk rk
k=1 k=1
which obtains
rp > |c1 |rp−1 + · · · + |cp |,
and hence r0 < r in view ofpthe above lemma. As r > r1 was arbitrary, we conclude
that r0 ≤ r1 = max1≤k≤p k λk |ck |, which is what we want. 

(b) Let f (z) := z p +c1 z p−1 +· · ·+cp−1 z+cp , g(z) := z p +c01 z p−1 +· · ·+c0p−1 z+c0p ,
and h(z) := g(z + zj ). First, we prove that the absolute value of the product of the
68 2. SOLUTIONS

roots of h, i.e., |h(0)|, is less than (2K)p δ. To this end, we can write
p
X
(c0i − ci )zjp−i

h(0) = g(zj ) = g(zj ) − f (zj ) =
i=1
p p
X 0 p−i X p−i
≤ ci − ci zj
≤ K i δ zj
i=1 i=1
Xp p
X
p
≤ δ K i (2 max k |ck |)p−i < δ K i (2K)p−i
1≤k≤p
i=1 i=1
p
X
= δK p 2p−i = δK p (2p − 1) < (2K)p δ,
i=1

yielding h(0) < (2K)p δ, as desired. Now, let z10 , . . . , zp0 be the roots of g(z) = 0.
It follows that z10 − zj , . . . , zp0 − zj are the roots of h(z) = g(z + zj ) = 0. Obviously,
there is a 1 ≤ k ≤ p such that |zk0 − zj | = min1≤i≤p |zi0 − zj |. Consequently,
Y
|zk0 − zj |p ≤ zi0 − zj = h(0) < (2K)p δ,

1≤i≤p

implying that |zk0 − zj | < 2K δ, which is what we want.
p

(c) We have not been able to prove the assertion. However, we prove the
following which can be thought of as a counterpart of the assertion at the local
level.

Under the hypothesis of the (c) part of the problem, prove that for every t ∈
[t1 , t2 ], there exists a δ = δ(t) > 0 such that if s ∈ [t1 , t2 ] and |s − t| < δ, then
p
|f (s) − f (t)| < 2K p |s − t|α .

For a given t ∈ [t1 , t2 ], let z1 = f (t), z2 , . . . , zk (1 ≤ k ≤ p) be distinct roots of


z p + b1 (t)z p−1 + · · · + bp−1 (t)z + bp (t) = 0. If k = 1, in fact f need not be continuous
and yet the assertion is a quick consequence of (b). To see this, note that by (b),
for the root f (s) of z p + b1 (s)z p−1 + · · · + bp−1 (s)z + bp (s) = 0, where s ∈ [t1 , t2 ],
there is a root of z p + b1 (t)z p−1 + · · · + bp−1 (t)z + bp (t) = 0, which must be the only
root of the equation, namely f (t), such that
p
f (s) − f (t) < 2K p |s − t|α ,
as desired. So, we may without loss of generality assume that k p > 1. Now, set
d0 = 21 min1≤i<j≤k |zi − zj |. Choose a δ = δ(t) > 0 such that 2K p |s − t|α < d0
and |f (s) − f (t)| < d0 whenever |s − t| < δ and s ∈ [t1 , t2 ]. It follows from (b) that
for every s ∈ [t1 , t2 ] with |s − t| < δ, there is an is ∈ {1, . . . , k} such that
p
f (s) − zis < 2K p |s − t|α < d0 .
On the other hand,
f (s) − f (t) < d0 .
Consequently,

z1 − zi = f (t) − zi ≤ f (t) − f (s) + f (s) − zi < 2d0 ,
s s s
2.3. THIRD COMPETITION 69

implying that z1 = f (t) = zis . Thus,


p
f (s) − f (t) < 2K p |s − t|α ,

whenever s ∈ [t1 , t2 ] and |s − t| < δ, which is what we want. 

2.3.2. Algebra. 1. (a) Just apply the First Isomorphism Theorem for mod-
ules to the mapping ϕ : M2 → M1M+M
1
2
defined by ϕ(x) = x + M1 , where x ∈ M2 .

(b) Suppose that {αi }m m


i=1 is a basis for M1 ∩ M2 . Enlarge {αi }i=1 to bases
m n m p
{αi }i=1 ∪ {βi }i=1 and {αi }i=1 ∪ {γi }i=1 for M1 and M2 , respectively. It is easily
p
seen that {αi }m n
i=1 ∪ {βi }i=1 ∪ {γi }i=1 is a basis for M1 + M2 . We can write

dim M1 + dim M2 = (m + n) + (m + p) = (m + n + p) + m
= dim(M1 + M2 ) + dim(M1 ∩ M2 )
= dim M3 + dim M4 ,
which is what we want. 

2. If G is abelian, we see that the function f : G → G defined by f (x) = x2


is a homomorphism. Conversely, if f , defined by f (x) = x2 for all x ∈ G, is a
homomorphism, then (xy)2 = x2 y 2 for all x, y ∈ G. Multiplying both sides by x−1
and y −1 on the left and right, respectively, we obtain xy = yx for all x, y ∈ G.
That is, G is abelian, which is what we want. 

3. (a) It is easily verified that the semigroup S defined by



S := (aij ) ∈ M2 (R) : a11 = a21 = 0, a12 , a22 ∈ R \ {0}
 
0 1
has a right identity element, namely , with respect to which every element
0 1
of S has a left inverse. And yet S is not a group, for its right identity element is
not a left identity element.

(b) Let x ∈ S be an arbitrary element with a left inverse x ∈ S so that xx = e.


To prove the assertion, it suffices to show that xx = e. To this end, let x be a left
inverse of x. We have
ex = (x.x)x = x(xx) = xe = x.
By showing that xx is a right identity element of S, we conclude that xx = e,
finishing the proof. Suppose that y ∈ S is arbitrary. We can write
y.(xx) = (ye)(xx) = y(ex)x = y(x.x) = ye = y.
Thus, xx = e, which is what we want. 
70 2. SOLUTIONS

N’ S
R M’

M
N
Figure 5

2.3.3. General. 1. Letting z = x in the second condition, we see that


F (y, x) ≤ F (x, x) + F (x, y) = F (x, y),
for all x, y ∈ R. Therefore, F (y, x) ≤ F (x, y) for all x, y ∈ R, implying that
F (x, y) ≤ F (y, x) for all x, y ∈ R. That is, F (x, y) = F (y, x) for all x, y ∈ R. This
proves (b). Letting y = x in the second condition, we get
0 = F (x, x) ≤ F (x, z) + F (z, x),
from which, in view of (b), we obtain 2F (x, z) ≥ 0, implying that F (x, z) ≥ 0 for
all x, z ∈ R. This proves (a), completing the proof. 

2. The identity (1 + x)2n = (1 + x)n (1 + x)n in R[x] together with the Binomial
Theorem implies that
X2n X n n
 X 
k k
C2n x = Cnk xk Cnk xk ,
k=0 k=0 k=0
n!
for all n ∈ N, where Cnk = is the binomial coefficient. Using the formula
k!(n − k)!
for Cauchy product of two polynomials, noting that Cnk = Cnn−k , and finally
equating the coefficients of xn of both sides of the above identity, we see that
n n 2
(2n)! n
X
k 2
 X n!
= C2n = Cn = ,
(n!)2 k!(n − k)!
k=0 k=0
proving the assertion. 

3. It is plain that if from the point M , we shine a beam of light onto a point R
of the mirror ∆ so that the reflected light beam hits the mirror ∆0 at a point S,
then the three points S , R, and M 0 are collinear, where M 0 is the image of M in
the mirror ∆. Hence the problem is solved as follows. Find the image of M and
N in the mirrors ∆ and ∆0 , respectively, to obtain the points M 0 and N 0 of the
plane P . The line joining M 0 and N 0 intersects ∆ and ∆0 at the points R and S,
respectively. It is now obvious that if we shine a beam of light onto the point R on
the mirror ∆, then the reflected light beam after hitting the mirror ∆0 at S would
pass through the point N (see Figure 5). 
2.4. FOURTH COMPETITION 71

(-2,0) (-1,0) (0,0) (1,0) (2,0)

Figure 6

2.4. Fourth Competition

2.4.1. Analysis. 1. Define the function f : R → R2 by



 g(t) t < −2,
f (t) = (t, 0) −2 ≤ t < 2,
h(t) 2 ≤ t,

   
−2 2(t+2) 2 2(t−2)
where g(t) = 1+(t+2) 2 , 1+(t+2)2 and h(t) = 1+(t−2) 2 , 1+(t−2)2 . It is straight-
forward to see that f is continuous and one-to-one. In fact, f is a one-to-one
parametrization of the curve consisting of the upper semicircle with radius one cen-
tered at (−1, 0) going from (0, 0) to (−2, 0), the line segment joining (−2, 0) and
(2, 0), and the upper semicircle with radius one centered at (1, 0) going from (2, 0)
to (0, 0) (see Figure 6).

We claim that the function f −1 is discontinuous at (0, 0), proving the assertion.
To see this, define the sequence (an )+∞
n=1 by
1
( 
n, 0 n even,
an =
 
2 2n
1+n2 , 1+n2 n odd.
It is obvious that limn an = (0, 0) = f (0) but
 1
n even,
f −1 (an ) = n
n+2 n odd,
whose limit does not exist as n → +∞. Therefore, f −1 is not continuous at (0, 0),
proving the claim. 

2. Set S := n ∈ N : αn > 0 . If the set S is finite, there is nothing to prove. So
assume that S is an infinite set. For each k ∈ S with k ≥ 2, define
n o
Tk := N ∈ S|N ≥ k  αn ≤ αk ∀n ≥ N .
As limn αn = 0, letting ε = ak in the definition of the limit for limn αn = 0, we
see that Tk 6= ∅. Let Mk be the initial element of Tk . If Mk = k, we will have
αn ≤ αk for all n ≥ k, in which case set Nk := k. If Mk > k, set Nk to be the
greatest integer subject to k ≤ Nk < Mk and Nk ∈ S. It follows that Nk ∈ / Tk
and that αn = 0 for all Nk < n < Mk . Consequently, αNk > αk > 0, for Mk
is the initial element of Tk . Now, if n ≥ Nk , then either n = Nk , in which case
72 2. SOLUTIONS

αn = αNk ≤ αNk , or Nk < n < Mk , which yields αn = 0 < αNk , or n ≥ Mk , in


which case αn ≤ αk < αNk . In other words, αn ≤ αNk for all n ≥ Nk . On the
other hand, S is an infinite set and Nk ≥ k for all k ∈ S. This shows that there are
an infinite number of Nk ’s such that αn ≤ αNk for all n ≥ Nk . 

2.4.2. Algebra. 1. (a) Let x, y ∈ R(B) and a ∈ A be arbitrary. It follows


that there are M, N ∈ N such that xM ∈ B and xN ∈ B. Noting that in a
commutative ring the Binomial Theorem holds, (ab)n = an bn for all a, b ∈ A and
n ∈ N, and that B is an ideal of A, we see that
M
X +N
M +N −i
(x − y)M +N = i
CM i
+N x (−y) ∈ B, (ax)M = aM xM ∈ B,
i=0

yielding x − y ∈ R(B) and ax ∈ R(B). That implies R(B) is an ideal of A, which


is what we want.

(b) First proof. Let B be a prime ideal of A. To prove that R(B) is prime,
suppose for given x, y ∈ A, we have xy ∈ R(B). We need to show that x ∈ R(B)
or y ∈ R(B). As xy ∈ R(B), there is an n ∈ N such that xn y n = (xy)n ∈ B. But
B is prime. So we must have xn ∈ B or y n ∈ B. In other words, x ∈∈ R(B) or
y ∈ R(B), as desired.

Second proof. We prove the assertion by showing that R(B) = B. Evidently,


B ⊆ R(B). To show that R(B) ⊆ B, let x ∈ R(B) be arbitrary. It follows that
there is an n ∈ N such that xn ∈ B. Now, a straightforward induction on n
together with the hypothesis that B is prime reveals that x ∈ B. This proves that
R(B) ⊆ B, and hence R(B) = B, as desired.

(c) Remark. We must have a 6= 0, ±1.


mk
Without loss of generality, assume that a > 1. Write a = pm 1 · · · pk , where
1

mi ∈ N and pi ’s are distinct prime numbers (1 ≤ i ≤ k). We show that R(B) =


hp1 p2 · · · pk i. First, let x ∈ hp1 p2 · · · pk i be arbitrary. It follows that x = p1 p2 · · · pk r
for some r ∈ Z. Obviously, we can write xm1 +···+mk = as ∈ B for some s ∈ Z,
implying that x ∈ R(B). This yields hp1 p2 · · · pk i ⊆ R(B). Next, let x ∈ R(B)
mk
be arbitrary. It follows that xn ∈ B = hpm 1 · · · pk i, form which, we obtain x =
1 n
m1 mk n
p1 · · · pk r for some r ∈ Z. Thus, pi |x for all 1 ≤ i ≤ k, yielding pi |x because
pi is prime. This implies p1 · · · pk |x, for pi ’s are distinct primes. That is, x ∈
hp1 p2 · · · pk i, and hence R(B) ⊆ hp1 p2 · · · pk i. Therefore, R(B) = hp1 p2 · · · pk i,
finishing the proof. 

2. Let {x, y} be a linearly independent set in E, which is a vector space over a field
F . Use Zorn’s Lemma to enlarge the independent sets {x, y} and {x, x + y} to
{x, y}∪{αi }i∈I and {x, x+y}∪{βi }i∈I , respectively, each of which is a Hamel basis
for E. It is well-known that there exists a linear transformation u : E → E satisfying
u(x) = x, u(y) = x + y, and u(αi ) = βi for all i ∈ I. The linear transformation u
is invertible because it maps the elements of the Hamel basis {x, y} ∪ {αi }i∈I onto
those of the Hamel basis {x, x + y} ∪ {βi }i∈I . This means u is an automorphism
of the vector space E. Now, suppose that a linear transformation f : E → E, also
called an endomorphism of the vector space E, commutes with any invertible linear
2.4. FOURTH COMPETITION 73

transformation v : E → E, which is also called an automorphism of E. We claim,


first of all, that for any nonzero vector x, we have f (x) = λx x for some λx ∈ F .
To prove this by contradiction, letting ux : E → E be an automorphism such that
ux (x) = x and ux (f (x)) = x + f (x), we must have f ux = ux f . In particular, we
have f (ux (x)) = ux (f (x)) which means f (x) = x + f (x), yielding x = 0, which is
impossible. Therefore, for any x ∈ E there exists λx ∈ F such that f (x) = λx x.
Next, fix a nonzero x0 ∈ E. By showing that λx = λx0 , we prove the assertion.
Without loss of generality, assume that {x, x0 } is linearly independent. Therefore,
there is an automorphism ux : E → E such that ux (x) = x and ux (x0 ) = x + x0 .
But f commuting with ux implies that

λx x + λx0 x0 = f (x + x0 ) = f ux (x0 )

= ux f (x0 ) = λx0 ux (x0 )
= λx0 x + λx0 x0 ,
yielding λx x = λx0 x. From this, we obtain λx = λx0 because x 6= 0. So we have
shown that f (x) = λx0 x for all x ∈ E, or equivalently f = λx0 IE , which is what we
want, finishing the proof. 

3. It suffices to show that any nonzero x ∈ R has a multiplicative inverse. To this


end, let a nonzero x ∈ R be given. It follows from the hypothesis that the following
descending chain of the ideals of R
xR ⊇ x2 R ⊇ x3 R ⊇ · · ·
must terminate. That is, there exists N ∈ N such that xn R = xN R for all n ≥ N .
In particular, xN +1 R = xN R, implying that xN ∈ xN +1 R. Hence, there exists
y ∈ R such that xN = xN +1 y, yielding xN (1 − xy) = 0. This, in view of the fact
that R is an integral domain, implies that 1 − xy = 0, yielding xy = yx = 1. In
other words, x is invertible, which is what we want. 

2.4.3. General. 1. Doing the substitution x = π2 − t, we can write


Z π2
sinn ( π2 − t)
Z 0
sinn x
I := dx = (−dt)
0 cosn x + sinn x π cosn (
2
π n π
2 − t) + sin ( 2 − t)
Z π2
cosn t
= dt.
0 sinn t + cosn t
We have
π
sinn x + cosn x  1 π
Z
1  1 2 π
I = I +I = dx = × = ,
2 2 0 cosn x + sinn x 2 2 4
for all n ∈ R. 

2. (a) Define the events a, b, c, a1 , b1 , c1 as follows:

a, b, c: The event that the prize is in box A, B, or C, respectively.

a1 , b1 , c1 : The event that the box A, B, or C is opened, respectively.


74 2. SOLUTIONS

It is obvious that {a, b, c} is a partition of the probability space and that the
desired probability is P (b|a1 ). Using the Inverse Probability Theorem or Bayes’
Theorem, we can write
P (b)P (a1 |b) P (b)P (a1 |b)
P (b|a1 ) = =
P (a1 ) P (a)P (a1 |a) + P (b)P (a1 |b) + P (c)P (a1 |c)
1 1
3 × 2 1
= 1 1 1 1 = .
3 ×0+ 3 × 2 + 3 ×1
3
So the probability that the chosen box, i.e. B, contains the prize is 31 .

(b) Plainly, the desired probability is P (c|a1 ). Again, using Bayes’ Theorem,
we can write
P (c)P (a1 |c) P (c)P (a1 |c)
P (c|a1 ) = =
P (a1 ) P (a)P (a1 |a) + P (b)P (a1 |b) + P (c)P (a1 |c)
1
3 × 1 2
= 1 1 1 1 = .
3 × 0 + 3 × 2 + 3 × 1 3
That is, the probability that the other box, i.e., C, contains the prize is 32 . 

2.5. Fifth Competition

2.5.1. Analysis. 1. No, it is not. We present a counterexample on the closed


interval [0, 1] which will work on the open interval (0, 1) as well. Define the functions
f, g : [0, 1] → [0, 1] as follows
x ∈ Q ∩ (0, 1], x = pq , p, q ∈ N, gcd(p, q) = 1,
 1
f (x) = q
0 x ∈ [0, 1] \ (Q ∩ (0, 1]).
1
x ∈ { q |q ∈ N},

1
g(x) =
0 x∈ / { 1q |q ∈ N}.
Obviously,

1 x ∈ Q ∩ (0, 1],
g ◦ f (x) =
0 x ∈ [0, 1] \ (Q ∩ (0, 1]).
We claim that f, g are Riemann integrable on [0, 1] but g ◦ f is not integrable on
[0, 1]. In fact, it turns out that g ◦ f is not integrable on any closed interval [ε, δ],
where 0 < ε < δ < 1, from which it follows that g ◦ f in not integrable on the open
interval (0, 1). In view of Solution 3 of 2.1.1, we see that
∀a ∈ (0, 1) : lim f (x) = 0, lim f (x) = 0, lim f (x) = 0.
x→a x→0+ x→1−

Thus, the function f is continuous at the irrational points of (0, 1) and the set of
discontinuity points of f , i.e., Q ∩ (0, 1), is countable. It is easily seen that g is
discontinuous at any point of the set { 1q |q ∈ N} ∪ {0} and it is continuous on the
set (0, 1] \ { 1q |q ∈ N}. Finally, using the fact that Q ∩ (0, 1] is dense in [0, 1], we see
that g ◦ f is discontinuous at any point of the interval [0, 1]. From this point on,
we present two proofs for the claim we made in the above. The first one uses some
standard theorem from analysis and the second proof is self-contained.
2.5. FIFTH COMPETITION 75

First proof. We will make use of Lebesgue’s Integrability Criterion for inte-
grals in the Riemann sense which asserts that a bounded function f : [a, b] → R
is integrable in the Riemann sense if and only if the set consisting of the points at
which f is discontinuous has measure zero. Recall that a set A ⊆ R is said to be
a set of measure zero if for any ε > 0, there exists a sequence (In )+∞n=1 of intervals
S+∞ P+∞
such that A ⊆ n=1 In and n=1 `(In ) < ε, where `(In ) := βn − αn and αn and βn
are the end-points of the interval In . It is easy to see that any finite or countable
subset of reals has measure zero and that a countable union of sets of measure
zero has measure zero. In view of all that, we see that the functions f and g are
Riemann integrable on [0, 1] and hence on (0, 1) but g ◦ f is not integrable (0, 1)
because it is not integrable on any interval [ε, δ], where 0 < ε < δ < 1, for g ◦ f is
continuous nowhere.

Second proof. Define the sequences (fn )+∞ +∞


n=1 and (gn )n=1 , with fn , gn :
[0, 1] → [0, 1], as follows
x = pq , p, q ∈ N, gcd(p, q) = 1, q ≤ n,
 1
fn (x) = q
0 otherwise.
 1
1 x = q , q ∈ N, q ≤ n,
gn (x) =
0 otherwise.
It is easily verified that the sequences (fn )+∞ +∞
n=1 and (gn )n=1 converge uniformly on
[0, 1] to f and g, respectively. As fn and gn (n ∈ N) are discontinuous at only
finitely many points of the interval [0, 1], it follows that f and g are integrable in
the Riemann sense on [0, 1]. Now, we see from the following lemma that g ◦ f is
not integrable on any interval [ε, δ], where 0 < ε < δ < 1, which is what we want.

Lemma. Let f : [a, b] → R be bounded and integrable on [a, b] in the Riemann


sense. Then the set of points at which f is continuous is infinite.
Proof. Since f integrable on any closed subinterval of [a, b], it suffices to prove
that f is continuous at some point of the interval [a, b], for it would then follow
that any subinterval of [a, b] contains a point at which f is continuous. And hence,
the set of points at which f is continuous would be infinite, which is what we want.
Taking ε = b − a in Riemann’s criterion for integrability on the interval I0 := [a, b],
we obtain a partition P0 : a = t00 < · · · < t0n0 = b of I0 such that
U (P0 , f ) − L(P0 , f ) < b − a,
where
n0
X n0
X
U (P0 , f ) = M0i ∆t0i , L(P0 , f ) = m0i ∆t0i , ∆t0i = t0i − t0(i−1) ,
i=1 i=1

M0i = sup f (x), m0i = inf f (x).


t0(i−1) ≤x≤t0i t0(i−1) ≤x≤t0i

Hence, there exists 1 ≤ i0 ≤ n0 such that


M0i0 − m0i0 < 1,
for otherwise it would follow that U (P0 , f ) − L(P0 , f ) ≥ b − a, which is impossible.
Set I1 := [a1 , b1 ] = [t0(i0 −1) , t0i0 ]. As f is integrable on I1 , taking ε = b1 −a
2 , a
1

similar argument shows that there exists a subinterval I2 := [a2 , b2 ] of I1 such that
supx∈I2 f (x) − inf x∈I2 f (x) < 21 . Continuing this way, we obtain a sequence of
76 2. SOLUTIONS

nested closed interval I1 ⊃ I2 ⊃ · · · such that 0 ≤ supx∈In f (x) − inf x∈In f (x) < n1 .
T+∞
Since n=1 In 6= ∅ because I0 is compact, in view of the above inequality, it follows
T+∞
that f is continuous at any point of n=1 In 6= ∅, finishing the proof. 

2. Remark. Adjusting the proof below, it is not difficult to prove the following
problem, which characterizes all continuous real functions having the property that
they do not assume any value more than twice. Let f : R −→ R be a continuous
function that is neither increasing nor decreasing on R and that it assumes any
value at most twice. Prove that one of the following statements holds.
(i) There exists an a ∈ R such that f or −f is strictly increasing on (−∞, a]
and is strictly decreasing on [a, +∞).
(ii) There exist a, b ∈ R with a < b such that f or −f is strictly increasing
on (−∞, a], is strictly decreasing on [a, b], and is strictly increasing on [b, +∞).
Moreover, limx→−∞ f (x) ≥ limx→+∞ f (x).

If the function f is strictly increasing or strictly decreasing, there is nothing to


prove. So we may assume that f is neither strictly increasing nor strictly decreasing.
At this point, we need the following lemma.

Lemma. Let R be a totally ordered set with at least three elements and f :
R → R a function that is neither strictly increasing nor strictly decreasing. Then,
there exist x1 , x2 , x3 ∈ R with x1 < x2 < x3 such that f (x2 ) ≥ max f (x1 ), f (x3 )
or f (x2 ) ≤ min f (x1 ), f (x3 ) .
Proof. To prove the assertion by contradiction, suppose that for all x1 , x2 , x3 ∈ 
R with x1 < x2 < x3 , we havef (x2 ) < max f (x1 ), f (x3 ) and f (x2 ) > min f (x1 ), f (x3 ) .
Now, since max f (x1 ), f (x3 ) = f (x1 ) or max f (x1 ), f (x3 ) = f (x3 ), we see that
for all x1 , x2 , x3 ∈ R with x1 < x2 < x3 , we have
f (x1 ) < f (x2 ) < f (x3 ) or f (x1 ) > f (x2 ) > f (x3 ). (∗)
Pick a, b, c ∈ R with a < b < c. It follows that either f (a) < f (b) < f (c) or
f (a) > f (b) > f (c). Suppose f (a) < f (b) < f (c). We show that f is strictly
increasing, a contradiction. To see this, let x, y ∈ R with x < y be arbitrary. There
are five cases to consider. (i) x < y < a, (ii) x < y = a, (iii) x < a < y, (iv)
x = a < y, and (v) a < x < y. In each case, in view of (∗), it is easy to see
that f (x) < f (y), from which, we conclude that f is strictly increasing, which is
a contradiction. Likewise, if f (a) > f (b) > f (c), one can see, in a similar fashion,
that f is strictly decreasing which is again a contradiction. So the proof is complete
by contradiction. 

In view of the lemma, there are two cases to consider. 


(i) There are x1 , x2 , x3 ∈ R with x1 < x2 < x3 such that f (x2 ) ≥ max f (x1 ), f (x3 ) .
In this case, from f (x2 ) ≥ max(f (x1 ), f (x3 )) and the continuity of f , we see
that the function f attains its absolute maximum on the closed interval [x1 , x3 ]
at an internal point of the interval, say, at xM ∈ (x1 , x3 ). By proving that f
attains its only absolute maximum on R at xM , we complete the proof in this case.
To this end, we first show that f (xM ) > f (x) for all x ∈ [x1 , x3 ] with x 6= xM .
Suppose to the contrary that there exists an x0 ∈ [x1 , x3 ] with x0 6= xM such
that f (x0 ) = f (xM ). Again, there are two cases to consider. (a) x1 ≤ x0 < xM
and (b) xM < x0 ≤ x3 . Let x1 ≤ x0 < xM and pick a x0 < t < xM . As f
2.5. FIFTH COMPETITION 77

assumes the value f (xM ) at most twice and f (xM ) is the absolute maximumof f
f (xM )+max f (x3 ),f (t)
on [x1 , x3 ], we see that f (t) < f (x0 ) = f (xM ). Set λ = 2 . It
is obvious that f (t) < λ < f (x0 ), f (t) < λ < f (xM ), and f (x3 ) < λ < f (xM ).
Thus, it follows from the Intermediate Value Theorem that there exist ξ1 , ξ2 , ξ3
with x0 < ξ1 < t , t < ξ2 < xM , and xM < ξ3 < x3 such that f (ξi ) = λ for each
i = 1, 2, 3, contradicting the hypothesis. Similarly, if xM < x0 ≤ x3 , we obtain a
contradiction. That is, we have shown that f (xM ) > f (x) for all x ∈ [x1 , x3 ] with
x 6= xM . Next, we prove that f (x) < f (xM ) for all x ∈ R with x 6= xM . Again
suppose to the contrary that there is a t ∈ R \ {xM } such that f (t) ≥ f (xM ). From
what we have proved so far, it follows that t < x1 or t > x3 . If t < x1 , noting
that f (x1 ) < f (xM ) ≤ f (t), we see that there exists an x0 ∈ R with  t ≤ x0 < x1
f (xM )+max f (x1 ),f (x3 )
such that f (x0 ) = f (xM ). Now, set λ = 2 . Again, it is
obvious that f (x1 ) < λ < f (x0 ), f (x1 ) < λ < f (xM ), and f (x3 ) < λ < f (xM ).
Hence, by the Intermediate Value Theorem we obtain ξ1 , ξ2 , ξ3 with x0 < ξ1 < x1
, x1 < ξ2 < xM , and xM < ξ3 < x3 such that f (ξi ) = λ for each i = 1, 2, 3,
contradicting the hypothesis. Similarly, if x3 < t, we obtain a contradiction. This
finishes the proof in this case. 
(ii) There are x1 , x2 , x3 ∈ R with x1 < x2 < x3 such that f (x2 ) ≤ min f (x1 ), f (x3 ) .
Adjusting the proof of case (i) or replacing f by −f and following the line of
argument presented in case (i), we can see that the absolute minimum of f on R
occurs in the interval (x1 , x3 ) and that it is unique. We omit the proof for the sake
of brevity. 


3. Let B = (p, q) : p, q ∈ Q, p < q , where (p, q) denotes the open interval with
end points p and q in R. The set B is a countable basis for the ordinary topology
of R. Write B = {In }n∈N , where In = (pn , qn ) for some pn , qn ∈ Q, and set

T := {n ∈ N : card(In ∩ A) ≤ ℵ0 }.

Obviously, T & N because otherwise A would be countable which is impossible.


Now, set
[  [
A1 = In ∩ A = (In ∩ A).
n∈T n∈T

The set A1 is countable because so is In ∩ A for all n ∈ T . By showing that any


point x ∈ A \ A1 is a congestion point of A, we finish the proof. To prove this by
contradiction, suppose that there exists an ε0 > 0 such that (x − ε0 , x + ε0 ) ∩ A
is countable. As B = {In }n∈N is a basis for the topology of R, we see that there
exists n0 ∈ N such that x ∈ In0 ⊆ (x − ε0 , x + ε0 ), from which, we obtain

In0 ∩ A ⊆ (x − ε0 , x + ε0 ) ∩ A,

implying that In0 ∩ A is countable because so is (x − ε0 , x +S ε0 ) ∩ A. That is,


card(In0 ∩ A) ≤ ℵ0 , yielding n0 ∈ T , and hence x ∈ In0 ∩ A ⊆ n∈T (In ∩ A) = A1 .
Consequently, x ∈ A1 , which contradicts the hypothesis that x ∈ A \ A1 . Thus, x
is a congestion point of A, which is what we want. 
78 2. SOLUTIONS

2.5.2. Algebra. 1. Let A : V → V be such that A 6= I , A2 6= I, A3 = I. If


x 6= 0 is an eigenvector corresponding to the eigenvalue λ ∈ C, we can write
Ax = λx =⇒ A2 x = λ2 x =⇒ A3 x = λ3 x,
which, in view of A3 = I, implies that λ3 = 1. That is, the set of the eigenvalues of
A is a subset of the third roots of unity. Likewise, if A 6= I, . . . , Ak−1 6= I, Ak = I,
then the set of the eigenvalues of A is a subset of the kth roots of unity. The
eigenvalues of A do not necessarily form a group under multiplication. To see this,
let ωk = cos 2π 2π
k + i sin k , where k − 1 ∈ N, and just note that ωk 6= 1 is the only
eigenvalue of the linear transformation ωk In , where In : Cn → Cn is the identity
transformation. Obviously, {ωk } does not form a group under multiplication. As
for a necessary and sufficient condition for the set of eigenvalues of A to form a
group, nothing interesting can be said. Having said that, it is easily seen that the
set of eigenvalues of A forms a group under multiplication, in fact a cyclic subgroup
of the multiplicative group of the kth roots of unity, if and only if the set of the
eigenvalues of A is closed under multiplication. 

2. We have ord(a) = 7 because a7 = 1, a 6= 1, and 7 is prime. Induction on k


together with the relations ab = b2 a and a = b2 ab−1 yields
k
ak = b2 ak b−1 ,
7
for all k ∈ N. In particular, a7 = b2 a7 b−1 together with a7 = 1 implies that
7
b2 −1 = 1. That is, b127 = 1. But 127 is prime and b 6= 1. Hence, ord(b) = 127.
Therefore, ord(a) = 7 and ord(b) = 127, which is what we want. 

3. (a) We can write


(−x)3 + 2(−x)2 + (−x) = 0 =⇒ −x3 + 2x2 − x = 0,
for all x ∈ R. On the other hand, x3 + 2x2 + x = 0 for all x ∈ R. Adding up the
two equalities, we obtain 4x2 = 0 for all x ∈ R. We can also write
(2x)3 + 2(2x)2 + (2x) = 0 =⇒ 8x3 + 8x2 + 2x = 0,
for all x ∈ R. But 8x3 = 8x2 = 0 because 4x2 = 0 for all x ∈ R. This together
with the above yields 2x = 0 for all x ∈ R, which is what we want.

(b) Using (a), we can write


(x + 1)3 + 2(x + 1)2 + (x + 1) = 0 =⇒ x2 + x = 0,
for all x ∈ R. Consequently, x2 = −x = x because 2x = 0. Hence, x2 = x for
all x ∈ R. Therefore, R is a Boolean ring, which is commutative by the following
argument.
(x + y)2 = x + y =⇒ x2 + y 2 + xy + yx = x2 + y 2
=⇒ xy = −yx = yx,
for all x, y ∈ R, which is what we want. 
2.5. FIFTH COMPETITION 79

2.5.3. General. 1. We need the following lemma

Lemma. The product of k consecutive integers is divisible by k!.


Proof. It suffices to show that the product of k natural numbers is divisible by
k!. To this end, let k, n ∈ N with k ≤ n be given. To show that n(n−1) · · · (n−k+1)
n!
is divisible by k!, it is enough to prove that Cnk := is an integer because
k!(n − k)!
n(n − 1) · · · (n − k + 1) = k!Cnk .
By proving the polynomial identity below, which is also known as the Binomial
Theorem
Xn
(1 + x)n = Cnk xk , (∗)
k=0
we conclude that Cnk is an integer, finishing the proof. Recall that, by Taylor’s
Formula, for two polynomial functions p, q : R → R of degree n, we have p = q
if and only if p(k) (0) = q (k) (0) for each k = 0, 1, . . . , n, where p(k) , q (k) denote the
kth derivative of p and q, respectively, and the zeroth derivative of a function, by
definition, is just the function. Also, it is readily verified that

 k(k − 1) · · · (k − (i − 1))(a + x)k−i 1 ≤ i < k,
k
 (i)
(a + x) = k! i = k,
0 i > k,

where a ∈ R is a constant. Use p and q to denote the left and right hand side of
(∗). Obviously, p(0) = q(0). We can write
(k)
p(k) (0) = (1 + x)n (0) = n(n − 1) · · · (n − k + 1)(1 + 0)n−k = k!Cnk ,
n
X (k)
q (k) (0) = Cni xi = Cnk (xk )(k) = Cnk k!,
i=0

for each k = 1, . . . , n. So we have shown that p(k) (0) = q (k) (0) for each k =
0, 1, . . . , n, implying that p = q, which is what we want. 

Now to prove the assertion, note first that


 
m−1
Y Y n
(mn)! =  (kn + j) .
k=0 j=1

But
n
Y n−1
Y
(kn + j) = (kn + j) × n(k + 1).
j=1 j=1
Qn−1
It follows from the lemma that j=1 (kn + j) is divisible by (n − 1)!, whence
Qn
j=1 (kn + j) is divisible by (n − 1)! × n(k + 1) = n!(k + 1). This implies (mn)! =
Qm−1 Qn  Qm−1 m
k=0 j=1 (kn + j) is divisible by k=0 n!(k+1) = (n!) m!. Thus, the number
(mn)!
is an integer, which is what we want. 
m!(n!)m
2. The assertion is known as the Gauss-Lucas Theorem . As is shown in what
follows, it suffices to prove that if the roots of a polynomial p lie in a closed half
80 2. SOLUTIONS

H’ z

x’ x

Figure 7

plane, then so do those of p0 . For a subset S of C, the closed convex hull of S,


denoted by the symbol C(S), is said to be the intersection of all closed convex
subsets of C which include S, i.e.,
\
C(S) := C : S ⊆ C, C is closed and convex .

It is straightforward to see that if S = {zi }ni=1 , where n ∈ N, then C(S) = C(S).


We claim that
\
C(S) = C(S) = H : S ⊆ H, H is a closed half plane .
T
Set C1 := H : S ⊆ H, H is a closed half-plane . As C1 is closed and convex, it
c
follows that C(S) = C(S) ⊆ C1 . We establish our claim by showing that C(S) =
C(S)c ⊆ C1c . To see this, let x ∈
/ C(S) = C(S) be arbitrary. As C(S) is closed and
{x} is compact, it follows that there exists an x0 ∈ C(S) such that
|x − x0 | = inf |x − y|.
y∈C(S)

Now, consider the perpendicular bisector ∆ of the line segment joining x and x0 .
Let H 0 be the closed half-plane containing the point x0 ∈ S. The half plane H 0
includes S because if, contrary to this, there exists a point z ∈ S in the other half
plane, then in the triangle x0 xz the perpendicular foot h of the altitude passing
through the vertex x belongs S, for x0 , z ∈ S, and moreover |x − h| < |x − x0 |, which
is a contradiction. Therefore, the closed half plane H 0 includes S, implying that
C1 ⊆ C(S), which is what we want.

To complete the proof we need the following lemma.

Lemma. If the roots of a polynomial with complex coefficients lie in a closed


half plane, then so do those of the derivative of the polynomial.
Proof. Suppose z1 , . . . , zn are the roots of a polynomial p so that p(z) =
c(z − z1 ) · · · (z − zn ), where c ∈ C. Also suppose that the closed half plane H =
{z ∈ C : Im z−a b ≤ 0} includes the roots of p. We show that H includes the roots of
p0 , the derivative of p. If z = zi , for some 1 ≤ i ≤ n, is a root of p of multiplicity
2.5. FIFTH COMPETITION 81

greater than one, then zi is a root of p0 as well, in which case there is nothing to
prove. So, without loss of generality, we may assume that
p0 (z) 1 1
= + ··· + .
p(z) z − z1 z − zn
Now, suppose z ∈ / H. We complete the proof by showing that z cannot be a root
of p0 . To this end, as z ∈/ H and zi ∈ H, we obtain
z − zi z−a zi − a
Im = Im − Im > 0,
b b b
b
for all 1 ≤ i ≤ n. It follows that Im z−zi
< 0 for all 1 ≤ i ≤ n. From this, we obtain
n
bp0 (z) X b
Im = Im < 0,
p(z) i=1
z − zi
implying that p0 (z) 6= 0, proving the lemma. 

Now, assume that f is a polynomial of degree greater than or equal to 2 and


A and A0 denote the sets of the roots of the equations f (z) = 0 and f 0 (z) = 0,
respectively. Suppose that H is an arbitrary closed half plane including A. It
follows from the lemma that H includes A0 . This yields
\
A0 ⊆

H : A ⊆ H, H is a closed half plane = C(A).
But since C(A) is a closed convex set which includes A0 , we see that
C(A0 ) = C(A0 ) ⊆ C(A),
which is what we want. 

3. Remark. The lemma below together with the proof following the lemma shows
that if the initial ball trajectory intersects the line segment joining the two foci, then
there exists a hyperbola confocal with the original ellipse to which all trajectory
segments are tangent.
We need the following lemma.

Lemma. (i) Let F and F 0 be two points and l a line in a plane not intersecting
the line segment F 0 F (resp. intersecting the line segment F F 0 between its end points
except the perpendicular bisector of F 0 F ). Then, there exists a unique ellipse (resp.
hyperbola) whose foci are F 0 and F and to which the line l is tangent.
(ii) Let F and F 0 be two points and P m and P n two half-lines through a point
P in the plane such that the half-lines do not intersect the line segment F 0 F (resp.
intersect F F 0 between its end points, i.e., between F and F 0 ). Then, there exists a
unique ellipse (resp. hyperbola) whose foci are F 0 and F and to which the half-lines
P m and P n are tangent if and only if ∠F P m = ∠F 0 P n.
Proof. (i) Since the foci, i.e., F and F 0 , of the desired ellipse (resp. hyperbola)
are fixed, it suffices to show that the sum (resp. the difference) of the focal radii of
any point of the ellipse (resp. hyperbola) is uniquely determined by the given line
l. To this end, suppose that the line l is tangent to an ellipse (resp. a hyperbola)
whose foci are F and F 0 . By the optical property of ellipses (resp. hyperbolas) ,
the tangent point is obtained as follows. Reflect the focus F in the line l to get F1 .
Then, join F1 and F 0 to intersect the line l at the tangent point, say, A ∈ l. It is
plain that AF +AF 0 = F1 F 0 = F2 F (resp. |AF −AF 0 | = F1 F 0 = F2 F ), where F2 is
82 2. SOLUTIONS

F1 l
l

A
F1
A F’
F
F
F’
Figure 8

Figure 9

the reflection of F 0 across the line l. In other words, the sum (resp. the difference)
of the focal radii of the point A, and hence any point, of the desired ellipse (resp.
hyperbola) is F1 F 0 = F2 F , which is uniquely determined by the given line l. This
proves (i).
(ii) The uniqueness follows from (i). Let F1 and F2 be the reflection of F and
F 0 across the half-lines P m and P n, respectively.
First, suppose there exists an ellipse (resp. a hyperbola) whose foci are F 0 and
F and to which the half-lines P m and P n are tangent. It is easily seen that the sum
(resp. the difference) of the focal radii of any point of the ellipse (resp. hyperbola)
is equal to F2 F = F1 F 0 . Now, the triangles 4F2 P F and 4F 0 P F1 are congruent
because F2 F = F1 F 0 , F2 P = F 0 P , and P F = P F1 . This, in particular, implies
∠F2 P F = ∠F 0 P F1 , yielding
∠F 0 P F + 2∠F 0 P n = ∠F 0 P F + 2∠F P m
2.6. SIXTH COMPETITION 83

(resp.
∠F 0 P F − 2∠F 0 P n = ∠F 0 P F − 2∠F P m).
Thus, ∠F 0 P n = ∠F P m, which, in turn, yields
∠F P n = ∠F 0 P F + ∠F 0 P n = ∠F 0 P F + ∠F P m = ∠F 0 P m,
(resp.
∠F P n = ∠F 0 P F − ∠F 0 P n = ∠F 0 P F − ∠F P m = ∠F 0 P m, )
as desired.
Next, suppose ∠F P n = ∠F 0 P m. We can write
∠F2 P F = ∠F 0 P F + 2∠F P n = ∠F 0 P F + 2∠F 0 P m = ∠F1 P F 0 ,
(resp.
∠F2 P F = ∠F 0 P F − 2∠F P n = ∠F 0 P F − 2∠F 0 P m = ∠F1 P F 0 ).
That is, ∠F2 P F = ∠F1 P F 0 . It thus follows that the triangles 4F2 P F and 4F 0 P F1
are congruent because F2 P = F 0 P , ∠F2 P F = ∠F1 P F 0 , and P F = P F1 . In
particular, we must have F2 F = F1 F 0 . In other words, the two ellipses (resp.
hyperbolas) whose foci are F and F 0 and are tangent to the half-lines P m and P n
coincide, proving the “if part” of the assertion. 

Let F and F 0 be the foci of the ellipse ξ and P Q, QR, RS be three consecutive
segments of the ball trajectory. It is well known that in any ellipse the focal radii of
any point on the ellipse form equal angles with the tangent at the point. In other
words, the normal and tangent lines at any point on an ellipse bisect the angles
between the focal radii of the point. From this, it follows that if a billiard segment
intersects the interior of the line segment F 0 F , so does all other billiard segments;
if a billiard segment passes through one of the foci, then so does all other billiard
segments; and finally, if a billiard segment does not intersect the line segment F 0 F ,
then neither does any other billiard segment. Now, with P, Q, R, S as in the above,
it follows that ∠F 0 QP = ∠F QR and ∠F 0 RQ = ∠F RS. This together with the
lemma implies that there are two ellipses ηQ and ηR confocal with ξ each of which
is tangent to QR. We see from part (i) of the lemma that ηQ = ηR for any billiard
segments QR. That is, there is an ellipse η confocal with the ellipse ξ such that
the ball trajectory is always tangent to η, which is what we want. 

2.6. Sixth Competition

2.6.1. Analysis. 1. To prove the assertion by contradiction, suppose that


there are three rational points A = (x1 , y1 ) , B = (x2 , y2 ), and C = (x3 , y3 ) on
the circle. Since A, B, C ∈ Q2 , it follows that the equations of the perpendicular
bisectors of the line segments AB and BC are of the form a1 x + b1 y = c1 and
a2 x + b2 y = c2 , respectively, where ai , bi , ci ∈ Q (i = 1, 2) and a1 b2 − b1 a2 6= 0. It
is obvious that the center (x, y) of the circle is the solution of

a1 x + b1 y = c1
,
a2 x + b2 y = c2
which must be rational because ai , bi , ci ∈ Q (i = 1, 2). This is a contradiction,
proving the assertion, which is what we want. 
84 2. SOLUTIONS

2. As the sequence (fn )+∞


n=1 is uniformly convergent to f on M , for a given ε > 0,
there exists a natural number N1 such that for all n ≥ N1 and x ∈ M we have
 ε
d fn (x), f (x) < .
6
Now, since the function fN1 is continuous at the point x ∈ M , for the given ε > 0,
there exists a δ > 0 such that for all y ∈ M with d(x, y) < δ we have
 ε
d fN1 (y), fN1 (x) < .
3
From limn xn = x, we see that for δ > 0 obtained from the above, there exists a
natural number N2 such that for all n ≥ N2
d(xn , x) < δ.
Let N = max(N1 , N2 ). For the given ε > 0 and for all n ≥ N , we can write
  
d fn (xn ), f (x) ≤ d fn (xn ), f (xn ) + d f (xn ), fN1 (xn )
 
+d fN1 (xn ), fN1 (x) + d fN1 (x), f (x)
ε ε ε ε
< + + + = ε.
6 6 3 3
That is, limn→+∞ fn (xn ) = f (x), which is what we want. 
R +∞
3. Since 0 f (x) dx < +∞, we see that for given ε > 0, there exists a natural
number N1 such that for all M ≥ N1
Z +∞
f (x) dx < ε .

M 3
The function f is continuous on the compact interval [0, N1 + 1]. Hence, it is
uniformly continuous on [0, N1 + 1]. Therefore, for given ε > 0, there exists a δ > 0
such that for all x, y ∈ [0, N1 + 1] with |x − y| < δ, we have
f (x) − f (y) < ε .

3N1
Now, for a given ε > 0, find N1 > 0 and δ > 0 as in the above and pick N ∈ N such
that N1 < δ. For all n ≥ N , we can write
Z +∞ Z N1 Z +∞
f (x + 1 ) − f (x) dx = f (x + 1 ) − f (x) dx + f (x + 1 ) − f (x) dx

0 n 0 n N1 n
Z +∞ Z +∞
ε f (x + 1 ) dx +

< (N1 − 0) + f (x) dx
3N1 N1 n N1
ε ε ε
< + + = ε.
3 3 3
That is,
Z +∞
f (x + 1 ) − f (x) dx − 0 < ε,



0 n
R +∞
for al n ≥ N . Therefore, limn→+∞ 0 f (x + n1 ) − f (x) dx = 0, which is what we
want. 
2.6. SIXTH COMPETITION 85

2.6.2. Algebra. 1. Proceed by contradiction. Suppose that AB is a subgroup


of G. Using Lagrange’s Theorem , we see that |AB| = pn b, where b ∈ N and b|a.
Since gcd(p, a) = 1 and b|a, gcd(p, b) = 1. On the other hand, we can write
|A| × |B|
|AB| = .
|A ∩ B|
But A ∩ B $ B because B * A. Hence, |A ∩ B| < |B|. Now, in view of Lagrange’s
Theorem, let |A ∩ B| = pi for some 0 ≤ i < m. We can write
pn .pm
pn b = =⇒ b = pm−i ,
pi
implying that gcd(p, b) = p, which is a contradiction. Therefore, AB cannot be a
subgroup of G, finishing the proof. 

2. We prove the assertion for matrices over a general division ring ∆. Induct on m.
If m = 1, the assertion is easily seen to hold. Assuming that the assertion holds for
matrices of size less than m, we prove the assertion for matrices of size m. To this
end, note first that r := rank(A1 ) ≤ dim ker A1 because A21 = 0. Now, from the
Rank-Nullity Theorem, we see that r ≤ m 2 . Let K := ker A1 . Since Ai ’s commute,
K is invariant under all Ai ’s (1 ≤ i ≤ n). Therefore, after a similarity, one can
write  
Bi Ci
Ai = ,
0 Di
where Bi ∈ Mm−r (∆) with B1 = 0, Ci ∈ M(m−r)×r (∆), and Di ∈ Mr (∆) for all
1 ≤ i ≤ n. We have m < 2n , yielding r ≤ m 2 < 2
n−1
. As Ai ’s commute, so do
Di ’s in Mr (∆) (1 ≤ i ≤ n). Hence, from the induction hypothesis, we see that
D2 · · · Dn = 0, which, in turn, implies that
 
B C
A2 · · · An = , (∗)
0 0
for some appropriate matrices B ∈ Mm−r (∆) and C ∈ M(m−r)×r (∆). On the other
hand,  
0 C1
A1 = . (∗∗)
0 D1
It now easily follows from (∗) and (∗∗) that
A1 A2 · · · An = A1 (A2 · · · An−1 An ) = 0,
which is what we want. 

3. Since e2 = e, f 2 = f , and ef = f e, using the Binomial Theorem, we see that


for all n ∈ N, there exists a Kn ∈ N such that (e − f )n = e + (−1)n f + Kn ef .
Suppose (e − f )n = 0. Without loss of generality, we may assume that n is even.
It follows that
(e − f )n = e + f + Kn ef = 0.
Multiplying both sides of the above by e and f , respectively, and noting that e2 = e,
f 2 = f , and ef = f e, we obtain
e + ef + Kn ef = 0 = f + ef + Kn ef,
yielding e = f = −(1 + Kn )ef , which proves the assertion. 
86 2. SOLUTIONS

2.6.3. General. 1. Noting that (−1)|n| = (−1)n for all n ∈ N, we can write
(−1)|n1 −m1 |+···+|nk −mk | = (−1)|n1 −m1 | × · · · × (−1)|nk −mk |
= (−1)n1 −m1 × · · · × (−1)nk −mk
= (−1)(n1 +···+nk )−(m1 +···+mk ) = (−1)0 = 1,
implying that |n1 − m1 | + · · · + |nk − mk | is an even number. 

2. Proceed by induction on n. If n = 1, the assertion is easy. Assuming that the


assertion holds for n×n matrices, we prove it for (n+1)×(n+1) matrices. Suppose
that A is a (n + 1) × (n + 1) matrix such that the sum of the elements on any row
and any column corresponding to any nonzero element of A is at least n + 1. As
the entries of A are all nonnegative integers, if the entries are all nonzero, then the
2
sum of the entries of the matrix A is at least (n + 1)2 which is no less than (n+1)2 .
Now, assume that for some 1 ≤ i, j ≤ n + 1, aij = 0. Let Âij be the n × n matrix
which is obtained from eliminating row i and column j of the matrix A. Obviously,
the sum of the elements on any row and any column corresponding to any nonzero
element of the matrix Âij is at least n. So it follows from the induction hypothesis
2
that the sum of the entries of Âij is no less than n2 . Noting that the sum of the
entries of row i and column j of A is at least n + 1, we see that the sum of the
entries of the matrix A is at least
n2 (n + 1)2 + 1 (n + 1)2
+n+1 = > ,
2 2 2
which is what we want, finishing the proof. 

3. The assertion is a special case of the following: Suppose k + m points inside


a circle are given. Then, there exists a straight line not passing through any of
the points that divides the circle into two sections one of which containing k points
and the other the remaining m points. To see this, note that the number of lines
passing through a pairof points from these k + m points is finite; in fact it is less
than or equal to k+m 2 . As the number of directions determined by these lines is
finite, there is a line d whose direction is not parallel to any of the lines determined
by any pair of points from the given points. Let AA0 be a diameter of the circle
whose direction is perpendicular to the line d. Project the given points on the line
segment AA0 to get k + m distinct points. Going from A to A0 , if P is the kth point
and Q the (k + 1)st point, then any line perpendicular to AA0 at any point between
P and Q does not pass through any of the given points inside the circle and yet
it divides the circle into two sections one of which contains k points, including the
point whose image is P , and the other contains the remaining m points. 

2.7. Seventh Competition

2.7.1. Analysis. 1. First of all, the hypothesis that S is closed is redundant


because it follows from the other hypothesis of the problem that such an S is closed.
The assertion is a consequence of the following. Let n ∈ N. Then there is no proper
subset S of Rn with the property that for all x ∈ Rn \ S there exists nx ∈ N with
nx > 1 such that there are exactly nx points in S as the closest point of S to x. To
2.7. SEVENTH COMPETITION 87

prove this by contradiction, pick a point x ∈ Rn \ S. It follows from the hypothesis


that there exists an nx ∈ N with nx > 1 and nx points s1 , . . . , snx in S such that
inf ||x − s|| = ||x − si || = δx ,
s∈S

for all 1 ≤ i ≤ nx . It is obvious that there is no point of S in Bδx (x), the open ball
centered at x with radius δx . Let y = x+s 2
1
and δ = δ2x . Plainly, the only point of
S in B δ (y), the closed ball centered at y with radius δ, is s1 . As Bδ (y) ⊆ Bδx (x),
it follows that for the point y ∈ Rn \ S there exists exactly one point s1 ∈ S such
that
inf ||x − s|| = ||x − s1 ||,
s∈S

which is a contradiction, finishing the proof. 

2. We need the following lemma.

Lemma. Let (Ir )r∈Q+ and A and f be as in the statement of the problem.
Then,
(i) for all x ∈ A with x ∈ I r , we have f (x) ≤ r.
(ii) for all x ∈ A with x ∈ / Ir , we have f (x) ≥ r.
Proof. Set Sx = {r : x ∈ Ir }. We have f (x) = inf Sx . To prove (i), note that
if x ∈ I r , then I r ⊆ Is whenever r < s where s ∈ Q, from which, we obtain x ∈ Is
for all s ∈ Q with r < s. Consequently, Sx includes all rational numbers greater
than r, whence f (x) = inf Sx ≤ r.
To prove (ii), note that if x ∈ / Ir , then I s ⊆ Ir whenever s < r where s ∈ Q.
This implies x ∈ / Is for all s ∈ Q with s < r. Therefore, Sx contains no rational
number less than r, whence f (x) = inf Sx ≥ r, proving the lemma. 

We now prove that f is continuous. Let x0 be an arbitrary point of A. It suffices


to prove that for any open interval (c, d) with f (x0 ) ∈ (c, d) there exists an open
neighborhood N around x0 such that f (N ) ⊆ (c, d). To this end, as c < f (x0 ) < d,
pick rational numbers p and q such that c < p < f (x0 ) < q < d. We claim that
the open set N = Iq \ I p contains x0 and moreover f (N ) ⊆ [p, q] ⊆ (c, d), finishing
the proof. Firstly, x0 ∈ N because, in view of the lemma, x0 ∈ Iq and x0 ∈ / I p.
Secondly, f (N ) ⊆ [p, q] ⊆ (c, d). To see this, let x ∈ N = Iq \ I p be arbitrary. It
follows that x ∈ Iq ⊆ I q , which, in view of the lemma, implies f (x) ≤ q. On the
other hand, since x ∈/ I p , we conclude from the lemma that f (x) ≥ p. That is, we
have proved f (x) ∈ [p, q] for all x ∈ N . Consequently, f (N ) ⊆ [p, q] ⊆ (c, d), which
is what we want. 

3. First solution: To find the limit, we need the following lemma whose first and
second parts are known as Stolz’s First and Second Theorems on limits.

Lemma. (i) Let (pn )+∞


n=1 be a sequence of reals whose elements are eventually
nonnegative, sn = p1 + · · · + pn , and limn sn = +∞. If (an )+∞
n=1 is a sequence of
real numbers such that limn an ∈ R ∪ {±∞}, then
p1 a1 + · · · + pn an
lim = lim an .
n p1 + · · · + pn n
88 2. SOLUTIONS

(ii) Let (xn )+∞ +∞


n=1 and (yn )n=1 be sequences of real numbers with limn yn = +∞
and such that (yn )n=1 is eventually increasing. If limn xynn −x
+∞
−yn−1 ∈ R ∪ {±∞}, then
n−1

xn xn − xn−1
lim = lim .
n yn n yn − yn−1

Proof. (i) There are two cases to consider.


(a) limn an = ±∞.
As p1 (−ap11)+···+pn (−an )
+···+pn = − p1 ap11+···+pn an
+···+pn , it suffices to prove the assertion for
the case when limn an = +∞. To this end, it follows from the hypothesis that for
given M > 0, there exists an N1 > 0 such that an > 2M whenever n > N1 . Let
AN1 = p1 a1 + · · · + pN1 aN1 . We can write
p1 a1 + · · · + pn an AN1 pN +1 aN1 +1 + · · · + pn an
= + 1
p 1 + · · · + pn sn sn
AN1 sn − sN1
> + 2M
sn sn
AN1 sn −sN1
But limn sn = 0 and limn sn = 1. Thus, there exists N2 > 0 such that
AN1 −M sn −sN1 2
sn > 3 and sn > 3 whenever n > N2 . Letting N = max(N1 , N2 ), for all
n > N , we have
p1 a1 + · · · + pn an −M 4M
> + = M.
p1 + · · · + pn 3 3
As M > 0 is arbitrary, this means limn p1 ap11+···+p n an
+···+pn = +∞, proving the assertion
in this case.
(b) limn an = a ∈ R.
First, let limn an = 0. It follows that for given ε > 0 there exists N1 > 0 such
that |an | < 2ε whenever n > N1 . Let AN1 be as in (a). For all n > N1 , we can write

p a + · · · + p a
1 1 n n
AN1 ε sn − sN AN1 ε
1
≤ + × ≤ + .
p 1 + · · · + pn sn 2 sn sn 2


AN1 AN
Now, as limn sn = 0, we see that there exists an N2 > 0 such that sn < 2ε 1

whenever n > N2 . Letting N = max(N1 , N2 ), for all n > N , we have


p a + · · · + p a ε ε
1 1 n n
< + = ε.
p1 + · · · + pn 2 2

That is, limn p1 ap11+···+p


+···+pn
n an
= 0, which is what we want.
Next, let limn an = a 6= 0. As limn (an − a) = 0, from what we just proved, it
follows that
 
p1 a1 + · · · + pn an p1 (a1 − a) + · · · + pn (an − a)
lim −a = lim
n p 1 + · · · + pn n p 1 + · · · + pn
= lim(an − a) = 0.
n

This implies that limn p1 ap11+···+pn an


= a, finishing the proof.
+···+pn
(ii) Define the two sequences (pn )+∞ +∞
n=1 and (an )n=1 , inductively, as follows

pn = yn − yn−1 , p1 = y1 , (n ≥ 2),
xn − xn−1 = pn an , x1 = p1 a1 , (n ≥ 2).
2.7. SEVENTH COMPETITION 89

Firstly, pn ’s are eventually nonnegative. Secondly, yn = p1 + · · · + pn → +∞ as


n → +∞. Thirdly, an = xynn −x−yn−1 and p1 a1 +· · ·+pn an = xn , and p1 +· · ·+pn = yn .
n−1

Therefore, (i) applies, proving the assertion. 

Now, note that


21 2n
1 + ··· + n Xn
xn = 2n+1
= .
n+1
Yn
2n+1
First of all, it is easily seen that the sequence (Yn )+∞
n=1 , where Yn = n+1 for all
n ∈ N, is increasing. Secondly, limn Yn = +∞. Thirdly, we can write
2n
Xn − Xn−1 n 1
lim = lim 2n+1 2n
= lim 2n = 1.
n Yn − Yn−1 n
n+1 − n
n
n+1−1
Xn
Therefore, it follows from the lemma that limn xn = limn Yn exists, and moreover
Xn Xn − Xn−1
lim xn = lim = lim = 1,
n n Yn n Yn − Yn−1

which is what we want.

Second solution: It is readily checked that


n+1
n + 2 X 2k n+2 
xn+1 = n+2
= xn + 1 ,
2 k 2(n + 1)
k=1
from which, we obtain
(n + 2)2 (xn+1 − xn ) − xn+1 − 1
xn+2 − xn+1 = ,
2(n + 1)(n + 2)
for all n ∈ N. As xn > 0 for all n ∈ N, in view of the above equality, we see that
xn+2 −xn+1 < 0 whenever xn+1 −xn ≤ 0. But a straightforward calculation reveals
that x3 = x4 = 53 . Thus, by an easy induction on n ≥ 3, we see that the sequence
(xn )+∞
n=1 is decreasing for n ≥ 3. From this, together with the fact that xn > 0 for
all n ∈ N, we conclude that limn→+∞ xn exists, and hence limn→+∞ xn = L for
some L ∈ R. Consequently,
n+2  1
L = lim xn+1 = lim xn + 1 = (L + 1) =⇒ L = 1.
n→+∞ n→+∞ 2(n + 1) 2
Therefore, limn→+∞ xn = 1, which is what we want. 

2.7.2. Algebra. 1. (a) Let x0 ∈ R be arbitrary. We show that x0 e = x0 . To


this end, note that for all x ∈ R, we can write
(x0 + e − x0 e)x = x0 x + ex − x0 (ex) = x0 x + x − x0 x = x.
That is, (x0 + e − x0 e)x = x for all x ∈ R. As the left identity element is unique, we
obtain x0 + e − x0 e = e, implying x0 e = x0 . This means e a right identity element
as well. Consequently, e is the identity element of R.

(b) Let a be an idempotent of R, i.e. a2 = a, and x ∈ R be arbitrary. It


is straightforward to see that (axa − ax)2 = 0. That is, axa − ax is a nilpotent
element of R, and hence axa−ax = 0, by the hypothesis. Likewise, (axa−xa)2 = 0,
90 2. SOLUTIONS

implying axa − xa = 0 for all x ∈ R. So we have shown that ax = xa = axa for all
x ∈ R, which is what we want. 

2. First solution: Let H be an arbitrary subgroup of G. Set H1 = H ∩ G1 . We


show that H1 is commutative, H1 E H, and that HH1 is commutative. First, H1
is commutative because H1 ⊆ G1 and G1 is commutative. Next, H1 ≤ H because
H1 = H ∩ G1 and H and G1 are subgroups of G. That H1 E H follows from
G1 E G and H1 = H ∩ G1 . Finally, to show that HH1 is commutative, using the
Second Isomorphism Theorem for groups, we can write
H H ∼ HG1
= = .
H1 H ∩ G1 G1
But HG1 ≤ G. Hence, HG G HG1
G1 ≤ G1 . Consequently, G1 is commutative because so
1

is GG1 . This, in view of the above isomorphism, completes the proof.

Second solution: Use G0 to denote the derived subgroup of G. Since G


G1 is
abelian, it follows that G0 ≤ G1 . Now for a given subgroup H of G, set
H1 = H 0 := h1 h2 h−1 −1


1 h2 : h1 , h 2 ∈ H .
It is plain that H1 is a normal subgroup of H and that HH1 is abelian because H1 is
the derived subgroup of H. To see that H1 is abelian, just note that H1 ≤ G0 ≤ G1
and that G1 is abelian by the hypothesis. This completes the proof. 

3. For the given θ : V → V , it is obvious that


ker θ ⊆ ker θ2 ⊆ · · ·
is an ascending chain of the subspaces of V . As V is finite-dimensional, it follows
that there exists an s ∈ N such that ker θs = ker θs+1 .
We claim that if ker θs = ker θs+1 , then ker θs = ker θs+k for all k ∈ N. We
prove this by induction on k. If k = 1, there is nothing to prove. Suppose the
assertion holds for k. To prove the assertion for k + 1, note first that ker θs ⊆
ker θs+k+1 . Next, let x ∈ ker θs+k+1 be arbitrary. We obtain θs+k (θx) = 0. That
is, θx ∈ ker θs+k , which, in view of the induction hypothesis, yields θx ∈ ker θs .
This implies θs+1 x = 0 which means x ∈ ker θs+1 . But ker θs+1 = ker θs . Thus,
x ∈ ker θs . As x ∈ ker θs+k+1 was arbitrary, we see that ker θs+k+1 ⊆ ker θs .
Therefore, ker θs+k+1 = ker θs , proving the claim by induction on k.
We now show that V = im(θs ) ⊕ ker(θs ), where s is as in the above. Note that
by the Rank-Nullity Theorem
dim im(θs ) + dim ker(θs ) = dim V.
On the other hand, by Problem 1(a) of 1.3.2, we can write
dim(im(θs ) + ker(θs )) = dim im(θs ) + dim ker(θs ) − dim im(θs ) ∩ ker(θs ) ,


which, in view of the preceding equality, implies V = im(θs ) ⊕ ker(θs ) as soon as we


show that im(θs ) ∩ ker(θs ) = {0}. To see this, let x ∈ im(θs ) ∩ ker(θs ) be arbitrary.
It follows that there exists x1 ∈ V such that x = θs x1 , from which, we obtain
θs+s x1 = 0 because x ∈ ker θs . On the other hand, by the claim we made in the
above, we have ker θs+s = ker θs , yielding x1 ∈ ker θs . In other words, x = θs x1 = 0.
Since x ∈ im(θs ) ∩ ker(θs ) was arbitrary, we conclude that im(θs ) ∩ ker(θs ) = {0},
which is what we want. Therefore, V = im(θs ) ⊕ ker(θs ), completing the proof. 
2.8. EIGHTH COMPETITION 91

2.8. Eighth Competition

2.8.1. Analysis. 1. (a) Define the function h : F ∪ G → R by



f (x) x ∈ F,
h(x) =
g(x) x ∈ G.
As f (x) = g(x) on F ∩ G, the function h is well-defined. It is obvious that h is an
extension of f and g to F ∪ G. To prove that h is continuous, it suffices to show
that h−1 (C) is a closed subset of F ∪ G whenever C is a closed subset of R. To this
end, note that for any closed subset C of R, we can write
h−1 (C) = f −1 (C) ∪ g −1 (C).
Now, as f and g are continuous and C is a closed subset of R, we see that f −1 (C)
and g −1 (C) are closed subsets of F and G, respectively. On the other hand, F and
G are closed in F ∪ G. Hence, f −1 (C) and g −1 (C) are closed subsets of F ∪ G. This
together with the above equality implies that h−1 (C) is a closed subset of F ∪ G,
which is what we want.
 
(b) For given
n ∈ N, let F = B1 (0) := x ∈ Rn : ||x|| < 1 and G = F c = x ∈
Rn : ||x|| ≥ 1 . Obviously, F ∩ G = ∅ and F ∪ G = Rn . Now, define the functions
f : F → R and g : G → R by f (x) = 0 and g(x) = 1 whenever x ∈ F and x ∈ G,
respectively. For this f and g, we show that the conclusion of (a) does not hold.
Suppose to the contrary that h : Rn = F ∪ G → R is an extension of f and g to
Rn = F ∪ G. We must have

0 x ∈ F,
h(x) =
1 x ∈ G,
which is not continuous on Rn = F ∪ G, for otherwise h(Rn ) = {0, 1}, which is
disconnected whereas Rn is connected. This leads to a contradiction, proving the
assertion. 

2. Remark. It is worth mentioning that using a proof almost identical to the


proof below one can show that if (an )+∞
n=1 is a decreasing sequence of real numbers
P+∞ P+∞
with limn an = 0, then n=1 an is convergent if and only if n=1 n(an − an+1 ) is
convergent. Also, see Problem 2 of 1.12.1.
P+∞
We have limn an = 0 because n=1 an is convergent. Since (an )+∞n=1 is decreasing,
it follows that an ≥ an+k for all n, k ∈ N. Letting k → +∞, we obtain an ≥ 0 for
P+∞
all n ∈ N. Now, as n=1 an is convergent, it follows that it is Cauchy. Thus, for
given ε > 0, there exists an N > 0 such that
n
X ε
ak < ,
2
k=m

for all n ≥ m ≥ N . Let m = [ n2 ]


> N . We obtain n > m ≥ N . This together with
the fact that an ’s are nonnegative implies
n
X ε
ak < .
2
k=[ n
2]
92 2. SOLUTIONS

Pn
As an ’s are decreasing, we have (n − [ n2 ] + 1)an ≤ k=[ n ak , yielding
2]

nan nan n n 
≤ + − [ ] + 1 an
2 2 2 2
n
n  X
= n − [ ] + 1 an ≤ ak
2 n
k=[ 2 ]
ε
< ,
2
for all n > 2N + 2. Consequently, nan < ε for all n > 2(N + 1). That is,
limn nan = 0. We can write
+∞
X +∞ 
X 

n(an − an+1 ) = nan − (n + 1)an+1 + an+1 .
n=1 n=1
P+∞ 
Since limn nan = 0, we see that the telescopic series n=1 nan − (n + 1)an+1
P+∞
converges to a1 . On the other hand, n=1 an+1 is convergent. Therefore, the
P+∞
series n=1 n(an − an+1 ) converges and we have
+∞
X +∞ 
X 

n(an − an+1 ) = nan − (n + 1)an+1 + an+1
n=1 n=1
+∞
X +∞
 X
= nan − (n + 1)an+1 + an+1
n=1 n=1
+∞
X
= a1 + an+1 ,
n=1

finishing the proof. 

3. First solution: As the function g is continuously differentiable on [0, 1], it


follows that g is of bounded variation on [0, 1]. We show that g can be written as
the difference of two increasing functions g1 and g2 each of which is continuously
differentiable on [0, 1]. To see this, let g 0 = f , where f is continuous on [0, 1]. We
can write f = f + − f − , where f + = max(f, 0) = f2 + |f2 | and f − = max(−f, 0) =
−f |f |
2 + 2 . Obviously, f
+
and f − are nonnegative continuous functions on [0, 1].
Using the Second Fundamental Theorem of Calculus, we can write
Z x Z x Z x
f+ − f − = g1 (x) − g2 (x),

g(x) = g(0) + f = g(0) +
0 0 0
Rx Rx
where g1 (x) = g(0) + 0 f + and g2 (x) = 0
f − . Applying the First Fundamental
Theorem of Calculus, we obtain
g10 (x) = f + (x) ≥ 0, g20 (x) = f − (x) ≥ 0,
for all x ∈ [0, 1]. Therefore, g = g1 − g2 , where g1 and g2 are continuously differen-
tiable and increasing on [0, 1]. We can write
Z 1 Z 1 Z 1 Z 1
n n n
x dg = x d(g1 − g2 ) = x dg1 − xn dg2 .
0 0 0 0
2.8. EIGHTH COMPETITION 93

Now, as g1 and g2 are continuously differentiable on [0, 1], we have


Z 1 Z 1 Z 1 Z 1
xn dg = xn g10 (x)dx − xn g20 (x)dx = xn g10 (x) − g20 (x) dx.

0 0 0 0
Hence,
Z 1 Z 1
n
x dg = xn g 0 (x)dx.
0 0
Now, as g 0 is continuous on [0, 1], there is an M > 0 such that |g 0 | ≤ M on [0, 1],
from which, we see that
Z 1 Z 1 Z 1
n 0
n
xn g 0 (x) dx


x dg =
x g (x)dx ≤
0 0 0
Z 1
M
≤ M xn dx = ,
0 n+1
R1 M
implying that limn 0 xn dg = 0 because limn n+1 = 0.

Second solution: As g is continuously differentiable on [0, 1], it is of bounded


variation on [0, 1]. Hence, there exists increasing functions g1 and g2 on [0, 1] such
that g = g1 − g2 . By the definition of the Riemann-Stieltjes integrals with respect
to integrands of bounded variations, we have
Z 1 Z 1 Z 1
xn dg = xn dg1 − xn dg2 .
0 0 0
Using integration by parts, we obtain
Z 1 1 Z 1 Z 1
n n n
x dgi = x gi (x) − gi (x)dx = gi (1) − ngi (x)xn−1 dx,

0 0 0 0
for each i = 1, 2. So we have
Z 1 Z 1 Z 1
xn dg = g(1) − ng(x)xn−1 dx = n g(1) − g(x) xn−1 dx.

0 0 0
Now, for given ε > 0, as g is continuous at 1, there exists a 0 < δ < 1 such that
|g(x) − g(1)| < 2ε whenever 1 − δ < x < 1. We can write
Z 1 Z 1 Z 1
n
 n−1
n g(1) − g(x) xn−1 dx


x dg =
n g(1) − g(x) x dx ≤

0 0 0
Z 1−δ Z 1
n−1
n g(1) − g(x) xn−1 dx.

= n g(1) − g(x) x dx +
0 1−δ

By the continuity of g on [0, 1], there exists M > 0 such that |g| ≤ M on [0, 1].
With this in mind, we have
Z 1 Z 1−δ
nε 1 n−1
Z
n n−1

x dg ≤ 2M n x dx + x dx

0

0 2 1−δ
ε ε
= 2M (1 − δ)n + 1 − (1 − δ)n < 2M (1 − δ)n + .

2 2
Consequently,
Z 1
ε
x dg < 2M (1 − δ)n + .
n



0 2
94 2. SOLUTIONS

Now, limn (1 − δ)n = 0 because 0 < 1 − δ < 1. Hence, there exists N > 0 such that
ε
(1 − δ)n < 4M for all n ≥ N . Therefore,
Z 1
n < 2M × ε + ε = ε,

x dg

0
4M 2
R1 n
for all n ≥ N . This means limn 0 x dg(x) = 0, which is what we want. 

2.8.2. Algebra. 1. Note that b = I + N , where I is the identity matrix,


N = b − I, and N 2 = 0. From this, using the Binomial Theorem and noting that
b−1 = I − N , it is easily verified that
 
1 n
bn = ,
0 1
for all n ∈ Z. Therefore,
  
1 n
H = {bn : n ∈ Z} = :n∈Z .
0 1
1
 
−1 −1 2 0
To find aHa , note that a = . We can write
0 1
1
     
2 0 1 n 0 1 2n
abn a−1 = 2 = .
0 1 0 1 0 1 0 1
Therefore,   
1 2n
aHa−1 = :n∈Z .
0 1
This implies that aHa−1  H. That is, aHa−1 is a proper subgroup of H, which
is what we want. 

2. We need the “only if part” of the following standard lemma.

T  Lemma. Let R be a commutative


ring with identity. Then, x ∈ J(R) :=
m C R : m is maximal in R if and only if 1 − xy is a unit in R for all y ∈ R.
Proof. “⇐=” Let x ∈ J(R). Suppose to the contrary that there exists a
y0 ∈ R such that 1 − xy0 is not a unit. As 1 − xy0 is not invertible, a standard
argument using Zorn’s Lemma shows that there exists a maximal ideal m0 such
that 1 − xy0 ∈ m0 . On the other hand, since x ∈ J(R), x ∈ m0 , and hence
xy0 ∈ m0 . So, we obtain 1 = (1 − xy0 ) + xy0 ∈ m0 , yielding m0 = R, which is a
contradiction. Thus the assertion follows by contradiction.
“=⇒” Again, we proceed by contradiction. Suppose that there exists a maximal
ideal m0 such that x ∈/ m0 . As m0 $ m0 + xR and m0 is maximal, we see that
m0 + xR = R. Hence, there exist a ∈ m0 and y0 ∈ R such that 1 = a + xy0 .
This together with the hypothesis implies that a = 1 − xy0 ∈ m0 is a unit in R,
which, in turn, implies m0 = R, a contradiction. Therefore, the assertion follows
by contradiction. 

To prove the assertion by contradiction, suppose that R has finitely many


maximal ideals m1 , . . . , mn for some n ∈ N. Firstly, {0} $ mi for each i = 1, . . . , n
because otherwise {0} and R are the only ideal of R, implying that R is a field,
2.8. EIGHTH COMPETITION 95

which is a contradiction, for R would then have infinitely many units. On the other
hand, we have
\n
mi ⊇ m1 · · · mn .
i=1
We also have m1 · · · mn 6= {0} because {0} $ mi for each Tni = 1, . . .T
, n and R is
an integral domain. Now, pick 0 6= a ∈ m1 · · · mn ⊆ i=1 mi = {m C R :
m is maximal in R}. It follows from the lemma that 1 − ax is a unit in R for x ∈ R.
But, as R is an integral domain, 1 − ax 6= 1 − ay whenever x 6= y. This together
with the fact that R is an infinite integral domain implies that R has infinitely
many units, which is a contradiction. So the assertion follows by contradiction. 

3. Let A = (aij ) ∈ Mn (R). Use the symbol (A2 )ij to denote the ij entry of the
matrix A2 . Using A2 = I, we can write
Xn X n n X
X n
(A2 )ij = (I)ij = n.
i=1 j=1 i=1 j=1

On the other hand,


Xn X n X n X
n X n n X
X n n
X
(A2 )ij = aik akj = aik akj
i=1 j=1 i=1 j=1 k=1 i=1 k=1 j=1
Xn X n n X
X n Xn
= aik a = a aik = a a = na2 .
i=1 k=1 i=1 k=1 i=1
2
Therefore, na = n, yielding a = ±1. Note that both cases a = 1 or a = −1 can
happen because A = ±I yields a = ±1, respectively. 

2.8.3. General. 1. Note that f (x, y, z) = (x − 3)2 + 4(y + 12 ) + z 2 − 10. This


yields f (x, y, z) ≥ −10 for all x, y, z ∈ R and that f (x, y, z) = −10 if and only if
x = 3 , y = − 12 , and z = 0. Therefore, the function f assumes it absolute minimum
at (3, − 12 , 0), which is what we want. 

2. Let x1 < · · · < x5 be the amount of money


owned by these five people in
increasing order. We have x1 x2 , x2 x3 , x3 x4 , and x4 x5 . Hence, there exist
natural numbers ki (1 ≤ i ≤ 4) such that xi+1 = ki xi . Using the hypothesis, we
can write
x1 (1 + k1 + k1 k2 + k1 k2 k3 + k1 k2 k3 k4 ) = 719.
But 719 is prime. So, we obtain
x1 = 1, 1 + k1 + k1 k2 + k1 k2 k3 + k1 k2 k3 k4 = 719.
Consequently, k1 (1 + k2 + k2 k3 + k2 k3 k4 ) = 718 = 2 × 359. As 2 and 359 are primes,
we must have
k1 = 2, 1 + k2 + k2 k3 + k2 k3 k4 = 359, or,
k1 = 1, 1 + k2 + k2 k3 + k2 k3 k4 = 718, or,
k1 = 359, 1 + k2 + k2 k3 + k2 k3 k4 = 2, or,
k1 = 718, 1 + k2 + k2 k3 + k2 k3 k4 = 1.
The last three cases are easily seen to be refuted. Thus,
k1 = 2, 1 + k2 + k2 k3 + k2 k3 k4 = 359.
96 2. SOLUTIONS

Simplifying, we obtain k2 (1 + k3 + k3 k4 ) = 358 = 2 × 179. Since 2 and 179 are


primes, we see that
k2 = 2, 1 + k3 + k3 k4 = 179, or,
k2 = 1, 1 + k3 + k3 k4 = 358, or,
k2 = 179, 1 + k3 + k3 k4 = 2, or,
k2 = 358, 1 + k3 + k3 k4 = 1.
Once again, refuting the last three cases, we obtain k2 = 2, 1 + k3 + k3 k4 = 179,
which yields k3 (1 + k4 ) = 178 = 2 × 89. Likewise, noting that 2 and 89 are primes,
we get
k3 = 2, 1 + k4 = 89, or,
k3 = 1, 1 + k4 = 178, or,
k3 = 89, 1 + k4 = 2, or,
k3 = 178, 1 + k4 = 1.
Again, refuting the last three cases, we obtain k3 = 2, 1 + k4 = 89, which yields
k4 = 88. Therefore, the only solution of x1 + · · · + x5 = 719 subject to the imposed
conditions is x1 = 1, x2 = k1 x1 = 2, x3 = k2 x2 = 4, x4 = k3 x3 = 8, and
x5 = k4 x4 = 704. That is, these five people in increasing order have
x1 = 1, x2 = 2, x3 = 4, x4 = 8, x5 = 704,
amounts of money, which is what we want. 

3. Without loss of generality, we may assume that starting with every minute, the
traffic-light of the crossroad B is 30 seconds red and 30 seconds green, respectively.
If t denotes the tth second of a particular minute at the crossroad B, then the stop
time function of the bus at the crossroad, denoted by f , on that particular minute,
is 
30 − t 0 ≤ t ≤ 30,
f (t) =
0 30 ≤ t ≤ 60.
Let µf denote the average value of the function f , which means the average of the
stop time at the crossroad B. By definition, we have
Z 60 Z 30
1 1
µf = f (t)dt = (30 − t)dt = 7.5.
60 0 60 0
But the bus passes through the crossroad B of the city with the probability of
1
3 . Therefore, the average of the stop time of the bus at the crossroad is equal to
1 7.5
3 µf = 3 = 2.5, which is what we want. 

2.9. Ninth Competition

2.9.1. Analysis. 1. Suppose that C is an equivalence class of the equivalence


relation ∼. Pick an x0 ∈ C. It is plain that

C = x0 + r|r ∈ Q .
It suffices to show that C ∩ (a, b) 6= ∅ for all open intervals (a, b) of R. To this
end, for a given open interval (a, b), as Q is dense in R, we see that there exists an
r ∈ Q ∩ (a − x0 , b − x0 ). This yields x0 + r ∈ C ∩ (a, b), implying C ∩ (a, b) 6= ∅,
which is what we want. 
2.9. NINTH COMPETITION 97

2. First, we claim that the function f is strictly increasing or strictly decreas-


ing. To prove this by contradiction, suppose that f is neither strictly increasing
nor strictly decreasing. By the lemma presented in Solution 2 of 2.5.1, there ex-
ist x1 , x2 , x3 ∈ R with x1 < x2 < x3 such that f (x2 ) ≥ max f (x1 ), f (x3 ) or
f (x2 ) ≤ min f (x1 ), f (x3 ) . As f is one-to-one and x1 < x2 < x3 , we see that
f (x2 ) > max f (x1 ), f (x3 ) or f (x2 ) < min f (x1 ), f (x3 ) . First, suppose f (x2 ) >
 f (x2 )+max f (x1 ),f (x3 )
max f (x1 ), f (x3 ) . Letting λ = 2 , we see that f (x1 ) < λ <
f (x2 ) and f (x3 ) < λ < f (x2 ). Hence, by the Intermediate Value Theorem,
there exist x1 < ξ1 < x2 and x2 < ξ2 < x3 such that f (ξ1 ) = λ = f (ξ2 ).
As f is one-to-one, this yields ξ1 = ξ2 , which is a contradiction.  Likewise, if
 f (x2 )+min f (x1 ),f (x3 )
f (x2 ) < min f (x1 ), f (x3 ) , considering λ = 2 , we obtain a
contradiction, proving the claim. Thus, the function f is either strictly increasing
or strictly decreasing. We prove the assertion in the case when f is strictly increas-
ing. The case when f is strictly decreasing can be done similarly or by replacing f by
−f and repeating the above argument. To prove that f −1 is continuous, it suffices
to show that f is an open map. To this end, note first that f (−∞) = limx→−∞ f (x)
and f (+∞) = limx→+∞ f (x) exist, where f (±∞) ∈ R ∪ {±∞},and that, in view
of the Intermediate Value Theorem, f (R) = f (−∞), f (+∞)  . Analogously, if
(a, b) is an open interval in R, we obtain f (a, b) = f (a), f (b) . Now, suppose that
G is an open subset of R. It follows
S that there exists a family {(ai , bi )}i∈I of open
intervals of R such that G = i∈I (ai , bi ). We can write
[  [ [ 
f (G) = f (ai , bi ) = f (ai , bi ) = f (ai ), f (bi ) .
i∈I i∈I i∈I
 
But f (ai ), f (bi ) is an open interval, hence an open subset, of f (R) = f (−∞), f (+∞) .
This, in view of the above equality, implies that f (G) is an open subset of f (R).
Therefore, f is an open map, which is what we want. 

3. To show f (E) = E, assume first that x ∈ E is arbitrary. We need to show


that x ∈ f (E). To see this, note that f (x) ≥ x. If f (x) = x, there is nothing
to prove. If f (x) > x, we have f (a) = a ≤ x < f (x). It thus follows from the
Intermediate Value Theorem that there exists a c ∈ [a, x) such that f (c) = x,
implying x ∈ f [a, b]. By showing c ∈ E, we conclude that x ∈ f (E), which is
what we want. If c ∈/ E, then f (c) < c, from which, as f is increasing, we see that
f f (c) < f (c). But f (c) = x, which obtains f (x) < x. This is a contradiction,
yielding x ∈ f (E), implying E ⊆ f (E).
Next, suppose that y = f (x) ∈ f (E), where x ∈ E, is arbitrary.
 As x ∈ E, we
have f (x) ≥ x, from which, as f is increasing, we obtain f f (x) ≥ f (x). That is,
y ∈ [a, b] and f (y) ≥ y, which means y = f (x) ∈ E. In other words, f (E) ⊆ E,
proving the assertion. 

4. Remark. The function f , as defined, in not well-defined at 0. Since the value


of f at any point has no affect on its integrability, we redefine f : [0, 1] → R by

x = pq , p, q ∈ N, gcd(a, b) = 1,
 1
f (x) = q
0 otherwise.
98 2. SOLUTIONS

First solution: Just as explained in Solution 1 of 2.5.1, the sequence (fn )+∞
n=1 ,
with fn : [0, 1] → [0, 1], defined by
x = pq , p, q ∈ N, gcd(p, q) = 1, q ≤ n,
 1
fn (x) = q
0 otherwise,
converges uniformly to f on [0, 1]. As fn (n ∈ N) is zero everywhere but at finitely
many points of the interval [0, 1], it follows that fn ’s are integrable on [0, 1]. Hence,
f is Riemann integrable on [0, 1], and moreover
Z 1 Z 1
f (x)dx = lim fn (x)dx = 0.
0 n 0

Second solution: As shown in Solution 3 of 2.1.1, we have limx→a f (x) = 0 for


all a ∈ (0, 1), limx→0+ f (x) = 0, and limx→1− f (x) = 0. Thus, the function f is
continuous on the set [0, 1] \ Q ∩ (0, 1] and it is discontinuous on Q ∩ (0, 1], which
is a countable set. From this point on, we present two proofs for the integrability
of f on [0, 1].
First proof. Just note that the set of points at which f is discontinuous is
countable and hence has measure zero. Therefore, using the Lebesgue’s Integrability
Criterion for Riemann integrals, we see that f is Riemann integrable, which is what
we want.

Second proof. Using Lebesgue’s Number Lemma, we directly prove that f is


Riemann integrable on [0, 1]. Recall that Lebesgue’s Number Lemma asserts that
if an open cover of a compact metric space X is given, then there exists a number
δ > 0, called a Lebesgue number for the cover, such that every subset of X of
diameter less than δ is contained in some element of the cover. We use Riemann’s
criterion for integrability to show that f is integrable. We need to show that for
given ε > 0, there exists a partition P ∈ P[0, 1], the set of all partitions of [0, 1],
such that
n
X
U (P, f ) − L(P, f ) = (Mi − mi )∆xi < ε,
i=1
where P : 0 = x0 < · · · < xn = 1, Mi = supxi−1 ≤x≤xi f (x), mi = inf xi−1 ≤x≤xi f (x),
and ∆xi = xi − xi−1 . To this end, for given ε > 0, choose N ∈ N such that
1 ε +∞ 1 1

2N
< 2 and let Q ∩ (0, 1] = {r }
i i=1 . Set N i = r i − 2N +i
, r i + 2N +i
∩ [0, 1].
P+∞ 1 ε
Obviously, i=1 l(Ni ) ≤ 2N < 2 , where l(Ni ) denotes the length of the interval
S+∞
Ni . On the other hand, Q ∩ (0, 1] = {ri }+∞ i=1 ⊆ i=1 Ni . Therefore, for any
S+∞
x ∈ C := [0, 1] \ i=1 Ni , the function f is continuous at x. Consequently, for given
ε > 0, there exists an open interval Nx containing x such that
ε
MNx (f ) − mNx (f ) < ,
2
where MNx (f ) = supt∈Nx f (t) and mNx (f ) = inf t∈Nx f (t). It is obvious that
{Ni }+∞
i=1 ∪ {Nx }x∈C is an open cover for the compact interval [0, 1]. Let δ > 0 be a
Lebesgue number for this open cover. Choose P ∈ P[0, 1] such that the length of
its subintervals are all less than δ. That is, if
P : 0 = x0 < · · · < xn = 1,
2.9. NINTH COMPETITION 99

then ∆xi = xi − xi−1 < δ for all 1 ≤ i ≤ n. Therefore, for all 1 ≤ i ≤ n, we have
[xi−1 , xi ] ⊆ Nj for some j ∈ N or [xi−1 , xi ] ⊆ Nx for some x ∈ C. Letting

A = i ∈ N : 1 ≤ i ≤ n, ∃j ∈ N  [xi−1 , xi ] ⊆ Nj ,

B = i ∈ N : 1 ≤ i ≤ n, ∃x ∈ C  [xi−1 , xi ] ⊆ Nx ,
we can write
n
X
U (P, f ) − L(P, f ) = (Mi − mi )∆xi
i=1
X X
≤ (Mi − mi )∆xi + (Mi − mi )∆xi
i∈A i∈B
Xε +∞ n
X X εX
≤ 1 × ∆xi + ∆xi ≤ l(Ni ) + ∆xi
2 i=1
2 i=1
i∈A i∈B
1 ε
≤ N
+ < ε.
2 2
This means, U (P, f ) − L(P, f ) < ε, which is what we want.
To find the value of the integral, it is obvious that
n
X
L(P, f ) = mi ∆xi = 0,
i=1

for all P ∈ P[0, 1]. Since f is integrable, we have


Z 1
f (x)dx = sup L(P, f ) = 0. 
0 P ∈P[a,b]

5. First, we show that no function f : [0, 1] → [0, 1] × [0, 1] can have the three
properties, namely, continuity, injectivity, and surjectivity. Suppose to the contrary
that f is a continuous and one-to-one function from [0, 1] onto [0, 1] × [0, 1]. As
f is invertible and [0, 1] is compact, it follows that f is a closed map, and hence
f −1 : [0, 1] × [0, 1] → [0, 1] is a continuous function. Let f ( 12 ) = (a, b) for some
a, b ∈ [a, b]. Obviously, f −1 (a, b) = 12 . Letting C = [0, 1] × [0, 1] \ {(a, b)}, we have
1
f −1 (C) = [0, 1] \ { }.
2
Since C is connected and f −1 is continuous, it follows that [0, 1] \ { 21 } is connected,
which is a contradiction. Therefore, the assertion follows by contradiction.
We now prove that f can have any two properties of the aforementioned prop-
erties. There are three cases to consider.

(i) f : [0, 1] → [0, 1] × [0, 1] can be continuous and one-to-one.

The function f : [0, 1] → [0, 1] × [0, 1] defined by f (t) = (t, 0) is continuous and
one-to-one.

(ii) f : [0, 1] → [0, 1] × [0, 1] can be continuous and onto.


100 2. SOLUTIONS

Such a function exists. We present the well-known example due to Schoenberg


(1938). First, define the function φ on [0, 2] by
0 ≤ t ≤ 31 or 53 ≤ t ≤ 2,


 0
1 2
3t − 1 3 ≤ t ≤ 3,

φ(t) = 2 4

 1 3 ≤ t ≤ 3,
4 5
−3t + 5 3 ≤ t ≤ 3.

Then, extend φ to R via φ(t + 2) = φ(t). It is plain that φ : R → R is 2-periodic


and that 0 ≤ φ(t) ≤ 1 for all t ∈ R. Now, define f1 , f2 : R → R by
+∞ +∞
X φ(32n−2 t) X φ(32n−1 t)
f1 (t) = n
, f2 (t) = .
n=1
2 n=1
2n
As 0 ≤ φ ≤ 1 on R, we see that the series defining f1 and f2 converge uniformly
and absolutely on R, and hence f1 and f2 are continuous on R and moreover, 0 ≤
f1 , f2 ≤ 1 on R because 0 ≤ φ ≤ 1 on R. Thus, the function f : [0, 1] → [0, 1] × [0, 1]
defined by f (t) = (f1 (t), f2 (t)) is well-defined and continuous. To prove that f
is onto, let (a, b) ∈ [0, 1] × [0, 1] be arbitrary. We need to show that there exists
a c ∈ [0, 1] such that f (c) = (a, b). To this end, write the binary expansions of
a, b ∈ [0, 1] as follows
+∞ +∞
X an X bn
a = n
, b = ,
n=1
2 n=1
2n
where an , bn = 0 or 1 for all n ∈ N. Let
+∞
X cn
c = 2 n
,
n=1
3
where 
a n+1 n odd,
cn = 2
b n2 n even.
P+∞ 1
Thus, cn = 0 or 1 for all n ∈ N. Consequently, as 2 n=1 3n = 1, we obtain
0 ≤ c ≤ 1. We claim that
φ(3k c) = ck+1 , (∗)
for all k ∈ Z with k ≥ 0. Suppose that (∗) holds. We would then have
φ(32n−2 c) = c2n−1 = an , φ(32n−1 c) = c2n = bn ,
for all n ∈ N. This easily yields f (c) = (f1 (c), f2 (c)) = (a, b), which is what we
want. Now, to prove (∗), suppose k ∈ Z with k ≥ 0. We can write
k +∞
X cn X cn
3k c = 2 n−k
+ 2 n−k
= 2Nk + dk ,
n=1
3 3
n=k+1
Pk k−n
P+∞ cn
where Nk = n=1 3 cn ∈ N and dk = 2 n=k+1 3n−k . As φ is periodic and its
period is 2, we have
φ(3k c) = φ(dk ).
If ck+1 = 0, then
+∞
X 1 1
0 ≤ dk ≤ 2 n
= ,
n=2
3 3
2.9. NINTH COMPETITION 101

2
in which case, φ(dk ) = 0, whence φ(3k c) = ck+1 = 0. If ck+1 = 1, then 3 ≤ dk ≤ 1,
in which case, φ(dk ) = 1, whence φ(3k c) = ck+1 = 1. That is,
φ(3k c) = ck+1 ,
for all k ∈ Z with k ≥ 0, proving the claim and hence finishing the proof.

(iii) f : [0, 1] → [0, 1] × [0, 1] can be one-to-one and onto.

Such a function exists. First, define f : [0, 1] → [0, 1] × [0, 1] by


f (t) = (0.t1 t3 t5 . . . , 0.t2 t4 t6 . . .),
where 0.t1 t2 t3 . . ., with ti = 0 or 1 for all i ∈ N, denotes the binary expansion of
the number t. We set the convention that in the binary expansion of a number
t the number 1 is not allowed to be repeated for all the digits after any digit
except when t = 1, in which case its binary expansion, by definition, is set to be
0.111 . . .. It is obvious that f is onto. Next, as mentioned in (i), the function
f : [0, 1] → [0, 1] × [0, 1] defined by f (t) = (t, 0) is (continuous and) one-to-one.
It thus follows from the Schroder-Bernstein Theorem that there is a function f :
[0, 1] → [0, 1] × [0, 1], which is one-to-one and onto, as desired. 

2.9.2. Algebra. 1. First solution: Proceed by contradiction. Suppose that


there exists a nonnormal subgroup H of index p in G so that |G| = p|H|, where p
is the smallest prime dividing the order of G. It follows that there is a g ∈ G such
that
K = H g := g −1 Hg = {g −1 hg : h ∈ H} 6= H.
It is obvious that |K| = |H|. Also, we have |H ∩ K| < |H| = |K|, for otherwise
|H ∩ K| = |H| = |K|, yielding H ∩ K = H = K, which is a contradiction. Now, we
|K| |K|
claim that |H∩K| ≥ p. To see this, let q be a prime dividing |H∩K| . It follows that
|K|
q divides |K|, and hence |G|. This obtains p ≤ q ≤ |H∩K| because p is the smallest
prime dividing |G|. On the other hand,
|K|
|G| ≥ |HK| = |H| ≥ |H|p = |G|,
|H ∩ K|
implying that HK = G. Now, let g = hk for some h ∈ H, k ∈ K. We can write
H g = H hk = (H h )k = H k .
Thus,
G = Gk = (HK)k = H k K k = H g K = KK = K,
which is a contradiction. So, the assertion follows by way of contradiction.

Second solution: Let H be a subgroup of G of index p in G, where p is the


/ H, then xi ∈
smallest prime dividing the order of G. First, we claim that if x ∈ /H
for all i ∈ N with 1 ≤ i ≤ p − 1. By way of contradiction, suppose that there exists
the smallest j ∈ N with 1 < j ≤ p − 1 such that xj ∈ H. It follows that xi ∈ / H
for all 1 ≤ i < j. Set n = |G| and m = ord(x). We have m|n. As j < p and p is
the smallest prime that divides n, it follows that m is not divisible by j. Hence,
102 2. SOLUTIONS

there are numbers r, q ∈ N with 0 < r < j such that m = jq + r. Noting that
m = ord(x), we can write
e = xm = xjq+r = (xj )q xr ,
yielding (xj )q xr ∈ H. But xj ∈ H, from which, we obtain xr ∈ H, a contradiction
because 0 < r < j. Consequently, xi ∈ / H for all 1 ≤ i ≤ p − 1, as we claimed.
Now, to prove the assertion, we proceed by contradiction again. Suppose that
there exist g ∈ G and h ∈ H such that ghg −1 ∈ / H. This yields g ∈ / H, which,
in turn together with the above claim, implies that g i ∈ / H for all i ∈ N with
1 ≤ i ≤ p − 1. On the other hand, g i H 6= g j H whenever 1 ≤ i < j ≤ p − 1 because
otherwise g j = g i h0 for h0 ∈ H, implying g j−i = h0 ∈ H, which is a contradiction,
for 1 ≤ j − i ≤ p − 1. Therefore, H, gH, . . . , g p−1 H are p distinct left cosets of H.
Consequently,
G
= H, gH, . . . , g p−1 H .

H
Let g1 = ghg −1 . Likewise, as g1 ∈ / H, we conclude that H, g1 H, . . . , g1p−1 H are p
distinct left cosets of H, implying
G
= H, g1 H, . . . , g1p−1 H .

H
Therefore, gH = g1r H for some 1 ≤ r ≤ p − 1, and hence g = g1r h1 for some h1 ∈ H.
On the other hand, g1r = ghr g −1 , from which, we obtain g = ghr g −1 h1 . This yields
g = h1 hr , implying g ∈ H, which is a contradiction. Therefore, H is a normal
subgroup of G, finishing the proof.

Third solution: Let H be a subgroup of G of index p in G, where p is the smallest


prime dividing the order of G. Use L to denote the set of all left cosets of H in
G, which has p elements by the hypothesis. For each g ∈ G define the mapping
τg : L −→ L by τg (xH) = gxH. In other words, G acts on L by multiplication from
the left. It is readily verified that τg ’s (g ∈ G) give rise to a group homomorphism
φ : G −→ S(L) ∼ = Sp which is defined by φ(g) = τg , where S(L) denotes the group
of all permutations of L which is isomorphic to Sp because |L| = p (here, as is
usual, Sp denotes the symmetric group of degree p, whose order is p!). Obviously,
K := ker φ ⊆ H. We prove the assertion by showing that H = K. By the First
G
Isomorphism
Theorem for groups, K is isomorphic to a subgroup of Sp implying
G G
that | K | p!. On the other hand, every divisor of | K | divides |G|. Also, of the prime
divisors of p! only p divides |G| because p is the smallest prime dividing the order
G G G G
of G. Thus, | K | = p or | K | = 1. The latter is impossible because | K | ≥ |H | = p.
G G
Therefore, | K | = p = | H |, implying that |H| = |K|. This obtains H = K because
K ⊆ H, completing the proof.

Fourth solution: By a standard result from Galois theory, e.g., see Proposition
V.2.16 of “Algebra” by T.W. Hungerford, there are fields E, K such that K is an
extension of E and that Gal(K/E) ∼ = G. In view of this, and two other standard re-
sults from Galois theory, e.g., see Theorem V.2.5 and Corollary V.3.15 of “Algebra”
by T.W. Hungerford, it suffices to prove the following.

Under the hypothesis of the problem, let K be an extension field of E such that
Gal(K/E) ∼ = G. If F ⊆ K is an extension field of E such that [F : E] = p, then F
is normal over E.
2.9. NINTH COMPETITION 103

To prove this, note first that every element of K is separable over E, i.e., the
minimal polynomial of every element of K over E splits into distinct linear factors,
for K is Galois over E. Therefore, F is a finite separable extension of E and hence,
by the Primitive Element Theorem (see Proposition V.6.15 of “Algebra” by T.W.
Hungerford, there is an α ∈ F such that F = E(α). Let g ∈ E[x] be an arbitrary
irreducible polynomial that has a root, say h(α) for some h ∈ E[x], in F = E(α).
It follows that the splitting field of g is contained in K, for h(α) ∈ F ⊆ K and K
is Galois, and hence normal, over E. Note that deg(g) = [E h(α) : E] ≤ [E(α) :
E] = p. We need to show that g splits into linear factors in E(α). Let β 6= h(α) be a
root of g in the splitting field of g, which, as we just saw, is contained in K. It suffices
to prove that β ∈ F = E(α).  To this end, we note that [F (β) : F ] < deg(g) ≤ p
because g(β) = 0, g h(α) = 0, and h(α) ∈ F . This yields [F (β) : F ] = 1 because p
is the smallest prime dividing |G| and [F (β) : F ] |G|, for β ∈ K and [K : E] = |G|
since Gal(K/E) ∼ = G. Consequently, β ∈ F , and hence F is normal over E, as
desired. 

Z3 [x] Z3 [x]
2. We determine all isomorphisms from the field onto the field 2 .
(x2 + 1) (x + x + 2)
[x] 3 [x]
Since f : hxZ23+1i → hx2Z+x+2i is an isomorphism of fields, we see that f (a+hx2 +1i) =
a + hx2 + x + 2i for all a ∈ Z3 . Thus, to determine f , we only need to determine
f (x + hx2 + 1i). To this end, suppose that f (x + hx2 + 1i) = ax + b + hx2 + x + 2i
for some a, b ∈ Z3 . We must have
  2
f x + hx2 + 1i = f x2 + hx2 + 1i = f − 1 + hx2 + 1i ,
 

implying that (ax + b)2 + 1 ∈ hx2 + x + 2i. But the polynomial (ax + b)2 + 1 =
a2 x2 + 2abx + b2 + 1 is divisible by x2 + x + 2 in Z3 [x] if and only if so is the
polynomial (2ab − a2 )x + (a2 + b2 + 1), which is obtained from a2 x2 + 2abx + b2 + 1
by letting x2 = −x − 2 = −x + 1, if and only if a and b satisfy the equations
a(2b − a) = 0 and a2 + b2 + 1 = 0. It is readily seen that a = 2, b = 1 and
a = 1, b = 2 are the only solutions of the above equations in Z3 . Therefore, an
[x] 3 [x]
isomorphism f : hxZ23+1i → hx2Z+x+2i must be given by

f ax + b + hx2 + 1i = a(2x + 1) + b + hx2 + x + 2i,




or by
f ax + b + hx2 + 1i = a(x + 2) + b + hx2 + x + 2i.


That is, either


f ax + b + hx2 + 1i = 2ax + (a + b) + hx2 + x + 2i,


or
f ax + b + hx2 + 1i = ax + (2a + b) + hx2 + x + 2i,


which are both easily seen to be monomorphisms of rings. It follows that such an f
Z3 [x] Z3 [x]
is an isomorphism from the field 2 onto the field 2 because they
(x + 1) (x + x + 2)
both have 9 elements. 

3. Note that the ring Z7 [x] is a Euclidean ring because Z7 is a field. With that in
mind, we can use the Euclidean algorithm to find the greatest common divisor of
104 2. SOLUTIONS

the two polynomials 4x4 − 2x2 + 1 and −3x3 + 4x2 + x + 1 in Z7 [x]. We have
4x4 − 2x2 + 1 = (−3x3 + 4x2 + x + 1)(x − 1) + x2 + 2,
3 2
−3x + 4x + x + 1 = (x2 + 2)(−3x + 4).
Therefore,
gcd(4x4 − 2x2 + 1, −3x3 + 4x2 + x + 1) = x2 + 2,
which is what we want. 

4. Let {1, y1 }, where y1 ∈ K, be a basis for the vector space K over F . If y12 ∈ F ,
there is nothing to prove. Suppose y12 ∈ / F . As y12 ∈ K, it follows that there exist
2
scalars a, b ∈ F with b 6= 0 such that y1 = a + by1 . We have
y12 − by1 = b.
On the other hand, since ch(F ) 6= 2, we have 2−1 = 1
2 ∈ F , and hence we can write
b b
y12 − by1 = (y1 − )2 − ( )2 ,
2 2
whence
b b
(y1 − )2 = a + ( )2 ∈ F.
2 2
Let y = y1 − 2b . It is easily verified that {1, y} is a basis for K over F and
y 2 = a + ( 2b )2 ∈ F , which is what we want. 

5. Let S = T − I. Obviously, W = ker S. On the other hand, by the Rank-Nullity


Theorem, we have
dim ker S + dim SV = dim V = n.
We claim that SV ⊆ ker S. To see this, let y = Sx ∈ SV , where x ∈ V , be
arbitrary. We have
Sy = S 2 x = (T − I)2 x = (T 2 − 2T + I)x = (2I − 2T )x = 0,
because T 2 = I and ch(F ) = 2. This means, y ∈ ker S, proving the claim. Now,
since SV ⊆ ker S, we can write
n = dim ker S + dim SV ≤ 2 dim ker S,
n
yielding dim W = dim ker S ≥ 2, which is what we want. 

2.9.3. General. 1. Let


 
r λ λ ··· λ λ
 λ r λ ··· λ λ 
.. .. ..
 
 .. .. .. 
 . . . . . . 
A =  .. .. ..
.
 .. .. .. 

 . . . . . . 

 λ λ ··· λ r λ 
λ λ ··· ··· λ r
2.9. NINTH COMPETITION 105

Subtracting the first row from all the other rows and then adding the jth column
(2 ≤ j ≤ n) to the first column, we obtain an upper triangular matrix whose
determinant is the product of the entries on its main diagonal. Thus, we can write
 
r λ λ ··· λ λ
 λ−r r−λ 0 ··· 0 0 
 
 λ−r 0 r−λ 0 ··· 0 
det A = det 
 
.. .. .. .. .. .. 

 . . . . . . 

 λ−r 0 ··· 0 r−λ 0 
λ−r 0 ··· ··· 0 r−λ
 
r + (n − 1)λ λ λ ··· λ λ

 0 r−λ 0 ··· 0 0 
 0 0 r−λ 0 ··· 0 
= det 
 
.. .. .. .. .. .. 

 . . . . . . 

 0 0 ··· 0 r−λ 0 
0 0 ··· ··· 0 r−λ
 n−1
= r + (n − 1)λ r − λ .
 n−1
That is, det A = r + (n − 1)λ r − λ , which is what we want. 

2. Let x denote the number of chickens, a the amount of food a chicken consumes
per day, and t the number of the days before the farmers run out of chicken food.
The amount of food in the chicken farm is equal to
(t + 20)(x − 75)a = (t − 15)(x + 100)a = txa.
Simplifying, we obtain
 
(t + 20)(x − 75) = (t − 15)(x + 100) x = 5t.
=⇒
(t − 15)(x + 100) = tx 20t − 3x − 300 = 0.
Substituting x = 5t into the second equation yields x = 300, which implies
t = 60. Therefore, there are x = 300 chickens in the farm. 

3. To prove the assertion by contradiction, suppose that such a function f exists.


We can write
Z 1 Z 1 Z 1 Z 1
2 2 2
(x − α) f (x)dx = x f (x)dx − 2α xf (x)dx + α f (x)dx
0 0 0 0
= α2 − 2α2 + α2 = 0.
R1
That is, 0 (x − α)2 f (x) = 0. As (x − α)2 f (x) is continuous and nonnegative on
[0, 1], it follows that (x − α)2 f (x) = 0 for all x ∈ [0, 1]. This implies f = 0 on [0, 1]
except possibly at x = α. Now, from the continuity of f , we see that f = 0 on
[0, 1], a contradiction. Therefore, no such function f exists, finishing the proof. 

4. First solution: Consider the points with integer coordinates in the closed
interval [0, n] of the real line. To any solution (x1 , . . . , xm ) of the equation
x1 + x2 + · · · + xm = n
in N, there corresponds m − 1 points p1 < · · · < pm−1 from the set {i}n−1
i=1 as follows
p1 = x1 , p2 = x1 + x2 , . . . , pm−1 = x1 + · · · + xm−1 .
106 2. SOLUTIONS

Conversely, for any m − 1 points p1 < · · · < pm−1 from the set {i}n−1
i=1 , a solution
(x1 , . . . , xm ) of the equation
x1 + x2 + · · · + xm = n
is determined as follows
x1 = p1 , x2 = p2 − p1 , . . . , xm−1 = pm−1 − pm−2 , xm = n − pm−1 .
Therefore, the number of the solutions of the equation x1 + x2 + · · · + xm = n in N
is equal to the number of ways of choosing m − 1 points {pi }m−1
i=1 from n − 1 points
n−1
{i}n−1
i=1 , which, as is well-known, is equal to m−1 .

Second solution: We only briefly elaborate on this solution. It is not difficult to


see that the number of the solutions of the equation
x1 + x2 + · · · + xm = n
in N is equal to the coefficient of xn in the expansion of
(x + x2 + · · · + xn−m )m
or in that of
+∞  
x m X m+n−1 n
(x + x2 + x3 + · · · )m = ( ) = xm x ,
1−x n=0
m−1
n−1

which is easily seen to be m−1 . 

5. Yes, just cut the paper along the heavy lines.

6. Since the time needed to get to work from home obeys the uniform distribution,
it follows that the density probability function is equal to

 0 x < 10
1
f (x) = 10 ≤ x ≤ 20 .
 10
0 x > 20

(a) As the worker needs 15 minutes to get to work on time, the desired proba-
bility is
Z 20
dx 1
P (x > 15) = = .
15 10 2
2.9. NINTH COMPETITION 107

(b) First solution. Noting that it takes the worker at least 10 minutes to get
to work, let x0 > 10 denote the time needed for the worker to get to work on time
and with the probability of 75% to have time to eat breakfast. We must have

Z min(x0 ,20)
dx
P (10 < x < x0 ) = = 0.75 ⇐⇒ min(x0 , 20) = 17.5
10 10
⇐⇒ x0 = 17.5.

Therefore, the latest time that this worker can leave home to get to work on time and
with the probability of 75% to have time to eat breakfast is equal to 7, 450 − 17.50 =
7, 27.50 = 7, 270 , 3000 .
Second solution. Suppose the worker heads off for work m minutes before 8
o’clock in the morning. In order for the worker to get to work on time and have
time to eat breakfast, we must have m − 15 ≥ 10, implying m ≥ 25. Assuming that
it takes the worker x minutes to get to work, the worker has m − x minutes to have
breakfast. Thus, to make sure that the m − x minutes, which is the remaining time
before the start hour of the factory, is enough for the worker to have breakfast with
the probability of 75%, we must have

P (m − x ≥ 15) = 0.75 ⇐⇒ P (x ≤ m − 15) = 0.75


Z min(m−15,20)
dx
⇐⇒ = 0.75
10 10
⇐⇒ min(m − 15, 20) = 17.5
⇐⇒ m = 32.5.

Therefore, if the worker heads off for work 32.5 minutes before 8 a.m., i.e., at
8, 000 − 32.50 = 7, 27.50 = 7, 270 , 3000 , s/he will get to work on time and with the
probability of 75% have time to eat breakfast. 

7. First solution: We solve the problem on a general field F . Let n ∈ N,


A = (ci cj ) ∈ Mn (F ), and In the identity matrix of size n. To evaluate det(In + A),
we may, without loss of generality, assume that ci 6= 0 for all 1 ≤ i ≤ n because
if ci0 = 0 for some i0 ∈ {1, . . . , n}, then det(In + A) = det(In−1 + A0 ) where
A0 = (c0i c0j ) ∈ Mn−1 (F ) with c0i ∈ {c1 , . . . , cn } \ {ci0 }, which has the same general
form of A = (ci cj ) ∈ Mn (F ). Use the basic properties of the determinant function
and perform the following operations: factor out ci of row i and of column i for
each i = 1, . . . , n; then subtract the first row from all the other rows; then multiply
column i by c2i (1 ≤ i ≤ n); and finally add row i to row 1 for each i = 2, . . . , n, to
obtain an upper triangular whose determinant is equal to that of In + A. We can
108 2. SOLUTIONS

write
1 + c12 1 ··· 1 1
 
1
 1 1 + c12 ··· 1 1 
 2 
2 2
 .. .. .. .. .. 
det(In + A) = c1 · · · cn det 
 . . . . .


1
1 1 ··· 1+ 1
 
 c2n−1 
1
1 1 ··· 1 1+ c2n
 
1 + c12 1 ··· 1 1
1
− c12 1
··· 0 0
 

1 c22 
.. .. .. ..
 
= c21 · · · c2n det 
 .. 
. . . . . 
− c12 1
 
 0 ··· c2n−1
0 
 1 
− c12 0 ··· 0 1
c2n
1

1 + c21 c22 c2n−1 c2n


 
···

 −1 1 ··· 0 0

= det 
 .. .. .. .. ..

 . . . . .


 −1 0 ··· 1 0 
−1 0 ··· 0 1
1 + c21 + · · · + c2n c22 c2n−1 c2n
 
···

 0 1 ··· 0 0

= det 
 .. .. .. .. ..

 . . . . .


 0 0 ··· 1 0 
0 0 ··· 0 1
= (1 + c21 + · · · + c2n ) × 1 × · · · × 1 = 1 + c21 + · · · + c2n .

Therefore, det(In + A) = 1 + c21 + · · · + c2n , which is what we want.

Second solution: Recall that if A ∈ Mn (F ), λi ’s (1 ≤ i ≤ n) are the eigenvalues


of A in the algebraic closure of F , and f (x) = det(xIn − A) is the characteristic
polynomial of A so that

f (x) = xn + fn−1 xn−1 + · · · + f0 ,


Pn Pn
then, fn−1 = − i=1 λi , f0 = (−1)n λ1 · · · λn , i=1 λi = tr(A),
 and λ1 · · · λn =
det(A). Also, if α ∈ F and g(x) = det xIn − (A + αIn ) is the characteristic
polynomial of A + αIn , then

g(x) = det (x − α)In − A = f (x − α).

With all that in mind, note first that the rank of A is 1 because row i is ci (c1 , . . . , cn )
which is a multiple of the fixed vector (c1 , . . . , cn ). It follows from the Rank-Nullity
Theorem that dim ker A = n − 1, and hence 0 is an eigenvalue of A of multiplicity
n − 1. Let λ be the only other eigenvalue of A. We must have
n
Y
f (x) = det(xIn − A) = (x − λi ) = xn−1 (x − λ) = xn − λxn−1 .
i=1
2.10. TENTH COMPETITION 109

On the other hand,


n
X n
X
λi = λ = tr(A) = c2i ,
i=1 i=1
Pn 2
implying λ = i=1 ci . If g denotes the characteristic polynomial of In + A, we
have
g(x) = f (x − 1)
= (x − 1)n − λ(x − 1)n−1
= xn + gn−1 xn−1 + · · · + g1 x + (−1)n (1 + λ).
Recall that
(−1)n (1 + λ) = (−1)n det(In + A),
whence
n
X
det(In + A) = 1 + λ = 1 + tr(A) = 1 + c2i ,
i=1
which is what we want. 

2.10. Tenth Competition

2.10.1. Analysis. 1. From (b), we see that f 0 (x) > 0 for all x ≥ 1, implying
that f is strictly increasing on [1, +∞) and hence f (t) > f (1) = 1 for all t > 1.
From this, we obtain
1 1
f 0 (t) = 2 < ,
t + (f (t))2 1 + t2
R +∞ 0 R +∞ dt π 0
for all t > 1. It thus follows that 1 f (t)dt < 1 1+t2 = 4 because f is
0 1
continuous and f (t) < 1+t2 on [1, +∞). Using the Second Fundamental Theorem
of Calculus , we can write
Z x Z x Z +∞
0 dt dt π
f (x) = 1 + f (t)dt < 1 + 2
<1+ 2
=1+ .
1 1 1+t 1 1+t 4
Therefore, f is bounded from the above. This together with the hypothesis that f
is increasing implies that limx→+∞ f (x) exists and that
Z +∞ Z +∞
1 π
lim f (x) = 1 + f 0 (t)dt < 1 + 2
=1+ ,
x→+∞ 1 1 1 + t 4
implying limx→+∞ f (x) < 1 + π4 , which is what we want. 

2. For an x ∈ (a, b), define g : [a, b] → R by


g(t) = (t − a)(t − b)f (x) − (x − a)(x − b)f (t).
Plainly, g(a) = g(x) = g(b) = 0. From this, applying Rolle’s Theorem twice to g
and g 0 , respectively, we obtain a c ∈ (a, b) such that g 00 (c) = 0. We can write
g 00 (c) = 2f (x) − (x − a)(x − b)f 00 (c) = 0,
which implies
f (x) ≤ M (x − a)(b − x) ,

2
110 2. SOLUTIONS

for all x ∈ [a, b]. Taking integrals of both sides of the above yields
Z b Z b
f (x) dx ≤ M (b − a)3
− x2 + (a + b)x − ab dx = M

.
a 2 a 12
Rb 3
That is, a f (x) dx ≤ M (b−a)

12 , which is what we want. 

3. First solution: We can write


Z 3 2 Z 3 2
x (1 − x)xn x |1 − x|xn



2n
dx ≤ dx

0 1+x
0 1 + x2n
Z 1 2 Z 3 2
x |1 − x|xn x |1 − x|xn
= dx + dx
0 1 + x2n 1 1 + x2n
Z 1 Z 3 n
x
≤ xn dx + 18 2n
dx
0 1 x
1 18 18
= + − .
n + 1 n − 1 3n−1 (n − 1)
It thus follows from the Squeeze Lemma that
Z 3 2
x (1 − x)xn
lim dx = 0,
n→+∞ 0 1 + x2n
which is what we want.

Second solution: In view of a standard theorem from classical analysis, it suffices


2
(1−x)xn +∞
to show that the sequence x 1+x 2n
n=1
uniformly converges to the zero function
on [0, 3]. Note first that
2
x (1 − x)xn

1 + x2n ≤ |1 − x|, (∗)

for all x ∈ [0, 3]. To see this, we have xn+2 ≤ 1 on [0, 1], implying xn+2 ≤ 1 + x2n
on [0, 1]. Also if n ≥ 2, then xn+2 ≤ x2n on [1, 3], which implies xn+2 ≤ 1 + x2n
on [1, 3]. Thus, xn+2 ≤ 1 + x2n on [0, 3] of which the above inequality is a quick
consequence. Now, let 0 < ε < 1 be given. It follows from (∗) that
2
x (1 − x)xn

1 + x2n < ε,

for all n ≥ 2 and x ∈ (1 − ε, 1 + ε). On the interval [0, 1 − ε], for all n > 2 we can
write 2
x (1 − x)xn xn+2

n+2
1 + x2n ≤ 1 + x2n ≤ x
≤ (1 − ε)n+2 .

As 0 < 1 − ε < 1, we have limn (1 − ε)n+2 = 0. Hence, there exists an N1 > 1 such
that (1 − ε)n+2 < ε for all n ≥ N1 . On the interval [1 + ε, 3], we have
2
x (1 − x)xn 2xn+2 2x2

2 2
1 + x2n < 1 + x2n = x−n + xn < xn−2 ≤ (1 + ε)n−2 ,

2 2
for all n ≥ 2. As limn (1+ε) n−2 = 0, there exists an N2 > 1 such that (1+ε)n−2 < ε

for all n ≥ N2 . Letting N = max(N1 , N2 ), for all x ∈ [0, 3] and n ≥ N we have


2
x (1 − x)xn

1 + x2n < ε.

2.10. TENTH COMPETITION 111

2
(1−x)xn +∞
That is, the sequence x 1+x 2n
n=1
uniformly converges to the zero function on
[0, 3], yielding
Z 3 2 Z 3
x (1 − x)xn x2 (1 − x)xn
lim 2n
dx = lim dx = 0,
n→+∞ 0 1+x 0 n→+∞ 1 + x2n
which is what we want. 

2.10.2. Algebra. 1. We prove the following lemma of which the assertion is


a quick consequence.

Lemma. Let V be a left (resp. right) vector space over a division ring D whose
characteristic is zero. Then the additive group of V has no maximal subgroup.
Proof. To prove the assertion by contradiction, suppose that M is a maximal
subgroup of the additive group of V . As M ≤ V , V is abelian, and M is maximal, it
V V
follows that M is a simple abelian group. Consequently, M is a finite cyclic group,
V
implying that there exists a prime number p such that M = Zp , and hence M is of
finite index in V . By proving that (V, +) has no proper subgroup of finite index, we
obtain a contradiction, proving the assertion. Suppose to the contrary that there
exists a proper subgroup H of the additive group of V such that [V : H] = n ∈ N.
V
As | H | = n, it follows from Lagrange’s Theorem that nx ∈ H for all x ∈ V . On the
other hand, x = n( nx ) and nx ∈ V for all x ∈ V , implying x ∈ H for all x ∈ V . This
yields V = H, a contradiction. Thus, V has no proper subgroup of finite index,
which is what we want, finishing the proof. 

To prove the assertion, just let V = D = R in the lemma. 

2. Since the ring R is unital, it follows from (∗) that there exist elements a, a0 , a00 ∈
A, b ∈ B, c ∈ C, and d ∈ D such that
a + b = 1, a0 + c = 1, a00 + d = 1.
Multiplying the above equalities and simplifying, we see that there is a u ∈ A such
that
1 = (a + b)(a0 + c)(a00 + d) = u + bcd,
implying 1 = u + bcd ∈ A + M because bcd ∈ M . This obviously yields A + M = R,
which is what we want.
Let R = Z, A = 2Z, B = 3Z, C = 5Z, and D = 7Z. It is easily verified that
M = B ∩ C ∩ D = 105Z and that R and its ideals A, B, C, D satisfy (∗). 

3. We solve the problem under the weaker hypothesis that gcd(n, c) = 1, where
c = lcm(1, 2, . . . , m − 1). It is obvious that F ⊆ F (αm ) ⊆ F (α). So we can write
[F (α) : F ] = [F (α) : F (αm )][F (αm ) : F ]. (∗)
As [F (α) : F ] = n, it follows that there exists a polynomial P ∈ F [x] of degree n
such that
P (α) = p0 + p1 α + · · · + pn αn = 0. (∗0 )
Using the division algorithm, dividing k by m for each k = 0, . . . , n, and collecting
terms appropriately in (∗0 ), we see that there exists a polynomial Q ∈ F (αm )[x]
with deg(Q) ≤ m − 1 such that
P (α) = Q(α) = q0 + q1 α + · · · + qm−1 αm−1 = 0.
112 2. SOLUTIONS

This clearly shows that [F (α) : F (αm )] ≤ m − 1, implying that [F (α) : F (αm )]
divides lcm(1, 2, . . . , m − 1) = c. But as, in view of (∗), [F (α) : F (αm )] divides n
and gcd(n, c) = 1, we conclude that [F (α) : F (αm )] = 1, whence F (α) = F (αm ),
yielding [F (αm ) : F ] = [F (α) : F ]. This, in turn, implies F (α) = F (αm ), which is
what we want. 

4. Recall that the trace of any nilpotent matrix N ∈ Mn (F ) is zero. With


that in mind, suppose by way of contradiction that there exist nilpotent matri-
ces N1 , . . . , Nk ∈ Mn (F ) (k ∈ N) that span Mn (F ). In particular, for the matrix
E11 ∈ Mn (F ), where E11 denote the matrix with 1 in the 11 place and zero else-
where, there are scalars c1 , . . . , ck ∈ F such that
E11 = c1 N1 + · · · + ck Nk ,
which implies
1 = tr(E11 ) = c1 tr(N1 ) + · · · + ck tr(Nk ) = 0,
a contradiction, proving the assertion. 

2.10.3. General. 1. As the tests are independent, we can write



P X = 0 = P (F S) + P (SSF S) + P (F F F S) + P (SSSSF S)
+P (SSF F F S) + P (F F SSF S) + P (F F F F F S) + · · ·
= qp + p2 qp + q 2 qp + p4 qp + p2 q 2 qp + p2 p2 qp + q 4 qp + · · ·
= qp 1 + (p2 + q 2 ) + (p2 + q 2 )2 + · · ·


1
qp 4 1
= = 1 = .
1 − p2 − q 2 2
2
1 1
 
That is, P X = 0 = 2. Similarly, one can show that P X = 1 = 2. 

2. We prove the assertion for the more general case where A is a subset of a
topological space. With that in mind, let A be a closed subset of a topological
space X such that A◦ = ∅. We prove the assertion by showing that A = ∂A = ∂Ac ,
where ∂A denotes the boundary of A. Recall that by definition ∂A = A ∩ Ac . So,
we can write
∂Ac = Ac ∩ Acc = Ac ∩ A = ∂A.
That is, ∂A = ∂Ac . On the other hand,
∂A = A \ A◦ = A \ ∅ = A = A.
So we have A = ∂A = ∂Ac , where Ac is an open set because A is a closed set,
which is what we want. 

3. Let x(t) be a solution of (∗). We have


x00 = f (t, x), x(0) = x0 , x0 (0) = y0 .
2.10. TENTH COMPETITION 113

As f is continuous, using the First Fundamental Theorem of Calculus and changing


the order of integration, we can write
Z t  Z t Z τ
0 0
 
x (t) = x (0) + f s, x(s) ds =⇒ x(t) = x(0) + y0 t + f s, x(s) ds dτ
0 0 0
Z t Z t 

=⇒ x(t) = x0 + y0 t + dτ f s, x(s) ds
0 s
Z t 
=⇒ x(t) = x0 + y0 t + (t − s)f s, x(s) ds,
0

which means x(t) is a solution of (∗∗).


Now, let x(t) be a solution of (∗∗). We can write
Z t 
x(t) = x0 + y0 t + (t − s)f s, x(s) ds,
0

implying that
Z t 
Z t 
x(t) = x0 + y0 t + t f s, x(s) ds − sf s, x(s) ds.
0 0

Again, using the First Fundamental Theorem of Calculus, we can write


Z t
0
  
x (t) = y0 + f s, x(s) ds + tf t, x(t) − tf t, x(t) ,
0

which obtains
Z t
0

x (t) = y0 + f s, x(s) ds.
0

Taking derivative of both sides of the above yields

x00 (t) = f t, x(t) .




It is obvious that x(0) = x0 and x0 (0) = y0 . Therefore, x(t) satisfies (∗), which is
what we want. 

4. First Solution: See Solution 1 of 2.5.3.

Second solution: Just as in the first solution, we settle the problem by proving
the polynomial identity
n
X
(1 + x)n = Cnk xk , (∗)
k=0
114 2. SOLUTIONS

n!
where Cnk = . Proceed by induction on n. If n = 1, the assertion is easy.
k!(n − k)!
Suppose that (∗) holds for n. To prove that (∗) holds for n + 1, we can write
n
X
(1 + x)n+1 = (1 + x)n (1 + x) = (1 + x) Cnk xk
k=0
n
X n
X
= Cnk xk + Cnk xk+1
k=0 k=0
n
X n
X
= 1+ Cnk xk + Cnk−1 xk + xn+1
k=1 k=1
Xn
= 1+ (Cnk + Cnk−1 )xk + xn+1 .
k=1

A straightforward calculation shows that Cnk + Cnk−1 = Cn+1


k
, from which, we easily
obtain
X n
(1 + x)n+1 = k
Cn+1 xk ,
k=0
proving the induction assertion, which is what we want.

Third Solution: For n, k ∈ N with k ≤ n, use Pnk to denote n(n−1) · · · (n−k +1).
n!
We have Pnk = k!Cnk for each n, k ∈ N with k ≤ n, where Cnk = . Thus,
k!(n − k)!
to prove the assertion, we need to show that Pnk is divisible by k! for all n, k ∈ N
with k ≤ n. We proceed by induction on n. If n = 1, there is nothing to prove.
Suppose that Pnk is divisible by k! for all k ≤ n. We need to show that Pn+1 k
is
k
divisible by k! for all k ≤ n + 1. If k = n + 1, then Pn+1 = k!, in which case, the
assertion is trivial. If k ≤ n, we can write
k
Pn+1 = (n + 1)n(n − 1) · · · (n + 1 − k + 1)
= (n − k + 1 + k)n(n − 1) · · · (n − k + 2)
= kPnk−1 + Pnk .
By the induction hypothesis, Pnk−1 and Pnk are divisible by (k − 1)! and k!, respec-
k
tively. Thus, Pn+1 = kPnk−1 + Pnk is divisible by k!, which is what we want.

Fourth solution: As we know, it suffices to prove that Cnk is an integer. To this


end, we need, first of all, the following lemma, which is due to Legendre (1808).

Lemma. Let p a prime number and n ∈ N. Then, the largest power of p


dividing n! is equal to
P h i
+∞ n
p i=1 pi ,
where [.] denotes the integer part function.
Proof. As n! = 1 × 2 × · · · × n, without loss of generality, we may assume
that p ≤ n. Among the consecutive integers 1, . . . , n those that are divisible by p
are
p, 2p, . . . , m1 p,
2.11. ELEVENTH COMPETITION 115

h i h i h i
n n n
where m1 = p . So, there are p such integers. Likewise, there are pi consec-
i i
utive integers from 1 up to n that are divisible by p (note that if p > n, then the
number of integers between and including 1 and n that are divisible by pi is zero).
Note that any multiple of pk which is not a multiple of pk+1 contributes as much as
k to the largest power of p that divides n!. Therefore, the largest power of p that
divides n! is equal to
P h i
+∞ n
p i=1 pi ,
proving the lemma. 

Now to prove the assertion, note first that


[a + b] ≥ [a] + [b],
for all a, b ∈ R. Thus, we can write
       
n n−k+k n−k k
= ≥ + i ,
pi pi pi p
for all i ∈ N. Adding up these inequalities, we obtain
+∞   +∞   X +∞  
X n X n−k k
≥ + .
i=1
pi i=1
pi i=1
pi
This means that the largest power of any prime p that divides n! is divisible by the
largest power of p that divides k!(n − k)!, proving the assertion. 

2.11. Eleventh Competition

2.11.1. Analysis. 1. To prove the assertion by contradiction, suppose that


the set of zeros of f , i.e.,
Z := x ∈ [0, 1]|f (x) = 0 = f −1 ({0})


is infinite. As the interval [0, 1] is compact, it follows that there exists a sequence
(xn )+∞
n=1 of distinct points in Z such that limn xn = x0 for some x0 ∈ [0, 1]. This
implies f (x0 ) = 0 because f is continuous. But the function f is differentiable at
x0 . So we can write
f (xn ) − f (x0 ) 0
f 0 (x0 ) = lim = lim = 0.
n xn − x0 n xn − x0

That is, f 0 (x0 ) = 0. On the other hand, f (x0 ) = 0, implying that x0 is a common
zero of f and f 0 , which is in contradiction with the hypothesis of the problem.
Thus, Z is finite, which is what we want. 

2. (a) To show that the sequence (fn )+∞ n=1 is pointwise convergent to a function f on
1 +∞
[1, e ], it suffices to prove that the sequence fn (a) n=1 is increasing and bounded
e
1 1
from the above for all a ∈ [1, e e ]. To this end, letting a ∈ [1, e e ] be arbitrary, we
prove these assertions by induction on n. We have
f1 (a) = a, fn+1 (a) = afn (a) ∀n ∈ N.
116 2. SOLUTIONS

First, by induction on n, we show that 1 ≤ fn (a) < e for all n ∈ N. If n = 1, then


1
1 ≤ f1 (a) = a ≤ e e < e. Assuming that 1 ≤ fn (a) < e, we can write
1
1 ≤ fn+1 (a) = afn (a) < (e e )e = e.
This proves 1 ≤ fn (a) < e for all n ∈ N by induction. We note that the function
g : R → R defined by g(x) = ax is increasing whenever a > 1. To prove that the
+∞
sequence fn (a) n=1 is increasing, again we use induction on n. If n = 1, we can
write
f2 (a) = g(a) > g(1) = f1 (a).
Now assuming that fn+1 (a) > fn (a), we have
 
fn+2 (a) = g fn+1 (a) > g fn (a) = fn+1 (a).
+∞
This proves that the sequence fn (a) n=1 is increasing. Thus, the sequence (fn )+∞
n=1
1
is pointwise convergent to a function f : [1, e e ] → R. In view of the continuity of
g, we can write
f (a) = lim fn+1 (a) = lim g fn (a) = g lim fn (a) = g f (a) = af (a) ,
  
n n n
1
f (a)
implying that f (a) = a and 1 ≤ f (a) ≤ e for all a ∈ [1, e e ], as desired.
1 ln x
(b) Define the function g : [1, e] → [1, e e ] by g(x) = e x . By inspecting the
1
derivative of g, one can easily verify that the function g : [1, e] → [1, e e ] is one-to-
1
one and onto. Moreover, g is continuous and so is its inverse g −1 : [1, e e ] → [1, e];
in fact g and, hence its inverse, g −1 are differentiable. From f (x) = xf (x) , we
1
easily obtain g(f (x)) = x for all x ∈ [1, e e ]. This shows that f = g −1 . Thus, f is
one-to-one and continuous because so is g −1 .
Now, the functional sequence (fn )+∞ n=1 of continuous functions converges point-
1
wise to the continuous function f on the compact interval [1, e e ] and moreover
1
fn (x) < fn+1 (x) for all x ∈ [1, e e ]. It thus follows from Dini’s Theorem that the
1
sequence (fn )+∞
n=1 converges uniformly to the function f on [1, e ], which is what
e

we want. 

3. Set
M = sup |φ00n (x)| : n ∈ N, x ∈ [−1, 1] .


It follows from the Mean Value Theorem that there exists a c between 0 and x
such that
φ0n (x) − φ0n (0) = xφ00n (c),
from which, we obtain
0
φn (x) = 1 + xφ00n (c) ≤ M + 1,

for all n ∈ N and x ∈ [−1, 1]. Using integration by parts, we get


Z 1 Z 1
≤ 1
0
φn (t) cos nπtdt φn (t) sin nπt dt

−1
nπ −1
M + 1 1
Z
M +1
≤ sin nπt dt = ,
nπ −1 nπ 2
2.11. ELEVENTH COMPETITION 117

implying that
1 1
Z
M +1
an = φn (t) cos nπtdt ≤ 2 2 ,
n −1 n π
for
P+∞ all n ∈ N. This, in view of the Limit Comparison Test for series, implies that
n=1 an is convergent, which is what we want. 

2.11.2. Algebra. 1. Set



I = xu − x : x ∈ R .
It is obvious that I is a two-sided ideal of R. It thus follows from the hypothesis
that I = {0} or I = R. If I = {0}, we see that xu = x, yielding xu = ux = x for
all x ∈ R, which means u = 1R . By showing that the case I = R is impossible, we
finish the proof. Suppose to the contrary that I = R. Thus, there exists an x0 ∈ R
such that u = x0 u − x0 . As u2 = u, we can write
u = u2 = (x0 u − x0 )u = x0 u − x0 u = 0,
and hence
x = ux = 0x = 0,
for all x ∈ R. This means R = {0}, implying that R has only one ideal, which is a
contradiction. Therefore, I = {0}, finishing the proof. 

2. To prove the assertion by contradiction, suppose that A is a finite extension of


Q so that [A : Q] = n for some n ∈ N. Pick a prime number p, e.g., p = 2, and note
that, by Eisenstein’s Criterion, the monic polynomial xn+1 − p is irreducible over
Z and hence over Q. If α is one of the roots of the equation xn+1 − p = 0, we have
Q ⊆ Q(α) ⊆ A.
On the other hand, [A : Q] = n < n + 1 = [Q(α) : Q], which is obviously impossible.
Thus, A is not a finite extension of Q, which is what we want. 

3. First we need to recall that if G is a finite p-group, where p is a prime number,


then the center of G, denoted by Z(G), is nontrivial, i.e., |Z(G)| > 1. To prove
this, first recall that the class equation of G is
k
X
|G| = |Z(G)| + [G : CG (xi )],
i=1

where {x1 , . . . , xk } is a maximal set of nonconjugate elements of G \ Z(G) and


CG (xi ) is the centralizer of xi in G (1 ≤ i ≤ k). Note that since xi ’s are in G\Z(G),
CG (xi )’s are proper subgroups of G, implying that whose orders are all powers of p,
and hence so are [G : CG (xi )]’s because G is a finite p-group. Consequently, from
the class equation of G, in view of |G| = pn for some n ∈ N, we conclude that p
divides |Z(G)|, yielding |Z(G)| > 1, as desired.
We now prove the assertion. It follows from the hypothesis that any two el-
ements of G which are different from the identity share the same orders. Now
suppose that p is a prime dividing |G|. It follows from Cauchy’s Theorem that
there exists an x ∈ G whose order is p. Hence, all of the elements of G but e have
order p, whence the order of G is a power of p. That is, G is a p-group.  Conse-
quently, the center of G is nontrivial, i.e., Z(G) 6= {e}. Since α Z(G) = Z(G) for
118 2. SOLUTIONS

all α ∈ Aut(G), from the hypothesis, we see that Z(G) = G. That is, G is abelian.
Therefore, by the Fundamental Theorem of finite abelian groups, we obtain

G ∼
= Zp ⊕ · · · ⊕ Zp ,

which is what we want. 

2.11.3. General. 1. Note that for an A ∈ M2 (Z), the evenness of det(A)


is equivalent to saying that det(A0 ) = 0, where A0 ∈ M2 (Z2 ) is the matrix whose
entries are obtained from those of A viewed as elements of Z2 . With that in mind,
the desired probability is equal to the probability that a randomly chosen matrix
A0 ∈ M2 (Z2 ) has determinant zero. Obviously, |M2 (Z2 )| = 24 . On the other hand,
the matrices in M2 (Z2 ) with zero determinants are as follows
         
0 0 0 1 1 0 0 0 1 1
, , , , ,
0 0 0 1 1 0 1 1 0 0
         
1 1 0 0 1 0 0 1 0 0
, , , , .
1 1 1 0 0 0 0 0 0 1
10
Therefore, the desired probability is equal to 16 = 58 , which is what we want. 

2. Plainly, the equation is equivalent to the following

4x3 − 3x = p,

where p = −4c. Now, letting x = cos z, where z ∈ C, the above equation becomes
equivalent to the following equation

cos 3z = p. (∗)

As the original equation has a root x0 in the closed interval [−1, 1], it follows that
p ∈ [−1, 1], whence there exists z0 ∈ R such that x0 = cos z0 . It is now obvious
that z0 , z0 + 2π 4π
3 , and z0 + 3 satisfy (∗). Thus, the corresponding x’s which are
2π 4π
cos z0 , cos(z0 + 3 ), cos(z0 + 3 ) ∈ [−1, 1] are all of the roots of the original equation,
proving the assertion. 

3. In view of the hypotheses of the problem, we see that C, endowed with the
symmetric difference of sets, denoted by ∆, which is defined by

A∆B = (A \ B) ∪ (B \ A)

forms a subgroup of P(X), where P(X) denotes the power set of X which obviously
forms a group under ∆ itself. As is well-known, |P(X)| = 2n , where n = |X|. Thus,
it follows from Lagrange’s Theorem that |C| divides |P(X)| = 2n , yielding |C| = 2k
for some k ≤ n, which is what we want. 
2.12. TWELFTH COMPETITION 119

2.12. Twelfth Competition

2.12.1. Analysis. 1. First Solution: Suppose that f (x) = f (y) for some
x, y ∈ R. It follows that M |x − y| ≤ 0, implying x = y. That is, f is one-to-one.
Now, as f is continuous and one-to-one, we see from Problem 2 of 1.9.1 that f is
strictly monotonic. Without loss of generality, we may assume that f is strictly
increasing. It now easily follows from the hypothesis that limx→−∞ f (x) = −∞ and
limx→+∞ f (x) = +∞. This, in view of the Intermediate Value Theorem, implies
that f is onto, which is what we want.

Second Solution: Just as we saw in the first solution, the function f is one-to-
one and strictly monotonic, and hence the inverse of f , i.e., f −1 : f (R) → R is
continuous. This implies that f is both an open and closed map. Now, since R is
both open and closed in R, we see that so is f (R), yielding f (R) = R because R is
connected. This proves the assertion. 
P+∞
2. “=⇒” Since n=1 fn converges uniformly on S, it is uniformly Cauchy on S,
and hence for given ε > 0, there exists an N > 0 such that
n
X ε
fk < ,
2
k=m

for all m, n ≥ N . Now, letting m = n2 > N , it follows that n > m ≥ N , from


 

which, in view of the fact that (fn )+∞


n=1 is a decreasing sequence of nonnegative
functions on S, we see that
n
X ε
fk <
2
k=[ n
2]

n
nfn hni X ε
=⇒ ≤ (n − + 1)fn ≤ fk < ,
2 2 2
2]
k=[ n

for all n ≥ 2N + 1. This obviously implies nfn < ε for all n ≥ 2N + 1. That is, the
sequence (nfn )+∞
n=1 converges uniformly to zero on S. On the other hand,
+∞
X +∞ 
X 

n(fn − fn+1 ) = nfn − (n + 1)fn+1 + fn+1 . (∗)
n=1 n=1

But (n + 1)fn+1 converges uniformly to zero on S. Hence, the telescopic series


P+∞
n=1 nfn − (n + 1)fn+1 converges uniformly to f1 on S. This together with the
P+∞ P+∞
hypothesis and (∗) implies that n=1 n(fn −fn+1 ) converges uniformly to n=1 fn
on S, which is what we want.

“⇐=” Note that as pointed out in the footnote of the problem in Section 1.12.1,
for this implication we need to assume further that the sequence (fn )+∞
n=1 converges
uniformly to zero on the set S. That is because for the numerical sequence (fn )+∞
n=1 ,
P+∞ P+∞
where fn = 2+ n12 , the series n=1 n(fn −fn+1 ) converges but n=1 fn is divergent.
120 2. SOLUTIONS

x=1

t
t=1
Figure 10

Now, since (fn )+∞ converges uniformly to zero on S, it follows that the tele-
P+∞ n=1
scopic series n=1 (fn − fn+1 ) converges uniformly to f1 on S. We can write
+∞
X +∞
X +∞
X
nfn = n (fk − fk+1 ) = n(fk − fk+1 ) ≤ k(fk − fk+1 ), (∗∗)
k=n k=n k=n
P+∞
for all n ∈ N. By the hypothesis, the series n=1 n(fn − fn+1 ) is uniformly Cauchy
on S. This, in view of (∗∗), easily implies that the sequence (nfn )+∞n=1 converges
P+∞ 
uniformly to zero on S. Thus, the telescopic series n=1 nfn − (n + 1)fn+1
converges uniformly to f1 on S. We can write
+∞ +∞ 
X X 
fn+1 = n(fn − fn+1 ) − nfn − (n + 1)fn+1 ,
n=1 n=1
P+∞ P+∞
implying that n=1 fn+1 converges uniformly on S, for so do the series n=1 n(fn −
P+∞  P+∞
fn+1 ) and
P+∞ n=1 nfn − (n + 1)fn+1 on S. Therefore, n=1 fn+1 , and hence
n=1 fn , converges uniformly on S. Moreover,
+∞
X +∞
X
fn = n(fn − fn+1 ).
n=1 n=1

So the proof is complete. 

3. First solution: Using integration by parts, we can write


Z 1 Z 1  Z 1 1 Z 1 Z 1 

g(t)dt dx = x g(t)dt − xd g(t)dt
0 x x x=0 0 x
Z 1
= 0+ xg(x)dx
0
Z 1
= xg(x)dx,
0

which is what we want.


2.12. TWELFTH COMPETITION 121

Second solution: Changing the order of integration, we can write


Z 1 Z 1  Z 1 Z t 
g(t)dt dx = g(t)dx dt
x=0 t=x t=0 x=0
Z 1 Z t 
= g(t) dx dt
t=0 x=0
Z 1
= tg(t)dt,
0

which is what we want. 

2.12.2. Algebra. 1. By Sylow’s Third Theorem, the number of subgroups


of order p is equal to 1 + kp for some k ∈ N ∪ {0} and that 1 + kp divides |G|. That
is, 1 + kp|2p. This yields 1 + kp = 1, implying k = 0. Thus, G has only one Sylow
p-subgroup. Therefore, G has only one subgroup of order p.
Using Sylow’s Third Theorem again, we see that the number of subgroups of
order 2 is equal to 1 + 2k for some k ∈ N ∪ {0} and that 1 + 2k divides |G|. That is,
1 + 2k|2p, from which we obtain 1 + 2k = 1 or 1 + 2k = p. Therefore, G has either
one and only one subgroup of order 2 or it has p subgroups of order 2.
Now, suppose that G has only one subgroup of order 2. Let H and K be the
only Sylow 2-subgroup and Sylow p-subgroup, respectively. We have |H| = 2 and
|K| = p. This implies |H ∩ K| = 1. In other words, H ∩ K = e. Pick h ∈ H
and k ∈ K such that ord(h) = 2 and ord(k) = p. As H ∩ K = e, we see that
hk = kh because hkh−1 k −1 ∈ H ∩ K, and hence ord(hk) = 2p, for ord(h) = 2,
ord(k) = p, and gcd(2, p) = 1 (for a more detailed proof of ord(hk) = 2p, see the
lemma presented in Solution 1 of 2.25.2). This implies that the cyclic subgroup
generated by hk is G. Therefore, G is a cyclic group, which is what we want. 

2. Note first that G has no elements of even order, in particular of order 2, because
G is of odd order. We shall show that the map ψ : G → G defined by ψ(g) = g 2 is
an automorphism of G. To see that ψ is one-to-one, suppose ψ(a) = ψ(b) for some
a, b ∈ G. It easily follows that (ab−1 )2 = e, where e is the identity element of G.
But ord(ab−1 ) 6= 2. So ab−1 = e, yielding a = b. That is, ψ is one-to-one. It now
follows that ψ is an automorphism of G because G is finite and abelian. Thus, for
an arbitrary g ∈ G, there exists an a ∈ G such that g = a2 . Set x = aφ(a) and
y = aφ(a−1 ). Noting that G is abelian, we can write

xy = aφ(a) aφ(a−1 ) = a2 φ(aa−1 ) = a2 = g.


 

On the other hand,

= φ aφ(a) = φ(a)φ2 (a) = aφ(a) = x,



φ(x)
= φ aφ(a−1 ) = φ(a)φ2 (a−1 ) = a−1 φ(a) = y −1 ,

φ(y)

as desired. To show that such x, y are unique, suppose that for g ∈ G, there
are x, y, z, t ∈ G such that g = xy = zt, φ(x) = x, φ(z) = z, φ(y) = y −1 , and
122 2. SOLUTIONS

φ(t) = t−1 . We have

φ(g) = φ(xy) = φ(zt) =⇒ φ(x)φ(y) = φ(z)φ(t)


=⇒ xy −1 = zt−1 =⇒ xyy −2 = zt−1
=⇒ zty −2 = zt−1 =⇒ y 2 = t2
=⇒ ψ(y) = ψ(t) =⇒ y = t,

which, in view of xy = zt, yields x = z. Therefore, any element g ∈ G can, uniquely,


be written as g = xy, where φ(x) = x and φ(y) = y −1 , finishing the proof. 

3. As every Boolean ring is commutative, it suffices to show that R is a Boolean


ring. To this end, pick x ∈ R such that x 6= 0, 1. As ch(R) = 2, there exists
x1 ∈ R \ {0, 1} such that x = 1 + x1 . It follows from the hypothesis that xy = xy 2
for all y ∈ R. In other words, (1 + x1 )y = (1 + x1 )y 2 , from which, in view of
x1 y = x1 y 2 , we obtain y = y 2 for all y ∈ R. That is, R is a Boolean ring, and hence
commutative, which is what we want. 

4. First, we claim that every ascending chain of ideals of R terminates. To this


end, suppose
I1 ⊆ I2 ⊆ · · ·
S+∞
is an ascending chain of ideals of R. Set I = n=1 In . It is easily seen that I is an
ideal of R, and hence there exists an x0 ∈ R such that I = hx0 i = x0 R. As x ∈ I,
it follows that there exists an N ∈ N such that x0 ∈ IN . We can write
+∞
[ +∞
[
IN ⊆ I = Ik = x0 R ⊆ IN =⇒ IN = Ik
k=1 k=1
=⇒ ∀n ∈ N : In ⊆ IN .

On the other hand, IN ⊆ In for all n ≥ N , from which, in view of the above
inclusion, we obtain IN = In for all n ≥ N . This proves the claim. Now, suppose
that f : R → R is a surjective homomorphism. To see that f is one-to-one, we note
that
ker f ⊆ ker f 2 ⊆ ker f 3 ⊆ · · ·
is an ascending chain of ideals of R. It follows from the claim that there exists an
N ∈ N such that ker f n = ker f N for all n ≥ N . In particular, ker f N +1 = ker f N .
Suppose x ∈ ker f is arbitrary so that f (x) = 0. As f is onto, so is f n for all
n ∈ N, and hence there exists an x0 ∈ R such that x = f N (x0 ). Consequently,
f N +1 (x0 ) = 0, yielding x0 ∈ ker f N +1 = ker f N . Therefore, x = f N (x0 ) = 0,
implying ker f = {0}. That is, f is one-to-one, which is what we want. 

5. First solution: Let λi ’s (1 ≤ i ≤ 3) be the eigenvalues of the matrix A in the


algebraic closure of F . As A is invertible, λ−1
i ’s (1 ≤ i ≤ 3) are the eigenvalues of
A−1 . If f is the characteristic polynomial of A, we have

f = (x − λ1 )(x − λ2 )(x − λ3 )
= x3 − (λ1 + λ2 + λ3 )x2 + (λ1 λ2 + λ1 λ3 + λ2 λ3 )x − λ1 λ2 λ3 .
2.12. TWELFTH COMPETITION 123

By the hypothesis,
tr(A) = λ1 + λ2 + λ3 = 0,
tr(A ) = λ−1
−1 −1 −1
1 + λ2 + λ3 = 0,
det(A) = λ1 λ2 λ3 = 1.
On the other hand, λ1 λ2 + λ1 λ3 + λ2 λ3 = λ−1 −1 −1
1 + λ2 + λ3 = 0. So, we must have
f (x) = x − 1. Now, by the Cayley-Hamilton Theorem, A3 = I, which is what we
3

want.

Second solution: Let λi ’s (1 ≤ i ≤ 3) be as in the first solution. We see that


λi ’s (1 ≤ i ≤ 3) are distinct because λ1 + λ2 + λ3 = 0 = λ−1 −1
1 + λ2 + λ3
−1
and
3 2
λ1 λ2 λ3 = 1. With that in mind, suppose f (x) = x +ax +bx+c is the characteristic
polynomial of A. We have a = tr(A) = 0 and c = det(A) = −1. By the Cayley-
Hamilton Theorem, A3 + bA − I = 0, yielding (A−1 )3 − b(A−1 )2 − I = 0. As λi ’s
(1 ≤ i ≤ 3) are distinct, the characteristic polynomial of A−1 , denoted by g, is
equal to g(x) = x3 − bx2 − 1. But b = tr(A−1 ) = 0. Therefore, (A−1 )3 − I = 0,
implying A3 = I, which is what we want. 

6. It is obvious that α and β are roots of the polynomial


p!
f (x) = p! + p!x + x2 + · · · + xp ,
2!
which is irreducible over Q by Eisenstein’s Criterion. Thus, f is the minimal poly-
nomial of α and β over Q. We prove the assertion by way of contradiction.
First suppose that α − β = r ∈ Q. We have f (β + r) = f (α) = 0. That is, β is a
root of the polynomial f (x + r) whose coefficients
are in Q. But f (x) is the minimal
polynomial of β over Q. It follows that f (x) f (x+r). As f (x) and f (x+r) are both
monic polynomials of the same degree, we obtain f (x + r) = f (x). Consequently,
if α1 , . . . , αp are all of the roots of f (x) = 0, then so are α1 + r, . . . , αp + r. In
particular, we must have
(α1 + r) + · · · + (αp + r) = α1 + · · · + αp =⇒ pr = 0
=⇒ r = 0
=⇒ α = β,
which is a contradiction. Thus, α − β = r ∈ / Q, as desired.
Now , assume that α + β = r ∈ Q. We have −f (r − β) = f (α) = 0. That is, β
is a root of the polynomial −f (r − x) whose coefficients are in Q. But f (x) is the
minimal polynomial of β over Q. Thus, f (x) − f (r − x). As f (x) and −f (r − x)
are both monic polynomials of the same degree, we obtain −f (r − x) = f (x).
Consequently, letting x = 2r , we obtain −f ( 2r ) = f ( 2r ), which yields f ( 2r ) = 0.
Hence, f (x) is divisible by x − 2r , which is in contradiction with the fact that f (x)
is irreducible over Q. Therefore, α + β = r ∈ / Q, as desired.
Next, suppose that αβ = r ∈ Q. Obviously, α, β 6= 0. We can write f ( βr ) =
p
f (α). It thus follows that β is a root of the polynomial xp! f ( xr ) whose coefficients
xp r p
are in Q. Again, we obtain f (x) p! f ( x ), from which, as f (x) and xp! f ( xr ) are
p
both monic polynomials of the same degree, we see that xp! f ( xr ) = f (x). There

are two cases to consider: (i) r > 0 and (ii) r < 0. If r > 0, substituting x = r
p √ √
into xp! f ( xr ) = f (x), we obtain f ( r) = 0. If r ∈ Q, then f (x) is divisible by
124 2. SOLUTIONS

A
R r
O M

Figure 11

√ √
x− r, which is in contradiction with the irreducibility
√ of f over Q. And if r ∈ / Q,
then √x2 − r would be the minimal polynomial of r over Q. From this, in view
of f ( r) = 0, we see that x2 − r f (x), implying that f is not irreducible over Q,
√ p
a contradiction again. If r < 0, substituting x = i −r into xp! f ( xr ) = f (x), we
√ √
obtain f (i −r) = 0. It follows
√ that x2 + r is the minimal
polynomial of r over Q.
From this, in view of f (i −r) = 0, we see that x2 + r f (x), implying that f is not
irreducible over Q, which is again a contradiction. Therefore, αβ = r ∈ / Q, which
is what we want. It might be worth mentioning that one can show, in a similar
fashion, that α
β ∈/ Q. 

2.12.3. General. 1. Let R and a, respectively, be the radius of the circle C


centered at O and the side length of the square whose√center M is on the circle.
2 √
From the hypothesis, we see that a2 < πR 2 , whence a 2 ≤ R π < 2R. That is,
the diameter of the square is smaller than that of the circle.

It follows that the vertices √of the square, with center M , are on the circle
centered at M with radius r = a 2 2 < R. Let A and B be the points at which these
two circle intersect one another. Consider two isosceles triangles OAM and OBM .
We have AM = M B < OA = OB = OM . It is plain that ∠AM O > 60◦ and
∠BM O > 60◦ , and hence ∠AM B > 120◦ . Therefore, the arc AB from the circle
centered at M is greater than 120◦ , and hence some vertex of the square must be
on this arc which itself lies insides the circle with radius R centered at O. This
proves the assertion. 

2. The answer is: yes, it does. To see this, we count the number of the desired
answer sheets in a test in which n questions are given. Suppose that there are n
questions in the test. Use 1 and 0 to denote a false and a true answer, respectively,
and An to denote the number of desired answer sheets. Then to any answer sheet,
there corresponds an n-tuple of 0’s and 1’s. Let’s call an answer sheet or n-tuple
admissible if there are no consecutive 1’s in it. First, for a given k (0 ≤ k ≤ n), we
count the number of admissible n-tuples each of which contains k ones and use Nk
to denote this number. To do so, note that in any such admissible n-tuple, there
are k ones and n − k zeros and that we want to insert a zero between any two
ones unless the zero occurs in the 1st position or the nth position. With that in
mind, there are n − k − 1 + 2 = n − k + 1 positions and of those we need to choose
k positions to each of which assign a one. Thus, the number Nk is equal to the
2.13. THIRTEENTH COMPETITION 125

number of ways of choosing k positions out of n − k + 1 positions which is equal to


n−k+1 n+1

k . Obviously, k ≤ p := 2 and that we have
p p  
X X n−k+1
An = Nk =
k
k=0 k=0
     
n+1 n n−p+1
= + + ··· + .
0 1 p
It thus follows that the number of desired answer sheets in the test in which 15
questions are given is equal to
     
16 15 8
A15 = + + ··· + = 1597.
0 1 8
As 1700 people have participated in the test, there are necessarily two equal answer
sheets. 

3. We have
n
X
(1 + i)n = Cnk in−k ,
k=0

where i ∈ C with i = −1. On the other hand, 1 + i = 2 cos π4 + i sin π4 , and
2


hence
n nπ nπ 
(1 + i)n = 2 2 cos + i sin .
4 4
So we have
n
X n nπ nπ 
Cnk in−k = 2 2 cos + i sin ,
4 4
k=0

whence
n nπ n nπ
Cn0 − Cn2 + Cn4 − · · · + i Cn1 − Cn3 + Cn5 − · · · = 2 2 cos
 
+ i2 2 sin ,
4 4
form which, we obtain
n nπ
Cn0 − Cn2 + Cn4 − · · · = 2 2 cos ,
4
n nπ
Cn1 − Cn3 + Cn5 − · · · = 2 2 sin ,
4
which is what we want. 

2.13. Thirteenth Competition

2.13.1. Analysis. 1. First solution: As f : R → R is integrable on any


closed interval, for all x, y ∈ R, we can write
Z x Z x Z x
f (t + y)dt = f (t)dt + f (y)dt,
0 0 0
126 2. SOLUTIONS

which easily yields


Z x Z x
xf (y) = f (t + y)dt − f (t)dt
0 0
Z x+y Z x
= f (t)dt − f (t)dt
y 0
Z x+y Z x Z y
= f (t)dt − f (t)dt − f (t)dt,
0 0 0

and hence xf (y) = yf (x) for all x, y ∈ R. Therefore, f (x)x = f (1)


1 = f (1) for all
x ∈ R with x 6= 0. This implies f (x) = f (1)x for all x ∈ R, as desired.

Second solution: As f : R → R is integrable, it follows from the lemma presented


in Solution 1 of 2.5.1 that the function f is continuous at infinitely many points
of any closed interval of R. On the other hand, f (x + y) = f (x) + f (y) for all
x, y ∈ R and hence f (r) = rf (1) for all r ∈ Q. From this, it is easily seen that f is
continuous on R and that f (x) = xf (1) for all x ∈ R, which is what we want. 

2. First solution: We need the following well-known lemma which is known as


Dedekind’s Extension of Abel’s Theorem.

Lemma. Let (an )+∞ and (bn )+∞


n=1 be sequences of real numbers. If the series
P+∞ P+∞ n=1 P+∞
n=1 an and n=1 |bn − b n+1 | converge, then so does the series n=1 an bn .
Proof. It is easily verified that
an+1 bn+1 + · · · + an+p bn+p =(sn+1 − sn )(bn+1 − bn+2 ) + · · · +
(sn+p−1 − sn )(bn+p−1 − bn+p ) + (sn+p − sn )bn+p ,
Pn P+∞
where sn = k=1 ak and n, p ∈ N. Note that as n=1 |bn − bn+1 | converges, so
P+∞
does the telescopic series n=1 (bn − bn+1 ), from which, we see that limn bn exists,
and hence the sequence (bn )+∞
n=1 is bounded. Now, the above equality, together with
Pn +∞
the facts that (sn )+∞
n=1 and k=1 k|b − b | +∞
k+1 n=1 are Cauchy and that (bn )n=1 is
Pn +∞
bounded, implies that ( k=1 ak bk )+∞
P
n=1 is Cauchy, and hence the series n=1 an bn
is convergent, which is what we want. 

an
To prove the assertion, letting cn = bn , we can write

an cn − c2n 2
 c2n 
= = cn − cn 1 + .
an + bn 1 − c2n 1 − c2n
c2n
Let An = cn − c2n and Bn = 1−c2n . We have
an
= An + An Bn .
an + bn
P+∞
As n=1 c2n is convergent, limn c2n = 0, and hence there exists an N ∈ N such that
c2n < 12 for all n ≥ N . It follows that for all n ≥ N we have
c2n
0 ≤ Bn = < 2c2n ,
1 − c2n
2.13. THIRTEENTH COMPETITION 127

P+∞
from which, in view of the Comparison Test, we see that n=1 Bn is convergent.
Note that
Bn − Bn+1 ≤ Bn + Bn+1 = Bn + Bn+1 ,
for all n ≥ N . Again the Comparison
P+∞ Test together
P+∞with the above inequality, in
view of the convergence of n=1 Bn , implies that n=1 |Bn − Bn+1 | is convergent.
P+∞
On the other hand, it follows from the hypothesis that n=1 An is convergent. Now,
P+∞
using Dedekind’s Extension of Abel’s Theorem, we see that n=1 An Bn converges.
P+∞ P+∞
This together with the convergence of n=1 An implies that n=1 (An + An Bn )
P+∞
converges. Thus, so does the series n=1 ana+b n
n
, which is what we want.

Second solution: We can write


an cn + c2n − c2n c2n
= = cn − ,
an + bn 1 + cn 1 + cn
where cn = abnn . As limn cn = 0, it follows that there exists an N ∈ N such that
c2n P+∞ 2
1+cn > 0 for all n ≥ N . This together with the convergence of n=1 cn , in view
P+∞ c2n
of the Limit Comparison Test, implies that the series n=1 1+cn converges. From
P+∞
the above equality and the hypothesis that n=1 cn is convergent, we see that the
P+∞ an
series n=1 an +bn is convergent and that we have
+∞ +∞ +∞
X an X X c2n
= cn − ,
a + bn
n=1 n n=1 n=1
1 + cn
finishing the proof. 

3. First solution: To prove the assertion by contradiction, suppose that the


sequence (fn )+∞
n=1 does not converge uniformly to zero on [0, 1]. It follows that
there are an  > 0, a sequence (nk )+∞ k=1 , with nk ≥ k, of natural numbers, and a
+∞
sequence (xk )k=1 in [0, 1] such that |fnk (xk )| ≥  for all k ∈ N. If necessary, by
passing to a subsequence of (xk )+∞ k=1 and replacing fnk by −fnk , without loss of
generality, we may assume that fnk (xk ) > 0 for all k ∈ N and that there is an
x0 ∈ [0, 1] such that limk xk = x0 . We can write
Z xk
fnk (xk ) − fnk (x0 ) = fn0 k (t)dt,
x0

from which, in view of ||fn0 ||∞ ≤ 1 for all n ∈ N, we obtain



fn (xk ) − fn (x0 ) ≤ xk − x0 ,
k k

for all k ∈ N. Let K1 ∈ N be such that |xk − x0 | < 2 for all k ≥ K1 . We must have
fn (x0 ) ≥ fn (xk ) − fn (xk ) − fn (x0 ) ≥  −  =  ,

k k k k
2 2
for all k ≥ K1 . Now, from this and
Z x
fnk (x) − fnk (x0 ) = fn0 k (t)dt,
x0

in a similar fashion, we see that


fn (x) ≥  ,

k
4
128 2. SOLUTIONS

for all x ∈ [0, 1] with |x − x0 | < 4 and k ≥ K2 , where K2 is such that |xk − x0 | < 4
for all k ≥ K2 . Since fnk ’s (k ≥ K2 ) are continuous functions, we conclude that
fnk (x) > 0, and hence,

fnk (x) ≥ ,
4
for all x ∈ [0, 1] with |x − x0 | < 4 and k ≥ K2 , for otherwise, by the Intermediate
Value Theorem, there must exist zk ∈ [0, 1] with |zk − x0 | < 4 and k ≥ K2 such
that fnk (zk ) = 0, which is impossible. Let [a, b] = [0, 1] ∩ [x0 − 4 , x0 + 4 ]. Now
define g : [0, 1] → R by

 0 0 ≤ x ≤ a,
3
a ≤ x ≤ a + b−a


 b−a
 (x − a) 3 ,
g(x) = 1 a + b−a
3 ≤ x ≤ a + 2 b−a
3 ,
−3 b−a
(x − b) a + 2 ≤ x ≤ b,


 b−a 3


0 b ≤ x ≤ 1.
The function g is continuous and nonnegative on [0, 1]. For all k ≥ K2 , we can
write
Z 1 Z a+ b−a
3
Z a+2 b−a
3
Z b
fnk g = fnk g + fnk g + fnk g
0 a a+ b−a
3 a+2 b−a
3

a+2 b−a
(b − a)
Z 3
≥ fnk ≥ ,
a+ b−a
3
12
R1
contradicting the hypothesis that limn 0
fnk g = 0. Therefore, the assertion follows
by contradiction.

Second solution: It suffices to show that every subsequence (gk )+∞ +∞


k=1 of (fn )n=1 ,
+∞
where gk = fnk (k ∈ N), in turn, has a subsequence (hj )j=1 , where hj = gkj
(j ∈ N), which converges uniformly to zero on [0, 1]. We first show that the sequence
(fn )+∞n=1 is uniformly bounded on [0, 1]. To this end, noting that fn : [0, 1] → R and
||fn0 ||∞ ≤ 1 for all n ∈ N, from the Mean Value Theorem, we see that

fn (x) ≤ 1 + fn (0) ,
+∞
for all x ∈ [0, 1] and n ∈ N. We claim that the sequence fn (0) n=1 is bounded.
+∞ +∞
Suppose to the contrary that there exists a subsequence fnk (0) n=1 of fn (0) n=1
such that limk fnk (0) = +∞ or limk fnk (0) = −∞. If necessary, replacing fn by
−fn , we may assume that limk fnk (0) = +∞. Thus, there exists a K ∈ N such
that fnk (0) ≥ 2 for all k ≥ K. The sequence (fn )+∞ n=1 is uniformly equicontinuous
on [0, 1] because ||fn0 ||∞ ≤ 1 for all n ∈ N. It follows that the sequence (fnk )+∞
n=1 is
uniformly equicontinuous on [0, 1] as well. So choosing ε = 1, we obtain 0 < 2δ < 1
such that
fn (x) − fn (0) < 1,
k k

whenever 0 < x < 2δ and k ∈ N. This easily yields


fnk (x) ≥ 2 − 1 = 1,
for all 0 < x < 2δ and k ∈ N with k ≥ K. Now, define g : [0, 1] → R by

 1 0 ≤ x ≤ δ,
g(x) = − xδ + 2 δ ≤ x ≤ 2δ,
0 2δ < x ≤ 1.

2.13. THIRTEENTH COMPETITION 129

Obviously, g is continuous and nonnegative on [0, 1]. So it follows from the hypoth-
R1
esis that limn 0 fn g = 0, from which, we obtain
Z 1
lim fnk g = 0.
k 0
Note that as g ≥ 0 on [δ, 2δ], for all k ≥ K, we have
Z 1 Z 2δ
fnk g = fnk g
0 0
Z δ Z 2δ
= fnk + fnk g
0 δ
≥ 1 × (δ − 0) = δ.
R1 R1
That is, 0 fnk g ≥ δ for all k ≥ K, which is in contradiction with limk 0 fnk g = 0
+∞
and 0 < δ. Therefore, fn (0) n=1 is bounded and hence so is (fn )+∞ n=1 because
|fn (x)| ≤ 1 + |fn (0)| for all x ∈ [0, 1] and n ∈ N.
Now suppose (gk )+∞ k=1 , with gk = fnk (k ∈ N), is an arbitrary subsequence of
(fn )+∞ +∞
n=1 . The sequence (gk )k=1 is uniformly bounded and equicontinuous on [0, 1]
+∞
because so is (fn )n=1 on [0, 1]. It thus follows from Arzela’s Theorem that the
sequence (gk )+∞ +∞
k=1 has a subsequence (hj )j=1 , with hj = gkj (j ∈ N), such that
+∞
(hj )j=1 converges uniformly to a function h : [0, 1] → R on [0, 1]. The function h
is continuous because it is a uniform
R1 limit of continuous functions. This together
with the hypothesis yields limj 0 hj h = 0. But it is plain that (hj h)+∞ j=1 converges
uniformly to h2 on [0, 1]. So, we can write
Z 1 Z 1 Z 1
2
h = lim hj h = lim hj h = 0,
0 0 j j 0
R1 2
implying 0 h = 0, which, in turn, implies h = 0 because h is continuous on
[0, 1]. That is, we have shown that every subsequence of (fn )+∞
n=1 , in turn, has a
subsequence converging uniformly to zero on [0, 1]. This proves the assertion. 

2.13.2. Algebra. 1. Note first that every nonzero ideal I of R is uncountable.


To see this, as R
I is countable, we have
R 
= an + I : an ∈ R .
I
S+∞
It follows that R = n=1 (an + I), implying that I is uncountable because other-
wise R would be countable, which is impossible. Now, to prove the assertion by
contradiction, suppose that 0 6= a ∈ R is a divisor of zero in R. Define

I := r ∈ R : ar = 0 .
It is plain that I is a nonzero ideal of R and hence it is uncountable. Also note that
aR is a nonzero ideal of R. Define the map f : aR → R I by f (ax) = x + I. The
map f is obviously onto. Suppose that f (ax1 ) = f (ax2 ). We have x1 + I = x2 + I,
implying that x1 − x2 ∈ I, whence a(x1 − x2 ) = 0. That is, ax1 = ax2 which
means f is one-to-one. Therefore, f is a one-to-one correspondence between the
uncountable set aR and the countable set R I , a contradiction. Thus, R has no
divisor of zero, and hence R is an integral domain, which is what we want. 
130 2. SOLUTIONS

2. Let S = M2 (Q). We have QI2 ≤ R ≤ S and dimQ R ≤ dimQ S ≤ 4, where I2


denotes the 2 × 2 identity matrix. Suppose that I is a nonzero left ideal of R. As R
includes QI2 , the left ideal I can be viewed as a vector space over Q and we have
dimQ I ≤ 4. Thus, there are xi ∈ I (1 ≤ i ≤ k ≤ 4) such that I = Qx1 + · · · + Qxk .
This together with QI2 ≤ R implies that I = Rx1 + · · · + Rxk . In other words,
I is finitely generated. Likewise, one can see that every right ideal of R is finitely
generated as well. To show that the condition (∗) cannot be dropped, define
 
0 G
J = ,
0 0
where G = h 21i i+∞ 1
i=1 ≤ Q is the additive group generated by 2i ’s in Q. It is easily
verified that J is a left ideal of the ring T which is not finitely generated, which is
what we want. 

3. First solution: Let G = GL2 (Q) be the multiplicative group of all 2 × 2


invertible matrices over the field Q and
   
1 1 −1 1
a= , b= .
0 −1 0 1
It is easily seen that a, b ∈ G, a2 = b2 = I2 , and ab = −I2 + 2N , where
   
1 0 0 1
I2 = , N= .
0 1 0 0
We have
(ab)n = (−1)n I2 + (−1)n−1 2nN 6= I2 ,
for all n ∈ N. That is, ab has order infinity and yet both a and b have order two,
which is what we want.

Second solution: Let G be the group of isometries of the Euclidean plane under
composition of isometries. Let A, B ∈ R2 be two distinct points in the plane and
a, b the half turns around the points A and B, respectively. If e denotes the identity
isometry, we have a2 = b2 = e. On the other hand, it is easy to see that ab is a
−−→
translation isometry along the vector 2AB, and hence ab has order infinity because
it is a translation. This is what we want, finishing the proof. 

4. (a) It is well-known that


|GLn (Zp )| = (pn − 1)(pn − p) · · · (pn − pn−1 ).
To see this, note that a matrix A ∈ GLn (Zp ) if and only if the columns of A are
linearly independent. In view of this, there are pn − 1 choices for the first column of
an arbitrary element, say A, of GLn (Zp ) (the zero column vector being excluded).
For an 1 ≤ i < n, assuming that the first i columns of an arbitrary element, say A,
of GLn (Zp ) are chosen, there are exactly pn − pi choices for the (i + 1)st column of
A, because the (i + 1)st column of A cannot be a linear combination of the first i
columns of A. Therefore, by the product rule of combinatorics, GLn (Zp ) has exactly
(pn −1)(pn −p) · · · (pn −pn−1 ) elements, as desired. Thus, |G| = (32 −1)(32 −3) = 48.
As K = Z(G) includes only scalar matrices, we see that K = {I2 , 2I2 }, where I2 is
the identity matrix, and hence |K| = 2. It is now obvious that K ≤ H ≤ G. To
calculate |H|, we note that a, b, c ∈ Z3 can be chosen independent of one another
2.13. THIRTEENTH COMPETITION 131

and that a, c can be 1 or 2 and b can be 0, 1, 2. Thus, using the product rule of
combinatorics, we have
|H| = 2 × 2 × 3 = 12.

(b) Let N = x∈G x−1 Hx. First, we claim that N is the largest subgroup of
T
H that is normal in G. We present two proofs for the claim.

First proof. Firstly, N ⊆ e−1 He = H. Secondly, N is a subgroup of G


because it is an intersection of subgroups of G, namely, x−1 Hx’s where x ∈ G. To
see that N is aTnormal subgroup of G, suppose that a ∈ N and g ∈ G are arbitrary.
As a ∈ N = x∈G x−1 Hx and xg −1 ∈ G, we see that a ∈ gx−1 Hxg −1 , yielding
g −1 ag ∈ x−1 Hx. It follows that g −1 ag ∈ x∈G x−1 Hx = N . That is, N is a
T
normal subgroup of G. Now, suppose that N1 ⊆ H is a normal subgroup of G. We
can write
N1 = x−1 N1 x ⊆ x−1 Hx,
for all x ∈ G, implying that N1 ⊆ x∈G x−1 Hx = N , as desired.
T

Second proof. We see from (a) that [G : H] = 4. Consequently, if X is used


to denote the set of left cosets of H in G, then there exist g1 = e, g2 , g3 , g4 ∈ H
such that

X = g1 H, . . . , g4 H .
Observe that G acts on X via left multiplication. That is, g(gi H) = ggi H ∈
X, where g ∈ G. Now, for a fixed g ∈ G, define the map `g : X → X by
`g (gi H) = ggi H. It is easily verified that `g is a one-to-one map which is onto
because |X| = 4 < ∞. In other words, `g defines a permutation on X, i.e., `g ∈
S(X) ∼ = S4 , where S4 denotes the symmetric group of degree four. Now, define the
map φ : G → S(X) ∼ = S4 by φ(g) = `g . As G acts on X via left multiplication, we
see that `g1 g2 = `g1 `g2 for all g1 , g2 ∈ G. This means
φ(g1 g2 ) = φ(g1 )φ(g2 ),
for all g1 , g2 ∈ G. Thus, φ is a homomorphism of groups. We claim that
\
ker φ = N = x−1 Hx ⊆ e−1 He = H,
x∈G

from which, we see that N is a normal subgroup of G which is contained in H. To


prove this last claim, suppose g ∈ ker φ. We have φ(g) = `g = `e , where e is the
identity element of G. It follows that `g (x−1 H) = `e (x−1 H) for all x ∈ G. That
is, gx−1 H =Tx−1 H, yielding xgx−1 ∈ H, and henceTg ∈ x−1 Hx for all x ∈ G. This
implies g ∈ x∈G x−1 Hx = N . Conversely, if g ∈ x∈G x−1 Hx = N , we conclude
that g ∈ ker φ. Therefore, ker φ = x∈G x−1 Hx = N , proving the claim.
T

We now show  that N= K. To this end, let n ∈ N be arbitrary. As N ⊆ H,


a1 b1
we have n = , where a1 , b1 , c1 ∈ Z3 and a1 c1 6= 0. Suppose g =
0 c1
 
a b
∈ G, where a, b, c, d ∈ Z3 with ad − bc 6= 0, is arbitrary. As N is
c d
normal in G, we must have h = g −1 ng ∈ N ⊆ H. This implies that h21 =
−aca1 −c2 b1 +acc1
ad−bc = 0, where h21 denotes the 21 entry of the matrix h. Simplifying,
132 2. SOLUTIONS

we obtain ac(c1 − a1 ) + c2 (−b1 ) = 0 for all a, b, c, d ∈ Z3 with ad − bc 6= 0. This


easily yields a1 = c1 and b1 = 0, proving that n ∈ K, which is what we want.

(c) We have shown that the map φ : G → S(X) ∼


= S4 defined by φ(g) = `g is a
homomorphism of groups and that
\
ker φ = x−1 Hx = N = K.
x∈G

In view of the First Isomorphism Theorem for groups, we can write

G ∼
= im(φ) ≤ S4 .
ker φ
G ∼
That is, K = im(φ) ≤ S4 . On the other hand, |im(φ)| = [G : K] = 24 = |S4 |.
Thus, im(φ) ∼ G ∼
= S4 , and hence K = S4 , which is what we want. 


5. Let Aν(ξ) i1 (1 ≤ i ≤ n) denote the i1 entry of the column matrix Aν(ξ). We
have
n
X n
X
   
Aν(ξ) i1 = aik ν(ξ) k1 = ai1 ν(ξ) 11 + aik ν(ξ) k1
k=1 k=2
n 
X 1 i = n,
= δi,n + δi,k−1 ξ k−1 =
ξi i < n.
k=2

In other words,
     
ξ ξ 1
 ξ2   ξ2   ξ 
ξ2
     
Aν(ξ) =  .. ..
=  = ξ  = ξν(ξ).
     
 .   .   .. 
 ξ n−1   ξ n−1   . 
1 ξn ξ n−1

That is, ν(ξ) is an eigenvector of A whose corresponding eigenvalue is ξ. 

2.13.3. General. 1. Let M be an arbitrary point on the unit circle in the


complex plane centered at the origin and A1 , . . . , An denote the vertices of a regular
n-gon which is inscribed in the circle. We have

M = exp iθ = cos θ + i sin θ,


Aj = exp iθj = cos θj + i sin θj ,
2.13. THIRTEENTH COMPETITION 133

where θ, θ1 ∈ R, and θj = θ1 + (j − 1) 2π
n (1 ≤ j ≤ n). We can write
n n
exp iθ − exp iθj 2
X X
M A2j

=
j=1 j=1
Xn

= 2 − exp i(θ − θj ) − exp i(θj − θ)
j=1
n n
X 2π j−1 X 2π j−1
= 2n − exp i(θ − θ1 ) exp −i − exp i(θ1 − θ) exp i
j=1
n j=1
n
n n
exp −i 2π exp i 2π
 
n −1 n −1
= 2n − exp i(θ − θ1 ) 2π − exp i(θ 1 − θ) 2π
exp −i n − 1 exp i n − 1
= 2n − exp i(θ − θ1 ) × 0 − exp i(θ1 − θ) × 0 = 2n.
Pn
That is, j=1 M A2j = 2n, which is what we want. 

2. Let
f (x) = an xn + an−1 xn−1 + · · · + a1 x + a0 ,
where ai ∈ Q (0 ≤ i ≤ n) and an > 0, be a polynomial with the property that f (x)
is irrational whenever x is irrational. Let p be a prime and
g(x) = lcm(a1 , . . . , an )f (x) − lcm(a1 , . . . , an )a0 − p
= gn xn + gn−1 xn−1 + · · · + g1 x − p,
where gi = lcm(a1 , . . . , an )ai (1 ≤ i ≤ n). As the leading coefficient of g is positive
and g(0) = −p < 0, it follows from the Intermediate Value Theorem that g has
a positive root which must be rational because, in view of the above equality, the
polynomial g has the property that g(x) is irrational whenever x is irrational. Now
by the Rational Root Theorem, the positive root is of the form d1 or dp for some
d dividing gn . If n > 1, it is obvious that by choosing p large enough none of the
numbers d1 and dp , where d divides gn , can be a root of g(x) = 0. Therefore, n ≤ 1,
yielding f (x) = ax + b for some a, b ∈ Q with a 6= 0. 

3. To prove the assertion, we need the following theorem which is known as


Sylvester’s problem.

Theorem. Let S be a finite set of the points of a plane such that the line
passing through any two points of S contains a third point of S. Then, all of the
points of S lie on a straight line.
Proof. To prove the assertion by contradiction, suppose that all of the points
of S are not collinear. As S is finite, there exist A, B, C ∈ S such that the smallest
height of the triangle ABC is minimal among all triangles with vertices from S.
Without loss of generality, assume that the altitude AH drawn from the vertex A
perpendicular to the base BC is the smallest height of the triangle ABC.
If H = B or H = C, then the altitude drawn from H to the base opposite to
it is smaller than AH, which is impossible. If not, then H is between B and C and
there is a third point D of S on the line BC. Thus, if necessary by renaming the
points B, C, and D, we may assume that B and C are both on the left or the right
of AH such that BH < CH. Assume, with no loss of generality, that B and C are
on the right of AH such that BH < CH. It is now easily seen that the altitude
134 2. SOLUTIONS

A A A
H’
B’

B C C
C=H B=H H B

Figure 12

drawn from B to AC is smaller than AH, a contradiction. Thus, all of the points
of S are collinear, finishing the proof. 
Remark. Motivated by Sylvester’s problem, we pose the following problem.
Let S be a finite set of the points of a plane containing no three collinear points
such that the circle passing through any three points of S contains a fourth point
of S. Then, all of the points of S lie on a circle. Hint. Perform an inversion with
center at one of the given points and any radius, say, radius one.

First, we show that there exists a plane which contains only two lines of the
given n parallel lines. To this end, intersect the n parallel lines with a plane to
obtain n points of intersection on the plane. It follows from the above theorem
that in the plane there is a line which contains exactly two points of the n points.
Now, this line and two lines of the given n parallel lines form a plane which passes
through only two lines of the n parallel lines. With this in mind, we prove the
assertion by induction on n. If n = 3, the assertion is obvious. Assuming that the
assertion holds for n − 1 parallel lines, to prove the assertion for n parallel lines, as
explained in the above find a plane that contains exactly two lines of the given n
lines. Now, one of these two lines and the n − 2 remaining lines are not coplanar.
It thus follows from the induction hypothesis that there exist at least n − 1 distinct
planes each of which includes at least two lines of these n − 1 lines. These n − 1
distinct planes together with the plane that contains only two lines of the given n
lines form n planes satisfying the desired property, which is what we want. 

2.14. Fourteenth Competition

2.14.1. Analysis. 1. To prove the assertion by contradiction, suppose that


f 0 (x0 ) exists. As f 0 (x) < 0 on (−∞, x0 ), f is strictly decreasing on (−∞, x0 ],
from which, we obtain f (x) > f (x0 ) for all x < x0 . Likewise, since f 0 (x) > 0 on
(x0 , +∞), we see that f (x) > f (x0 ) for all x > x0 . Thus, f assumes it absolute
minimum, and hence local minimum, at x0 , yielding f 0 (x0 ) = 0. Now, f 00 (x) < 0
on (x0 , +∞) implies f 0 is strictly decreasing on [x0 , +∞), which, in turn, implies
f 0 (x) < f 0 (x0 ) = 0 for all x > x0 , which is in contradiction with the hypothesis.
This shows that f 0 (x0 ) does not exist, which is what we want. 

2. We need the following lemma.


2.14. FOURTEENTH COMPETITION 135

Lemma. Let g : R → R be a uniformly continuous and nonnegative function


R +∞
on R such that −∞ g < ∞. Then
lim g(x) = 0, lim g(x) = 0.
x→−∞ x→+∞

Proof. We prove that limx→+∞ g(x) = 0. One can prove limx→−∞ g(x) = 0
in a similar fashion. Proceed by way of contradiction. Thus, there exists an ε0 > 0
such that for all n ∈ N, there is an xn > n for which

g(xn ) = g(xn ) ≥ ε0 .
From the hypothesis that g is uniformly continuous on R, we see that for ε20 > 0,
there exists a 0 < δ0 < 1 such that
g(x) − g(y) < ε0 ,

2
for all x, y ∈ R with |x − y| < δ0 . In particular, for all i ∈ N and x ∈ R with
|x − xi | < δ0 , we have
g(x) − g(xi ) < ε0 ,

2
from which, in view of |g(xi )| − |g(x)| ≤ |g(x) − g(xi )|, we obtain

ε0 g(xi ) g(xi ) ε0
g(x) = g(x) ≥ g(xi ) −
≥ = ≥ .
2 2 2 2
Thus, g(x) ≥ ε20 on (xi − δ0 , xi + δ0 ) for all i ∈ N, yielding
Z xi +δ0
ε0
g ≥ 2δ0 = ε0 δ0 .
xi −δ0 2
R xi +δ0 R xi −δ0 R xi +δ0
On the other hand, −∞ g − −∞ g = xi −δ0 g. Thus, letting i → +∞ in the
above, in view of the hypothesis, we obtain 0 ≥ ε0 δ0 , which is a contradiction.
Therefore,
lim g(x) = 0,
x→+∞
which is what we want. 

First solution: Suppose by way of contradiction that there exists an ε0 > 0 such
that for all n ∈ N there is an xn ∈ R with |xn | > n for which |f (xn )| ≥ ε0 . As f is
uniformly continuous on R, for ε20 > 0 there exists a δ0 > 0 such that
f (x) − f (y) < ε0 ,

2
whenever x, y ∈ R and |x − y| < δ0 . In particular, for all i ∈ N and |x − xi | < δ0 ,
we have
f (x) − f (xi ) < ε0 .

2
But
f (xi ) − f (x) ≤ f (x) − f (xi )
from which, we see that
f (x) ≥ f (xi ) − ε0 ≥ |f (xi )| ≥ ε0 .

(∗)
2 2 2
On the other hand, f is bounded on R. So there exists an M > 0 such that |f (x)| ≤
M for all x ∈ R. By the continuity of g on R, g is continuous on the compact
136 2. SOLUTIONS

interval [−M, M ], and hence it is uniformly continuous on [−M, M ]. Consequently,


for given ε > 0, there exists a δ1 > 0 such that

g(x) − g(y) < ε, (∗0 )
whenever x, y ∈ [−M, M ] and |x − y| < δ1 . It follows from the uniform continuity
of f on R that for the above δ1 > 0, which was obtained from the given ε > 0,
there exists a δ > 0 such that

f (x) − f (y) < δ1 ,
whenever x, y ∈ R and |x − y| < δ. Now, for all x, y ∈ R with |x − y| < δ, we have
|f (x) − f (y)| < δ1 . This together with the fact that f (x), f (y) ∈ [−M, M ] for all
x, y ∈ R with |x − y| < δ, in view of (∗0 ), implies

g(f (x)) − g(f (y)) < ε.
That is, g ◦ f is uniformly continuous on R. On the other hand, g ◦ f is integrable
on R. It thus follows from the lemma that
lim g ◦ f (x) = 0, lim g ◦ f (x) = 0.
x→−∞ x→+∞

Now, set
 ε0 ε0
m = inf g(x)|x ∈ [−M, − ] ∪ [ , M ] .
2 2
As g(x) > 0 whenever x 6= 0 and that g is continuous on the compact set [−M, − ε20 ]∪
[ ε20 , M ], we see that there exists a c ∈ [−M, − ε20 ] ∪ [ ε20 , M ] such that m = g(c), and
hence m = g(c) > 0. Now, from limx→∞ g ◦ f (x) = 0, it follows that for m > 0,
there exists an N > 0 such that
 
g f (x) = g f (x) − 0 < m,
whenever x ∈ R and |x| > N . On the other hand, we have limi |xi | = +∞, and
hence for the above N > 0, there exists an I > 0 such that |xi | > N whenever
i ≥ I. In particular, |xI | > N, implying g f (xI ) < m. But, by (∗), we have ε20 <
f (xI ) ≤ M , yielding g f (xI ) ≥ m, a contradiction. Therefore, limx→∞ f (x) = 0,
which is what we want.

Second solution: Again we proceed by contradiction and continue up to (∗) and


set  ε0 ε0
m = inf g(x)|x ∈ [−M, − ] ∪ [ , M ] .
2 2
As we saw in the first solution, there exists a c ∈ [−M, − ε20 ] ∪ [ ε20 , M ] such that
m = g(c), and hence m = g(c) > 0. It follows from (∗) that for all i ∈ N and
x ∈ R such that x ∈ (xi − δ0 , xi + δ0 ), we have M ≥ |f (x)| ≥ ε20 , implying
f (x) ∈ [−M, − ε20 ] ∪ [ ε20 , M ], whence g f (x) ≥ m > 0. Thus, we can write
Z xi +δ0
g ◦ f ≥ m(xi − δ0 − xi + δ0 ) = 2δ0 m (∗00 )
xi −δ0
On the other hand,
Z xi +δ0 Z xi +δ0 Z xi −δ0
g◦f = g◦f − g ◦ f.
xi −δ0 −∞ −∞

Thus, letting i → +∞ in (∗00 ), in view of the hypothesis, we obtain 0 ≥ 2δ0 m,


which is a contradiction. Therefore, limx→∞ f (x) = 0, which is what we want. 
2.14. FOURTEENTH COMPETITION 137

3. Define the sets A and B as follows


x ∈ [c, d] : f (x) = a = g −1 {a} ,
 
A :=
x ∈ [c, d] : f (x) = b = g −1 {b} ,
 
B :=
where g = f |[c,d] , which is continuous on [c, d] because so is f on [a, b]. It follows
that A = g −1 ({a}) and B = g −1 ({b}) are closed, and hence compact, subsets of
the compact interval [c, d]. Thus, there exist r ∈ A and s ∈ B such that

s − r = inf x − y : x ∈ A, y ∈ B .
It is plain that r, s ∈ [c, d] and r 6= s. Assume, without loss of generality, that
r < s. By showing that f ([r, s]) = [a, b], we settle the proof. To this end, note
first that f (r) = a and f (s) = b. Next, suppose that λ ∈ [a, b] is arbitrary. We
have f (r) = a ≤ λ ≤ b = f (s). From this and the Intermediate Value Theorem,
we see that there exists a c ∈ [r, s] such that f (c) = λ. That is, λ ∈ f ([r, s]),
implying [a, b] ⊆ f ([r, s]). Now, suppose that λ = f (c) ∈ f ([r, s]), where c ∈ [r, s],
is arbitrary. We need to show that λ ∈ [a, b]. Suppose to the contrary that λ ∈ / [a, b].
This implies λ < a or λ > b. If λ < a, we can write
f (c) = λ < a < b = f (s).
From this, in view of the Intermediate Value Theorem, we see that there exists
r1 ∈ (c, s) such that f (r1 ) = a. On the other hand, r ≤ c < r1 < s, f (r) = f (r1 ) =
a, f (s) = b, and |s − r1 | < |s − r|, which is in contradiction with our choice of r and
s. This implies a ≤ λ. Likewise, one can see that λ ≤ b. Thus, a ≤ λ ≤ b, yielding
f ([r, s]) ⊆ [a, b] because λ ∈ f ([r, s]) was arbitrary. Therefore, f ([r, s]) = [a, b],
which is what we want. 

2.14.2. Algebra. 1. First solution: As G is a finite p-group, we have |G| =


pn for some n ∈ N. It now follows from Sylow’s First Theorem that G has a normal
|G| G
subgroup H such that |H| = pn−1 . We have |H| = p. Thus, H is a cyclic, and
G
hence an ableian, group because p is a prime number. Since H is abelian, we obtain
G0 ⊆ H $ G, implying that G0 6= G, as desired.

Second solution: Recall that a group is solvable if and only if for some n ∈ N∪{0},
0
we have G(n) = {e}, where G(0) = G and G(n+1) = G(n) (n ∈ N ∪ {0}). It is
well-known that every finite p-group is solvable. With all that in mind, proceed
by contradiction. Suppose G0 = G. It follows that G(n) = G for all n ∈ N ∪ {0}.
Consequently, G(n) 6= {e} for all n ∈ N ∪ {0}, and hence G cannot be solvable,
which is in contradiction with the fact that every finite p-group is solvable. Thus,
G0 6= G, which is what we want. 

2. Let p be the smallest prime dividing |G| and K a normal subgroup of G with p
elements. As p is prime, it follows that K is cyclic so that K = hai for some a ∈ G
with ord(a) = p. Obviously, it suffices to prove that a ∈ Z(G). To this end, suppose
that g ∈ G is arbitrary. It follows that g −1 ag ∈ K because K is a normal subgroup
of G. Hence, there is an integer j with 0 ≤ j ≤ p − 1 such that g −1 ag = aj . Firstly,
j 6= 0 because otherwise a = e, which is impossible, for ord(a) = p. So 0 < j ≤ p−1
and we can write
aj = g −1 ag =⇒ (aj )j = g −1 aj g,
138 2. SOLUTIONS

2 n
whence aj = g −2 ag 2 . By an easy induction on n, we see that aj = g −n ag n for
p−1
all n ∈ N. In particular, if n = p − 1, we obtain aj = g −(p−1) ag p−1 . Now, since
0 < j ≤ p − 1 and p is prime, it follows that gcd(j, p − 1) = 1. This together with
p
Fermat’s Little Theorem yields j p−1 ≡ 1. That is, there exists an integer k such
that j p−1 = 1 + kp. As ord(a) = p, we have
p−1
(g p−1 )−1 ag p−1 = aj = a1+kp = a(ap )k = aek = a,
which yields
(g p−1 )−1 ag p−1 = a.
In other words, ag p−1 = g p−1 a for all g ∈ G. To finish the proof, we need to
show that the map ϕ : G → G defined by ϕ(g) = g p−1 is onto. As |G| is finite, it
suffices to show that the map ϕ is one-to-one. To this end, suppose ϕ(g1 ) = ϕ(g2 )
for some g1 , g2 ∈ G. We have g1p−1 = g2p−1 and that gcd(p − 1, |G|) = 1, for p is
the smallest prime that divides |G|. It follows that there exist r, s ∈ Z such that
r(p − 1) + s|G| = 1. Now, noting that g |G| = e for all g ∈ G, we can write
r(p−1)+s|G| (p−1) r |G| s (p−1) r |G| s r(p−1)+s|G|
g1 = g1 = g1 g1 = g2 g2 = g2 = g2 .
That is, g1 = g2 . In other words, ϕ : G → G is one-to-one, and hence onto, finishing
the proof. 

3. Recall that if R is a commutative ring and a, b ∈ R are nonzero elements of


it, then the greatest common divisor of a and b in R, denoted by gcd(a, b), is a
nonzero element d ∈ R satisfying the following. (i) d|a and d|b, that is, there exist
k1 , k2 ∈ R such that a = k1 d and b = k2 d. (ii) If d0 |a and d0 |b for some 0 6= d0 ∈ R,
then d0 |d. Also, the least common multiple of a and b, denoted by lcm(a, b), is a
nonzero element c ∈ R satisfying the following. (i) a|c and b|c. (ii) If a|c0 and b|c0
for some 0 6= c0 ∈ R, then c|c0 .
We claim that in Z6 , the ring of integers mod 6, the greatest common divisor
of 2 and 3 is 1, whereas 2 and 3 have no common multiple. First, it is obvious that
1|2 and 1|3. Next, suppose d0 |2 and d0 |3. It follows that d0 |3 − 2 = 1. Hence, 1 is
a greatest common divisor of 2 and 3. Use contradiction to prove that 2 and 3 has
no common multiple. Suppose to the contrary that for 0 6= c ∈ Z6 we have 2|c and
3|c. It follows that there exist k1 , k2 ∈ Z6 such that c = 2k1 and c = 3k2 . We can
write
3c = 6k1 = 0, 2c = 6k2 = 0,
whence c = 3c − 2c = 0 − 0 = 0. That is, c = 0, which is impossible. This proves
the claim by contradiction. 

4. First, if n ∈ N and n > 1, we show that xn y + xn−1 + 1 is irreducible in F [x, y].


Suppose to the contrary that there exist f, g ∈ F [x, y] such that xn y+xn−1 +1 = f g.
As degy (xn y+xn−1 +1) = 1, in view of the above equality, without loss of generality,
we may assume that degy (f ) = 0 and degy (g) = 1. Consequently, there exist a non-
invertible f0 ∈ F [x] and g0 , g1 ∈ F [x] such that f = f0 and g = g0 + yg1 . We can
write
xn y + xn−1 + 1 = f0 g0 + f0 g1 y,
yielding f0 g1 = xn and f0 g0 = xn−1 + 1. As f0 is not invertible, there exists 1 <
j0 < n such that f0 = xj0 and g1 = xn−j0 . From this, we obtain g0 xj0 = xn−1 + 1,
2.14. FOURTEENTH COMPETITION 139

which is impossible because j0 > 1 and n > 1. Thus, the polynomial xn y + xn−1 + 1
is irreducible in F [x, y] whenever n > 1.
Next, if n = 1, we show that xy + 1 + 1 = xy + 2 is irreducible in F [x, y] if
and only if ch(F ) 6= 2. To this end, note first that if ch(F ) = 2, then xy + 2 = xy
is reducible in F [x, y]. Now, suppose that xy + 2 is reducible in F [x, y]. We show
that ch(F ) = 2. Suppose to the contrary that ch(F ) 6= 2. It follows that 2 has an
inverse in F . Since xy + 2 is reducible in F [x, y], just as we saw in the above, there
exist a non-invertible f0 ∈ F [x] and g0 , g1 ∈ F [x] such that f = f0 , g = g0 + yg1 ,
and xy + 2 = f g. We must have
xy + 2 = f0 g0 + f0 g1 y,
from which, we obtain f0 g0 = 2 and f0 g1 = x. Now, since 2 is invertible in F , it
follows that so is f0 in F [x], a contradiction. Thus, ch(F ) = 2, which is what we
want. 

5. We may assume that V1 * V2 and V2 * V1 because otherwise, in view of


dim V1 = dim V2 , we see that V1 = V2 , in which case the assertion is easy to prove.
More precisely, pick a basis {αi }m
i=1 for V1 = V2 , where m = dim V1 , and enlarge it
to a basis {αi }m
i=1 ∪ {β }n
i i=1 for V , where m + n = dim V . It is plain that
U ⊕ V1 = U ⊕ V2 = V,
where U = hβ1 , . . . , βm i is the vector subspace spanned by βi ’s (1 ≤ i ≤ n). We
prove the assertion by induction on dim V − dim V1 + 1. If dim V − dim V1 + 1 = 1,
then dim V1 = dim V2 = dim V . Hence, for U = {0}, we obviously have U ⊕ V1 =
U ⊕ V2 = V . Assuming that the assertion holds whenever dim V − dim V1 + 1 = n,
we prove it whenever dim V − dim V1 + 1 = n + 1. As V1 * V2 and V2 * V1 , we see
that V1 ∪ V2 $ V , and hence there exists a vector β ∈ V \ (V1 ∪ V2 ). Set
V10 = V1 ⊕ hβi, V20 = V2 ⊕ hβi.
We have dim V10 = dim V20 = dim V1 + 1, yielding dim V − dim V10 + 1 = dim V −
dim V1 = n. It thus follows from the induction hypothesis that there exists a
subspace U1 such that
U1 ⊕ V10 = U1 ⊕ V20 = V.
Letting U = U1 ⊕ hβi, we obviously obtain
U ⊕ V1 = U ⊕ V2 = V,
proving the induction assertion, which is what we want. 

2.14.3. General. 1. First solution: Let A = {A1 , . . . , An } be a family


containing n distinct subsets of the set S with n elements. Define the edge-labeled
graph G with vertices in A as follows. For 1 ≤ i, j ≤ n with i 6= j, connect the
distinct vertices Ai and Aj with an edge labeled s ∈ S whenever Ai = Aj ∪ {s} or
Aj = Ai ∪ {s}. Now, define the relation ∼ on A as follows. For 1 ≤ i, j ≤ n, we
write Ai ∼ Aj whenever i = j or there exists a path in the graph G joining the
vertices Ai and Aj of the graph. It is easily verified that ∼ is an equivalence relation
on A and that the equivalence classes of the relation ∼ are connected components
of the graph G. Let {Gi }ki=1 (1 ≤ k ≤ n) denote the connected components of G.
Also, let Ti (1 ≤ i ≤ k) be a maximal tree of Gi (recall that a tree is connected
graph not having any cycle). That is, Ti is a subgraph of Gi which is a tree and that
140 2. SOLUTIONS

it is maximal as a tree in Gi (from this, we immediately see that Ti is a spanning


tree of Gi , i.e., Ti a subgraph of Gi that contains all the vertices of Gi and is a
tree). For a subgraph H of G, use L(H) to denote the set of labels of the edges
of H. We claim that L(Ti ) = L(Gi ) for all 1 ≤ i ≤ k. Obviously, it suffices to
show that L(Gi ) ⊆ L(Ti ). To this end, let s ∈ L(Gi ) be arbitrary. It follows that
s ∈ S is the label of an edge Ai Aj of the subgraph Gi . If the edge Ai Aj is already
in the tree Ti , there is nothing to prove. If not, as Ti is a maximal tree of Gi , we
see that the graph obtain by adding the edge Ai Aj to the tree Ti is not a tree.
It thus follows that the edge Ai Aj participates in a cycle whose all edges but the
edge Ai Aj come from the tree Ti . Now, s ∈ S being the label of Ai Aj , we see that
Ai = Aj ∪ {s} or Aj = Ai ∪ {s}. By symmetry, we may assume without loss of
generality, that Ai = Aj ∪ {s}. Note that s ∈ / Aj , for i 6= j. Now, going from Aj
to Ai along the other edges of the cycle, in which the edge Ai Aj participates, and
noting that s ∈ / Aj but s ∈ Ai , we see that there must be an edge Ak Al ∈ Ti , where
1 ≤ k, l ≤ n, having s as its label, because otherwise s ∈ / Ai , which is impossible.
Thus, s ∈ L(Ti ), whence L(Gi ) ⊆ L(Ti ), implying L(Gi ) = L(Ti ), as desired. It is
plain that L(Ti ) ≤ ε(Ti ), where ε(Ti ) denotes the number of the edges of Ti . On
the other hand, since Ti is a tree, by a standard theorem from the theory of graphs,
we have
ε(Ti ) = ν(Ti ) − 1,
where ν(Ti ) denotes the number of vertices of Ti . The number k being the number
of connected components of G, we can write
k
X k
X k
X
L(G) = L(Gi ) ≤ ε(Ti ) = ν(Ti ) − k,
i=1 i=1 i=1

implying
k
X
L(G) ≤ ν(Ti ) − k = ν(G) − k,
i=1
where ν(G) denotes the number of vertices of G. This implies L(G) ≤ ν(G) − k <
ν(G). Thus, there exists an s0 ∈ S such that s0 ∈ / L(G). It is now plain that the
sets
A1 ∪ {s0 }, . . . , An ∪ {s0 }
are distinct, for otherwise we must have Ai = Aj ∪ {s0 } or Aj = Ai ∪ {s0 } for some
1 ≤ i, j ≤ n with i 6= j, yielding s0 ∈ L(G), which is impossible. This completes
the proof of the assertion.

Second solution: This solution is taken from Linear Algebra Methods in Com-
binatorics by L. Babai and P. Frankl. Without loss of generality, assume that
S = {1, . . . , n}. It suffices to show that there exists an element x ∈ S such that the
sets
A1 \ {x}, . . . , An \ {x}
are all distinct because so will then be the sets
A1 ∪ {x}, . . . , An ∪ {x}.
To see this, just note that for some x ∈ S and 1 ≤ i, j ≤ n, Ai ∪ {x} = Aj ∪ {x}
if and only if Ai \ {x} = Aj \ {x}. Define the n × n matrix M = (mij ) with
2.14. FOURTEENTH COMPETITION 141

mij ∈ {0, 1} (1 ≤ i, j ≤ n) as follows



1 j ∈ Ai ,
mij =
0 j∈/ Ai .
By the hypothesis, the rows of M are all distinct. We need to show that this remains
to be the case after omitting an appropriate column from M . There are two cases
to consider. (i) det M = 0; and (ii) det M 6= 0. If det M = 0, there is a column
of M , say, column j for some 1 ≤ j ≤ n, which is linearly dependent on the other
columns of M . We see that after deleting column j of M , the remaining rows are
all distinct. Suppose on the contrary that this is not the case. That is, the matrix
obtained from deleting column j of M has two equal rows, say rows i1 and i2 for
some 1 ≤ i1 < i2 ≤ n. It follows that rows i1 and i2 of M are equal as well because
column j of M depends linearly on the other columns of M . This is a contradiction,
proving the assertion in this case. If det M 6= 0, if necessary by interchanging two
rows of M , we may assume that the first row of M is a row with the minimum
number of ones. Note that there might be several rows with the minimum number
of ones. Expanding det M by the first row, we see that m1j det M1j 6= 0, where
M1j is the matrix obtained by eliminating row 1 and column j from the matrix
M . Consequently, m1j = 1 and no two rows of M1j are the same. It thus follows
that deleting column j leaves no two equal rows. Suppose on the contrary that two
rows of the matrix obtained from deleting column j of M are equal. Since no two
rows of M1j are the same, we see that for some 1 < i ≤ n, row 1 and row i of the
matrix obtained from deleting column j of M must be equal. This easily implies
that mij = 0, for otherwise rows 1 and i of M would be equal, which is impossible.
Thus, the number of ones of row i of M is less than that of row 1, which is a
contradiction. Therefore, no two rows of the matrix obtained from deleting column
j of M are equal, which is what we want. 

2. We prove that the answer is 14 days. It is obvious that the maximum number of
the exhibition days is attained provided that there is exactly one common book for
any two days of the exhibition. With that in mind, first, we obtain the number of
books needed to exhibit the books for k days in such a way that 100 books are to
be exhibited in each day and that the exhibited books of any two days have exactly
one book in common. Then, we find the maximum number of the days subject
to the condition that the number of the books is less than or equal to 1369. In
doing so, suppose that we would like to run the exhibition for k days. First, choose
k − 1 books (b1i )ki=2 for the first day of the exhibition and set aside the book b1j to
be exhibited for a second time in the jth day. Then, choose k − 2 books (b2i )ki=3 ,
from the books not already chosen, for the second day and set aside the book b2j
(3 ≤ j ≤ k) to be exhibited for a second time in the jth day. Then, choose k − 3
books (b3i )ki=4 , from the books not already chosen, for the third day and set aside
the book b3j (4 ≤ j ≤ k) to be exhibited for a second time in the jth day. Continue
this way to finally choose one book bk−1 k , from the books not already chosen, for
the (k − 1)st day and set aside the book bk−1 k to be exhibited for a second time in
the kth day. So far, for each day we have k − 1 books to be exhibited and that the
exhibited books of any two days, say day i and day j where i < j, have exactly one
book in common, namely, the book bij . In order to exhibit 100 books per day, for
each day we need another 100 − (k − 1) = 101 − k books. So the number of the
142 2. SOLUTIONS

books needed is
k(k − 1)
(k − 1) + (k − 2) + · · · + 2 + 1 + k(101 − k) = + k(101 − k)
2
k(201 − k)
= .
2
Thus, for k = 14, the number of the books needed is 14(201−14)
2 = 1309 and for
k = 15 the number of the books needed is 15(201−15)
2 = 1395. Since we have only
1369 books, it follows that the maximum number of the days of the exhibition is
14, finishing the proof. 

3. The assertion is a quick consequence of the following lemma.

Lemma. Let F be a field and with ch(F ) = 0 or > n, K an extension of F ,


and {λ1 , . . . , λn } ⊆ K. Also, let m ∈ N ∪ {0} and c ∈ F . If
λk1 + · · · + λkn = ck−m (λm m
1 + · · · + λn ),
for each k = m, m + 1, . . . , m + n, then λi = 0 or c for all 1 ≤ i ≤ n.
Proof. We prove the assertion by induction on n. If n = 1, the proof is
obvious. Suppose that the assertion holds for all k < n. Let {λ1 , . . . , λn } ⊆ K be
a given subset as described in the lemma. There are two cases to consider.
(i) First, suppose c = 0 or λm m
1 + · · · + λn = 0. Then

λk1 + · · · + λkn = 0,
for each k = m + 1, . . . , m + n. We claim that λi = 0 for all 1 ≤ i ≤ n. Suppose,
by contradiction, that this is not the case. Let µi ∈ K (1 ≤ i ≤ l) be nonzero and
distinct such that {µ1 , . . . , µl } ∪ {0} = {λ1 , . . . , λn } ∪ {0}. It follows that there
exist n1 , . . . , nl ∈ N with n1 + · · · + nl ≤ n such that
n1 µk1 + · · · + nl µkl = λk1 + · · · + λkn = 0,
for each k = m + 1, . . . , m + n. In other words,
 m+1
· · · µm+1
   
µ1 l n1 0
.. .. ..   ..   ..  ,
 .  = 

 . . . . 
µm+n
1 · · · µm+n
l nl 0
implying that
 
1 ··· 1
µm+1 · · · µm+1 det  .. .. ..
 = 0.
 
1 l . . .
µn−1
1 ··· µn−1
l
This in turn, in view of Vandermonde’s determinant formula, implies µi = 0 for
some 1 ≤ i ≤ l or µi = µj for some 1 ≤ i < j ≤ l, a contradiction in any event.
Thus, λi = 0 for all 1 ≤ i ≤ n, which is what we want.
(ii) Next, suppose c 6= 0, and hence λm m
1 + · · · + λn 6= 0. For each k = 1, . . . , n,
let Sk denote the elementary symmetric polynomial in λ1 , . . . , λn of degree k, i.e.,
S1 = λ1 + · · · + λn , . . . , Sn = λ1 · · · λn . Obviously, we can write
xn − S1 xn−1 + · · · + (−1)n Sn = (x − λ1 ) · · · (x − λn ), (∗)
from which, we obtain
λn+m
i − S1 λn+m−1
i + · · · + (−1)n Sn λm
i = 0,
2.15. FIFTEENTH COMPETITION 143

for all 1 ≤ i ≤ n. Adding up these equations, we get


n
X n
X n
X
λn+m
i − S1 λn+m−1
i + · · · + (−1)n Sn λm
i = 0.
i=1 i=1 i=1

It now follows from the hypothesis that


n
X n
X n
X
cn λm
i − S1 c
n−1
λm n
i + · · · + (−1) Sn λm
i
i=1 i=1 i=1
n
X n
X n
X
= λn+m
i − S1 λn+m−1
i + · · · + (−1)n Sn λm
i = 0.
i=1 i=1 i=1
Thus,
n
X
λm
 n
c − S1 cn−1 + · · · + (−1)n Sn = 0,

i
i=1
yielding
cn − S1 cn−1 + · · · + (−1)n Sn = 0.
This together with (∗) implies that c = λi for some 1 ≤ i ≤ n. It is now plain that
the induction hypothesis can be applied to the set {λ1 , . . . , λn } \ {c}, completing
the proof. 

Now, to prove the assertion, just let F = K = R and c = m = 1 in the lemma


above. 

2.15. Fifteenth Competition

2.15.1. Analysis. 1. (a) Letting 0 ≤ x < y, we can write


  Z x+y Z x Z x+y Z x+y
x+y 2 2 2
2ϕ = 2 f= f+ f+ f.
2 0 0 0 x
y−x
As f is increasing on [0, +∞) and is positive, we obtain
2
x+y Z x+y Z y
y − x
Z 2 2
f ≤ f t+ dt = f.
x x 2 x+y
2

So, we have
x+y x+y
  Z x Z Z
x+y 2 2
2ϕ = f+ f+ f
2 0 0 x
x+y
Z x Z 2
Z y
≤ f+ f+ f
x+y
0 0 2
Z x Z y
= f+ f
0 0
= ϕ(x) + ϕ(y).
This proves (a).
144 2. SOLUTIONS

(b) Note first that f is integrable on any closed and bounded subinterval of
[0, +∞) because f is increasing. Consequently, ϕ is continuous on [0, +∞). On the
other hand, it easily follows from (a) that

ϕ λx + (1 − λ)y ≤ λϕ(x) + (1 − λ)ϕ(y), (∗)
k k
n

λ = 2k , 1 ≤ k ≤ 2 , and k, n ∈ N. Now, as the set 2k : k, n ∈ N, 1 ≤ k ≤
where
n
2 is dense in [0, 1], in view of the continuity of ϕ, we see that (∗) holds for all
λ ∈ [0, 1]. In other words, ϕ is convex, which is what we want. 

2. It is worth mentioning that the hypothesis g(0) = 0 is redundant. The function


g is continuous on the closed interval [0, 1], so it is bounded on [0, 1]. Hence, there
exists an M > 0 such that |g(x)| ≤ M for all x ∈ [0, 1]. Noting that 0 ≤ sin x ≤ 1
for all x ∈ [0, 1], we can write
g(x)(sin x)n M sin x

M
|fn (x)| = ≤ ≤ .
1 + nx 1 + nx n
From this, we conclude that the sequence (fn )+∞
n=1 uniformly converges to the zero
function on [0, 1]. So the assertion follows. 

3. (a) For all x, z ∈ (0, +∞), we have


2xz ≤ xf (x) + zf −1 (z).
Letting z = f (y), we obtain
2xf (y) ≤ xf (x) + yf (y),
for all x, y ∈ (0, +∞). We can write
y−x
xf (y) − xf (x) ≤ yf (y) − xf (y) =⇒ f (y) − f (x) ≤ f (y),
x
for all x, y ∈ (0, +∞). Interchanging x, y in the above, we obtain
x−y y−x
f (x) − f (y) ≤ f (x) =⇒ f (y) − f (x) ≥ f (x)
y y
for all x, y ∈ (0, +∞). This together with the preceding inequality proves (a).

(b) First, we prove that f is increasing on (0, +∞). To this end, we have
2xf (y) ≤ xf (x) + yf (y),
2yf (x) ≤ yf (y) + xf (x),
yielding  
2 xf (y) + yf (x) ≤ 2 xf (x) + yf (y) ,
for all x, y ∈ (0, +∞). Consequently,

(y − x) f (y) − f (x) ≥ 0,
for all x, y ∈ (0, +∞). This shows that f is increasing on (0, +∞). It thus follows
that f has limit from the left and limit from the right at any point and that the two
limits at any point coincide because f is increasing and onto. Thus, f is continuous
on (0, +∞). From (a), we have
y−x y−x
f (x) ≤ f (y) − f (x) ≤ f (y),
y x
2.15. FIFTEENTH COMPETITION 145

for all x, y ∈ (0, +∞). Dividing by y − x where x, y ∈ (0, +∞) with x < y, we can
write
f (x) f (y) − f (x) f (y)
≤ ≤ .
y y−x x
Letting y → x+ in the above, we obtain
f (x)
f 0 (x+ ) = ,
x
where f 0 (x+ ) denotes the right derivative of f at x. Likewise, dividing by y − x
where x, y ∈ (0, +∞) with x > y, we can write
f (x) f (y) − f (x) f (y)
≥ ≥ ,
y y−x x
from which by letting y → x− , we obtain
f (x)
f 0 (x− ) = ,
x
where f 0 (x− ) denotes the left derivative of f at x. That is, f 0 (x) exists for all
0 f (x) 0
x ∈(0, +∞)
 and moreover f (x) = x . This implies xf (x) − f (x) = 0 and hence
d f (x) xf 0 (x)−f (x)
dx x = x2 = 0 for all x ∈ (0, +∞). Therefore, there exists a c ∈ R
f (x)
such that x = c, yielding f (x) = cx for all x ∈ (0, +∞), which is what we want.



2.15.2. Algebra. 1. Let Ω = P1 , . . . , Pp+1 denote the set of all (distinct)
Sylow p-subgroups of G. The group G acts on Ω by conjugation. The kernel of
the action is
p+1
\
g ∈ G| ∀i = 1, . . . , p + 1 : g −1 Pi g = Pi =

H := NG (Pi ),
i=1

where NG (Pi ) denotes the normalizer of Pi in G. Recall that for a subset S of G,


the normalizer of S in G, denoted by NG (S), is defined by
NG (S) = g ∈ G : g −1 Sg = S .


We note that by Sylow’s Second Theorem H is a normal subgroup of G. From


G
the First Isomorphism Theorem for groups, we see that the group H is 2isomorphic
to a subgroup of the
symmetric group Sp+1 , and since p (p + 1)! but p - p + 1)!,

it follows that pα |H| or pα−1 |H|. Note that the subgroup
H ∩ Pi is a Sylow
HPi H
p-subgroup of H for each i = 1, . . . , p + 1, for = . Now, as H ∩ Pi
Pi H∩Pi
and H ∩ Pj are Sylow p-subgroups of H, by Sylow’s Second Theorem, they are
conjugate in H, and hence there exists an h ∈ H such that h−1 (H ∩ Pi )h = H ∩ Pj .
On the other hand, h ∈ H, implies h ∈ NG (Pi ) and hence
H ∩ Pi = (h−1 Hh) ∩ (h−1 Pi h) = h−1 (H ∩ Pi )h = H ∩ Pj .
That is,
H ∩ Pi = H ∩ Pj ,
146 2. SOLUTIONS

for all 1 ≤ i, j ≤ p + 1. From this, we obtain H ∩ Pi ⊆ Pj for all 1 ≤ i, j ≤ p + 1,


yielding
p+1
\
H ∩ Pi ⊆ Pj ,
j=1
for all 1 ≤ i ≤ p + 1. On the other hand,
p+1
\
Pi ⊆ H ∩ Pi ,
i=1
for all 1 ≤ i ≤ p + 1. Therefore,
p+1
\
Pj = H ∩ Pi ,
j=1
Tp+1
for all 1 ≤ i ≤ p + 1. This implies that j=1 Pj is a Sylow p-subgroup of H. Now,
Tp+1 Tp+1
if pα |H|, then j=1 Pj = pα = |Pi |, and hence j=1 Pj = Pi , implying that

Pi = Pj , a contradiction. Thus, pα−1 |H|. Consequently, a Sylow p-subgroup of
Tp+1
H must be of order pα−1 , and hence | i=1 Pi | = pα−1 , which is what we want. 

2. First, we claim that if P is a prime ideal of R and ab ∈ P for some a, b ∈ R,


then a ∈ P or b ∈ P . To see this, from ab ∈ P , we get Rab ⊆ P . Now, as the left
ideal Ra is also a right ideal of R, we see that RaR ⊆ Ra, implying RaRb ⊆ Rab.
This yields RaRb ⊆ P , from which, we obtain Ra ⊆ P or Rb ⊆ P , for P is a prime
ideal of R. This, in turn, implies a ∈ P or b ∈ P because R is unital.
Now to prove the assertion, first, suppose that x ∈ R is nilpotent. So there
exists an n ∈ N such that xn = 0. We have xn = 0 ∈ P for all prime ideals of R. In
viewTofthe above claim, we see that x ∈ P for all prime ideals of R. In other words,
x∈ P : P C R, P is prime . Conversely, suppose that x ∈ R and that xn 6= 0
for all n ∈ N. In other words, for the multiplicative set S := {xn : n ∈ N}, we have
S ∩ {0} = ∅. A standard argument using Zorn’s Lemma shows that there exists
T that P ∩ S = ∅. This yields x ∈
a prime ideal P such / P . In other words, we have
proved that if x ∈ P : P C R, P is prime , then x must be nilpotent. Therefore,
the intersection of all prime ideals of R is equal to the set of the nilpotent elements
of R, which is what we want. 

3. If we do not require the matrix AB to be a nonzero idempotent, then the


assertion is trivial. Just let B = 0. Then, AB = 0 is an idempotent, settling the
proof. However, we state and prove the following nontrivial problem.
Let D be a division ring, n ∈ N, and A ∈ Mn (D). Then, there exists a matrix
B ∈ Mn (D) such that AB is an idempotent whose rank is equal to that of A.
Let Dn denote the right vector space of all n × 1 column vectors with entries
in D; that is, the addition x + y is defined componentwise and the multiplication
of the scalar λ ∈ D into the vector x = (xi )ni=1 ∈ Dn is defined by xλ := (xi λ)ni=1 .
The members of Mn (D) can be viewed as linear transformations acting on the left
of Dn via the usual matrix multiplication; that is, we can write Mn (D) = L(Dn ),
where L(Dn ) is the ring of all right linear transformations acting on the left of
Dn . For x ∈ Dn and f ∈ (Dn )0 , where (Dn )0 = Dn denotes the dual of Dn
which is the left vector space of all 1 × n row vectors with entries in D, define the
rank-one linear transformation x ⊗ f ∈ L(Dn ) by (x ⊗ f )(y) := xf (y). Choose
2.15. FIFTEENTH COMPETITION 147

yi ∈ Dn (1 ≤ i ≤ r) such that {Ayi }1≤i≤r is a basis for the range of A. Set


xi := Ai yi and enlarge {xi }1≤i≤r to a basis B ∪ {xi }1≤i≤r for Dn , where the set
B is linearly independent. Now, let {fi }1≤i≤r be a dual subset with respect to
B ∪ {xi }1≤i≤r so that hBi ⊆ ker fi and fi (xj ) = δij for each i, j = 1, . . . , r. Let
B = y1 ⊗f1 +· · ·+yr ⊗fr . Since hBi ⊆ ker fi and fi (xj ) = δij for each i, j = 1, . . . , r,
we easily see that AB = A(y1 ⊗ f1 + · · · + yr ⊗ fr ) = x1 ⊗ f1 + · · · + xr ⊗ fr is an
idempotent whose rank is r = rank(A), proving the assertion. 

2.15.3. General. 1. We prove that a necessary and sufficient condition for


the product of two integers a, b to be divisible by their sum is that there exist
integers x, y, z such that x, y are relatively prime and that
a = x(x + y)z, b = y(x + y)z.

Sufficiency is easy. To prove necessity, from a + b ab, we see that
ab = m(a + b),
for some m ∈ Z. If one of a or b is zero, say, a = 0, then letting x = 0, y = 1,
z = b proves the assertion. So without loss of generality, assume that a and b are
nonzero. Set d = gcd(a, b) and
a b
, y= .
x=
d d
It is obvious that x and y are relatively prime. From ab = m(a + b), we obtain
dxy = m(x + y), implying x + y dxy. But xy and x + y are relatively prime because
so are x and y. Thus, x + y d, and hence there exists an integer z such that
d = (x + y)z. Form this, we obtain
a = x(x + y)z, b = y(x + y)z,
which is what we want. 
R1
2. Set β = 0 xf (x)dx. We prove that
Z 1 Z 1
2
(x − β) f (x)dx ≤ (x − α)2 f (x)dx, (∗)
0 0
R1 R1
for all α ∈ R. To this end, using 0 f (x)dx = 1 and 0 xf (x)dx = β, we can write
Z 1 Z 1
2
(x − α)2 f (x)dx = (x − β) + (β − α) f (x)dx
0 0
Z 1
= (x − β)2 f (x)dx + (β − α)2 + 2(β − α)β − 2(β − α)β
0
Z 1
= (x − β)2 f (x)dx + (β − α)2
0
Z 1
≥ (x − β)2 f (x)dx.
0

As (x − 12 )2 ≤ 1
4 on [0, 1], we can write
Z 1 Z 1
1 2 1 1
x− f (x)dx ≤ f (x)dx = .
0 2 0 4 4
148 2. SOLUTIONS

Now, in view of (∗), we have


Z 1 Z 1
1 2 1
(x − β)2 f (x)dx ≤ x− f (x)dx ≤ ,
0 0 2 4
which is what we want. 

3. (a) The desired probability is the ratio of the number of “favorable cases” to the
number of “total cases”. The number of “total cases” is obviously np . To calculate
the number of “favorable cases”, let Ai be the event that no one gets on the ith
wagon. It follows that the number of “disfavorable cases” is equal to |A1 ∪ · · · ∪ An |.
Using the inclusion-exclusion principle from combinatorics, we can write
X X
|A1 ∪ · · · ∪ An | = |Ai | − |Ai ∩ Aj | + · · · + (−1)n−1 |A1 ∩ · · · ∩ An |
i i,j
X X
p
= (n − 1) − (n − 2)p + · · · + (−1)n−1 0p
i i,j
     
n n n
= (n − 1)p − (n − 2)p + · · · + (−1)n−2 1p .
1 2 n−1

Thus, the number of “favorable cases” is equal to


     
p n p n p n−1 n
n − (n − 1) + (n − 2) + · · · + (−1) ,
1 2 n−1

and hence the desired probability is


      
1 p n p n p n−1 n
n − (n − 1) + (n − 2) + · · · + (−1) ,
np 1 2 n−1

which is what we want.

(b) First, if p < n, the probability calculated in (a) is obviously


 zero,implying
that the number of “favorable cases” is zero. This, in view of nr = n−r n
, yields
       
n p n p n p n−1 n
1 − 2 + 3 − · · · + (−1) np = 0.
1 2 3 n

Next, if p = n, then the number of wagons is equal to that of the passengers. In this
case, in order not to have any empty wagon, we must have one passenger in each
wagon. Thus, the number of “favorable cases” is equal n!. On the other hand, if
we let p = n in the above formula, which we obtained for the number of “favorable
cases”, and equate these two values, we obtain
       
p p p p p p p p
1 − 2 + 3 − · · · + (−1)p−1 p = (−1)p−1 p!,
1 2 3 p

finishing the proof. 


2.16. SIXTEENTH COMPETITION 149

2.16. Sixteenth Competition

2.16.1. Analysis. 1. As g is continuous on the compact interval [0, 1], it


follows that there exists an M > 0 such that g(x) ≤ M for all x ∈ [0, 1]. From
the continuity of g at 1 and g(1) = 0, we see that for given ε > 0, there exists a
0 < δ < 1 such that

g(x) < ε,
whenever x ∈ [0, 1] and 0 < 1 − x < δ. On the other hand, for given ε > 0, there
exists an N ∈ N such that (1 − δ)n M < ε for all n ≥ N . It thus follows that for a
given ε > 0, for all x ∈ [0, 1] and n ≥ N , we have
 
fn (x) − 0 = xn g(x) ≤ max (1 − δ)n M, sup g(x) < ε,

1−δ≤x≤1

proving the assertion. 

2. (a) Let x = [x] + 0.a1 a2 a3 . . .. We have


x + 0.1 = [x] + a0 .a01 a2 a3 . . . ,
where a0 = 0 or 1 and a01 ∈ {0, 1, . . . , 9} and hence
f (x) = f (x + 0.1) = a2 .
Hence, 0.1 is a period for f . To prove that 0.1 is the period of f , suppose to the
contrary that 0 < α < 0.1 is a period of f . If we let x1 = 0.1 − (0.1)α, we have
0.09 < x1 < 0.1. Consequently, x1 = 0.09α1 α2 . . ., where all of αi ’s are not equal to
9. It follows that f (x1 ) = 9. On the other hand, if we let x2 = α+x1 = 0.1+(0.9)α,
we have
0.0999 . . . = 0.1 < x2 < 0.19 = 0.18999 . . . .
Thus, x2 = 0.1β1 β2 . . ., where β1 ≤ 8. Hence, f (x2 ) = β2 ≤ 8, whence f (x1 ) 6=
f (x2 ), which is a contradiction. This shows that α cannot be the period of f .

(b) Using integration by parts, we can write


Z c Z 0.1 Z 0.1
xdf (x) = xdf (x) = 0.1f (0.1) − f (x)dx
0 0 0
9
X 1
= (0.1).9 − k = 0.45,
100
k=0

which is what we want. 

3. We prove the assertion in the normed linear space setting. Let X be a real
or complex normed linear space and f : X → X a uniformly continuous function.
Then, there exists a, b ∈ R+ such that
||f (x)|| ≤ a||x|| + b,
for all x ∈ X, where ||.|| denotes the norm of X. As f is uniformly continuous on X,
for ε = 1, there exists δ0 > 0 such that ||f (x) − f (y)|| < 1 whenever x, y ∈ Rn and
||x−y|| ≤ δ0 . We show that for a = δ10 and b = |f (0)|+1, we have ||f (x)|| ≤ a||x||+b
150 2. SOLUTIONS

h i
kxk
for all x ∈ X. To this end, for x ∈ X, set N = δ0 , where bracket stands for the
integer part function. We can write
||f (x)||
≤ ||f (0)|| + ||f (x) − f (0)||

≤ ||f 0 ||
N
X x  x   x 
+ f kδ0 ||x|| − f (k − 1)δ0 ||x|| + f x − f N δ0 ||x|| .

k=1
h i
||x||
From N = δ0 , we obtain

kxk
N≤ < N + 1,
δ0
yielding

x − N δ0 x <
kxk
< δ0 ,
||x|| N +1

kδ0 x − (k − 1)δ0 x =
δ0
kxk x = δ0 ≤ δ0 ,

||x|| ||x||
for all 1 ≤ k ≤ N . So, for all x ∈ X, we can write
1
kf (x)k ≤ kf (0)k + N + 1 ≤ kxk + (kf (0)k + 1),
δ0
whence
||f (x)|| ≤ a||x|| + b,
for all x ∈ X, where a, b are as in the above, which is what we want. 

2.16.2. Algebra. 1. First, recall that Inn(G) ∼ G


= Z(G) , where Z(G) denotes
the center of the group G. To see this, it is easily checked that the map f :
G −→ Aut(G) defined by fg (x) = gxg −1 is a homomorphism of groups and that
ker f = Z(G). It thus follows from the First Isomorphism Theorem for groups
that Inn(G) ∼ G
= Z(G) . Next, we need to recall that if p is a prime, then every
2
group G of order p is abelian. To prove this by contradiction, suppose that G
is not abelian. Note first that |Z(G)| > 1 because GG is a p-group (see Solution 3
of 2.11.2). Consequently, |Z(G)| = p. But then Z(G) = p, implying that G is
Z(G)
cyclic and hence G is abelian, a contradiction. Thus, G is abelian, as desired.
We now prove the assertion. To this end, from the hypothesis that [G : A] =
[G : B] = p, it follows that |A| = |B|. And since p is the smallest prime that divides
|G|, from Problem 1 of 2.9.2, we see that A and B are both normal in G. It thus
follows that AB ≤ G. By the Second Isomorphism Theorem for groups, we have
AB B
A = A∩B . So we can write

|G| [G : A] p
[G : AB] = = = .
|AB| [B : A ∩ B] [B : A ∩ B]
If [B : A∩B] = 1, we obtain A = B, which is a contradiction. Thus, [B : A∩B] = p,
and hence G = AB. We now prove that A ∩ B = Z(G). To this end, for x ∈ A ∩ B,
2.16. SIXTEENTH COMPETITION 151

as is usual, use CG (x) to denote the centralizer of the element x of G. As A and B


are abelian, we have A, B ≤ CG (x), and hence
p = [G : A] = [G : CG (x)][CG (x) : A],
p = [G : B] = [G : CG (x)][CG (x) : B],
from which, we see that CG (x) = G. That is, x ∈ Z(G), and hence A ∩ B ≤ Z(G).
Now, as
[G : A ∩ B] = [G : B][B : A ∩ B] = [G : B][AB : A] = [G : B][G : A] = p2 ,

we conclude that [G : Z(G)] = 1 or p or p2 because [G : Z(G)] [G : A ∩ B]. As G
is nonabelian, the cases [G : Z(G)] = 1 or p are impossible. Thus, [G : Z(G)] = p2 ,
from
which,
in
Gview
of [G : A ∩ B] = p2 , we see that A ∩ B = Z(G). Consequently,
Inn(G) = 2
Z(G) = p , and hence Inn(G) is abelian. This together with the

Fundamental Theorem of finite abelian groups implies Inn(G) ∼ G
= Z(G) ∼
= Zp2
∼ G ∼
or Zp ⊕ Zp . From Inn(G) = Z(G) = Zp2 , it follows that G is abelian, which is
impossible. Therefore, Inn(G) ∼ = Zp ⊕ Zp , which is what we want. 

2. There exists an n ∈ N such that (r2 − r)n = 0, for r2 − r is nilpotent. It is plain


that there exists a g ∈ Z[x] such that
0 = (r2 − r)n = rn − rn+1 g(r),
whence rn = rn+1 g(r). Setting f (x) = xn g(x)n , we have
f (r)2 = r2n g(r)2n ,
and hence we can write
rn = rn+1 g(r) = rg(r)rn = rg(r)rn+1 g(r) = rn+2 g(r)2 = r2 g(r)2 rn
= r2 g(r)2 rn+1 g(r) = rn+3 g(r)3 = · · · = rn+n g(r)n = r2n g(r)n .
That is, rn = r2n g(r)n , yielding
f (r)2 = r2n g(r)2n = r2n g(r)n g(r)n = rn g(r)n = f (r).
If f (r) = 0, then
0 = rn f (r) = rn rn g(r)n = r2n g(r)n = rn ,
implying rn = 0, which contradicts the hypothesis that r is not nilpotent. There-
fore, f (r) is a nonzero idempotent element of R, which is what we want. 

3. Remark. Adjusting the proof below one can prove that



det lcm(i, j) = n!f (1) · · · f (n),
P
where f (n) = d|n dµ(d) and µ denotes the Möbius function. Hint. Use lcm(i, j) =
ij
gcd(i,j) .

We show that the matrix A is invertible. To this end, define the matrices B = (bij )
and C = (cij ) in Mn (Q) as follows
 
ϕ(i) i=j 1 j|i
bij = , cij = ,
0 i 6= j 0 j-i
152 2. SOLUTIONS

where ϕ is the Euler’s totient function. Let C t denote the transpose of C. We


have
Xn n
X n
X
(CBC t )ij = cik (BC t )kj = cik bkk0 cjk0
k=1 k=1 k0 =1
Xn X X
= cik ϕ(k)cjk = ϕ(k)cik cjk = ϕ(k)
k=1 k|i, k|j k|gcd(i,j)

=
gcd(i, j) = aij = (A)ij ,
P
for all 1 ≤ i, j ≤ n (for a proof of d|n φ(d) = n for all n ∈ N, see Solution 1 of
2.2.2). Consequently, A = CBC t . So we can write
det A = det CBC t = det C det B det C t
= 1. det B.1 = det B = ϕ(1)ϕ(2) · · · ϕ(n),
implying that det A 6= 0. Thus, A is invertible, which is what we want. 

2.17. Seventeenth Competition

2.17.1.
Analysis. 1. To prove the assertion by contradiction, suppose that
|f 0 (x)| ≤ f (b)−f (a)

b−a for all x ∈ (a, b). There are two cases to consider.
f (b)−f (a)
(i) b−a ≥ 0.
Define the function g : [a, b] → R by g(x) = f (x) − f (b)−f
b−a
(a)
(x − a). Now, from
the contradiction hypothesis, we see that
f (b) − f (a)
g 0 (x) = f 0 (x) − ≤ 0,
b−a
for all x ∈ (a, b). Thus, g is non-increasing on [a, b]. On the other hand, we
have g(a) = f (a) = g(b), and hence f (a) = g(b) ≤ g(x) ≤ g(a) = f (a) for all
x ∈ [a, b]. Therefore, g(x) = f (a) for all x ∈ [a, b]. In other words, f (x) =
f (a) + f (b)−f
b−a
(a)
(x − a) for all x ∈ [a, b]. That is, the graph of f is a line segment,
which is a contradiction. This proves the assertion in this case.
(ii) f (b)−f
b−a
(a)
≤ 0.
Adjusting the above proof or replacing f by −f and repeating the above argu-
ment, one can prove the assertion in this case as well. 

2. Let Z x Z x Z x Z x
p(x) = p1 p3 p2 p 4 − p1 p4 p 2 p3 .
−1 −1 −1 −1
It is plain that p ∈ R[x] and p(−1) = 0. Form this, it follows that p is divisible by
x + 1. We can write
Z x Z x
p0 (x) = p1 (x)p3 (x) p2 p4 + p2 (x)p4 (x) p1 p3
−1 −1
Z x Z x
−p1 (x)p4 (x) p2 p3 − p2 (x)p3 (x) p1 p4 ,
−1 −1
2.17. SEVENTEENTH COMPETITION 153

from which, we obtain p0 (−1) = 0. Thus, p(x) is divisible by (x + 1)2 . Taking


derivative of the both sides of the above equality yields
Z x Z x
p00 (x) = (p1 p3 )0 (x) p2 p4 + (p1 p3 p2 p4 )(x) + (p2 p4 )0 (x) p1 p3
−1 −1
Z x
+(p2 p4 p1 p3 )(x) − (p1 p4 )0 (x) p2 p3 − (p1 p4 p2 p3 )(x)
−1
Z x
+(p2 p3 )0 (x) p1 p4 − (p2 p3 p1 p4 )(x),
−1

implying p00 (−1) = 0, and hence p(x) is divisible by (x + 1)3 . Finally, we can write
Z x Z x
000 00 0 00

p (x) = (p1 p3 ) (x) p2 p4 + (p1 p3 ) p2 p4 (x) + (p2 p4 ) (x) p 1 p3
−1 −1
Z x
+ (p2 p4 )0 p1 p3 (x) − (p1 p4 )00 (x) p2 p3 − (p1 p4 )0 p2 p3 (x)
 
−1
Z x
−(p2 p3 )00 (x) p1 p4 − (p2 p3 )0 p1 p4 (x),

−1
which obtains
p000 (−1) (p1 p3 )0 p2 p4 + (p2 p4 )0 p1 p3 (−1) − (p1 p4 )0 p2 p3 + (p2 p3 )0 p1 p4 (−1)
 
=
= (p1 p2 p3 p4 )0 (−1) − (p1 p2 p3 p4 )0 (−1) = 0.
Therefore, p(x) is divisible by (x + 1)4 , which is what we want. 

3. “ =⇒” Suppose that Z(f ) := {x ∈ X : f (x) = 0} = f −1 ({0}) is an open subset


of X. It follows that Z(f ) is both open and close, and hence so is Z(f )c = X \Z(f ).
Now, since X = Z(f ) ∪ Z(f )c and Z(f ) and Z(f )c are both open sets, the function
g : X → R defined by
 1
f (x) x ∈ Z(f )c
g(x) =
0 x ∈ Z(f )
is continuous on X and furthermore satisfies f = gf 2 on X. This is what we want.

“ ⇐=” Suppose that there exists a continuous function g : X → R such that


f = gf 2 . To show that Z(f ) := {x ∈ X : f (x) = 0} = f −1 ({0}) is an open subset
of X, we prove that Z(f )c := X \ Z(f ) is closed. It suffices to show that for any
sequence (xn )+∞ c c
n=1 , with xn ∈ Z(f ) , we have x∞ = limn xn ∈ Z(f ) . To this end,
2
first note that f (xn ) = g(xn )f (xn ) for all n ∈ N. As f (xn ) 6= 0 for all n ∈ N, we
see that f (xn )g(xn ) = 1 for all n ∈ N. Now, in view of the continuity of f and g,
letting n → +∞, we obtain f (x∞ )g(x∞ ) = 1, yielding f (x∞ ) 6= 0. In other words,
x∞ ∈ Z(f )c , which is what we want. 

4. Define the function g : [0, 12 ] → R by g(x) = f (x + 21 ) − f (x). We have


1 1  1
g(0) = f ( ) − f (0) = − f (1) − f ( ) = −g( ),
2 2 2
yielding g(0)g( 12 ) = −g(0)2 ≤ 0. It thus follows from the Intermediate Value
Theorem that there exists a c ∈ [0, 12 ] such that g(c) = 0. That is, f (c + 12 ) = f (c).
Letting a = c and b = c + 12 , we have b − a = 12 and f (a) = f (a), which is what we
want. 
154 2. SOLUTIONS

2.17.2. Algebra. 1. For a, b ∈ G, define a ∼ b if and only if h1 ah2 = b for


some h1 , h2 ∈ H. It is readily verify that ∼ is indeed an equivalence relation on
G. Also, for any x ∈ G, we have [x] = HxH, where [x] denotes the equivalence
class of x. With all that in mind, let e denote the identity element of G and
{x1 = e, x1 , . . . , xn } be a maximal set of nonequivalent elements of G. It follows
from the hypothesis that n > 1. We have
n
[
G = Hxi H.
i=1

This yields
n −1 n
X
G =
H x Hxi
i
X |H|2
= ,
H ∩ (x−1 Hxi ) H ∩ (x−1 Hxi )

i=1 i i=1 i
from which, together with the hypothesis, we obtain
n
2
H = H + (n − 1) H 2 .
X
G = H +
i=2
|G|
This implies [G : H] = |H| = 1+(n−1)|H|. In other words, [G : H]−(n−1)|H| = 1,
which easily yields gcd([G : H], |H|) = 1, which is what we want. 

2. The assertion is a special case of the following. Let R be a unital ring with the
property that ab = 1 implies ba = 1 whenever a, b ∈ R. Then, the ring R[x] has
the same property, i.e., if a, b ∈ R[x] and ab = 1, then ba = 1. If D is a division
ring and n ∈ N, it follows from the Rank-Nullity Theorem that for A, B ∈ Mn (D),
we have AB = In if and only if BA = In , where In denotes the identity matrix.
Thus, the ring R = Mn (F ) satisfies the above property. As a matter of fact, it
can be shown that any left or right Noetherian ring satisfies the above property.
To prove the above more general assertion, suppose that for f, g ∈ R[x] we have
f g = 1. We prove that gf = 1. Suppose to the contrary that gf 6= 1. Note
first that (gf )2 = g(f g)f = gf . Assuming that f = a0 + a1 x + · · · + an xn and
g = b0 + b1 x + · · · + bm xm , we obtain a0 b0 = 1 because f g = 1. Thus, b0 a0 = 1.
So we can write gf = 1 + cxk + · · · , where k is the least exponent of x such that
c 6= 0. Now, from (gf )2 = gf , we see that
(1 + cxk + · · · )2 = 1 + cxk + · · · ,
from which, we obtain 2c = c, yielding c = 0, which is a contradiction. Thus
gf = 1, which is what we want. 

3. We prove the counterpart of the problem for left (resp. right) finite-dimensional
vector spaces over a division ring D. So assume that V is a left (resp. right)
vector space over a division ring D and T : V −→ V a left (resp. right) linear
transformation on V . As T 2 V ⊆ T V , the linear transformation S : T V → T V
defined by S(T x) = T 2 x defines a left (resp. right) linear transformation on the
left (resp. right) vector space T V . By the Rank-Nullity Theorem, we can write
dim T V = dim S(T V ) + dim ker S.
As S(T V ) = T 2 V and ker S = ker T ∩ T V , we see that
dim(ker T ∩ T V ) = dim T V − dim T 2 V.
2.18. EIGHTEENTH COMPETITION 155

Therefore,
dim(ker T ∩ T V ) = rank(T ) − rank(T 2 ),
as desired. 

2.18. Eighteenth Competition

2.18.1. Analysis. 1. We have an = An − An−1 . This together with the


hypothesis easily yields limn AAn−1
n
= 1, and hence limn AAn−1n
= 1 because An > 0
for all n ∈ Z P
with n ≥ 0. From this, it follows that the radius of convergence of the
+∞
power series n=0 An xn is 1. In particular, the series absolutely converges for all
x ∈ (−1, 1). Now, as a0 = A0 and an < An for all n ∈ N, it follows that
n n
0 ≤ an x ≤ An x ,
P+∞ n
for all n ∈ N ∪ {0} and x ∈ (−1, 1). Thus, by the Comparison Test, n=0 an |x|
P+∞ n
converges for all x ∈ (−1, 1) because so does n=0 An |x| for all x ∈ (−1, 1). So, if
P+∞
R is the radius of convergence of the series n=0 an xn , then R ≥ 1. But R > 1 is
P+∞
impossible, for otherwise the series n=0 an must be convergent, which contradicts
the hypothesis that limn An = +∞. Therefore, R = 1, which is what we want. 

2. We prove the assertion under the weaker hypothesis that the functions f and g
are Riemann integrable on [0, 1]. In the following integrals, perform the substitu-
tions nx = t and s = t − k + 1, respectively, to obtain
Z 1 Z n
1 t
f (x)g(nx)dx = f g(t)dt
0 0 n n
n Z k
X 1 t
= f g(t)dt
n n
k=1 k−1
n Z k
X 1 s k − 1 
= f + g s + (k − 1) ds.
k=1k−1 n n n
But the period of g is one. So we can write
Z 1 Z 1
f (x)g(nx)dx = fn (s)g(s)ds,
0 0
Pn
where fn (s) = n1 k=1 f ns + k−1

n . Suppose we have proved that the sequence
+∞
R1
(fn )n=1 uniformly converges to 0 f on [0, 1]. Since g is bounded on [0, 1], we see
R1
that the sequence (fn g)+∞
n=1 uniformly converges to g 0 f on [0, 1]. Hence, we can
write
Z 1 Z 1
lim f (x)g(nx)dx = lim fn (s)g(s)ds
n 0 n 0
Z 1 Z 1 Z 1 
= lim fn (s)g(s)ds = f g(s)ds
0 n 0 0
Z 1  Z 1 
= f g ,
0 0
156 2. SOLUTIONS

proving the assertion. It remains to show that the sequence (fn )+∞ n=1 uniformly
R1
converges to 0 f on [0, 1]. To this end, as f is Riemann integrable, it follows from
Riemann’s criterion for integrability that for given ε > 0, there exists a δ > 0 such
that for any partition P ∈ P[0, 1], where P[0, 1] denotes the set of all partitions of
the interval [0, 1], with ||P || < δ, we have
n
X
U (P, f ) − L(P, f ) = (Mi − mi )∆xi < ε,
i=1

where

P : x0 = 0 < x1 < · · · < xn = 1, ||P || = max1≤i≤n ∆xi


Pn Pn
U (P, f ) = i=1 Mi ∆xi , L(P, f ) = i=1 mi ∆xi ,

mi = inf x∈[xi−1 ,xi ] f (x), Mi = supx∈[xi−1 ,xi ] f (x).

We also know that if U (P, f ) − L(P, f ) < ε for some P ∈ P[0, 1], then for any
xi−1 ≤ ξi ≤ xi , we have
Z 1
X n
f (ξi )∆xi − f < ε.



i=1 0

Note that fn (s) corresponds to the Riemann sum with respect to the uniform
partition of [0, 1] with n subintervals of the same length, i.e., ∆xi = n1 for all
1 ≤ i ≤ n, and the mid points ξi = ns + i−1 n . In view of this, for given ε > 0, find
δ > 0 from the above and let N ∈ N such that N1 < δ. Now, for each n ≥ N , let Pn
be the uniform partition of [0, 1] with n subintervals of the same length. We have
||Pn || = n1 < N1 < δ. It thus follows that
Z 1 n Z 1

fn (s) −
1 X s i − 1 
f = f + − f


0 n n n 0
i=1
X n Z 1
= f (ξi )∆xi − f


i=1 0
≤ U (Pn , f ) − L(Pn , f ) < ε.
That is,


Z 1

fn (s) − f < ε,

0

for all s ∈ [0, 1] and n ≥ N . In other words, the sequence (fn )+∞
n=1 uniformly
R1
converges to 0 f on [0, 1], which is what we want. 

3. We need the following lemma.

Lemma. If α ∈ [0,P1], then there exists a sequence (ai )+∞


i=1 with ai ∈ {0, 1}
n
a
i=1 i
(i ∈ N) such that lim = α.
n→+∞ n
2.18. EIGHTEENTH COMPETITION 157

Proof. If α = 0, let ai = 0 for all i ∈ N. If not, let


1 if i = b m

ai = α c for some m ∈ N,
0 otherwise,
where b.c stands for the integer part function. We have
Pn
i=1 ai 1 n jmk o
= card m ∈ N : ≤n
n n α
1 n o bα(n + 1)c
= card m ∈ N : m < α(n + 1) = .
n n
This obtains Pn
ai bα(n + 1)c
lim i=1 = lim = α,
n n n n
as desired. 

To prove Pthe assertion, note first that if a sequence (ai )+∞


i=1 has the property
n
a
i=1 i
that lim = α, then the property remains intact under changing a finite
n→+∞ n
number of the terms of (ai )+∞ i=1 . Now, let I be a nonempty open interval of R so
that I = (x − r, x + r) for some x ∈ I and r > 0. Clearly, g(I) ⊆ [0, 1]. Let α ∈ [0, 1]
1
be arbitrary. Choose n ∈ N such that n < ε. And choose x0 ∈ R such that the
2
first n digits of its binary expansion are the same of those of x and from its (n+1)st
digit onward its digits are equal to ai ’s. That is,
x0 = 0.x1 . . . xn a1 a2 a3 . . . ,
where x = 0.x1 . . . xn xn+1 . . .. It is obvious that x0 ∈ I and that g(x0 ) = α.
Consequently, [0, 1] ⊆ g(I), and hence g(I) = [0, 1], as desired.
For the rest, note first that any open subset G of R includes an open interval I,
implying that g(G) ⊇ g(I) = [0, 1]. This implies g(G) = [0, 1] because g(G) ⊆ [0, 1].
That is, for any open subset G of R, we have g(G) = [0, 1]. Now, define the function
h : R → R by h = f ◦ g, where the function f : [0, 1] → R is defined by

x 0 < x < 1,
f (x) = 1
2 x ∈ {0, 1}.
It is obvious that for any open subset G of R, we have
 
h(G) = f g(G) = f [0, 1] = (0, 1).
In other words, h is an open map. By proving that h has no limit at any point of
R, we show that h is not continuous on R. Suppose to the contrary that there exist
x0 , ` ∈ R such that
lim h(x) = `.
x→x0
It follows that for every ε > 0, there exists a δ > 0 such that
` − ε < h(x) < ` + ε,
whenever x ∈ (x0 − δ, x0 + δ). We have 0 ≤ l ≤ 1 because 0 < h < 1. Consequently,
we can choose ε > 0 such that (` − ε, ` + ε) ∩ (0, 1) $ (0, 1). For this ε > 0, find
δ > 0 from the above. Thus, for all x ∈ (x0 − δ, x0 + δ), we have
h(x) ∈ (` − ε, ` + ε). (∗)
158 2. SOLUTIONS

Letting G = (x0 − δ, x0 + δ), we obtain



h(G) = f g(G) = (0, 1),
from which, in view of (∗), we see that
(0, 1) ⊆ (` − ε, ` + ε),
implying (` − ε, ` + ε) ∩ (0, 1) = (0, 1), contradicting our choice of ε > 0. Therefore,
the function h has no limit at any point of R, which is what we want. 

2.18.2. Algebra. 1. The subgroup H ∩K is normal in K because K is normal


in G. Since K is simple, we have H ∩ K = K or H ∩ K = {e}, where e is the
identity element of G. If H ∩ K = K, then K ⊆ H, implying that K = H because
|G| < ∞ and H ∼ = K. To finish the proof, we show that H ∩ K 6= {e}. Suppose to
the contrary that H ∩ K = {e}. As K is normal in G, we have HK = KH, whence
HK ≤ G, and hence |HK| |G|. But

HK = H K = K 2 .

H ∩ K

In other words, |K|2 |G|, contradicting the hypothesis that the square of the order
of K does not divide that of G. Thus, H ∩ K 6= {e}, which is what we want. 

2. It suffices to show that any right ideal of R is finitely generated. To this end,
let I be a right ideal in R. If I = 0, the assertion is trivial. Suppose I 6= 0. As is
usual, use Eij to denote the 2 × 2 matrix whose ij entry is 1 and zero elsewhere.
There are two cases to consider.
(i) There exists A = a0 E11 + b0 E12 + c0 E22 ∈ I such that a0 6= 0.
Using the well-ordering principle of natural numbers, let am be the least pos-
itive integer a for which there are b, c ∈ Q such that aE11 + bE12 + cE22 ∈ I. It
is easily verified that if aE11 + bE12 + cE22 ∈ I for some a ∈ Z and b, c ∈ Q,
then am divides a. We claim that I is generated by {am E11 , E12 , E22 }. Suppose
am E11 + bE12 + cE21 ∈ I for some b, c ∈ Q. It follows that
(am E11 + bE12 + cE22 )E12 = am E12 ∈ I,
implying
1 
(am E12 ) E22 = E12 ∈ I.
am
That is, E12 ∈ I. We can write
(am E11 + bE12 + cE22 )E11 = am E11 ∈ I,
yielding am E11 ∈ I. Now, as am E11 , E12 ∈ I, we see that if A = kam E11 +
bE12 + cE22 ∈ I for some k ∈ Z and b, c ∈ Q, then cE22 ∈ I. If c = 0 for all
A = kam E11 + bE12 + cE22 ∈ I, then I is generated by {am E11 , E12 }. Otherwise,
just as we saw in the above, cE22 ∈ I for some nonzero c ∈ Q, yielding
1 
(cE22 ) E22 = E22 ∈ I.
c
That is, E22 ∈ I, in which case the right ideal I is generated by {am E11 , E12 , E22 }.
This proves the assertion in this case.
(ii) Every A ∈ I is of the form A = bE12 + cE22 , where b, c ∈ Q.
2.18. EIGHTEENTH COMPETITION 159

Suppose A0 = b0 E12 +c0 E22 ∈ I, where b0 , c0 ∈ Q are such that (b0 , c0 ) 6= (0, 0).
Set J = QA0 . It is easily verified that J is a right ideal of R. If I = J, then I is
generated by A0 . If not, then there exist b1 , c1 ∈ Q such that b1 c0 − b0 c1 6= 0 and
b1 E12 + c1 E22 ∈ I. It follows that (b0 c1 − b1 c0 )E12 = (b0 E12 + c0 E22 )(b1 E12 +
c1 E22 ) − (b1 E12 + c1 E22 )(b0 E12 + c0 E22 ) ∈ I, and hence
1 
(b0 c1 − b1 c0 )E12 E22 = E12 ∈ I.
b0 c1 − b1 c0
That is, E12 ∈ I. So we have
E12 (b0 E22 ) = b0 E12 ∈ I.
From this, we obtain c0 E22 ∈ I because b0 E12 + c0 E22 ∈ I. If c0 = 0 whenever
b0 E12 + c0 E22 ∈ I, then I is generated by E12 . If not, then c0 E22 ∈ I for some
nonzero c0 ∈ Q. This implies E22 = (c0 E22 )( c10 E22 ) = E22 ∈ I. Thus, I is
generated by {E12 , E22 } in this case. So in any event, I is finitely generated.
We now prove that every ascending chain of right ideals of R necessarily termi-
nates. To this end, let (In )+∞ n=1 be an ascending sequence of right ideals of R. Set
I = ∪+∞n=1 In . As (I n )+∞
n=1 is an ascending sequence of right ideals of R, it is easily
verified that I is a right ideal of R. It follows from the above that there are at most
three, not necessarily distinct, elements A1 , A2 , A3 ∈ I such that I = hA1 , A2 , A3 i.
Since I = ∪+∞ n=1 In , there exist n1 , n2 , n3 ∈ N such that Ai ∈ Ini for each i = 1, 2, 3.
Letting N = max(n1 , n2 , n3 ), we have Ai ∈ In for all n ≥ N and i = 1, 2, 3. This
implies

In ⊆ I = A1 , A2 , A3 ⊆ In ,
for all n ≥ N . Therefore, In = I for all n ≥ N . In other words, In = IN for all
n ≥ N , which is what we want. 

3. We need the following lemma.

Lemma. Let V be a finite-dimensional vector space over a field F and {Vi }i∈I
a family of proper subspaces of V such that |I| < |F |. Then,
[
Vi $ V.
i∈I

Proof. Without loss of generality, we may assume that {Vi }i∈I is a family
of distinct proper subspaces of V . With that in mind, we prove the assertion by
induction on dim V . If dim V = 1, we must have dim Vi = 0, implying Vi = {0} for
all i ∈ I, in which case the assertion is trivial. Assuming that the assertion holds
for any vector space V with dim V < k, we prove the assertion for any vector space
V with dim V = k. Let Vi (i ∈ I) be as in the lemma. Set

S = dim Vi |i ∈ I .
Plainly, S ⊆ N and S is nonempty and bounded from above by k = dim V . It
follows that S has a terminal element. That is, there exists i0 ∈ I such that
dim Vi ≤ dim Vi0 ,
for all i ∈ I. Set W = Vi0 , J = I \ {i0 }, and finally Wj = Vj ∩ Vi0 . Firstly, for all
j ∈ J we have Wj $ Vi0 . Suppose to the contrary that Wj = Vj ∩Vi0 = Vi0 for some
j ∈ J, then Vi0 ⊆ Vj , implying dim Vi0 ≤ dim Vj . On the other hand, dim Vj ≤
dim Vi0 , yielding Vi0 = Vj which is impossible because j 6= i0 . Thus, Wj $ W = Vi0
160 2. SOLUTIONS

for all j ∈ J. We have dim W = dim Vi0 < dim S V = k and |J| ≤ |I| < |F |. So
it follows from the induction hypothesis that j∈J Wj $ W = Vi0 . Consequently,
there exists a vector v0 ∈ Vi0 such that v0 ∈ / Wj = Vj ∩ Vi0 for all j ∈ J. That is,
v0 ∈/ Vj for all j ∈ I with j 6= i0 . On the other hand, since Vi0 $ V , there exists a
vector v1 ∈ V \ Vi0 . We claim that there exists an f0 ∈ F such that v1 + f0 v0 ∈ / Vi
for all i ∈ I. To prove this by contradiction, suppose that for each f ∈ F there
exists an if ∈ I such that v1 + f v0 ∈ Vif . Note that if f, f 0 ∈ F and f 6= f 0 , then
v1 + f v0 6= v1 + f 0 v0 . Also note that the hypothesis |I| < |F | implies that there
exist f, f 0 ∈ F with f 6= f 0 such that if = if 0 . Firstly, we observe that if 6= i0 , for
otherwise v1 + f v0 ∈ Vi0 , yielding v1 ∈ Vi0 , because v0 ∈ Vi0 , which is impossible.
Secondly, from v1 + f v0 , v1 + f 0 v0 ∈ Vif = Vif 0 , we obtain (f − f 0 )v0 ∈ Vif , yielding
v0 ∈ Vif , which, in turn, implies if = i0 , contradicting if 6= i0 as observed in
the above. That is, we obtain a contradiction in any event. Thus, there exists S an
f0 ∈ F such S that v1 + f v
0 0 ∈
/ V i for all i ∈ I. In other words, v1 + f v
0 0 ∈/ i∈I i ,
V
and hence i∈I Vi $ V , which is what we want. 

We now use the lemma to prove the assertion. From this point on, the proof is
almost identical to that of Problem 5 of 1.14.2.
Fix i0 ∈ I. We prove the assertion by induction
S on dim V − dim Vi0 . If dim V −
dim Vi0 = 1, then for U = {u}, where u ∈ V \ ( i∈I Vi ), we obviously have Vi ⊕ U =
V for all i ∈ I. Assuming that the assertion holds whenever dim V −dim SVi0 = n, we
prove it whenever dim V − dimSVi0 = n + 1. As, in view of the lemma, i∈I Vi $ V ,
there exists a vector β ∈ V \ ( i∈I Vi ). Set
Vi0 = Vi ⊕ hβi.
We have dim Vi0 = dim Vj0 = dim Vi0 + 1 for all i, j ∈ I, yielding dim V − dim Vi00 =
dim V − dim Vi0 = n for all i ∈ I. It thus follows from the induction hypothesis that
there exists a subspace U 0 such that
Vi0 ⊕ U 0 = V,
for all i ∈ I. Letting U = hβi ⊕ U 0 , we obviously obtain
Vi ⊕ U = V,
for all i ∈ I, proving the induction assertion, which is what we want. 

2.19. Nineteenth Competition

2.19.1. Analysis. 1. Define the function g : [0, 1] → R by


Z x
g(x) = 2x − 1 − f.
0
It suffices to show that g has only one zero on the interval [0, 1]. As f is continuous
on [0, 1], g is differentiable on (0, 1). Using the First Fundamental Theorem of
Calculus, we can write
g 0 (x) = 2 − f (x),
for all x ∈ (0, 1). But 0 ≤ f (x) ≤ 1 for all x ∈ (0, 1). In particular, we must have
g 0 (x) = 2 − f (x) ≥ 1 > 0 for all x ∈ (0, 1), implying that g is strictly increasing
on [0, 1]. Therefore, g has at most one zero on [0, 1]. With that in mind, note that
R1
g(0) = −1 < 0 ≤ 1 − 0 f = g(1). To see the second inequality, just note that
2.19. NINETEENTH COMPETITION 161

R1 R1
from f ≤ 1 on [0, 1], it follows that 0 f ≤ 1, yielding g(1) = 1 − 0 f ≥ 0. It thus
follows from the Intermediate Value Theorem that there exists a c ∈ (0, 1] such
that g(c) = 0. This completes the proof. 
x
2. First Solution: Note that limx→+∞ e h = +∞ whenever h > 0. With that in
mind, using L’Hopital’s rule, we can write
x x 0
e h f (x) e h f (x)
lim f (x) = lim x = lim x 0
x→+∞ x→+∞ eh x→+∞ eh
x x
1 h 0
h e f (x) + e f (x)
h
f (x) + hf 0 (x) = 0,

= lim 1 x = lim
he
x→+∞ h x→+∞

implying limx→+∞ f (x) = 0. This together with


f (x) + hf 0 (x) = 0,

lim
x→+∞

where h ∈ R , yields limx→+∞ f 0 (x) = 0, which is what we want. (We note that the
+

hypothesis that f is continuously differentiable is redundant. The differentiability


of f is enough for the assertion to be true.)

Second solution: It follows from the hypothesis that for given ε > 0, there exists
an M = M (ε) > 0 such that
f (x) + hf 0 (x) < ε ,

2
d x  ε hx
whenever x ≥ M . This easily implies dx e h f (x) < 2h
e whenever x ≥ M . We
can write
x M
Z x d t 
e f (x) − e h f (M ) = e h f (t) dt
h
M dt
Z x
ε t ε x M 
≤ e h dt = e h − e h ,
2h M 2
for all x ≥ M . Consequently, for all x ≥ M ,
f (x) − e Mh−x f (M ) < ε 1 − e Mh−x ,

2
which obtains
f (x) ≤ f (x) − e Mh−x f (M ) + e Mh−x f (M )

ε M −x  M −x
≤ 1 − e h + e h f (M )
2
ε M −x
< + e h f (M ) ,
2
for all x ≥ M . Consequently,
ε M −x  ε
lim sup f (x) ≤ lim sup + e h f (M ) = ,
x→+∞ x→+∞ 2 2
for all ε > 0. Thus, lim supx→+∞ f (x) ≤ 0, and hence limx→+∞ f (x) = 0.
As we pointed out in the first solution, this together with the hypothesis that
limx→+∞ f (x) + hf 0 (x) = 0, where h ∈ R+ , yields limx→+∞ f 0 (x) = 0, finishing
the proof. 
162 2. SOLUTIONS

3. Recall that A equipped with the uniform metric, induced by the uniform norm
of A, denoted by ||.||∞ , which is defined by

d(f, g) = ||f − g||∞ = sup f (x) − g(x) ,
x∈[0,1]

is a complete normed space or a real Banach space. Also recall that by Problem 2
of 1.6.1 the following lemma holds.
Lemma. Let X and Y be metric spaces and f, fn : X → Y continuous func-
tions such that the sequence (fn )n∈N uniformly converges to f on X. Also let
x0 , xn ∈ X be such that the sequence (xn )n∈N converges to x0 . Then, limn fn (xn ) =
f (x0 ).
With all that in mind, we now prove the assertion.

(a) Let N ∈ N be arbitrary. To show that EN is closed, suppose that f : [0, 1] →


R is a limit point of EN with respect to the topology induced by the uniform norm
of A. We show that f ∈ EN . To this end, as f is a limit point of EN , we see that
there exists a sequence (fn )n∈N in EN such that fn → f in A as n → +∞. In other
words, fn ’s uniformly converge to f on [0, 1]. Now, from fn ∈ EN for each n ∈ N,
it follows that there exists an an ∈ [0, 1] such that

fn (x) − fn (an ) ≤ N x − an ,
for all x ∈ [0, 1]. It is plain that the sequence (an )n∈N in the compact interval
[0, 1] has a subsequence (ank )k∈N converging to some a0 ∈ [0, 1]. Let bk = ank and
gk = fnk for all k ∈ N. We can write

gk (x) − gk (bk ) ≤ N x − bk ,
for all x ∈ [0, 1] and k ∈ N. Now, letting k → +∞ and using the above lemma, we
see that
f (x) − f (a) ≤ N x − a ,
for all x ∈ [0, 1]. That is, f ∈ EN , and hence EN is closed for all N ∈ N. To show
that the interior of EN is empty for all N ∈ N, we prove that the set A\EN is dense
in A. Suppose that f ∈ A is arbitrary and ε > 0 is given. From the continuity
of f on the compact interval [0, 1], we see that f is uniformly continuous on [0, 1].
Thus, there exists a δ > 0 such that
f (x) − f (y) < ε ,

2
whenever x, y ∈ [0, 1] and |x−y| < δ. Choose n ∈ N such that n1 < δ and set xi = ni ,
where 1 ≤ i ≤ n. By constructing a function g satisfying ||f −g||∞ < ε and g ∈ / EN ,
we show that A \ EN is dense in A, finishing the proof. To construct g, it suffices to
do so on any subinterval [xi−1 , xi ] (1 ≤ i ≤ n) in such a way that g is (piecewise)
continuous, ||f − g||∞ < ε, and g ∈ / EN . Here is a description of the graph of g on
the interval [xi−1 , xi ]. Let g be any function
  [xi−1 , xi ] (1 ≤ε i ≤ n) whose
on
ε
 graph
is a polygonal line in the rectangle
 xi−1 , x i  × f (xi−1 ) − 2 , f (xi ) + 2 joining
the two points xi−1 , f (xi−1 ) and xi , f (xi ) satisfying the following properties.
(i) The consecutive vertices of any segment of the polygonal line lie on the lines
y = f (xi−1 ) − 2ε and y = f (xi ) + 2ε , respectively. (ii) The absolute value of the
slope of any segment formed by any two consecutive vertices of the polygonal line
is greater than N . It is plain that for any function g : [0, 1] → R as described in
the above, we have ||f − g||∞ < ε and g ∈ / EN , which is what we want.
2.19. NINETEENTH COMPETITION 163

(b) As A is a complete metric space with respect to the metric induced by the
uniform norm of A, it follows from the Baire Category Theorem that A cannot be
written as a countable union of nowhere dense subsets of A. Recall that is subset
of a topological space X is called nowhere dense if the complement of its closure is
S EN is nowhere dense for all N ∈ N, and
dense in X. It thus follows from (a) that
hence, by the Baire Category Theorem, N ∈N EN $ A. Define the set E as follows
 [
E := f ∈ A|∃a ∈ [0, 1]  f ∈ Da = Da ,
a∈[0,1]

where Da , with a ∈ (0, 1), is the set of all functions that are differentiable at
a ∈ (0, 1); the set D0 (resp. D1 ) is defined to be the set of functions
S that are right
(resp. left) differentiable at 0 (resp. 1). We claim that E ⊆ N ∈N EN . To prove
this, suppose f ∈ E is given. It follows that there exists a ∈ [0, 1] such that f ∈ Da .
Define the function g : [0, 1] → R by
 f (x)−f (a)
x−a x ∈ [0, 1] \ {a},
g(x) = 0
f (a) x = a.
We have limx→a g(x) = limx→a f (x)−f x−a
(a)
= f 0 (a) = g(a) for all a ∈ (0, 1). Likewise,
limx→0+ g(x) = g(0) and limx→1− g(x) = g(1). Therefore, the function g is con-
tinuous on the compact interval [0, 1], and hence it is bounded on [0, 1]. So there
exists an M > 0 such that
g(x) ≤ M,
for all x ∈ [0, 1]. In other words,

f (x) − f (a) ≤ M x − a ,
for all x ∈ [0, 1]. Consequently, if we let N ∈ N be such that N ≥ M , then

f (x) − f (a) ≤ N x − a ,
S
for all x ∈ [0, 1]. That is, f ∈ EN , yielding S f ∈ N ∈N EN , proving the claim.
Now, to prove the assertion, recall that E ⊆ N ∈N EN $ A. This implies that
there exists a function f ∈ A \ E. In other words, the function f : [0, 1] → R is
continuous and yet it is nowhere differentiable because f ∈ / E. This proves (b). 

2.19.2. Algebra. 1. To prove the assertion by contradiction, suppose that


N0 6= {e} is a maximal element of A. We have NG0 ∼
= G. It follows that there exists
G
an isomorphism φ : G → N0 . Since N0 6= {e} is a normal subgroup of G and φ
is an isomorphism, we see that φ(N0 ) 6= {e} is a normal subgroup of NG0 . From
this, we obtain a normal subgroup M0 % N0 of G such that φ(N0 ) = M N0 . Thus,
0

φ|N0 : N0 → N0 is an isomorphism of groups, yielding N0 ∼


M0 M
= N00 . Now, the Third
Isomorphism Theorem for groups together with N0 = G and N0 ∼
G ∼
= M N0 implies
0

that
G
G ∼ N0 ∼ G ∼
= M0 = = G.
M0 N0
N0
In other words, G ∼
M0= G. But N0 $ M0 , contradicting the hypothesis that N0 is a
maximal element of A. Thus, the assertion follows by contradiction, which is what
we want. 
164 2. SOLUTIONS

2. (a) Note first that for any nonzero a ∈ R and any minimal left ideal J of R,
the left ideal Ja is also a minimal left ideal of R. To prove this by contradiction,
suppose that Ja is not a minimal left ideal of R for some nonzero element a ∈ R
and minimal left ideal J of R. It follows that there exists a nonzero left ideal I of
R such that I $ Ja. Define

J1 := j ∈ J|ja ∈ I .

It is plain that J1 is a left ideal of R, J1 $ J, because I $ Ja, and that J1 6= 0,


because I 6= 0. This is in contradiction with J being a minimal left ideal of R. Thus,
Ja is a minimal left ideal of R whenever J is a minimal left ideal of R and a ∈ R
is nonzero. To prove the assertion, if R has no minimal P left ideal, the assertion is
trivial because, by definition, S = 0. If not, as S = {J : J is a minimal left ideal
of R}, we see S is a left ideal in R. To see that S is a right ideal of R as well, let
r ∈ R and s ∈ S be arbitrary. We need to show that sr ∈ S. Since s ∈ S, it follows
that there are minimal left ideals J1 , . . . , Jn , where n ∈ N, and jk ∈ Jk (1 ≤ k ≤ n)
such that
s = j1 + · · · + jn ,
yielding
sr = j1 r + · · · + jn r.
But jk r ∈ P
Jk r and Jk r is a minimal left ideal of R for all 1 ≤ k ≤ n. It thus follows
that sr ∈ {J : J is a minimal left ideal of R} = S. That is, S is a right ideal of
R as well, and hence an ideal of R, as desired.

(b) It follows from (a) that


\
I ⊆ S,
ICR

for the two sided ideal S appears in the intersection. To prove the inclusion in the
opposite direction, it suffices to show that for any minimal left ideal J of R, we
have
\
J ⊆ I.
ICR

To this end, assuming that J and I are, respectively, an arbitrary minimal left ideal
and an arbitrary two-sided ideal of R, it is enough to show that J ⊆ I. To see this,
note first that
IJ ⊆ J,
for J is a left ideal of R. Secondly, IJ 6= 0 because otherwise I(JR) = (IJ)R = 0,
where I 6= 0 and JR 6= 0 are two-sided ideals of R, contradicting the hypothesis.
Thus, IJ 6= 0, and hence
0 6= IJ ⊆ J.
That is, the nonzero left ideal IJ is contained in the minimal left ideal J. This
implies IJ = J. Now, since I is a two-sided ideal of R, we see that IJ ⊆ I ∩ J,
whence J ⊆ I ∩ J, yielding J ⊆ I, which is what we want. 
2.20. TWENTIETH COMPETITION 165

3. Recall that the rank of a matrix is equal to its column rank as well as its row
rank. With that in mind, let Aj denote the jth column of the matrix A. We have
     
x1 + yj x1 1
 x2 + yj   x2   1 
Aj =   =  ..  + yj  ..  = α + yj β,
     
..
 .   .   . 
xn + yj xn 1
   
x1 1
 x2
  1 
where α =   and β =  .. . That is, Aj ∈ hα, βi, yielding hA1 , . . . , An i ⊆
   
..
 .  . 
xn 1
hα, βi. Thus, rank(A) = dimhA1 , . . . , An i ≤ dimhα, βi ≤ 2. Therefore, rank(A) ≤
2, as desired. 

4. First solution: Let A ∈ Mn (F ) be a rank one n × n matrix over F . Since


rank(A) = 1, it follows from the Rank-Nullity Theorem that the rank and nullity
of the linear transformation T : Mn×1 (F ) → Mn×1 (F ) defined by T X = AX, are,
respectively, equal to 1 and n−1. Let {αi }n−1 n−1
i=1 be a basis for ker T . Enlarge {αi }i=1
0 n n−1
to a basis B = {αi }i=1 for Mn×1 (F ), where αn is any vector in Mn×1 (F ) \ hαi ii=1 ,
where hαi in−1
i=1 denotes the vector subspace spanned by α1 , . . . , αn−1 . It is plain
that the matrix of T with respect to the basis B 0 , denoted by B, has the form
 
0 · · · 0 b1
 0 · · · 0 b2 
B = (bij )n×n =  . . . .
 
 .. · · · .. .. 
0 · · · 0 bn
On the other hand, the matrix of T with respect to the standard basis of Mn×1 (F )
is equal to A. It thus follows that there exists an invertible matrix P ∈ Mn (F )
such that B = P −1 AP , whence A = P BP −1 . We can write
det(I + A) = det(P (I + B)P −1 ) = det(I + B)
 
1 ··· 0 b1
 0 ··· 0 b2 
= det  .
 
. ..
 .. · · · ..

. 
0 ··· 0 1 + bn
= 1 + bn = 1 + tr(B) = 1 + tr(A).
That is, det(I + A) = 1 + tr(A), which is what we want.

Second solution: A proof identical to that of the second solution of Problem 7 of


1.9.3 shows that det(I + A) = 1 + tr(A), which is what we want. 

2.20. Twentieth Competition

1. If {(x, y), (z, t)} or {(x0 , y 0 ), (z 0 , t0 )} is linearly dependent, then there exist α, β ∈
C, which are not simultaneously zero, such that
θ = α(x, y) + β(z, t) = (0, 0) or θ0 = α(x0 , y 0 ) + β(z 0 , t0 ) = (0, 0).
166 2. SOLUTIONS

In this case, the assertion is trivial because θ = 0 or θ0 = 0. So suppose that


{(x, y), (z, t)} and {(x0 , y 0 ), (z 0 , t0 )} are linearly independent. Set
 0
x y0
  
x y
A= , A0 = .
z t z 0 t0
The matrices A and A0 are invertible because their rows are linearly independent.
It follows that the matrix A−1 A0 ∈ M2 (C) is also invertible. Now, suppose that
λ ∈ C is a nonzero eigenvalue of the invertible matrix A−1 A0 . We see that the
matrix λI2 − A−1 A0 is not invertible. Hence, neither is λA − A0 = A(λI2 − A−1 A0 ).
This implies that the rows of the noninvertible matrix
  0
x y0
 
x y
λA − A0 = λ −
z t z 0 t0
are linearly dependent. Thus, there exist α, β ∈ C, which are not simultaneously
zero, such that
α(λx − x0 , λy − y 0 ) + β(λz − z 0 , λt − t0 ) = (0, 0).
A straightforward calculation reveals that
λθ − θ0 = α(λx − x0 , λy − y 0 ) + β(λz − z 0 , λt − t0 ) = (0, 0),
where θ = α(x, y) + β(z, t) and θ0 = α(x0 , y 0 ) + β(z 0 , t0 ). In other words, the vectors
θ and θ0 are linearly dependent, which is what we want. 

2. Set xn = ln(1 + an ). It follows from the hypothesis that


xm+n ≤ xm + xn ,
for all m, n ∈ N. We prove that limn xnn exists. To see this, letting α = inf n∈N xnn ∈
R, we show that limn xnn = α. To this end, let ε > 0 be given. Since α = inf n∈N xnn ,
there exists a positive integer N1 = N1 (ε) such that
xN1 ε
α ≤ ≤ α+ .
N1 2
Now, for any n ≥ N1 , we can write n = N1 q + r, where q ∈ N and r ∈ N ∪ {0} with
0 ≤ r < N1 . In view of the hypothesis, we can write
xn qxN1 + rx1 qxN1 rx1
α−ε<α≤ ≤ ≤ +
n N1 q + r N1 q n
xN1 N1 x1 ε N1
< + ≤α+ + x1 .
N1 n 2 n
Now, pick N2 ∈ N such that N2 > 2ε N1 x1 . Letting N = max(N1 , N2 ), for all
n ≥ N , we have
xn ε N1
α−ε< <α+ + x1 < α + ε.
n 2 n
That is,
x
n
− α < ε,

n

xn √
for all n ≥ N . In other words, lim = α, implying that limn n 1 + an = eα ,
n n
which is what we want. 
2.20. TWENTIETH COMPETITION 167

3. It follows from the hypothesis that σ 2 = id for all σ ∈ aut(G). Consequently,


σ −1 = σ for all σ ∈ aut(G), from which we obtain
σ1 σ2 = (σ1 σ2 )−1 = σ2−1 σ1−1 = σ2 σ1 ,
for all σ1 , σ2 ∈ aut(G). In other words, aut(G) is abelian. On the other hand, just
as we saw in Solution 1 of 2.16.2 using the First Isomorphism Theorem for groups,
G ∼
we have Z(G) = In(G), where In(G) denotes the set of all inner automorphisms of
G, which is a normal subgroup of aut(G). It follows that In(G) is commutative and
hence so is Z(G)G
. This yields G0 ⊆ Z(G), where G0 denotes the derived subgroup
(a.k.a. the commutator subgroup) of G. From this, we see that G00 = {e}, implying
that G is solvable, which is what we want. 

4. Note first that f (x) > 0 for all x ∈ ( 41 , 1) because f (x) = xf (x) for all such x’s.
Next, since f (x) > 0, we see that 0 < ( 41 )f (x) < xf (x) < 1f (x) = 1, implying that
0 < f (x) < 1 for all x ∈ ( 14 , 1). Now, noting that xf (x) = f (x) and f (x) > 0 for
all x ∈ ( 14 , 1), and taking ln, we obtain f (x) ln x = ln f (x) for all x ∈ ( 14 , 1). This
implies
ln f (x)
x = e f (x) ,
ln x
for all x ∈ ( 14 , 1). Define g : (0, 1) → (0, 1) by g(x) = e x . The function g is
x lnxx
differentiable and we have g 0 (x) = 1−ln x2 e . Thus, g 0 (x) > 0 for all 0 < x < 1,
and hence g is strictly increasing on (0, 1). The function g : (0, 1) → (0, 1) is
surjective because limx→0+ g(x) = 0 and limx→1− g(x) = 1. From this, it follows
that g has an inverse, say, g −1 : (0, 1) → (0, 1), and that g −1 is differentiable
and strictly increasing on (0, 1). In particular, g −1 is continuous on (0, 1). Also,
it is easily verified that g( 12 ) = 14 . With all that in mind, define the function
ge : [0, 1] → [0, 1] by 
 0 x = 0,
ge(x) = g(x) 0 < x < 1,
1 x = 1.

It is now plain that the function ge : [0, 1] → [0, 1] is surjective, one-to-one and
continuous and that its inverse ge−1 : [0, 1] → [0, 1] is also continuous, and hence
uniformly continuous, because [0, 1] is a compact interval. Noting 0 < f (x) < 1 for
all x ∈ ( 41 , 1), we can write
 
ge f (x) = g f (x) = x,
from which, we obtain f (x) = ge−1 (x) for all x ∈ ( 14 , 1). In other words, f =
ge−1 |( 14 ,1) : ( 41 , 1) → ( 12 , 1). From this, it follows that f : ( 41 , 1) → ( 12 , 1) is uniformly
continuous because so is ge−1 : [0, 1] → [0, 1], which is what we want. 

5. Let I 6= 0 and Z, respectively, denote the intersection of all nonzero ideals of


R and the set of all zero divisors of R including zero. By showing that Z is a
maximal ideal of R and that Z = I, we settle the proof, for, from this, it follows
that Z = I is the only nontrivial ideal of R. First, we show that I ⊆ Z. Suppose
/ Z. We see that x20 6= 0
to the contrary there exists a nonzero x0 ∈ I such that x0 ∈
because x0 is not a zero divisor of R. As R is unital and I is the intersection of
all nonzero ideals of R, we obtain I ⊆ x0 R 6= 0 and I ⊆ x20 R 6= 0. On the other
hand, x0 R ⊆ I and x20 R ⊆ I, for x0 ∈ I. It thus follows that I = x0 R = x20 R.
In particular, we must have x0 ∈ x20 R, which yields x0 = x20 r for some r ∈ R.
168 2. SOLUTIONS

This obtains x0 (1 − x0 r) = 0, which, in turn, implies 1 = x0 r ∈ I, for x0 ∈ / Z, a


contradiction. Thus, {0} 6= I ⊆ Z, and hence Z 6= {0}.
Next, we show that Z is a maximal ideal of R. To this end, let x ∈ Z \ {0}
and Ann(x) = {r ∈ R : rx = 0}. Note that Z ⊆ Ann(x) because Z 2 = 0. On
the other hand, Ann(x) ⊆ Z whenever x 6= 0. It thus follows that Ann(x) = Z
whenever x ∈ Z \ {0}. Now, let 0 6= x0 ∈ I ⊆ Z = Ann(x) be arbitrary. As we saw
in the above Rx0 = I. It is plain that the map φ : R → Rx0 defined by φ(r) = rx0
is an epimorphism of the left modules R and Rx0 . From the First Isomorphism
R ∼ ∼ R ∼ R
Theorem for modules, we see that ker φ = Rx0 , implying that Rx0 = Ann(x) = Z
because ker φ = Ann(x) = Z. We now show that Rx0 is a simple left module,
i.e., it has no nontrivial submodules. To this end, suppose that M 6= 0 is a left
submodule of Rx0 = I. Choose a nonzero y0 ∈ M . As M ≤ Rx0 = I, there exists
an r0 ∈ R such that y0 = r0 x0 . We have y0 R ⊆ I because x0 ∈ I. On the other
hand, I ⊆ y0 R, for R is unital and I is the intersection of all nonzero ideals of R.
Therefore, I = y0 R. Consequently, we have M ⊆ I and I = y0 R ⊆ M because
y0 ∈ M , yielding M = I = y0 R. This proves that Rx0 = I is simple as a left
module. This together with Rx0 ∼ = R Z implies that Z is a maximal left ideal, and
hence a maximal ideal of R because R is commutative.
Finally, we show that Z ⊆ I. To this end, suppose that 0 6= x0 ∈ Z is
arbitrary. We have x0 R 6= {0}, from which, we obtain {0} 6= I ⊆ x0 R. Now,
choose an arbitrary nonzero element y0 ∈ I. It follows that there exist r0 ∈ R such
that r0 x0 = y0 ∈ I ⊆ Z. Note that r0 ∈ / Z, for otherwise y0 = r0 x0 = 0 which is
impossible. If r0 = 1, then x0 = y0 ∈ I yielding Z ⊆ I, as desired. If r0 6= 1, we
obtain r0 R + Z = R because Z is a maximal ideal and r0 ∈ / Z. Consequently, there
exist r1 ∈ R and x1 ∈ Z such that r0 r1 + x1 = 1, whence 1 − r1 r0 = x1 ∈ Z. On
the other hand, x0 ∈ Z. Since Z 2 = {0}, we see that x0 (1 − r1 r0 ) = 0. This yields
x0 = x0 r0 r1 = y0 r1 , which, in turn, implies x0 ∈ I because y0 ∈ I. That is, x0 ∈ I,
and hence Z ⊆ I, for x0 was arbitrary. Therefore, Z = I and Z is a maximal ideal
of R, proving the assertion, which is what we want. 

6. First solution: We need the following lemmas.

Lemma 1 (Cauchy’s criterion for the convergence of improper inte-


grals). Let f, α : [a, +∞) → R be two functions such that α is increasing on
[a, +∞) and f is integrable with respect to α in the sense of Riemann-Stieltjes, i.e.,
f ∈ R(α), on any closed interval [a, x], where x ∈ R with x > a. Then, a necessary
R +∞
and sufficient condition for the convergence of a f dα is that for every ε > 0,
there exists an x0 > a such that
Z x2


f dα < ε,

x1

whenever x2 > x1 > x0 . 


As the lemma is standard, we omit its proof. The interested reader may consult
standard books on analysis to see a proof of the lemma. The following lemma is
also standard. We however include a proof for the reader’s convenience.

Lemma 2 (Second Mean Value Theorem for integrals). Let g :


[a, b] → R be a Riemann integrable function on [a, b].
2.20. TWENTIETH COMPETITION 169

(a) If f : [a, b] → R is a nonnegative and decreasing function on [a, b], then


there exists a number ξ ∈ [a, b] such that
Z b Z ξ
f g = f (a) g.
a a

(b) If f : [a, b] → R is a nonnegative and increasing function on [a, b], then


there exists a number η ∈ [a, b] such that
Z b Z b
f g = f (b) g.
a η

Proof. (a) Set M = supx∈[a,b] |g(x)|. Obviously, g(x) + M ≥ 0 for all x ∈ [a, b].
With that in mind, suppose that P ∈ P[a, b] is a partition of the closed interval
[a, b] which is given by
P : x0 = a < x1 < · · · < xn = b.
We can write
Z b n Z
X xi n
X Z xi
f (g + M ) = f (g + M ) ≤ f (xi−1 ) (g + M )
a i=1 xi−1 i=1 xi−1

Xn Z xi n
X
= f (xi−1 ) g+M f (xi−1 )(xi − xi−1 ).
i=1 xi−1 i=1
Rx
Now, let G : [a, b] → R be defined by G(x) = a g. As G(a) = 0 and f is
nonnegative and decreasing on [a, b], we have
Xn Z xi Xn

f (xi−1 ) g = f (xi−1 ) G(xi ) − G(xi−1 )
i=1 xi−1 i=1
n
X n−1
X
= f (xi−1 )G(xi ) − f (xi )G(xi )
i=1 i=1
Xn

= f (xi−1 ) − f (xi ) G(xi ) + f (b)G(b)
i=1
n
X 
≤ f (xi−1 ) − f (xi ) sup G(x) + f (b) sup G(x)
i=1 x∈[a,b] x∈[a,b]

= f (a) − f (b) sup G(x) + f (b) sup G(x)
x∈[a,b] x∈[a,b]

= f (a) sup G(x).


x∈[a,b]

So we can write
Z b n
X
f (g + M ) ≤ f (a) sup G(x) + M f (xi−1 )(xi − xi−1 ).
a x∈[a,b] i=1

On the other hand, by a result due to Darboux, we have


Xn Z b
lim f (xi−1 )(xi − xi−1 ) = f,
||P ||→0 a
i=1
170 2. SOLUTIONS

where ||P || = max1≤i≤n (xi − xi−1 ). So letting ||P || → 0 in the above inequality,
we obtain Z b Z b
f (g + M ) ≤ f (a) sup G(x) + M f,
a x∈[a,b] a
implying
Z b
f g ≤ f (a) sup G(x).
a x∈[a,b]
Replacing g by −g in the above inequality, we obtain
Z b
f (−g) ≤ f (a) sup −G(x),
a x∈[a,b]

yielding
Z b
f g ≥ f (a) inf G(x).
a x∈[a,b]
These inequalities together with the facts that f (a) ≥ 0 and that G is continuous,
in view of the Intermediate Value Theorem, imply that there exists a number
ξ ∈ [a, b] such that
Z b Z ξ
f g = f (a)G(ξ) = f (a) g,
a a
which is what we want.
(b) A proof similar to that of (a) settles (b). However, here is a slick proof of
(b) using (a). Define functions f1 , g1 : [0, b − a] → R as follows
f1 (x) = f (b − x), g1 (x) = g(b − x).
It is plain that f1 and g1 satisfy the hypotheses of (a). It thus follows from (a) that
there exists ξ ∈ [0, b − a] such that
Z b−a Z ξ
f1 g1 = f1 (0) g1 .
0 0
R b−a Rξ
That is, 0 f (b − x)g(b − x)dx = f (b) 0 g(b − x)dx. Performing the substitution
b − x = t, we easily obtain
Z b Z b
f g = f (b) g,
a η

where η = b − ξ, as desired. 
Rp
Now, to prove the assertion, let M = 0 |g|. If M = 0, the assertion is trivial
Rb
because we would have a f g = 0 for all b > a, from which, the assertion trivially
follows. So suppose M > 0. As limx→+∞ f (x) = 0 and f is decreasing on [0, +∞),
we see that f (x) ≥ 0 on [0, +∞) and that for given ε > 0, there exists an x0 > 0
such that ε
f (x) = f (x) < ,
2M
whenever x ≥ x0 . It now follows from Lemma 2 that for all x0 < x1 < x2 , there
exists ξ ∈ [x1 , x2 ] such that
Z x2 Z ξ Z
ξ


f g = f (x1 ) g = f (x1 ) g .


x1 x1 x1
2.20. TWENTIETH COMPETITION 171

Since g is periodic with period p, so is |g|. And since


Z p
g = 0,
0
we see that Z jp Z ip
Z p
g = 0, g = g ,
ip (i−1)p 0
for all i, j ∈ N. Now, set T = {kp : k ∈ N} ∩ [x1 , ξ]. It is plain that T is a finite set.
For x0 < x1 < x2 , there are two cases to consider. (i) T = ∅. (ii) T 6= ∅.
First, if T = ∅, then we have [x1 , ξ] ⊆ [(i − 1)p, ip] for some i ∈ N. So we can
write
Z x2 Z
ξ Z ξ

f g = f (x1 ) g ≤ f (x ) g

1


x1 x1 x1
Z ip Z p
ε g = ε

g = ε M = ε < ε.


2M (i−1)p 2M 0 2M 2
That is, Z
x2


f g < ε.
x1
Next, if T 6= ∅, let ip and jp be the closest points of T to x1 and ξ, respectively.
Note that i, j ∈ N and i ≤ j. We can write
Z x2 Z Z Z ξ
ξ ip Z jp
f g = f (x1 ) g = f (x1 ) g+ g+ g


x1 x1 x1 ip jp
Z Z ξ !
ip Z ξ
ε
Z ip

= f (x1 ) g+ g ≤ g + g .

x1 jp 2M x1 jp

But [x1 , ip] ⊆ [(i − 1)p, ip] and [jp, ξ] ⊆ [jp, (j + 1)p]. So we can write
Z x2 Z ip Z (j+1)p !
ε
f g ≤ g + g

x1 2M (i−1)p jp
Z p Z p 
ε
g = ε (M + M ) = ε.

= g +
2M 0 0 2M
That is, Z
x2


f g ≤ ε.
x1
Thus, in any case, Z
x2


f g < 2ε,
x1
R +∞
and hence from Lemma 1, it follows that 0
f g converges, which is what we want.
Rx
Second solution: Let G : [0, +∞) → R be defined by G(x) = 0
g. It follows
R x+p
from the hypothesis that x g = 0, and hence
Z x Z x−np
G(x) = g= g,
np 0
172 2. SOLUTIONS


where n = xp and b.c denotes the integer part function. Consequently, G(x) ≤
 
Rp
g = M for all x ≥ 0. Thus, the function G is bounded. Now, we prove that the
0
assertion holds true under this hypothesis as well. That is, the assertionR will remain
p
true if in the problem one replaces “g is a p-periodic function such that 0 g = 0” by
Rx
“the function G : [0, +∞) → R, defined by G(x) = 0 g, is bounded”. It is worth
mentioning that the idea of this proof is very much like that of Dirichlet’s Test for
convergent series. Just as in the first R y solution,
it suffices to show that for given
ε > 0 there is an N > 0 such that x f g < ε whenever x, y ∈ R with N < x < y.
By the First Fundamental Theorem of Calculus, G0 = g. So we can write
Z y Z y Z y Z y
0
y
f g = f G = f dG = f G − Gdf
x

x x x x
Z y Z y

≤ |f (y)G(y) − f (x)G(x)| + Gd(−f ) ≤ 2M f (x) + Gd(−f )
x x
Z y

≤ 2M f (x) + M d(−f ) = 2M f (x) + M f (x) − f (y)
x
≤ 3M f (x).
ε
But limx→+∞ f (x) = 0. Thus, there is an N > 0 such that f (x) < 3M whenever
x > N . Therefore, Z y


f g < ε,
x
R +∞
whenever N < x < y, and hence 0 f g converges, as desired. 

2.21. Twenty First Competition

2.21.1. Analysis. 1. (a) Suppose α is a limit point of the set S such that for
every ε > 0 the set {x ∈ S : |f (x)| ≥ ε} is finite. For given ε > 0, let

δ = min α − x : f (x) ≥ ε .
It is easily verified that |f (x)| < ε whenever 0 < |x − α| < δ. This proves the
assertion.

(b) To prove the assertion by contradiction, suppose there exist an ε > 0 and
an infinite sequence (xi )+∞
i=1 of S such that |f (xi )| ≥ ε for all i ∈ N. Since the set
S is compact, the subset {xi }+∞
i=1 has a limit point in S, say, α ∈ S. It is obvious
that limx→α f (x) 6= 0, a contradiction. Thus, the assertion follows. 

2. (a) To prove the assertion by contradiction, suppose that there exists an ε0 > 0
such that to each n ∈ N, there corresponds an xn ∈ R with xn > n and a tn ∈ K
satisfying
g(xn + tn ) − g(xn ) ≥ ε0 ,
for all n ∈ N. Since K is compact, if necessary, by passing to a subsequence of
(tn )+∞
n=1 , we may assume that limn tn = t0 for some t0 ∈ K. Define f : [0, +∞) → R
by f (x) = g(x + t0 ) − g(x). As limx→+∞ f (x) = 0 and g is continuous, we easily
see that f is uniformly continuous on [0, +∞). Thus, for given ε0 > 0, there exists
a δ0 > 0 such that
g(x + t0 ) − g(y + t0 ) = f (x) − f (y) < ε0 ,

2
2.21. TWENTY FIRST COMPETITION 173

whenever x, y ∈ [0, +∞) and |x − y| < δ0 . On the other hand, it follows from the
hypothesis that for given ε0 > 0, there exists an M > 0 such that
g(x + t0 ) − g(x) < ε0 ,

2
whenever x > M . Now, since K is compact, and hence bounded, and limn tn = t0 ,
choose n ∈ N large enough so that xn + tn − t0 > 0, xn > M , and |tn − t0 | < δ0 ,
where M > 0 and δ0 are as in the above. We can write

ε0 ≤ g(xn + tn ) − g(xn )

≤ g (xn + tn − t0 ) + t0 − g(xn + t0 ) + g(xn + t0 ) − g(xn )
ε0 ε0
< + = ε0 ,
2 2
implying ε0 < ε0 . So the assertion follows by way of contradiction.
R 
x+1
(b) To prove limx→+∞ x g(u)du − g(x) = 0, note first that by the Mean
Value Theorem for integrals, for all x ∈ R, there exists a tx ∈ [0, 1] such that
Z x+1
g(u)du = g(x + tx ).
x
Now, letting K = [0, 1] in (a), for given ε > 0, find M > 0 from (a). We have
Z x+1


g(u)du − g(x) = g(x + tx ) − g(x) < ε,
x
R 
x+1
whenever x > M . That is, limx→+∞ x g(u)du − g(x) = 0, as desired.
g(x) g(x)
To prove limx→+∞ x = 0, it suffices to show that limx→+∞ [x] = 0, where
[.] denotes the integer part function, for limx→+∞ [x]
= 1. To this end, note that
x
we can write   
g(x) g [x] + tx − g [x] g [x]
= + ,
[x] [x] [x]
where tx = x − [x] ∈ [0, 1) for all x ∈ R  with x > 1. We need to show that
g [x]+tx −g [x] g [x]
limx→+∞ [x] = limx→+∞ [x] = 0. To this end, letting K = [0, 1] in
(a), for given ε > 0, there exists an M > 0 such that

g(x + t) − g(x) < ε,
whenever x > M and t ∈ [0, 1]. In particular, if x > M + 1, then [x] > M , implying
that 
g [x] + tx ) − g([x] < ε,

because tx ∈ [0, 1]. Thus, limx→+∞ g(x) − g([x]) = 0, and hence
 
g [x] + tx − g [x]
lim = 0.
x→+∞ [x]

g [x]
To prove limx→+∞ [x] = 0, it suffices to show that limn→+∞ g(n) n = 0. To see
this, note that part (ii) of the lemma presented in Solution 3 of 2.7.1, which is
known as Stolz’s Second Theorem, together with the hypothesis yields
g(n) g(n) − g(n − 1) 
lim = lim = lim g(n) − g(n − 1) = 0,
n→+∞ n n→+∞ n − (n − 1) n→+∞
174 2. SOLUTIONS

g(x)
as desired. Therefore, limx→+∞ x = 0, which is what we want. 

3. We need the following lemma which is (essentially) due to Leo M. Levine (1977).

Lemma. Let f : (a, b) → R be a bounded function. Let D, L, and R denote


the set of points at which f is discontinuous, has a left hand limit, and has a right
hand limit, respectively. Then, D ∩ (L ∪ R) is countable.
Proof. Define ω : (a, b) → [0, +∞) by
  
ω(x) = lim+ sup f [x − δ, x + δ] − inf f [x − δ, x + δ] .
δ→0
S
It is readily verified that x ∈ D if and only if ω(x) > 0. This implies D = n∈N Dn ,
where Dn = {x ∈ (a, b) : ω(x) > n1 }, and hence
[ [ [ 
D ∩ (L ∪ R) = (Dn ∩ L) (Dn ∩ R) .
n∈N n∈N
Thus, to establish the lemma, it suffices to show that Dn ∩ L and Dn ∩ R are
countable for all n ∈ N. To this end, suppose x0 ∈ Dn ∩ L (resp. x0 ∈ Dn ∩ R). As
x0 ∈ L (resp. x0 ∈ R), there exists a δ > 0 such that |f (x) − f (x− 1
0 )| < 2n (resp.
+ 1
|f (x) − f (x0 )| < 2n ) whenever x ∈ (x0 − δ, x0 ) (resp. x ∈ (x0 , x0 + δ)). From this,
we obtain
f (x1 ) − f (x2 ) < 1 ,

n
whenever x1 , x2 ∈ (x0 − δ, x0 ) (resp. x1 , x2 ∈ (x0 , x0 + δ)). Therefore, ω(x) ≤ n1 for
all x ∈ (x0 − δ, x0 ) (resp. x ∈ (x0 , x0 + δ)), implying x ∈ / Dn . It thus follows that
any point of Dn ∩ L (resp. Dn ∩ R) is the right (resp. left) end point of an open
interval which contains no point of Dn ∩ L (resp. Dn ∩ R). But these intervals are
obviously disjoint and hence form a countable set. So Dn ∩ L (resp. Dn ∩ R) is
countable for all n ∈ N, as desired. 

It is now obvious that the lemma together with Lebesgue’s Integrability Cri-
terion for Riemann integrals implies that a bounded function f : [a, b] → R is
integrable in the Riemann sense if and only if [a, b] \ (L ∪ R) is a set of measure
zero. This clearly proves the assertion. It is also worth mentioning that, in view
of the lemma, one can mimic the second proof presented for Solution 4 of 2.9.1 to
give a direct proof of the assertion. We omit the details for the sake of brevity. 

2.21.2. Algebra. 1. It is easily checked that if I, J, K are ideals of R with


s s
J ⊆ I, then I ∩ (J + K) = J + I ∩ K. Let A ⊆ B and C ⊆ D. We prove that
s
A+C ⊆ B +D. To see this, suppose E is an ideal of R such that A+C +E = B +D.
We need to show that E = B + D. To this end, we can write
B ∩ (A + C + E) = B ∩ (B + D).
But A ⊆ B and B ⊆ B + D. So, we have
A + B ∩ (C + E) = B,
s
implying that B ∩ (C + E) = B because A ⊆ B. Thus, B ⊆ C + E. Likewise,
from D ∩ (A + C + E) = D ∩ (B + D), we obtain D ⊆ A + E. Note that A ⊆ B
and B ⊆ C + E. Thus, A ⊆ C + E, and hence B + D = A + C + E = C + E.
2.21. TWENTY FIRST COMPETITION 175

Now, from C + E = B + D, we obtain D ∩ (C + E) = D ∩ (B + D), yielding


s
C + (D ∩ E) = D, which, in turn in view of C ⊆ D, implies D ∩ E = D. Therefore,
D ⊆ E. Analogously, since C ⊆ D ⊆ A + E, we have B + D = C + A + E = A + E,
s
yielding B ∩(A+E) = B ∩(B +D). Hence, A+B ∩E = B, which in view of A ⊆ B,
implies B ∩E = B. Therefore, B ⊆ E. It follows that B +D ⊆ E +E = E ⊆ B +D.
In other words, E = B + D, as desired. 

2. First we need to recall that if p is a prime, then every group G of order p2 is


abelian (for a proof see Solution 1 of 2.16.2).
We now prove the assertion. From H ≤ Z(G) ≤ G, we see that
p2 = [G : H] = [G : Z(G)][Z(G) : H],
implying that [G : Z(G)] = 1 or p or p2 . If [G : Z(G)] = 1, then G = Z(G). In
other words, G is abelian and hence G0 = {e}, which is cyclic. If [G : Z(G)] = p,
G
then Z(G) is cyclic, implying that G is abelian. This again yields G0 = {e}, which
G
is cyclic. Finally, if [G : Z(G)] = p2 , then Z(G) is abelian because every group of
2
order p is abelian whenever p is a prime. It thus follows from the Fundamental
G
Theorem of finite abelian groups that Z(G) is either cyclic or there are x, y ∈ G
G G
such that Z(G) = hxZ(G), yZ(G)i. Just as we saw in the above, if Z(G) is cyclic,
then G is abelian, in which case the assertion trivially holds. So assume that
G
Z(G) = hxZ(G), yZ(G)i for some x, y ∈ G. It follows that there is a z ∈ Z(G) such
that xy = yxz, for Z(G) G
is abelian. Note that z = x−1 y −1 xy ∈ Z(G). By proving
that G0 = hx−1 y −1 xyi = {z k : k ∈ Z}, we complete the proof. A straightforward
induction on i + j reveals that
xi y j = y j xi z ij ,
for all i, j ∈ N. Also, as yx = xyz −1 and y −1 x = xy −1 z, inducting on i + j, we
obtain
y j xi = xi y j z −ij , y −j xi = xi y −j z ij ,
G
for all i, j ∈ N. Again, as Z(G) = hxZ(G), yZ(G)i is abelian, for all g ∈ G there are
i, j ∈ N and zg ∈ Z(G) such that g = xi y j zg . Therefore, for given g, g 0 ∈ G, there
0 0
are i, j, i0 , j 0 ∈ N and zg , zg0 ∈ Z(G) such that g = xi y j zg and g 0 = xi y j zg0 . We
can write
0 0  0 0
g −1 g 0−1 gg 0 = y −j x−i y −j x−i xi y j xi y j

0 0  0 0 0 0
= z −ij x−i y −j y −j x−i xi y j y j xi z i j

0 0 0 0 0 0
= x−i y −(j+j ) x−i xi y j+j xi z i j −ij
0 0 0 0 0
)i0 i0 j 0 −ij
= x−i y −(j+j ) x−i xi xi y j+j z −(j+j z
0 0 0
= x−i y −(j+j ) xi y j+j z −j(i+i )

0 0 0 0
= x−i xi y −(j+j ) y j+j z i(j+j ) z −j(i+i )
0
−i0 j
= z ij .
In other words, g g gg = (x y xy)k , where k = ij 0 − i0 j ∈ Z. Consequently,
−1 0−1 0 −1 −1

G0 is the cyclic group generated by z = x−1 y −1 xy ∈ Z(G), completing the proof.



176 2. SOLUTIONS

3. (a) Let At ∈ Mn (R) denote the transpose of the matrix A ∈ Mn (R). For all
1 ≤ i, j ≤ n, use (AAt )ij to denote the ij entry of the matrix AAt . It follows from
the hypothesis that
(AAt )ij = tr(A)aij ,
for all 1 ≤ i, j ≤ n. It is now obvious that tr(A) 6= 0, for otherwise (AAt )ij = 0
for all 1 ≤ i, j ≤ n, yielding tr(AAt ) = 0, which obtains A = 0, contradicting the
hypothesis. Thus, tr(A) 6= 0, which is what we want.

(b) In view of (a), we have


1
A= AAt .
tr(A)
Since AAt is symmetric, so is A, as desired.

(c) In view of the second proof presented for Solution 7 of 2.9.3, it suffices to
show that rank(A) = 1. To this end, suppose A ∈ Mn (R) satisfies aik ajk = akk aij
for all 1 ≤ i, j, k ≤ n. For 1 ≤ j ≤ n, let us use Aj to denote the jth column of the
matrix A. As tr(A) 6= 0, there exists a 1 ≤ k ≤ n such that akk 6= 0. It thus follows
ajk
from the hypothesis that for all 1 ≤ j ≤ n, we have Aj = bjk Ak , where bjk = akk .
This obviously proves that rank(A) = 1, which is what we want. 

2.22. Twenty Second Competition

2.22.1. Analysis. 1. By the Mean Value Theorem, there exists a c ∈ (0, 1)


such that f 0 (c) = f (1)−f
1−0
(0)
= 1. If f 0 (x) ≥ 1 (resp. f 0 (x) ≤ 1) for all x ∈ (0, 1), we
see that f (x) = x for all x ∈ [0, 1], in which case the assertion is trivial. To see this,
suppose to the contrary that there exists c ∈ (0, 1) such that f (c) > c or f (c) < c.
Suppose f (c) > c. It follows from the Mean Value Theorem that there exists d ∈
(c, 1) (resp. d ∈ (0, c)) such that f 0 (d) = f (1)−f
1−c
(c)
< 1 (resp. f 0 (d) = f (c)−f
c−0
(0)
> 1),
a contradiction in any event. Likewise, if f (c) < c, we obtain a contradiction. Thus,
there exist a, b ∈ (0, 1) with a < c < b such that (f 0 (a)−1)(f 0 (b)−1) < 0. It follows
from Darboux’s Theorem that the range of f0 , denoted by R, includes the closed
interval [m, M ], where m = min f 0 (a), f 0 (b) < 1 < M = max f 0 (a), f 0 (b) . It


thus follows that [1 − ε, 1 + ε] ⊆ R, where ε = min(1 − m, M − 1). It is obvious


that for all y1 ∈ [1 − ε, 1 + ε], with y1 = f 0 (x1 ) < 1 for some x1 ∈ [0, 1], there exists
y2 ∈ [1 − ε, 1 + ε], with y2 = f 0 (x2 ) > 1 for some x2 ∈ [0, 1], such that
1 1
+ = 2. (∗)
f 0 (x1 ) f 0 (x2 )
This obviously proves the assertion when n ∈ N is even. If n ∈ N is odd, use (∗)
and the fact that f 01(c) = 1, where c is as in the above, to prove the assertion. 
P+∞
2. (a) We have fn ≤ i=1 fi = f 2 for all n ∈ N because fn ’s are all nonnegative.
Define gn : X −→ R by
(
0 x ∈ f −1 ({0}),
gn (x) = fn (x)
f (x) x ∈ X \ f −1 ({0}).
2.22. TWENTY SECOND COMPETITION 177

It is obvious that fn = gn f for all n ∈ N. It remains to show that gn is continuous


for all n ∈ N. To this end, note that gn is continuous on the open set X \ f −1 ({0})
because so are f and fn on X \ f −1 ({0}) (n ∈ N). Suppose that x0 ∈ X \ f −1 ({0})
is arbitrary. It follows that for given ε > 0, there exists δ > 0 such that

fn (x) fn (x0 )

f (x) − < ε,
f (x0 )
whenever x ∈ X \ f −1 ({0}) and |x − x0 | < δ. Since X \ f −1 ({0}) is an open set and
x0 ∈ X \ f −1 ({0}), choose δ small enough such that x ∈ X \ f −1 ({0}) whenever
|x − x0 | < δ. So we have

gn (x) − gn (x0 ) < ε,

whenever |x−x0 | < δ. That is, gn is continuous at x0 . Next, suppose x0 ∈ f −1 ({0})


is arbitrary. As f is continuous, we see that for given ε > 0, there exists a δ > 0
such that

f (x) = f (x) − f (x0 ) < ε,
whenever |x − x0 | < δ. Now if |x − x0 | < δ, we can write
(
0<ε x ∈ f −1 ({0})
gn (x) − g(x0 ) = gn (x) = fn (x) .

f (x) ≤ f (x) < ε
x ∈ X \ f −1 ({0})

In other words, |gn (x) − g(x0 )| < ε whenever |x − x0 | < δ. That is, gn is continuous
at x0 , as desired.
P+∞
(b) Suppose that n=1 gn converges uniformly on X and that the interior of
P+∞
f −1 ({0}) is empty. It follows that n=1 gn is a continuous function on X and
P+∞
that X \ f −1 ({0}) is a dense subset of X. We see from (a) that n=1 gn f = f 2 ,
P+∞  P+∞
yielding f f − n=1 gn = 0. This easily implies f (x) − n=1 gn (x) = 0 whenever
P+∞
x ∈ X \ f −1 ({0}). That is, the continuous function f − n=1 gn vanishes on
+∞
the dense subset X \ f −1 {0} . Thus, f − n=1 gn vanishes on X, and hence
 P
P+∞
f = n=1 gn , which is what we want. 

3. Note first that f is continuous on D because the sequence (fn )+∞ n=1 uniformly
converges to f on D as n tends to infinity and that fn ’s are all analytic, and
hence
R continuous. Thus, in view of Morera’s Theorem, it suffices to show that
γ0
f (z)dz = 0, where γ0 is any simple closed curve inside γ. We have
Z Z
f (z)dz = lim fn (z)dz = 0,
γ0 n γ0

because (fn )+∞


n=1 uniformly converges to f on γ0 and fn ’s are analytic. Assuming
that z0 is inside γ, we see from Cauchy’s Integral Formula that
Z Z
1 fn (z) 1 f (z)
fn (z0 ) = dz, f (z0 ) = dz,
2πi γ z − z0 2πi γ z − z0
proving the assertion because fn (z0 ) → f (z0 ) as n → +∞. 
178 2. SOLUTIONS

2.22.2. Algebra. 1. Suppose [G : K] = 2n − 1 for some n ∈ N. We have


[G : H] = [G : K][K : H] = 2(2n − 1). The group G acts on Ω = {Hg : g ∈ G}, the
set of all right cosets of H in G, by multiplication from the right. That is, the action
is given by the map ϕ : G × Ω → Ω defined by ϕ(a, Hg) = Hga. Recall that ϕ gives
rise to a group homomorphism ψ : G → S(Ω) defined by ψ(a) = ϕ(a, .) : Ω → Ω,
where S(Ω) denotes the group of the permutations of Ω, i.e., the set of all one-to-
one maps from Ω onto Ω, which forms a group T under composition of maps. The
kernel of the action, by definition, is ker ψ = g∈G g −1 Hg. It follows from the First
Isomorphism Theorem for groups that kerGψ ∼ = im(ψ) ≤ S(Ω) ∼ = S2(2n−1) . That
G
is, ker ψ is isomorphic to a subgroup of the symmetric group of degree 2(2n − 1).
Since k ∈ K has order 2, we see that for k ker ψ ∈ kerGψ , we have (k ker ψ)2 =
k 2 ker ψ = ker ψ. Thus, the order of k ker ψ, as an element of kerGψ , is equal to 1
or 2. But k ∈ / ker ψ because otherwise, in particular, k ∈ H, which is impossible.
Consequently, ord(k ker ψ) = 2. Note that kerGψ acts on Ω via (a ker ψ, Hg) → Hga
and that the element (k ker ψ, .) ∈ S(Ω) ∼ = S2(2n−1) has no fixed point, for otherwise
Hgk = Hg for some g ∈ G, yielding gkg −1 ∈ H, contradicting the hypothesis.
Therefore, the element (k ker ψ, .) ∈ S(Ω) corresponds to an odd permutation of
S(Ω) because, in view of (k ker ψ, .)2 = id ∈ S(Ω), it decomposes into the product
of 2n − 1 transpositions or 2-cycles. Thus, the subgroup of kerGψ , say, kerJ ψ for some
J ⊇ ker ψ, which consists of all elements that correspond to even permutations of
S(Ω) forms a subgroup of kerGψ of index 2. Since [G : ker ψ] = [G : J][J : ker ψ], we
can write
h G J i
[G : J] = : = 2,
ker ψ ker ψ
which is what we want. 

2. If J = R or J = {0}, the assertion is trivial. So suppose that J is a nontrivial


ideal of R and J has a maximal ideal, say, M . By obtaining a contradiction,
J
we settle the proof. Let f : J → M , defined by f (x) = x + M , be the natural
J J
homomorphism from J onto M . As M is a maximal ideal of J, it follows that M
J
has no nontrivial ideal. This implies that M is a field or it is a finite ring with zero
multiplication. The latter is impossible because it would mean (x+M )(y+M ) = M
J
for all x, y ∈ J, yielding J 2 ⊆ M , which is impossible. Thus, M is a field. Let e+M
denote the identity element of the field M , where e ∈ J \ M . Define f ∗ : R → M
J J
by
∗ ∗
f (x) = f (xe + M ). It is readily seen that f is a homomorphism of rings. So, we
see from the First Isomorphism Theorem for rings that kerRf ∗ ∼ =M J
. Consequently,
∗ J ∗
ker f is a maximal ideal of R, for M is field. Thus, J ⊆ ker f . This, in particular,
implies f ∗ (e) = M . On the other hand, f ∗ (e) = f (e2 + M ) = (e + M )2 = e + M .
So, we must have M = e + M , yielding e ∈ M , which is impossible. The assertion
thus follows by contradiction. 

3. To prove the assertion by contradiction, suppose that rank(L) = 1 so that


LV = hαi for some α ∈ V . With no loss of generality, we may assume that the
characteristic polynomial of T is irreducible over F . We claim that

T k α ∈ ker L,
2.23. TWENTY THIRD COMPETITION 179

for each k = 0, 1, 2, . . .. As the rank of LT k is at most one and


tr(LT k ) = tr(T ST k − ST k+1 ) = 0,
for each k = 0, 1, 2, . . ., it follows that LT k is nilpotent. And hence its eigenvalues
are all zero. But LT k α ∈ LV = hαi for all nonnegative integers k. That is, α is an
eigenvector of LT k , and hence LT k α = 0 for all nonnegative integers k. Now, set
W = T k α : k = 0, 1, 2, . . . .



It is plain that W is invariant under T and that W is a nontrivial subspace of


V because α is nonzero and W ⊆ ker L. It thus follows that the characteristic
polynomial of T |W , the restriction of T to the invariant subspace W , divides the
minimal polynomial of T , which is equal to the characteristic polynomial of T ,
for the characteristic polynomial of T is irreducible over F . This contradicts the
hypothesis that the characteristic polynomial of T is irreducible. Thus, the assertion
follows by contradiction. 

2.22.3. General.
a b c d a b c d a b c d
1. F 15. F 29. F
2. F 16. F 30. F
3. F 17. F 31. F
4. F 18. F 32. F
5. F 19. F 33. F
6. F 20. F 34. F
7. F 21. F 35. F
8. F 22. F 36. F
9. F 23. F 37. F
10. F 24. F 38. F
11. F 25. F 39. F
12. F 26. F 40. F
13. F 27. F 41.
14. F 28. F 42.

2.23. Twenty Third Competition

2.23.1. Analysis. 1. (a) We prove the assertion by way of contradiction.


Suppose that the set {fα : α ∈ R} is compact with respect to the uniform norm
of Cb (R) and yet f is not uniformly continuous on R. It follows that there are
 > 0 and sequences (xn )+∞ +∞
n=1 and (yn )n=1 in R such that limn xn = limn yn = ±∞,
limn (xn − yn ) = 0, and |f (xn ) − f (yn )| ≥  for all n ∈ N. By the hypothesis,
the sequence (gn )+∞ +∞
n=1 , where gn = fxn , has a subsequence, say (gnk )k=1 , which
converges uniformly to fα on R for some α ∈ R. It follows that limk gnk (ynk −xnk ) =
fα (0). On the  other hand, limk gnk (0) = fα (0). Consequently, limk gnk (ynk −
xnk ) − gnk (0) = 0. But

gn (yn − xn ) − gn (0) = f (yn ) − f (xn ) ≥ ,
k k k k k k
180 2. SOLUTIONS

for all k ∈ N, which is in contradiction with



lim gnk (ynk − xnk ) − gnk (0) = 0.
k

Thus, f is uniformly continuous on R, as desired.

(b) We disprove the proposition by showing that for the function f : R −→ R


2
defined by f (x) = e−x , the sequence {fα : α ∈ R} is not compact with respect to
the uniform norm of Cb (R). Note first that f is bounded and uniformly continuous
on R because limx→±∞ f (x) = 0. To disprove the proposition by contradiction,
suppose that {fα : α ∈ R} is compact with respect to the uniform norm of Cb (R). It
follows that the sequence (fn )+∞ +∞
n=1 has a subsequence, say (fnk )k=1 , which converges
2 2
uniformly on R to fα for some α ∈ R. Thus, 0 = limk e−(x+nk ) = e−(x+α) ,
2
implying that e−(x+α) = 0 for all x ∈ R, which is obviously impossible. So the
proposition is disproved by way of contradiction. 

2. First solution:
P+∞ First we need to recall Abel’s Continuity Theorem which
asserts that if n=0 an is a convergent series of complex numbers, then
+∞
X +∞
X
lim an rn = an .
r→1−
n=0 n=0

Without loss of generality, if necessary by adding or subtracting multiples of


2π, we may assume that 0 < α < 2π. Set
n
X eikα
Sn (α) = .
k
k=1

We have
n
X ei(n+1)α − eiα
Sn0 (α) =i eikα = i .
eiα − 1
k=1

Consequently,
α Z α
ei(n+1)t
Z
1 
Sn (α) − Sn (π) = i it
dt − i 1 + it dt
π e −1 π e −1
Z α  ei(n+1)t  Z α
1 (cos t − 1) − i sin t
= it − 1
d − i(α − π) − i dt
π e n + 1 π 2(1 − cos t)
α
1 ei(n+1)t α ei(n+2)t i α
Z Z
i
= + dt − i(α − π) + dt
n + 1 eit − 1 π n + 1 π (eit − 1)2 2 π

Z α
1  t
− t d sin
π sin 2 2
Z α i(n+2)t
1 ei(n+1)α (−1)n i e i(α − π)
= iα
+ + it 2
dt −
n+1 e −1 2(n + 1) n + 1 π (e − 1) 2
α
− ln sin .
2
2.23. TWENTY THIRD COMPETITION 181

So we can write
α π − α  1 ei(n+1)α
Sn (α) − − ln 2 sin +i = ln 2 + Sn (π) +
2 2 n + 1 eiα − 1
n Z α i(n+2)t
(−1) i e
+ + dt.
2(n + 1) n + 1 π (eit − 1)2
On the other hand, we have
+∞
X (−1)k−1
lim Sn (π) = − = − ln 2.
n k
k=1
P+∞ n−1
To see this, just apply Abel’s Continuity Theorem to the power series n=1 (−1)n xn =
P+∞ n
ln(1 + x) and note that the alternating series n=1 (−1) n converges by Leibniz’s
Theorem. Also, it is plain that
1 ei(n+1)α (−1)n
 
lim + = 0.
n n + 1 eiα − 1 2(n + 1)
Finally,
Z i(n+2)t
α
ei(n+2)t 1 α
Z e
i
dt ≤ 2 dt
n + 1 (eit − 1)2 n+1 π

π eit − 1
Z
1 α 1
= ,
dt

n+1 π eit − 1 2

1
for all n ∈ N and limn n+1 = 0, implying that
Z α i(n+2)t
i e
lim dt = 0.
n n + 1 π (eit − 1)2

Consequently,
 α π − α 
lim Sn (α) − − ln 2 sin + i = 0.
n 2 2
In other words,
+∞ inα
X e α π−α
= − ln 2 sin + i .
n=1
n 2 2
Now, equating the real and imaginary parts of the above equality obtains
+∞
X cos(nα) α
= − ln 2 sin ,
n=1
n 2
+∞
X sin(nα) π−α
= ,
n=1
n 2

which is what we want.

Second solution: For this solution, first we need to recall Dirichlet’s Theorem
which asserts that if (an )+∞ +∞
n=1 and (bn )n=1 are sequences of complex and real num-
bers, respectively, such that the sequence of partial sums of (an )+∞
n=1 is bounded
182 2. SOLUTIONS

P+∞
and that (bn )+∞
n=1 is monotonic and tends to zero as n → +∞, then n=1 an bn
converges. Note that, as we saw in the first solution, we have
n
X ei(n+1)α − eiα
eikα = ,
eiα − 1
k=1

implying that
n
X 2
eikα ≤ iα

,
k=1
e − 1
for all n ∈ N. On the other hand, the sequence ( n1 )+∞
n=1 is decreasing and tends to
zero as n → +∞. Thus, by Dirichlet’s Theorem, the series
+∞ inα
X e
n=1
n
converges whenever α ∈ R \ {2kπ : k ∈ Z}.
Again, without loss of generality, we may assume that 0 < α < 2π. Let
z = reiα , where 0 ≤ r < 1. Note that 1 − z = (1 − r cos α) + i(−r sin α). We can
write
+∞ n +∞ inα
X z X e
− ln(1 − z) = = rn ,
n=1
n n=1
n
where ln denotes the principal value of the natural logarithm, which is defined by
ln z = ln |z| + i arg z, where arg z = arctan Im(z) π π
Re(z) ∈ (− 2 , 2 ). On the other hand,

− ln(1 − z) = − ln |1 − z| + i arg(1 − z)
 p 
2
−r sin α
= − ln 1 − 2r cos α + r + i arctan .
1 − r cos α
Letting r → 1− , in view of Abel’s Continuity Theorem, we obtain
+∞ inα
X e √ − sin α
= − ln 2 − 2 cos α − i arctan
n=1
n 1 − cos α
r
α α
= − ln 4 sin2 + i arctan cot( )
2 2
α π−α
= − ln 2 sin + i .
2 2
Thus,
+∞ inα
X e α π−α
= − ln 2 sin + i .
n=1
n 2 2
Equating the real and imaginary parts of the above equality, we obtain
+∞
X cos(nα) α
= − ln 2 sin ,
n=1
n 2
+∞
X sin(nα) π−α
= ,
n=1
n 2
which is what we want. 
2.23. TWENTY THIRD COMPETITION 183

3. It is worth mentioning that the hypothesis that f 0 and fn0 ’s are continuous is
redundant. Let x ∈ [a, b) be arbitrary. In view of the definition of the derivative,
the Mean Value Theorem, and the continuity of g, we can write
1
f 0 (x) =

lim n f (x + ) − f (x)
n→+∞ n
1
lim lim n fk (x + ) − fk (x) = lim lim fk0 (xn )

=
n→+∞ k→+∞ n n→+∞ k→+∞
= lim g(xn ) = g(x),
n→+∞

where x < xn < x + n1 . Thus, f 0 (x) = g(x) for all x ∈ [a, b). If x = b, just
write f 0 (b) = limn→+∞ −n f (b − n1 ) − f (b) and repeat the above argument to get
f 0 (b) = g(b). So the proof is complete. 

2.23.2. Algebra. 1. This problem is wrong! For n ∈ N, use Dn to denote


the dihedral group of order 2n. A presentation of Dn is
a, b : an = e, b2 = e, ak 6= e (0 < k < n), ab = ba−1 ,



and hence Dn = ai bj : 0 ≤ i < n, j = 0, 1 . We show that D8 is a counterexample.
First, recall that Inn(G) ∼ G
= Z(G) , where Z(G) denotes the center of the group G
(see Solution 1 of 2.16.2). Therefore, Inn(D8 ) = ∼ D8 . To show that D8 is a
Z(D8 )
counterexample, it suffices to prove that Z(D D2n ∼
= Dn for all n ∈ N. To this end,
 n 2n )

just note that Z(D2n ) = e, a , where a is the element of order 2n in the above
presentation of D2n , and that
D2n  i j
= A B : A = aZ(D2n ), B = bZ(D2n ), 0 ≤ i < n, j = 0, 1
Z(D2n )

= Dn ,
for one can easily verify that An = Z(D2n ) = B 2 , AB = BA−1 , and that Ak 6=
Z(D2n ) for all 0 < k < n. 

2. First, we prove that H is a two-sided ideal of R by showing that for every r ∈ R


and x ∈ H, there is a t ∈ R such that rx = xt. To this end, note first that rR is a
right ideal of R. If rR = {0}, there is nothing to prove. If rR 6= {0}, then x ∈ rR,
and hence there is a y ∈ R such that x = ry. Likewise, considering the right ideal
yR, we obtain a t ∈ R such that x = yt. So we can write rx = (ry)t = xt. That
is, for r ∈ R and x ∈ H, there is a t ∈ R such that rx = xt, as desired. So far,
we have actually shown that the intersection of all nonzero right ideals of R is the
same of those of the nonzero left ideals of R.
Next, suppose that H 2 6= 0. We prove that R is a division ring. To this end,
we first show that R has no zero divisors. Suppose to the contrary that there are
nonzero elements x, y ∈ R such that xy = 0. As H 2 6= 0, there are a, b ∈ H such
that ab 6= 0. For a, b ∈ R, just as we saw in the above, there are s, t ∈ R such that
xs = a and yt = b because a, b ∈ H. Again, for s ∈ R and b ∈ H, there is a u ∈ R
such that sb = bu. We can write
xy = 0 =⇒ xyt = 0 =⇒ xb = 0 =⇒ xbu = 0 =⇒ xsb = 0 =⇒ ab = 0,
184 2. SOLUTIONS

a contradiction. Thus, R has no zero divisors. To show that R has a multiplicative


identity, choose a nonzero a ∈ H and note that a ∈ aR, implying that a = ae for
some e ∈ R. Since R has no zero divisors and a(ea − a) = 0, we obtain ea = a. Now
for an arbitrary x ∈ R, we have (xe − x)a = a(ex − x) = 0, from which, we obtain
xe = ex = x. That is, e is the multiplicative identity of R. For the rest, since H
is an ideal of R, we need to prove that every nonzero element of H is invertible.
To this end, for a nonzero a ∈ H, note that a2 6= 0, and hence a = a2 t for some
t ∈ R. It follows that a(e − at) = 0, which, in turn, yields at = e. This implies
that ta(e − ta) = 0, from which, we obtain e = ta. Thus e = ta = at. That is, a is
invertible, which is what we want. 

3. Let A = (aij ) ∈ Mn (F ). Use Eij and In to denote the matrix whose ij entry
is 1 and zero elsewhere and the identity matrix, respectively. If 1 ≤ i, j ≤ n
with i 6= j, then tr(Eij ) = 0. By the hypothesis, aji = tr(AEij ) = 0 for all
1 ≤ i, j ≤ n with i 6= j. Thus, A is a diagonal matrix. For 1 ≤ i ≤ n, let
Bi = diag(1, 0, . . . , 0)−diag(δ1i , δ2i , . . . , δni ), where δji denotes the Kronecker delta.
Since tr(Bi ) = 0, the hypothesis implies aii − a11 = tr(ABi ) = 0 for all 1 ≤ i ≤ n.
Therefore, A = a11 In , as desired. 

2.24. Twenty Fourth Competition

Rx
2.24.1. First Day. 1. Define F : [a, b] −→ R by F (x) = a f (t)dt. By
the First Fundamental Theorem of Calculus, F 0 (x) = f (x) for all x ∈ [a, b]. Let
h = F − g. Obviously, h is differentiable on [a, b] and we have h0 = f − g 0 . It
follows from the hypothesis that h0 (a)h0 (b) < 0. This together with Darboux’s
Theorem implies that there exists a c ∈ (a, b) such that h0 (c) = 0. In other words,
f (c) = g 0 (c), which is what we want. 

2. Note first that all real harmonic functions satisfy the Mean Value Property.
That is, if D is a domain such that Br0 (a0 ) ⊆ D for some a0 = (x0 , y0 ) ∈ D and
r0 > 0 and u : D −→ R is a harmonic function, then
Z 2π
1
u(a0 ) = u(a0 + r0 eiθ )dθ.
2π 0
To see this, let B be an open ball such that Br0 (a0 ) ⊆ B ⊆ D and f a holomorphic
function on the simply connected domain B such that u|B = Ref . It easily follows
from Cauchy’s Integral Formula that
Z 2π
1
f (a0 ) = f (a0 + r0 eiθ )dθ.
2π 0
Taking the real part of the both sides of the above equation proves the counterpart
of it with f replaced by u, as desired. Next, we need the following version of the
Maximum Modulus Theorem for harmonic functions, which in fact holds true for
all functions satisfying the Mean Value Property.

Theorem. Let D be a domain and u : D −→ R be a harmonic function. If


there is a point a0 = (x0 , y0 ) ∈ D such that u(z) ≤ u(a0 ) for all z ∈ D, then
u(z) = u(a0 ) for all z ∈ D.
2.24. TWENTY FOURTH COMPETITION 185

Proof. Set
L = {z ∈ D : u(z) = u(a0 )}.
It is plain that L is a nonempty closed subset of D. By proving that L is an open
subset of D as well, in view of the connectedness of D, we see that L = D, proving
the assertion. To this end, suppose to the contrary that there is a z0 ∈ L which
is not an interior point of L. Choose r > 0 such that Br (z0 ) ⊆ D. It follows
that there is a b0 ∈ Br (z0 ) such that u(b0 ) < u(a0 ) = u(z0 ). And hence, from the
continuity of u, we see that u(z) < u(z0 ) for all z in a small enough neighborhood
of b0 . Letting r0 = |z0 − b0 | and b0 = z0 + r0 eiθ0 for some 0 ≤ θ0 ≤ 2π, there thus
exists a proper subinterval I of [0, 2π] such that θ0 ∈ I and u(z0 + r0 eiθ ) < u(z0 )
for all θ ∈ I. This, in view of the Mean Value Property of harmonic functions,
yields
Z 2π
1
u(z0 ) = u(z0 + r0 eiθ )dθ < u(z0 ),
2π 0
which is a contradiction. Thus, L is open, completing the proof. 

To prove the assertion, let the set A be as in the statement of the problem.
By the Maximum Modulus Theorem for harmonic functions, the set A is open.
The assertion follows as soon as we prove that A is closed because D is connected
and A is nonempty by the hypothesis. To prove that A is closed, suppose that
(an )+∞
n=1 is a sequence of the elements of A such that limn an = a for some a ∈ D.
We need to show that a ∈ A. To this end, choose r > 0 such that Br (a) ⊆ D.
It follows that there is an n ∈ N such that an ∈ Br (z0 ). As an ∈ A, there
is an rn > 0 such that Brn (an ) ⊆ Br (a) ⊆ D and that u(z) ≤ u(an ) for all
z ∈ Brn (an ). Since Br (a) is simply connected, there is a holomorphic function
f on Br (a) such that u|Br (a) = Ref . By the Maximum Modulus Theorem for
harmonic functions, Ref |Brn (an ) = Ref (an ), which, in view of the Cauchy-Riemann
Equations, yields f |Brn (an ) = f (an ). Since f is holomorphic on Br (a), it follows
that f |Br (a) = f (an ) = f (a), which, in turn, implies u(z) = u(a) for all z ∈ Br (a).
Therefore, a ∈ A, which is what we want, completing the proof. 

3. First solution: Let p = t(3n − 2n ). In particular, p 3n − 2n . In other words,
p
3n ≡ 2n . Obviously, gcd(6, p) = 1. And hence, 2 has an inverse modulo p. That is,
p
there is an a ∈ N with 1 < a < p such that 2a ≡ 1. So we can write
p p p
3n an ≡ 2n an ≡ (2a)n ≡ 1,
p
implying (3a)n ≡ 1. Use m to denote the order of 3a modulo p, i.e., m is the
p
least positive integer satisfying (3a)m ≡ 1. It follows that m n. By Fermat’s Little
p
Theorem, (3a)p−1 ≡ 1, which obtains m p−1. Consequently, m gcd(n, p−1). Note
p p
that m > 1, for otherwise 3a ≡ 1, yielding 3 ≡ 2, which is impossible. This implies
that any prime divisor of m, say q, is a divisor of n as well and that q ≤ p − 1 < p.
Thus, t(n) ≤ q < p = t(3n − 2n ), as desired.

p
Second solution: Let m be the least positive integer such that 3m ≡ 2m . Ob-
p p
viously, 2m 3m ≡
6 0. Note that if 3k ≡ 2k for some k ∈ N, then m k. To see this,
186 2. SOLUTIONS

letting k = qm + r, where r + 1, q ∈ N and 0 ≤ r < k, we can write


p p p p
3r ≡ 3k−qm ≡ 3k (3m )−q ≡ 2k (2m )−q ≡ 2k−mq = 2r ,
p
implying that 3r ≡ 2r . Thus, r = 0, yielding k = qm, i.e., m k. This together with
p
the hypothesis implies that m n. On the other hand, as 6 ≡ 6 0, in view of Fermat’s
Little Theorem, we can write
p p
3p−1 ≡ 1 ≡ 2p−1 ,

yielding m p − 1. Consequently, m gcd(n, p − 1), which, as we saw in the first
solution, entails t(n) < t(3n − 2n ), as desired. 

4. Use Z to denote the center of G. It follows from the hypothesis that there exist
n ∈ N and gi ∈ G (1 ≤ i ≤ n) such that
G 
= g1 Z, . . . , gn Z .
Z
It is obvious that gi Z’s (1 ≤ i ≤ n) are all infinite subsets of G. It suffices to
show that gi gj = gj gi for all 1 ≤ i, j ≤ n. To this end, for given 1 ≤ i, j ≤ n
with i 6= j, it follows from the hypothesis that there exist zi , zj ∈ Z such that
(gi zi )(gj zj ) = (gj zj )(gi zi ). This easily yields (gi gj )(zi zj ) = (gj gi )(zi zj ), which, in
turn, implies gi gj = gj gi for all 1 ≤ i, j ≤ n, completing the proof. 

5. First solution: First, we show that if there are m n-tuples from the set {0, 1}
in such a way that every two of which differ at least in d components, then
mn
≥ (m − 1)d.
2
To this end, the given n-tuples form the following m × n matrix
 
a11 · · · a1n
 .. ..  .
 . ··· . 
am1 ··· amn
whose row i is the ith n-tuple (1 ≤ i ≤ m). Denote by A the total number
P of differ-
ences between any two rows of these m rows. In other words, A = 1≤i<j≤m dij ,
where dij is the number of components in which row i and row j differ. As dij ≥ d
for all 1 ≤ i < j ≤ m, we obtain A ≥ m 2 d. To find an upper bound for A, fix a
column, say column j, and use cj to denote the number of differences that
P these m
vector can mutually have in their jth component. It is obvious that A = 1≤j≤n cj .
2
If there are xj zeros in column j, then cj = xj (m − xj ) ≤ m m m
2 (m − 2 ) = 4 because
the polynomial function f (x) = x(m − x) assumes its maximum at x = m 2 . It
2 2
follows that A = 1≤j≤n cj ≤ 1≤j≤n m4 = m4 n . Consequently,
P P

m2 n
 
m m(m − 1)
≥A≥ d= d,
4 2 2
from which, we obtain
mn
≥ (m − 1)d,
2
as desired.
2.24. TWENTY FOURTH COMPETITION 187

Now, to prove the assertion, let m = M (2d − 1, d) and note that n = 2d − 1.


In view of the above inequality, we have
m(2d − 1)
≥ (m − 1)d,
2
yielding
2md − m ≥ 2md − 2d =⇒ m = M (2d − 1, d) ≤ 2d,
which is what we want.

Second solution: First we need to recall Turán’s Theorem from Graph Theory.
For the sake of completeness, we quote a simple proof of it which is due to William
Staton.

Theorem (Turán, 1941). Graphs with n vertices containing no Kr have no


2
more than (r−2)n
2r−2 edges, for r ≥ 2.

Proof (William Staton). Induct on r. If r = 2, the result is obvious. Now


if the statement is true for Kr -free graphs it must be shown that Kr+1 -free graphs
2
have no more than (r−1)n2r edges. Let G be such a graph, and let x be the number
of vertices in a largest Kr -free induced subgraph of G. Since the neighbors of any
vertex induce a Kr -free subgraph, no vertex of G has degree exceeding x. Let A be
2
a largest induced Kr -free subgraph of G. By induction, there are at most (r−2)x
(2r−2)
edges in A. Each edge of G not in A is incident with at least one of the n − x
vertices not in A, so summing the degrees of these vertices counts each such edge
at least once. Hence there are at most x(n − x) such edges and so G has at most
(r−2)x2
(2r−2) + x(n − x) edges. Since
 2
r−2 2 r−1 2 r (r − 1)n
(x ) + x(n − x) = n − x− ,
2r − 2 2r 2r − 2 r
the result follows. 

To prove the assertion by contradiction, suppose that M (2d−1, d) > 2d so that


there are 2d+1 (2d−1)-tuples any two of which differ in at least d components. Let
G be an edge-labeled graph as follows. The set of vertices is the 2d+1 (2d−1)-tuples.
For any two vertices, if the two vertices differ in their k component (1 ≤ k ≤ 2d−1),
then connect the two vertices with an edge with label k. Consequently, there are
at least d labeled edges connecting any two vertices of the graph G and hence the
2d+1

graph G has at least 2 × d = (2d + 1)d2 edges. Since 2d − 1 numbers are
assigned to the edges and there are at least (2d + h1)d2 edges, i we see that there is a
2
label 1 ≤ k0 ≤ 2d − 1 which is assigned to at least (2d+1)d
2d−1 + 1 edges of the graph,
where [.] denotes the integer part function. Now consider the induced graph on
these edges to each of which k0 ishassigned.i This induced graph is indeed a simple
2
graph having 2d + 1 vertices and (2d+1)d
2d−1 + 1 edges. Note that

(2d + 1)d2 (2d + 1)d2 (2d + 1)2


   
+1> > d2 + d = .
2d − 1 2d − 1 4
It thus follows from Turán’s Theorem that the induced graph has a K3 , i.e., a
triangle. This means that there are three vertices any two of which differ in their
188 2. SOLUTIONS

k0 component. This is a contradiction because the components are either 0 or 1,


completing the proof. 

6. First we need the following simple lemma.

Lemma. Let two lookalike boxes contain w1 white marbles and b1 black mar-
bles, and w2 white marbles and b2 black marbles, respectively. The probability of
picking a white marble from one of the boxes is equal to 21 w1w+b w2

1
1
+ w2 +b2 .

Proof. Use A to denote the event that one picks a marble from the box that
contains w1 white marbles and b1 black marbles, and use B to denote the event
that one picks a marble from the other box. If W denotes the event of picking
a white marble from one of the boxes, as {A, B} is a partition of the probability
space, we can write

P (W ) = P (A ∩ W ) ∪ (B ∩ W ) = P (A)P (W |A) + P (B)P (W |B)
1 w1 1 w2 1 w1 w2 
= × + × = + ,
2 w1 + b1 2 w2 + b2 2 w1 + b1 w2 + b2
proving the lemma. 

Suppose that the first person has picked a marble from one of the boxes, say,
box b. Use b0 to denote the other box. For each i = 1, 2, 3, define the following
events
Wi (resp. Wi0 ): the event that the ith person picks a white marble from b (resp.
0
b ).
Bi (resp. Bi0 ): the event that the ith person picks a black marble from b (resp.
0
b ).
By the lemma above, we have
1 1 2  1
P (W1 ) = + = .
2 1+2 1+2 2
Likewise, P (B1 ) = 12 .
0 0
Use B011 and B001 (resp. B011 and B001 ) to denote the events that box b (resp.
0
b ) contains one black marble and two white marbles and two black marbles and
one white marble, respectively.
First, suppose that the first person has picked a white marble from b. Using
Bayes’ Theorem, we can write
1 2
P (B011 )P (W1 |B011 ) 2 × 3 2
P (B011 |W1 ) = = 1 = ,
P (W1 ) 2
3
1 1
P (B001 )P (W1 |B001 ) 2 × 3 1
P (B001 |W1 ) = = 1 = .
P (W1 ) 2
3
0 0
And hence, P (B011 |W1 ) = 1 − P (B011 |W1 ) = 1 − 23 = 13 and P (B001 |W1 ) =
1 − P (B001 |W1 ) = 1 − 3 = 3 . Let U = B011 |W1 , V = B001 |W1 , U 0 = B011
1 2 0
|W1 , and
V 0 = B001
0
|W1 . We have P (U ) = P (V 0 ) = 32 and P (V ) = P (U 0 ) = 13 . Noting that
{U, V } is a partition of the probability space, we have
2 1 1 1
P (W2 ) = P (U )P (W2 |U ) + P (V )P (W2 |V ) = × + ×0= .
3 2 3 3
2.24. TWENTY FOURTH COMPETITION 189

That is, assuming that the first person has picked a white marble from b, the
probability that the second person picks a white marble from b is 13 . Analogously,
as {U 0 , V 0 } is a partition of the probability space, we can write
1 2 2 1 4
P (W20 ) = P (U 0 )P (W20 |U 0 ) + P (V 0 )P (W20 |V 0 ) = × + × = .
3 3 3 3 9
In other words, assuming that the first person has picked a white marble from b,
the probability that the second person picks a white marble from b0 is 49 , which is
greater than 31 . So, in this case, the second person should pick a marble from the
other box, i.e., b0 .
Now, assuming that the first person has picked a black marble from b, again us-
ing Bayes’ Theorem, we will have P (B011 |B1 ) = 13 and P (B001 |B1 ) = 23 , and hence
0 0
P (B011 |B1 ) = 1 − P (B011 |B1 ) = 1 − 13 = 23 and P (B001 |B1 ) = 1 − P (B001 |B1 ) =
0 0
2 1
1− 3 = 3 . So, letting S = B011 |B1 , T = B001 |B1 , S = B011 |B1 , and T 0 = B001
0
|B1 ,
we can write
1 2 1 2
P (W2 ) = P (S)P (W2 |S) + P (T )P (W2 |T ) = ×1+ × = ,
3 3 2 3
2 2 1 1 5
P (W20 ) = P (S 0 )P (W20 |S 0 ) + P (T 0 )P (W20 |T 0 ) = × + × = .
3 3 3 3 9
Since P (W2 ) = 32 is greater than P (W20 ) = 59 , the second person should pick a
marble from box b. Therefore, the probability of survival for the second person is
1 4 1 2 5
× + × = ,
2 9 2 3 9
for, by the above lemma, the first person picks a white (resp. black) marble with
the probability of 12 .
To investigate the probability of survival for the third person, there are two
cases to consider.
(i) the first person survives.
Without loss of generality, we may assume that the first person has picked a
white marble from box b. In view of what we showed in the above, there are two
subcases to consider. (a) The second person picks a white marble from box b0 ; and
(b) The second person picks a black marble from box b0 . In case (a), since the first
and second person have picked white marbles from b and b0 , respectively, the boxes
do not make any difference for the third person to pick a marble from. Thus, by the
lemma above, the third person picks a white marble from one of the boxes with the
probability of 14 . So if we use S1 to denote the event that the third person survives
in this case, we will have
1 4 1 1
P (S1 ) = × × = .
2 9 4 18
In case (b), letting C = W1 ∩ B20 , in view of Bayes’ Theorem, we can write
P (C|B011 )P (B011 )
P (B011 |C) =
P (B011 )P (C|B011 ) + P (B001 )P (C|B001 )
2 2 1
3 × 3 × 2 4
= 1 2 2 1 1 1 = 5.
2 × 3 × 3 + 2 × 3 × 3
190 2. SOLUTIONS

Thus, P (B001 |C) = 1 − P (B011 |C) = 15 . Consequently, the probability that the
third person picks a white marble from b is equal to
1 4 1 2
P (B001 |C)P (W3 B001 |C) + P (B011 |C)P (W3 B011 |C) = × 0 + × = .
5 5 2 5
Likewise, the probability that the third person picks a white marble from b0 is equal
to
1 4 1 3
P (B001 |C)P (W30 B001 |C) + P (B011 |C)P (W30 B011 |C) = × 1 + × = .

5 5 2 5
Thus, the third person should pick a marble from b0 , which is the box from which
the second person picked a marble. So, the probability of survival for the third
person in this case, denoted by P (S2 ), is equal to
1 5 3 1
P (S2 ) = × × = .
2 9 5 6
(ii) the first person does not survive.
Again, we may assume that the first person has picked a black marble from
box b. There are two subcases to consider. (a) The second person picks a white
marble from box b; and (b) The second person picks a black marble from box b. In
case (a), letting C = B1 ∩ W2 , in view of Bayes’ Theorem, we can write

P (C|B001 )P (B001 )
P (B001 |C) =
P (B001 )P (C|B001 ) + P (B011 )P (C|B011 )
2 1 1
3 × 2 × 2 1
= 1 2 1 1 1 = .
2 × 3 × 2 + 2 × 3 ×1
2

Thus, the boxes do not make any difference for the third person to pick a marble
from. So the probability of picking a white marble becomes 42 = 12 . And hence, if
S3 denotes the event that the third person survives in this case, we will have
1 2 1 1
P (S3 ) = × × = .
2 3 2 6
Finally, if the first and second person both pick black marbles from b, then the
third person can survive by picking the remaining white marble from box b. So the
probability of survival in this case, which is denoted by P (S4 ), is equal to
1 1 1
P (S4 ) = × ×1= .
2 3 6
Consequently, the probability of survival for this person, denoted by P (S), is
equal to
1 1 1 1 5
P (S) = P (S1 ) + · · · + P (S4 ) = + + + = .
18 6 6 6 9
That is, the probability of survival for the third person is the same as that of the
second person. 
2.24. TWENTY FOURTH COMPETITION 191

2.24.2. Second Day. 1. (a) Use B to denote the closure of B with respect
to the Euclidean norm of Rn . As B is compact because it is bounded and closed,
there is a positive real r such that

diam(B) := sup ||a − b|| : a, b ∈ B = 2r, .

As diam(B) = diam(B), in view of the hypothesis, we see that for all n ∈ N there
are xn , yn ∈ B and an open ball Brn (zn ), the ball centered at zn with radius rn ,
such that xn , yn ∈ Brn (zn ) ⊆ B and ||xn − yn || > 2r − n1 . Since zn ’s are in the
compact set B, if necessary by passing to a subsequence of (zn )+∞ n=1 , we may assume
that there is a z ∈ B such that limn zn = z. We claim that B = Br (z), the open ball
with radius r centered at z, proving the assertion. First, we prove that Br (z) ⊆ B.
To this end, let x ∈ Br (z) be arbitrary. We can write ||x − z|| = r −  for some
 > 0. As limn n1 = 0, there is an N ∈ N such that N1 <  and ||z − zN || < 2 . We
have

||x − zN || ≤ ||x − z|| + ||z − zN || < r −  +
2
 1 ||xN − yN ||
= r− <r− < ≤ rN ,
2 2N 2
implying that x ∈ BrN (zN ) ⊆ B. In other words, x ∈ B, and hence Br (z) ⊆ B.
To prove that B ⊆ Br (z), we argue by contradiction. Suppose that there is a point
x ∈ B \ Br (z) so that ||x − z|| > r. Join the two points x, z to intersect the closed
ball B r (z) at two antipodal points y, y 0 ∈ B r (z) so that y and y 0 are, respectively,
on and off the line segment that joins x and z. It is obvious that
||x − y 0 || = ||x − z|| + ||z − y 0 || = ||x − z|| + r > 2r = diam(B) = diam(B),
which is a contradiction because x, y 0 ∈ B. Therefore, B ⊆ Br (z), and hence
B = Br (z), completing the proof.

(b) First solution: Let X = (C[0, 1], ||.||∞ ) be the normed space of all real valued
continuous functions on the closed interval [0, 1] equipped with the uniform norm.
As is well-known, X is a Banach space, and hence in particular a complete metric
space. Let Y = (C[0, 1], d) be the metric space equipped with the metric d which is
defined on C[0, 1] by
||f − g||∞
d(f, g) := ,
1 + ||f − g||∞
where ||f − g||∞ = supx∈[0,1] |f (x) − g(x)|. It is readily verified that Y is a complete
metric space. We claim that the set B = {f ∈ C[0, 1] : f (x) > 0} is a bounded
subset of Y with the property that for each pair of points x, y in B, there exists
an open ball U such that U ⊆ B and x, y ∈ U and yet B is not an open ball in
Y . Plainly, Y is bounded, and hence  so is B. Now, suppose f, g ∈ B are  arbitrary.
Set M = supx∈[0,1] max f (x), g(x) and m = inf x∈[0,1] min f (x), g(x) . We have
0 < m, M ∈ R because f and g are continuous on the compact interval [0, 1].
x
Define t : [0, +∞) → [0, +∞) by t(x) = 1+x . First, we show that f, g ∈ Br (h) ⊆ B,
M− m
where r = t( 2 2 ) and h ∈ B is defined by h(x) = M +m
2 for all x ∈ [0, 1]. To
see f ∈ Br (h), we need to show that d(f, h) < r. To this end, note first that
m ≤ f (x) ≤ M implying that 3m m
4 < f (x) < M + 4 for all x ∈ [0, 1], from which,
192 2. SOLUTIONS

M− m
we easily obtain ||f − h||∞ < 2
2
. But t is strictly increasing on [0, +∞). This
yields
 M−m 2

d(f, h) = t ||f − h||∞ < t = r,
2
as desired. Likewise, we see that g ∈ Br (h). Next, we need to show that Br (h) ⊆ B.
To this end, let k ∈ Br (h) be arbitrary. It follows that
m
 M− 2

d(k, h) = t ||k − h||∞ < r = t ,
2
which yields
M−m 2 M +m M−m 2
− < k(x) − < ,
2 2 2
for all x ∈ [0, 1] because t is strictly increasing on [0, +∞). This obtains
3m m
0< < k(x) < M + ,
4 4
for all x ∈ [0, 1], implying that k ∈ B, which is what we want. It remains to prove
that B is not a an open ball. Suppose to the contrary that B = Bt(s) (l), for some
s > 0 and l ∈ B. Now, let k ∈ B = Bt(s) (l) be arbitrary. We see that

d(k, l) = t ||k − l||∞ < t(s),
which obtains
−s < k(x) − l(x) < s,
for all x ∈ [0, 1]. Consequently, supx∈[0,1] k(x) < s+M0 , where M0 = supx∈[0,1] l(x).
This is a contradiction because k ∈ B is arbitrary. Therefore, B is a bounded subset
of the complete metric space Y = (C[0, 1], d) with the property that for each pair
of points f, g in B there exists an open ball Br (h) such that f, g ∈ Br (h) ⊆ B and
yet B is not an open ball in Y , proving the claim.

Second solution: A simpler proof similar to that of first solution shows that the
|x−y|
open interval (0, +∞) of the complete metric space (R, d), where d(x, y) := 1+|x−y|
and |.| denotes the absolute value function, is a counterexample to (a) when Rn is
replaced by the complete metric space (R, d). We omit the details for the sake of
brevity.

Third solution: Let X = {A1 , . . . , A5 }, where A1 = (1, 0, 0), A2 = (0, 1, 0),


A3 = −A1 , A4 = −A2 , and A5 = (0, 0, 34 ). Note that if we use d and s to,
respectively, denote the diameter and side √ length of the square whose vertices are
A1 , A2 , A3 , A4 , we have d = 2 and s = 2. Also note that s < A5 Ai = 53 < d for
each i = 1, 2, 3, 4. It is readily checked that X with respect to the Euclidean metric
of R3 is a complete metric space. Set B = {A1 , . . . , A4 }. It is now easily verified
that B is a bounded subset of the complete metric space X with the property that
for each pair of points x, y in B, there exists an open ball Br (z), where r = s+A25 A1 ,
such that x, y ∈ Br (z) ⊆ B and yet B is not an open ball in X, which is what we
want. 

2. First solution: It might be worth noting that the hypothesis that f is continu-
ous is redundant. To prove the assertion, it is enough to assume that f is integrable
2.24. TWENTY FOURTH COMPETITION 193

Rb
on [a, b]. Without loss of generality, we may assume that a f (t)dt > 0. For given
k ∈ (0, 1), define the continuous function gk : [a, b] −→ R by
Z x Z b
gk (x) = f (t)dt − k f (t)dt.
a a
Rb Rb
We have gk (a) = −k a f (t)dt < 0 and gk (b) = (1 − k) a f (t)dt > 0. Thus, by the
Intermediate Value Theorem, there is a ck ∈ (a, b) such that g(ck ) = 0, proving
the assertion.

Second solution: Define g : [a, b] −→ R by


Rx
f (t)dt
g(x) = Rab .
a
f (t)dt
Plainly, g is continuous on [a, b] and g(a) = 0 < 1 = g(b). Thus, by the Intermediate
Rc
Value Theorem, there exists a ck ∈ (a, b) such that g(ck ) = k, implying a k f (t)dt−
Rb
k a f (t)dt = 0, as desired. 

3. (a) The inequality is known as the Frobenius Inequality. We prove the assertion
over division rings. Let D be a division ring and A ∈ Mm×n (D), B ∈ Mn×p (D), C ∈
Mp×q (D), where m, n, p, q ∈ N. View A (resp. B, C) as a linear transformations
acting on the left of Dn (resp. Dp , Dq ), the right vector space of all n × 1 (resp.
p × 1, q × 1) column vectors over D. Let A1 = A|B(Dp ) . We can write
dim ker A1 |BC(Dq ) ≤ dim ker A1 = dim B(Dp ) − dim A1 B(Dp ) ,


which, in view of A1 B(Dp ) = AB(Dp ), yields
dim ker A1 |BC(Dq ) ≤ r(B) − r(AB).
On the other hand,
dim BC(Dq ) − dim A1 BC(Dq )

dim ker A1 |BC(Dq ) =
= r(BC) − dim ABC(Dq ) = r(BC) − r(ABC).
Therefore,
r(BC) − r(ABC) ≤ r(B) − r(AB),
implying
r(BC) + r(AB) ≤ r(ABC) + r(B),
as desired.

(b) To prove the assertion which is known as the Sylvester Inequality, just set
A = A, B = In , and C = B in (a). 

4. Let I and K be a two-sided ideal and a left ideal of the ring R, respectively.
It is plain that KI ⊆ I ∩ K. To prove I ∩ K ⊆ KI, choose an arbitrary element
x ∈ I ∩ K. It suffices to show that x ∈ KI. To this end, by the hypothesis, we have
(I ∩ K)2 = I ∩ K because I ∩ K is a left ideal of R. It follows that x ∈P (I ∩ K)2 .
n
And hence there are n ∈ N and xi , yi ∈ I ∩ K (1 ≤ i ≤ n) such that x = i=1 xi yi .
But xi yi ∈ KI for all 1 ≤ i ≤ n. Therefore, x ∈ KI, which is what we want,
completing the proof. 
194 2. SOLUTIONS

5. First solution: First, suppose that the number of wins of no two teams are
equal. As there are n teams and any team has played against all of the other n − 1
teams, we see that for each i = 0, . . . , n − 1, there is exactly one team, which we
call the ith team, whose wins is equal to i. It is obvious that the (n − 1)st team has
won all the other n − 1 teams, the (n − 2)nd team losses to the (n − 1)st team and
wins all the remaining n − 2 teams, and so on and so forth. For the ith team, where
i ∈ {0, 1, . . . , n − 1}, let’s call i to be the label of the team. Thus, for any three
teams, the team with the smallest label has lost to the other two teams. Therefore,
there are no three teams A, B, C such that A wins B, B wins C, and C wins A.
Next, suppose that there are no such three teams. Define the relation ≥ on the
teams as follows. For two teams A and B, we write A ≥ B if and only if A = B
or A wins B. Since there are no such three teams, in view of the hypothesis, it is
easily verified that the relation ≥ is a linear order on the set of the participating
teams. So if we put the teams in the decreasing order, say T1 > T2 > · · · > Tn ,
then Ti , the ith team, has won exactly n − i + 1 teams, namely, Ti+1 > · · · > Tn .
Consequently, no two teams have scored the same number of wins, which is what
we want.

Second solution: First, suppose that there are two teams A and B that have
scored k wins, where k ∈ {0, . . . , n − 1}. Without loss of generality, we may assume
that A wins B. We claim that there is a team C such that A wins B, B wins C,
and C wins A. To see this, as B has won k teams, one of them, say C, must have
won A, for otherwise A must have won k + 1 teams, the k teams lost to B plus B
itself, which is impossible. Thus, such team C exists, settling the implication.
Next, suppose that there are three teams A , B, and C such that A has won
B, B has won C, and C has won A. We prove the assertion by induction on n, the
number of participating teams. If n = 3, the assertion is trivial. Assuming that the
assertion holds for n − 1, to prove the assertion for n, argue by contradiction. So
no two teams have scored the same number of wins. Consequently, there is exactly
one team, say D, that has scored n − 1 wins, for n teams have participated in the
game. Now, D is not one of A, B, or C because D has won them all. Exclude D
and consider the game between the remaining n − 1 teams which include A, B, C.
It follows from the induction hypothesis that there are two teams in the remaining
n − 1 teams whose wins are equal. But the two teams have both lost to D. Thus,
the two teams have scored the same number of wins in the original game with n
teams, which is a contradiction. So the assertion follows. 

6. We convert this problem into the following coin-flipping game of which the
problem is a special case. We then present a proof of the counterpart of the assertion
for the coin-flipping game, which we have taken from “Concrete Mathematics”, a
book by Ronald L. Graham, Donald E. Knuth, and Oren Patashnik. Here is the
counterpart of the problem.

Two persons, A and B, are playing the following coin-flipping game.


First, A chooses a pattern of length ` (` ≥ 3) of heads and tails (for instance,
HT H, where H stands for heads and T for tails). Then B, who knows the pattern
chosen by A, chooses a different pattern of the same length of heads and tails. Then
a fair coin is flipped until one of the patterns is first obtained, in which case the
the player whose pattern occurs first is to win the game. Show that no matter what
2.24. TWENTY FOURTH COMPETITION 195

choice is made by A, there is a choice for B so that the probability of winning the
game by B is greater than 21 .

Denote, respectively, by SA and SB the sum of A’s and B’s winning positions.
Use N to denote the sum of the patterns each of which does not contain any
occurrences of the patterns A and B chosen by A and B, respectively. For instance,
if A chooses A = HT H and B chooses B = T T H, we have
SA =
HT H + HHT H + T HT H + HHHT H + T T HT H + T HHT H + · · · ,
SB =
T T H + HT T H + T T T H + HHT T H + T T T T H + T HT T H + HT T T H + · · · ,
N =
1 + H + T + HH + T T + T H + HT + HHH + HHT + T HT + T T T + · · · .
Obviously, if we set H = T = 21 , the resulting values for SA and SB , respectively,
become the probability that A and B wins the game. We have
1 + N (H + T ) = N + SA + SB ,
`
X `
X
NA = SA A(`−k) δA(k) ,A(k) + SB A(`−k) δB(k) ,A(k) ,
k=1 k=1
`
X `
X
NB = SA B (`−k) δA(k) ,B(k) + SB B (k−`) δB(k) ,B(k) ,
k=1 k=1

where δα,β denotes the Kronecker delta, A and A(k) (resp. B (k) and B(k) ) denote
(k)

the last and the first k characters of A (resp. B). To see the first equality, just note
that every term on the left side of it either ends with A, or B, or it does not end
with either of A and B meaning that the term belongs to N ; conversely, every term
on the right of the first equality, is either empty or it belongs to N H or N T . The
second equality holds because every term on the left either completes a term of SA
in such a way that the last k characters of A coincides with its first k characters
for some 1 ≤ k ≤ `, or a term of SB in such a way that the last k characters of
B equals the first k characters of A for some 1 ≤ k ≤ `; and conversely because
every term on the right belongs to the left. Analogously, the third equality holds.
As noted in the above, by setting H = T = 12 , we obtain the wining probabilities
for A and B, which we denote by P (A) and P (B), respectively. It follows from the
first equality above that P (A) + P (B) = 1. Let
`
X `
X
A:A = 2k−1 δA(k) ,A(k) , B:A = 2k−1 δB(k) ,A(k) ,
k=1 k=1
`
X `
X
A:B = 2k−1 δA(k) ,B(k) , B:B = 2k−1 δB(k) ,B(k) .
k=1 k=1
Using the second and third equalities, we can write
 
N = 2 P (A)(A:A) + P (B)(B:A) ,
 
N = 2 P (A)(A:B) + P (B)(B:B) ,
196 2. SOLUTIONS

which obtains
P (A) B:B − B:A
= .
P (B) A:A − A:B
We now claim that if A chooses the pattern A = τ1 τ2 · · · τ` , then B has a bet-
ter chance of winning the game by choosing B = τ20 τ1 · · · τ`−1 , where τ20 is the
heads/tails opposite of τ2 . It suffices to show that P (A) < P (B). Suppose the
contrary, implying that
B:B − B:A ≥ A:A − A:B. (∗)
`−1 `−1 `−3 `−2 (`)
Note that A:A ≥ 2 and B:B < 2 + 2 , and B:A ≥ 2 , for A = A(`) ,
B (`) = B(`) but B (`−2) 6= B(`−2) , and B (`) 6= A(`) but B (`−1) = B(`−1) . It follows
that B:B − B:A < 2`−1 + 2`−3 − 2`−2 . Since A(`) 6= B(`) and A(`−1) 6= B(`−1) , we
conclude A(`−2) = B(`−2) , for otherwise A:B ≤ 2`−3 , from which, we obtain
A:A − A:B ≥ 2`−1 − 2`−3 ≥ 2`−1 + 2`−3 − 2`−2 > B:B − B:A,
which is impossible. Consequently, A(`−2) = B(`−2) , yielding τ20 = τ3 , τ1 = τ4 , τ2 =
τ5 , τ3 = τ6 , . . . , τ`−3 = τ` . But then, A:A ≥ 2`−1 + 2`−4 + · · · , A:B ≤ 2`−3 + 2`−6 +
· · · , B:A ≥ 2`−2 + 2`−5 + · · · , and B:B < 2`−1 + 2`−4 + · · · , implying that
A:A − A:B ≥ (2`−1 − 2`−3 ) + (2`−4 − 2`−6 ) + · · ·
> (2`−1 − 2`−2 ) + (2`−4 − 2`−5 ) + · · ·
> B:B − B:A.
In other words, A:A − A:B > B:B − B:A, which is in contradiction with (∗). There-
fore, P (A) < P (B), proving the assertion. 

2.25. Twenty Fifth Competition

2.25.1. First Day. 1. As |G| = n and [G : Z(G)] = 4, we have


Z(G) $ CG (x) $ G,
for all x ∈ G \ Z(G), where CG (x) := {g ∈ G : xg = gx} denotes the centralizer of
the element x in G. We can write
4 = [G : Z(G)] = [G : CG (x)][CG (x) : Z(G)],
which easily implies [G : CG (x)] = 2. It follows that the
size of any conjugacy class
of
the elements
of G\Z(G) is equal to 2, and
hence 2 G\Z(G) . On the other hand,

G \ Z(G) = 3n . Thus, 2 3n , implying 8 3n, which, in turn, yields 8 n. For a given
4 4
n ∈ N satisfying 8 n, set G := Q8 × Z n8 , where Q8 denotes the quaternionic group
with 8 elements. It is obvious that Z(G) = {−1, 1} × Z n8 , yielding [G : Z(G)] = 4,
as desired. 

2. From T 2 = T , we see that V = ker T ⊕ imT . Now, let α ∈ V be arbitrary. It


follows that there are β ∈ ker T and γ ∈ V such that α = β + T γ. In view of the
hypotheses, we can write
(T + S)α = (T + S)(β + T γ) = T β + T 2 γ + Sβ + ST γ = T γ + Sβ.
Since β ∈ ker T , by the hypothesis, there is a δ ∈ V such that β = Sδ. So we can
write
(T + S)α = T γ + Sβ = T γ + S 2 δ = T γ + Sδ = β + T γ = α.
2.25. TWENTY FIFTH COMPETITION 197

Thus, T + S is the identity transformation, for α ∈ V was arbitrary, finishing the


proof. 

3. By the Mean Value Theorem, there is a c ∈ (y, y + 1) such that


f (y + 1) − f (y) = f 0 (c).
As f 00 (t) < 0 for all t ∈ R, f 0 is strictly decreasing on R. Thus, f 0 (y) > f 0 (c)
because y < c. So we can write
f (y + 1) − f (y) = f 0 (c) < f 0 (y) < f (y + 1) − x,
yielding x < f (y), as desired. 

4. Define g : (a, b) −→ R by g(x) = ln 1 + f 2 (x) − x. We have
2f (x)f 0 (x) 2f (x)f 0 (x) − f 2 (x) − 1
g 0 (x) = − 1 = ≥ 0,
1 + f 2 (x) 1 + f 2 (x)
for all x ∈ (a, b). Thus, g is nondecreasing on (a, b), implying that −a = limx→a+ g(x) ≤
limy→b− g(y) = 1 − b. Therefore, −a ≤ 1 − b, yielding b − a ≤ 1. As for an example

for which b − a = 1, just let a = 0 = b − 1 and f (x) = ex − 1 on (0, 1). 

5. Note first that the game stops exactly when the 7th numbered one marble is
drawn from a box. To see this, it is obvious that if the 7th numbered one marble
is drawn from a box, the the game stops because one needs to draw a marble from
box one which is empty. Conversely, if the game stops when a marble numbered i
is drawn from a box, then i = 1. Because otherwise the box numbered i must be
empty for some i ≥ 2, which means seven marbles numbered i must have already
been drawn from the ith box, implying that there are 8 marbles numbered i, a
contradiction.
Now consider the following extended game. Assume that, after a stop in the
original game, the game is continued by choosing the box which is not empty yet
and whose number is minimal among all nonempty boxes, and that the game is
continued in this manner until all the marbles are drawn from all boxes. In this
extended game, to each permutation of the 49 marbles, there corresponds a round of
the game. Conversely, to each round of the game, there corresponds a permutation
of the 49 marbles. Also, a round of the original game continues until all the marbles
are drawn from all boxes only when the number on the last drawn marble, i.e., the
49th drawn marble, reads one, in which case the original game and the extended
game are the same. It is now obvious that the probability of the event that all the
marbles are drawn from all boxes in the original game is equal to that of that same
event in the extended game which is equal to
48!
6!7!6 1
49!
= ,
7!7
7
which is what we want. 

6. Let 0 < c ≤ b ≤ a with a, b, c ∈ N and a + b + c = n for some n ∈ N, be the


side lengths of a desired triangle. The following gives an algorithm for finding a
partition of the number n into the summands 2, 3, and 4 in which the summand 3
appears at least once. As the following algorithm is reversible, the assertion follows.
We have a < b + c, e.g., 7 < 6 + 3. We explain the algorithm in three stages.
198 2. SOLUTIONS

Stage (i): To create summands of 3, subtract one from the three side lengths,
i.e., a, b, c, one at a time and continue this for c + b − a times so that the inequality
a < b + c becomes an equality, e.g.,
7 < 6 + 3 → 6 < 5 + 2 → 5 = 4 + 1.
Stage (ii): To create summands of 4, in the equality obtained in stage (i),
subtract two units and one unit, one at a time, from the left hand side and right
hand side of the equality, respectively. Continue in this manner for a − b times till
a zero appears in the equality , e.g.,
5 = 4 + 1 → 3 = 3 + 0.
Stage (iii): To create summands of 2, in the last equality obtained in stage (ii),
subtract one unit at a time from each nonzero number of the equality and continue
in this way for b − c times till all numbers become zero, e.g.,
3 = 3 + 0 → 2 = 2 + 0 → 1 = 1 + 0 → 0 = 0 + 0.
So, the corresponding partition for a triangle with side lengths 0 < a < b < c
and a + b + c = n is given by n = 4(a − b) + 3(b + c − a) + 2(b − c), e.g., 16 =
4 + 3 + 3 + 2 + 2 + 2. 

2.25.2. Second Day. 1. The assertion is a quick consequence of the following


proposition.
Let R be a unital ring without zero divisors, i.e., ∀a, b ∈ R : ab = 0 ⇒ a = 0
or b = 0, and U (R) denote the multiplicative group of the units of R. Then, every
finite abelian subgroup of U (R) is cyclic.
We present two proofs for this proposition.
First proof. Let G be an abelian subgroup of U (R) and Z denote the prime
ring of R, i.e., Z = {k1 : k ∈ Z}. It follows from the hypothesis that
n
X
Z[G] := { ki gi : n ∈ N, ki ∈ Z, gi ∈ G (1 ≤ i ≤ n)}
i=1
is an integral domain and that G is a subgroup of the multiplicative group of the
units of Z[G]. So without loss of generality, we may assume that R is an integral
domain. To prove the assertion, we need the following lemma.

Lemma. (i) Let G be a group and g ∈ G with ord(g) = n1 · · · nk , where


k ∈ N and nj ’s (1 ≤ j ≤ k) are pairwise relatively prime. Then, there exist
unique g1 , . . . , gk ∈ G such that g = g1 · · · gk , gi gj = gj gi , and ord(gi ) = ni for
each i, j = 1, . . . , k. Conversely, if for g ∈ G, there are g1 , . . . , gk ∈ G satisfying
g = g1 · · · gk , gi gj = gj gi and ord(gi ) = ni for each i, j = 1, . . . , k such that nj ’s
(1 ≤ j ≤ k) are pairwise relatively prime, then ord(g) = n1 · · · nk .
(ii) Let G be a group and g1 , . . . , gk ∈ G be of finite order such that gi gj = gj gi 
for each i, j = 1, . . . , k. Then there is a g ∈ G such that ord(g) = lcm ord(g1 ), . . . , ord(gk ) .
Proof. (i) First, recall that if g ∈ G is such that ord(g) = n for some n ∈ N,
n
Qk
then ord(g i ) = gcd(i,n) for all i ∈ Z. Now, set mi = i6=j=1 nj . As nj ’s (1 ≤ j ≤ k)
are pairwise relatively prime, we see that gcd(m1 , . . . , mk ) = 1 and hence there are
Pk
cj ’s (1 ≤ j ≤ k) in Z such that j=1 cj mj = 1. We note that gcd(ci , ni ) = 1
for each i = 1, . . . , k. To see this, it suffices to show that no prime p divides
2.25. TWENTY FIFTH COMPETITION 199

gcd(ci , ni ). Suppose to the contrary that there is a prime p that divides gcd(ci , ni )
for some 1 ≤ i ≤ k. It follows that p ci and p ni and hence p ci and p mj for all
Pk
j ∈ {1, . . . , k} \ {i}. Consequently, p j=1 cj mj = 1, which is impossible. Thus,
gcd(ci , ni ) = 1 for each i = 1, . . . , k. Now, letting gi = g ci mi , we can write
ord(g) mi ni
ord(gi ) = ord(g ci mi ) = =
gcd(ci mi , ord(g)) gcd(ci mi , mi ni )
ni
= = ni .
gcd(ci , ni )
Also, we can write
P k
cj mj
g1 · · · gk = g c1 m1 · · · g ck mk = g j=1 = g.
Finally, it is obvious that gj ’s (1 ≤ j ≤ k) commute. To see that gj ’s (1 ≤ j ≤ k) are
unique, suppose that there are g10 , . . . , gk0 ∈ G such that g = g10 · · · gk0 , gi0 gj0 = gj0 gi0 ,
and ord(gi0 ) = ni for each i, j = 1, . . . , k. We can write
c mj c mj
g cj mj = (g10 · · · gk0 )cj mj = g 0 1j · · · g 0 kj .
mj Qk
But g 0 i = e whenever i 6= j because ord(gi0 ) = ni and mj = ni . Thus,
P
j6=i=1
k
c mj 1− ci mi
g cj mj = g 0 jj = g0 j j6=i=1
= gj0 ,
for all 1 ≤ j ≤ k, as desired.
As for the converse, suppose
that ord(g) = m. We need to show that m =
n1 · · · nk . Note first that m n1 · · · nk because g n1 ···nk = (g1 · · · gk )n1 ···nk = e. It
thus suffices to show that n1 · · · nk m. To this end, letting 1 ≤ i ≤ k be arbitrary
but fixed, we can write
k
Y
g m = e =⇒ (g1 · · · gk )m = e =⇒ gim = gj−m .
i6=j=1

As gini = e, we get (gim )ni = e, from which, we obtain ord(gim ) ni . On the other

hand,
Q k
k
Y Qk
nj
k
Y Q k
nj −m
(gim ) nj
gl−m
 i6=j=1 i6=j=1
i6=j=1 = = gl = e,
i6=l=1 i6=l=1
Qk Qk 
implying that ord(gim ) i6=j=1 nj . Thus, ord(gim ) gcd ni , i6=j=1 nj = 1, which

yields ord(gim ) = 1. This obtains gim = e for all 1 ≤ i ≤ k. Hence, ni m for all
1 ≤ i ≤ k. It thus follows that n1 · · · nk m because nj ’s (1 ≤ j ≤ k) are pairwise
relatively prime. Therefore, m = ord(g) = n1 · · · nk , which is what we want.

(ii) It is plain that there exists an l ∈ N and primes p1 , . . . , pl such that


ord(gi ) = pm
1
i1
· · · pm
l
il
,
where mij ’s are nonnegative integers (1 ≤ i ≤ k, 1 ≤ j ≤ l). For 1 ≤ i ≤ k, we see
from (i) that there are gi1 , . . . , gil ∈ G such that gi = gi1 · · · gil , that gij ’s (1 ≤ i ≤
m
k, 1 ≤ j ≤ l) all commute, and that ord(gij ) = pj ij . It is obvious that for each
200 2. SOLUTIONS


1 ≤ j ≤ l, there is an 1 ≤ ij ≤ k such that ord(gij j ) = max ord(g1j ), . . . , ord(gkj ) .
Ql
Let g 0 = j=1 gij j . It thus follows from (i) that
l 
0
Y max m1j ,...,mkj 
ord(g ) = pj = lcm ord(g1 ), . . . , ord(gk ) ,
j=1

which is what we want. 

Note that U (R), and hence G, is commutative. Let a ∈ G be such that its
order, say m, is maximal among the elements of G. Let b be an arbitrary element
of G. Since G is commutative, by part (ii) of the above lemma, there is a c ∈ G
whose order is equal to the least common multiple of ord(a) and ord(b). It follows
that ord(c) = ord(a) because ord(a) is maximal. Thus, ord(b) ord(c) = ord(a) = m,
yielding bm = 1. In other words, every element of G is a root of f = xm − 1 ∈ R[x].
As R is an integral domain, f has at most m roots, and hence |G| ≤ m. Therefore,
|G| = m = ord(a). That is, G is cyclic, as desired.

Second proof. Just as we saw in the first proof, we may assume that R is an
integral domain. Consequently, the equation xn = 1 has at most n solutions in G
for all n ∈ N. It thus follows from the lemma presented in Solution 1 of 2.2.2 that
G is cyclic, as desired. 

2. Let q be a prime factor of 2p + 1 which is different from 3. We have q 2p + 1, and
q
hence q 22p − 1, yielding 22p ≡ 1. Use t = ordq (2) to denote the order of 2 modulo
q. It follows that t divides 2p. By showing that t 6= 1, 2, p, we conclude that t = 2p.
q q
Firstly, t 6= 1 because 21 6≡ 1. Secondly, t 6= 2 because 22 6≡ 1, for q 6= 3. Thirdly,
q
t 6= p because otherwise 2p ≡ 1, which, in view of q 2p + 1, obtains
∗ q 2, which is

impossible, for q is odd. Thus,
t = 2p. Now, since t = ordq (2) |Zq | = q − 1, where

Zq = Zq \ {0}, we have 2p q − 1 or q − 1 = 2kp for some k ∈ N, which is what we

want. 

3. It is plain that f is a one-to-one function from X onto X. Thus, f is invertible.


Use f −1 to denote the inverse of f . It follows from the hypothesis that
d f −1 (x), f −1 (y) ≤ d(x, y),


for all x, y ∈ X. Thus, the function f −1 : X −→ X is a Lipschitz function, and


hence it is (uniformly) continuous. Now, since f −1 is continuous and X is compact,
we see that f −1 is a closed map, implying that f = (f −1 )−1 is continuous, which
is what we want. 

4. Let α = f (1) and β = f (i). By the hypothesis, |f (z)| = |z|, |f (z) − α| = |z − 1|,
and |f (z) − β| = |z − i| for all z ∈ C. In particular, by substituting z = 1, i in the
above equalities, we obtain

|α| = |β| = 1, |α − β| = 2.
We can write
α2 + β 2 = α2 β β̄ + β 2 αᾱ = αβ(αβ̄ + β ᾱ)
= αβ αᾱ + β β̄ − (α − β)(ᾱ − β̄) = αβ |α|2 + |β|2 − |α − β|2
 

= αβ(1 + 1 − 2) = 0,
2.25. TWENTY FIFTH COMPETITION 201

yielding β = εα, where ε = ±i. Simplify |f (z) − α| = |z − 1| and |f (z) − β| = |z − i|


to, respectively, obtain ᾱf (z) + αf (z) = z + z̄ and ᾱf (z) − αf (z) = −εiz + εiz̄ for
all z ∈ C. Adding up these two equalities, we obtain
2ᾱf (z) = (1 − εi)z + (1 + εi)z̄,
which, in view of α = f (1) and ε = ±i, yields f (z) = f (1)z for all z ∈ C or
f (z) = f (1)z̄ for all z ∈ C, as desired. 

5. Let n = 2k − 1 for some k ∈ N and Kn denote the n × n matrix whose elements


are all 1. For a given n × n matrix A with entries in {−1, 1}, let’s call
n
X n
X
C(A) := ai + bj
i=1 j=1

the content of the matrix A, where ai ’s and bj ’s are as in the statement of the
problem. We have
C(Kn ) = n + n = 4k − 2.
It is plain that any n × n matrix A, whose entries are in {−1, 1}, can be obtained
from Kn within a finite number of steps so that at each step a fixed entry, say,
aij = 1, of Kn is changed to −aij = −1. After each step, the number two will be
added to or will be subtracted from each of the two sums in the definition of the
content. Thus, the content remains intact or it will be increased or decreased by
four at each step. Therefore, the content of any n × n matrix A will always be of
the form 4k 0 − 2, for some k 0 ∈ Z, which is never zero. This proves the first part of
the assertion. As for the case when n is even, the assertion does not hold because
for the matrix A defined by
 
Km −Km
A = ,
Km Km
n
where m = 2, we have C(A) = 0. 

6. For a member a ∈ S, use A(a) to denote the set of all of the acquaintances
of a. It follows from the hypothesis that A(a) 6= A(b) whenever a 6= b. Proceed
by contradiction. Let x ∈ S be a member for which A(x) is maximal among all
members of S. Since S is ideal but S \ {x} is not ideal, there are two distinct
members p, q ∈ S \ {x} such that
A(p) = A(q) \ {x}, x ∈ A(q). (∗)
Likewise, since S is ideal but S \ {p} is not ideal, there are two distinct members
r, s ∈ S \ {p} such that
A(r) = A(s) \ {p}, p ∈ A(s). (∗∗)
We claim that r = x. By (∗∗), p and s are acquainted. This, in view of (∗), implies
that s 6= x and that s and q are acquainted. But since, by (∗∗), r is acquainted
with all of the acquaintances of s but p, we see that r and q are acquainted. This
implies r = x because r is not acquainted with p. Thus, A(x) = A(s) \ {p}. That
is, the number of the acquaintances of s is greater than those of x by one, which is
a contradiction, proving the assertion. 
202 2. SOLUTIONS

2.26. Twenty Sixth Competition

2.26.1. First Day. 1. First solution: Define g : [a, b] −→ R by g(x) =


f (x) + x − (a + b). The function g is continuous on [a, b] and differentiable on (a, b).
Moreover, g(a) = a − b < 0 and g(b) = b − a > 0. So, by the Intermediate Value
Theorem, there is a c ∈ (a, b) such that g(c) = 0, which means f (c) = a + b − c.
It now follows from the Mean Value Theorem that there are x1 , x2 ∈ (a, b) with
x1 6= x2 such that
f (a) − f (c) c−b f (c) − f (b) a−c
f 0 (x1 ) = = , f 0 (x2 ) = = ,
a−c a−c c−b c−b
yielding f 0 (x1 )f 0 (x2 ) = 1, proving the assertion.

Second solution: Let g = f ◦ f : [a, b] −→ R. We have g(a) = a and g(b) = b. By


the Mean Value Theorem, there is a c ∈ (a, b) such that
g(b) − g(a)
1= = f 0 (c)f 0 (f (c)),
b−a
yielding f 0 (c)f 0 (f (c)) = 1. There are two cases to consider. If f (c) 6= c, then the
assertion is proved by letting x1 = c and x2 = f (c). If f (c) = c, then using the
Mean Value Theorem for f on the intervals [a, c] and [c, b], respectively, we see that
there are x1 ∈ (a, c) and x2 ∈ (c, b) such that
f (c) − f (a) f (b) − f (c)
f 0 (x1 ) = = 1, f 0 (x2 ) = = 1,
c−a b−c
yielding f 0 (x1 ) = f 0 (x2 ) = 1, and hence f 0 (x1 )f 0 (x2 ) = 1, proving the assertion. 

2. Suppose to the contrary that f has no zero in U . Define g : U −→ C by


1
g(z) = f (z) . It follows that g is analytic on U . Moreover, g(0) = 1 and |g(z)| ≤ 21 on
|z| = 1. This contradicts the Maximum Modulus Principle, proving the assertion.


3. Since gcd(p, 2p−2) = 1, it suffices to prove the congruence modulo p and modulo
p
2p − 2. Note that, by Fermat’s Little Theorem, (2p − 2)p−1 ≡ 1. This, together
p p
with pp−1 ≡ 0, yields pp−1 + (2p − 2)p−1 ≡ 1. Also, note that
pp−1 − 1 = (p − 1)(pp−2 + · · · + p + 1) = 2k(p − 1),
2p−2 2p−2
which implies pp−1 ≡ 1. This, together with (2p − 2)p−1 ≡ 0, yields pp−1 +
2p−2 p(2p−2)
(2p − 2)p−1 ≡ 1. Therefore, pp−1 + (2p − 2)p−1 ≡ 1, as desired. 

4. We need to show that for all x, y ∈ R, x divides y or y divides x. To prove


this by contradiction, suppose that there are x, y ∈ R such that x - y and y - x. It
follows from the hypothesis that there is a nonzero z ∈ R such that Rx + Ry = Rz.
Thus, there are r, s ∈ R such
that x = rz and y = sz. The elements r, s ∈ R are not
units because otherwise x y or y x, which is not possible. Therefore, the elements
r and s belong to M , the only maximal ideal of R. From Rx + Ry = Rz, we see
that there are a, b ∈ R such that z = ax + by. This, together with x = rz and
y = sz, yields z = (ar + bs)z, implying 1 − (ar + bs) z = 0. On the other hand,
1 − (ar + bs) is a unit in R, for otherwise 1 − (ar + bs) ∈ M , yielding 1 ∈ M , which
is a contradiction. Therefore, z = 0, a contradiction. So the assertion follows. 
2.26. TWENTY SIXTH COMPETITION 203

5. For the sake of brevity, we say a k-cover for the set S to mean a k-element cover
for the set S. It is plain that any element of a minimal cover of S is nonempty and
that it includes exactly one element of S by itself. Therefore, a minimal (n − 1)-
cover for the set S is either a partition of the set S, or there exists exactly one
element x ∈ S which belongs to two or more elements of the (n − 1)-cover. In the
first case, the number of such (n − 1)-covers of S, i.e., such partitions of S into
n n

(n − 1) subsets, is equal to 2 . In the latter case, there are 1 ways to choose
x ∈ S and 2n−1 − n1 = 2n−1 − n ways to put x in the elements of the (n − 1)-cover


so that x is put in at least two elements of the cover. So, we must have
   
n n  n
M (n, n − 1) = + 2n−1 − n = (2n − n − 1),
2 1 2
which is what we want. 

f (Y ) : Y ∈ X {m} , R ⊆ Y ⊆ S . We prove the following more


P
6. Let g(R, S) =
general proposition.
Under the hypotheses of the problem, g(R, S) = 0 whenever
R ⊆ S and S \R ≥ k. The assertion is a consequence of the above proposition. To
see this, note that for every T ∈ X {m} , there is an S ∈ X {m+k} such that T ⊆ S.
This implies f (T ) = g(S, T ) = 0, as desired. We prove the above proposition by
induction on |R|. If |R| = 0, assuming that |S| ≥ k, we have
 
|S| X
g(∅, S) = g(∅, T ) = 0,
k {k}
T ∈X ,T ⊆S

for, by the hypothesis, g(∅, T ) = Y ∈X {m} ,Y ⊆T f (Y ) = 0 for all T ∈ X {k} such


P
that T ⊆ S. On the other hand, the following recurrence equation holds.
 
g(R, S) = g R \ {α}, S − g R \ {α}, S \ {α} ,
for all α ∈ R. Therefore, the assertion follows by induction. 

2.26.2. Second Day. 1. Let a ∈ [0, 1] \ Q be arbitrary. We show that f is


P+∞
continuous at a. To this end, for a given ε > 0, choose k ∈ N such that i=k 21i < ε
and let 
δ = min a − r1 , . . . , a − rk .
Note that δ > 0. If a − δ < x ≤ a, we can write
+∞

f (x) − f (a) =
X 1 X 1
≤ < ε.
2i 2i
i∈{n∈N:x<rn ≤a} i=k+1

Analogously, if a ≤ x < a + δ, then


+∞

f (x) − f (a) =
X 1 X 1
≤ < ε.
2i 2i
i∈{n∈N:a<rn ≤x} i=k+1

That is, limx→a f (x) − f (a) = 0. Thus, f is continuous at a, which is what we
want. 

2. It easily follows from the hypothesis that f is one-to-one. Since f is continuous,


we conclude that f is strictly increasing or decreasing on R. But f is not strictly
decreasing, for otherwise f ◦ f would be strictly increasing and hence f ◦ f ◦ f = I
204 2. SOLUTIONS

would be strictly decreasing, which is impossible. Thus, f is strictly increasing on


R. Now, to prove the assertion by contradiction, suppose that there is an a ∈ R
such that f (a) 6= a. Two cases
 to consider. If a < f (a), we see that f (a) < f (f (a)),
 
yielding f (a) < f f (a) < f f f (a) = a, a contradiction. Likewise, if a > f (a),
we obtain a contradiction. So, the assertion follows by way of contradiction. 

3. Suppose by way of contradiction that there is an x ∈ H ∩ K \ {e}, where e


denotes the identity element of G. Since H is abelian and Z(G) = {e}, we have
H ⊆ CG (x) & G, where CG (x) denotes the centralizer of the element x in G. Thus,
H = CG (x) because H is maximal. Likewise, we see that K = CG (x). Therefore,
H = K, which is a contradiction. So the assertion follows. 

4. We prove the assertion by showing that An = λI, where I denotes the identity
matrix of size n. Let F n denote the n-dimensional vector space of all n × 1 column
vectors with entries in F . By the hypothesis, the vectors x, Ax, . . . , An−1 x are
linearly independent and hence they form a basis for F n . We can write
An (Ai x) = Ai (An x) = λAi x,
implying
(An − λI)(Ai x) = 0,
for all 0 ≤ i ≤ n − 1. This yields An − λI = 0, and hence An = λI, because
{x, Ax, . . . , An−1 x} forms a basis for F n .


5. Let (a1 , . . . , a6 ) and (b1 , . . . , b6 ) be the sequences of the numbers appearing on


two extended dice. Let f (x) = xa1 + · · · + xa6 and g(x) = xb1 + · · · + xb6 . It follows
from the hypothesis that
f (x)g(x) = (x1 + · · · + x6 )2 = x2 (1 + x)2 (1 + x + x2 )2 (1 − x + x2 )2 .
As f (0) = g(0) = 0, the factor x must appear in the factorizations of both of f
and g. Since f (1) = g(1) = 6, the factors 1 + x and 1 + x + x2 must appear in the
factorizations of both of f and g as well. Therefore, the only choices which do not
lead to two ordinary dice are as follows
f (x) = x(1 + x)(1 + x + x2 ) = x + 2x2 + 2x3 + x4 ,
g(x) = x(1 + x)(1 + x + x2 )(1 − x + x2 )2 = x + x3 + x4 + x5 + x6 + x8 .
That is, aside from the ordinary dice, the sequences of the numbers on the desired
extended dice are (1, 2, 2, 3, 3, 4) and (1, 3, 4, 5, 6, 8). 

6. In view of the definition of a balanced matrix and the hypothesis that the
balanceness remains intact under interchanging the rows, it follows that for all
j1 < j2 and i1 < i < i2 , we have
 
ci1 j1 + cij2 ≤ ci1 j2 + cij1
⇐⇒ ci1 j1 + cij2 = cij2 + cij1
cij1 + ci1 j2 ≤ cij2 + ci1 j1
⇐⇒ ci1 j1 + cij1 = ci1 j2 + cij2 .
That is, the difference of two rows i and i0 is a constant vector whenever i1 ≤ i ≤
i0 ≤ i2 . Conversely, it follows from the above relations that if the difference of two
rows is a constant vector, then the balanceness remains intact under interchanging
the two rows. 
2.27. TWENTY SEVENTH COMPETITION 205

2.27. Twenty Seventh Competition

2.27.1. First Day. 1. First solution: See Problem 2 of 2.1.1.

Second solution: Since all norms on the n2 -dimensional normed space Mn (R) are
equivalent, it suffices to show that GLn (R) is open and disconnected with respect to
the operator norm of Mn (R). To this end, let A ∈ GLn (R) be arbitrary. By showing
that B ∈ GLn (R) whenever ||B − A|| < ||A−1 ||−1 , where ||.|| denotes the operator
norm of Mn (R), we prove that GLn (R) is open. It is plain that B ∈ GLn (R) if and
only if BA−1 ∈ GLn (R). But ||I − BA−1 || ≤ ||A − B|| ||A−1 || < 1. Thus,
+∞
−1 X
(BA−1 )−1 = I − (I − BA−1 ) = (I − BA−1 )n .
i=1
−1
Consequently, BA ∈ GLn (R), implying B ∈ GLn (R), as desired. To show that
GLn (R) is disconnected, suppose to the contrary that it is connected. Since GLn (R)
is open and connected, it is path connected. Choose A, B ∈ GLn (R) with det(A) =
− det(B) = 1. It follows that there is a continuous function φ : [0, 1] −→ GLn (R)
with φ(0) = A and φ(1) = B. Since  det : Mn (R) −→ R is continuous, so is
det(φ) : [0, 1] −→ R. But det φ(0) = det(A) = 1 and det φ(1) = det(B) = −1.
It thus follows from the Intermediate Value Theorem that there is a c ∈ (0, 1)
such that det φ(c) = 0. This implies φ(c) ∈ / GLn (R), which is a contradiction.
Therefore, GLn (R) is disconnected, proving the assertion. 

2. Let 0 ≤ θ < π be the angle of inclination of the line L, i.e., θ is the angle between
L and the x-axis. Define the function g : C −→ C by g(z) = f (eiθ z + A) − A e−iθ .


It is plain that g is analytic on C, g takes the x-axis to the x-axis and the y-axis
to the y-axis, and that g(0) = 0. We can write
+∞
X
g(z) = an z n ,
n=1

g (n) (0)
where an = n! . Set
+∞
X
ge(z) = g(z̄) = ān z n ,
n=1
Since g maps the x-axis into the x-axis, we obtain
+∞
X
(g − ge)(z) = (an − ān )z n = 0,
n=1

whenever z ∈ R. It thus follows from the Uniqueness Theorem for analytic functions
that g = ge because the zeros of the analytic function g − ge has a limit point in C.
Therefore, an = ān which, in turn, implies an ∈ R for all n ∈ N. Define h : C −→ C
by
+∞
X
h(z) = ig(iz) = an in+1 z n .
n=1
Since g maps the x-axis into the x-axis and the y-axis into the y-axis, it follows
that the analytic function h maps the x-axis into the x-axis. Hence, just as we saw
206 2. SOLUTIONS

in the above, we see that an in+1 ∈ R for all n ∈ N. This yields a2n = 0 for all
n ∈ N. Therefore,
+∞
X
g(z) = a2n−1 z 2n−1 ,
n=1

where a2n−1 ∈ R for all n ∈ N. Obviously, g(−z) = −g(z) for all z ∈ C. That is, if
z1 , z2 ∈ C are symmetric with respect to 0, then so are g(z1 ) and g(z2 ) with respect
to 0. Now, suppose z1 and z2 are symmetric with respect to A. So, z1 + z2 = 2A.
We can write
f (z2 ) = eiθ g e−iθ (z2 − A) + A


= eiθ g − e−iθ (z1 − A) + A = −eiθ g e−iθ (z1 − A) + A


 

= −f (z1 ) + 2A,
implying f (z1 ) + f (z2 ) = 2A. Thus, f (z1 ) and f (z2 ) are symmetric with respect to
A, finishing the proof. 
x −x
3. Note first that sinh : R −→ R defined by sinh(x) = e −e 2 is one-to-one and
onto. Thus, for each n ∈ N, there is a unique αn ∈ R such that an = sinh(αn ). We
can write
q q
sinh(αn+1 ) = sinh(αn ) 1 + sinh2 (αn−1 ) + sinh(αn−1 ) 1 + sinh2 (αn )
= sinh(αn ) cosh(αn−1 ) + sinh(αn−1 ) cosh(αn )
= sinh(αn−1 + αn ),
implying αn+1 = αn−1 + αn for each n ∈ N. The characteristic√
polynomial of
2 1± 5
this recurrence equation is x − x − 1 = 0, yielding x = 2 . Thus, αn =
√  √ 
n n
A 1+2 5 + B 1−2 5 for some A, B ∈ R, where n ∈ N ∪ {0}. To determine A
and B, we have
0 = sinh−1 (0) = α0 = A + B,
√ √
1 + 5 1 − 5
sinh−1 (b) = A +B ,
2 2
sinh−1 (b)
implying A = −B = √
5
. So we have
√ √ !
−1 sinh−1 (b) 1 + 5 n 1 − 5 n
sinh (an ) = αn = √ − ,
5 2 2
yielding
√ √ !!
sinh−1 (b) 1 + 5 n 1 − 5 n
an = sinh √ − ,
5 2 2
for all n ∈ N, which is what we want. 

4. To prove the assertion by contradiction, suppose that there is an i0 ∈ I such that


i0 ∈
/ Ai0 . It follows that {i : i ∈
/ Ai } 6= ∅, from which, in view of the hypothesis, we
see that {i : i ∈
/ Ai } = Ai1 for some i1 ∈ I. Two cases to consider. If i1 ∈ Ai1 , then
i1 ∈
/ {i : i ∈
/ Ai } = Ai1 , a contradiction. If i1 ∈/ Ai1 , then i1 ∈ {i : i ∈
/ Ai } = Ai1 , a
contradiction again. Therefore, i ∈ Ai for all i ∈ I, as desired. 
2.27. TWENTY SEVENTH COMPETITION 207

5. It is easily verified that (AB)2 = 9(AB). We can write


(BA)3 = B(AB)2 A = 9B(AB)A = 9(BA)2 ,
yielding (BA)2 (BA − 9I2 ) = 0. If we use f to denote the minimal polynomial of
BA, we see that f divides x2 (x − 9). As the degree of f is less than or equal to
2, there are four cases to consider. If f = x, then BA = 0, yielding AB = 0, for
A(BA)B = (AB)2 = 9(AB), which is impossible. If f = x2 , then (BA)2 = 0,
implying tr(AB) = tr(BA) = 0, which is again impossible. If f = x(x − 9), then
tr(AB) = tr(BA) = 9, which is impossible. If f = x − 9, we obtain BA − 9I2 = 0.
Thus, BA = 9I2 , which is what we want. 

6. We first show that H is normal in G. To prove this by contradiction, suppose


that there exist g ∈ G and h ∈ H such that g −1 hg ∈ / H. Since g −1 hg ∈ G \ H,
it follows from the hypothesis that there is a u ∈ H such that g(g −1 hg)g −1 =
u−1 (g −1 hg)u, from which, we obtain g −1 hg = uhu−1 ∈ H, a contradiction. Thus,
H is normal in G. To prove that G/H is abelian, it suffices to show that g1 g2 H =
g2 g1 H, or equivalently, g2−1 g1−1 g2 g1 ∈ H for all g1 , g2 ∈ G. If g2 ∈ H, we see
that g2−1 (g1−1 g2 g1 ) ∈ H because H is normal in G. So we may, with no loss of
generality, assume that g2 ∈ / H. Hence, for g1 ∈ G, there is a u ∈ H such that
g1−1 g2 g1 = u−1 g2 u. This implies g2−1 g1−1 g2 g1 = (g2−1 u−1 g2 )u ∈ H, for H is normal
in G. It thus follows that G/H is abelian, as desired. 

2.27.2. Second Day. 1. First solution: Since f 0 (a) > 0, f 0 (b) > 0, and
f (a) = f (b) = 0, there is a δ > 0 such that a + δ < b − δ, f (x) > 0 whenever
a < x < a + δ, and that f (x) < 0 whenever b − δ < x < b. It follows that
f (a + δ) ≥ 0 and f (b − δ) ≤ 0. So by the Intermediate Value Theorem, there is a
c ∈ [a + δ, b − δ] such that f (c) = 0. We have f (a) = f (c) = f (b) = 0. Applying the
Rolle’s Theorem, we obtain c1 ∈ (a, c) and c2 ∈ (c, b) such that f 0 (c1 ) = f 0 (c2 ) = 0,
as desired.

Second solution: Recall that by Darboux’s Theorem, the derivative function


has the intermediate value property. If there is a c ∈ (a, b) such that f 0 (c) < 0,
then f 0 (a)f (c) < 0 and f 0 (c)f 0 (b) < 0. Hence, by Darboux’s Theorem, there are
c1 ∈ (a, c) and c2 ∈ (c, b) such that f 0 (c1 ) = f 0 (c2 ) = 0, proving the assertion in
this case. If f 0 (x) ≥ 0 for all x ∈ (a, b), then f is increasing on [a, b]. This implies
0 = f (a) ≤ f (x) ≤ f (b) = 0, whence f (x) = 0 for all x ∈ [a, b], in which case the
assertion is trivial. So the proof is complete. 

2. First we need to recall the following lemma from theory of metric spaces. Also
recall that a subset of a metric space is said to be perfect if it is closed and has no
isolated points.

Lemma. Every nonempty perfect subset of a complete metric space is uncount-


able.
Proof. Let P be a nonempty perfect subset of a complete metric space X.
As P is closed, P equipped with the metric X induces on P forms a complete
metric space itself. Now to prove the assertion by contradiction, suppose that P
is countable so that P = {xi }+∞i=1 , where xi ∈ X for all i ∈ N. As P is perfect,
each singleton {xi } (i ∈ N) is a nowhere dense subset of P , implying that P is a
208 2. SOLUTIONS

countable union of nowhere dense subsets, namely {xi }’s where i ∈ N. This is in
contradiction with the Baire Category Theorem which asserts that every complete
metric space is of second category in itself, i.e., the space cannot be written as a
countable union of nowhere dense subsets of it. Thus, P is uncountable, which is
what we want. 

To prove the assertion by contradiction, suppose that [0, 1] is written as a


union of mutually disjoint closed intervals each of which having a positive length
less than one. There are a countable number of the intervals participating in the
union because they are mutually disjoint. So we may assume that
+∞
[
[0, 1] = In ,
n=1
where In = [an , bn ] for some an , bn ∈ R with 0 < bn − an < 1 and Im ∩ In = ∅
whenever m, n ∈ N and m 6= n. Set
 +∞  +∞
P = an n=1 ∪ bn n=1 .
The set P is closed. Because if x ∈ [0, 1] \ P , then x ∈ (an , bn ) for some n ∈ N, and
hence x is an interior point of [0, 1] \ P . We now show that any point x ∈ P \ {0, 1}
is a limit point of P . Note that 0 and 1 are isolated points of P . If x = an > 0
for some n ∈ N, then for every 0 < δ < an , there exists an m ∈ N such that
(an − δ, an ) ∩ Im 6= ∅. Since In ∩ Im = ∅, we see that bm ∈ (an − δ, an ). Thus,
x = an is a limit point of P , as claimed. Likewise, if x = bn < 1 for n ∈ N, then x
is a limit point of P . Therefore, P \ {0, 1} is a nonempty perfect subset of R, which
is a complete metric space endowed with the ordinary metric of R. By the lemma
above, the set P \ {0, 1} is uncountable, and hence so is P , which is obviously a
contradiction. This completes the proof. 

3. As D is countable, there are a countable number of lines which are all parallel
to the x-axis so that each of which interests D. Use {`xi }N
i=1 to denote these lines,
x

where `xi : x = xi and Nx ∈ N ∪ {+∞}. Likewise, there are a countable number


Ny
of lines, denoted by {`yj }j=1 , where `yj : y = yj and Ny ∈ N ∪ {+∞}, which are
all parallel to the y-axis so that each of which intersects D. It is obvious that
Nx Ny
D ⊆ A × B, where A = {xi }i=1 and B = {yj }j=1 . Set

Γ = D ∩ (xi , yj ) : j ≤ i ,

∆ = D ∩ (xi , yj ) : j > i .
As D ⊆ A × B, {Γ, ∆} is a partition of D into two subsets. Now, any line parallel
to the x-axis whose equation is given by x = a, where a ∈ R, intersects Γ at no
point or at most at the i points (xi , y1 ), . . . , (xi , yi ) depending on whether a ∈
/ A
or a = xi for some 1 ≤ i ≤ Nx < +∞ or for some i ∈ N if Nx = +∞. Analogously,
any line parallel the y-axis whose equation is given by y = b, where b ∈ R, intersects
∆ at no point or at most at the j − 1 points (x1 , yj ), . . . , (xj−1 , yj ) depending on
whether b ∈/ B or b = yj for some 1 ≤ j ≤ Ny < +∞ or for some j ∈ N if Ny = +∞.
So the assertion follows. 

4. For a subset I ⊆ Nn := {1, . . . , n}, set BI = ( i∈I Ai )c = S \( i∈I Ai ). Also, let


S S
K := {I ⊆ Nn : |I| = k − 1}. We claim that if I, J ∈ K, then BI ∩ BJ = ∅ whenever
I 6= J. To see this, note first that BI ∩ BJ = BI∪J . Since I, J ∈ K and I 6= J,
2.27. TWENTY SEVENTH COMPETITION 209

S
we see that |I ∪ J| ≥ k, which, in view of the hypothesis, yields i∈I∪J Ai = S,
and hence BI ∩ BJ = BI∪J = S \ S = ∅. It is now plain that the number of BI ’s,
n n
where I ∈ K, is equal to k−1 and that such BI ’s form a partition of S into k−1
subsets. In other words,
[˙ 
S= BI : I ∈ K , (∗)
where ˙ stands for the disjoint union.
S
n
 Thus, |BI | = 1 for all I ∈ K, because the
number of such BI ’s is equal to k−1 = |S| and that they are all nonempty. Now,
fix an arbitrary i0 ∈ I. Note that K is a disjoint union of K0 := {I ∈ K : i0 ∈ I}
and K00 := {I ∈ K : i0 ∈ / I}. It is plain that |K0 | = n−1
k−2 and that if x ∈ BI for
some I ∈ K0 (resp. I ∈ K00 ), then x ∈ / Ai0 (resp. x ∈ Ai0 ). This together with (∗),
in view of the fact that |BI | = 1 for all I ∈ K, implies that
     
n n−1 n−1
|Ai0 | = − = .
k−1 k−2 k−1
In other words, Ai = n−1

k−1 for all i ∈ Nn , which is what we want. 

5. Let k = ordab (a + b), m = ordb (a), n = orda (b), and [m, n] = lcm(m, n). We
b a
have am ≡ 1 and bn ≡ 1. Also, [m, n] = mq = nq 0 for some q, q 0 ∈ N. So we can
write
a a 0 a
(a + b)[m,n] ≡ b[m,n] ≡ bnq ≡ 1.
Analogously,
b b b
(a + b)[m,n] ≡ a[m,n] ≡ amq ≡ 1.
ab
Since gcd(a, b) = 1, it follows that (a + b)[m,n] ≡ 1, which implies ordab (a + b) ≤
ab
[m, n] = lcm[ordb (a), orda (b)]. On the other hand, (a + b)k ≡ 1. So, in particular,
a b a a
(a + b)k ≡ 1 and (a + b)k ≡ 1, which, in turn, yields bk ≡ (a + b)k ≡ 1 and
b b
ak ≡ (a+b)k ≡ 1. Thus, m k and n k, and hence lcm[ordb (a), orda (b)] = [m, n] k =
ordab (a + b). Therefore,
ordab (a + b) = lcm[ordb (a), orda (b)],
as desired. 

6. The rings R and R0 are both abelian and have characteristic 2 because they are
Boolean rings. Since f is onto and 0 ∈ R0 , there is an x ∈ R such that f (x) = 0.
We can write
f (0) = f (0x) = f (0)f (x) = f (0)0 = 0.
That is, f (0) = 0. Now, let a, b ∈ R be arbitrary. By the surjectivity of f , there is
an x ∈ R such that f (a + b) − f (a) − f (b) = f (x). We need to show that x = 0.
We can write

f (ax) f (a + b) − f (a) − f (b) = f (ax)f (x) = f (ax),
implying that
f (ax + abx) − f (ax) − f (abx) = f (ax),
and hence
f (ax + abx) = f (abx).
210 2. SOLUTIONS

Since f is one-to-one, we obtain ax + abx = abx, implying that ax = 0. Likewise,


we see that bx = 0. Now, in view of ax = bx = 0, we can write

f (x) f (a + b) − f (a) − f (b) = f (x)f (x) = f (x),
which yields
f (ax + bx) − f (ax) − f (bx) = f (x).
Hence, f (x) = 0 = f (0), from which, in view of the injectivity of f , we see that
x = 0, which is what we want. 

2.28. Twenty Eighth Competition

P+∞
2.28.1. First Day. 1. Let f (z) = n=0 cn z n . If f is a polynomial, then,
by the Fundamental Theorem of Algebra, we have f (z) = c0 + c1 z because f is
one-to-one. If not, then f ( z1 ) would have an essential singularity at zero. Thus, by
Picard’s Great Theorem, f ( z1 ) is not one-to-one in any neighborhood of zero, and
hence neither is f , a contradiction. This proves the assertion. 

2. We prove the assertion by showing that f −1 {(0, 0)} is not bounded, and


hence has infinitely many points. As f is surjective, f −1 {(0, 0)} is nonempty.


Suppose to the contrary that f −1 {(0, 0)} is bounded. Thus, f −1 {(0, 0)} is


a nonvoid closed boundedsubset of R. Consequently,  there exist a, b ∈ R such


that a = min f −1 {(0, 0)} < b = max f −1 {(0, 0)} . Let A1 = {0} × (0, +∞),
A2 = {0} × (−∞, 0), A2 = (0, +∞) × {0}, and A4 = (−∞, 0) × {0}. As f is
continuous and [a, b] is a compact subset of R, we see that f [a, b] is a bounded
subset of A. Now, since f is continuous and f (x) 6= 0 for all x ∈ (−∞, a) ∪ (b, +∞),
there are i1 , i2 ∈ {1, 2, 3, 4}, not necessarily distinct, such that f (−∞, a) ⊆ Ai1
and f (b, +∞) ⊂ Ai2 . Therefore, f (R) ∩ Ai is a bounded subset Ai for each i ∈
{1, 2, 3, 4}\{i1 , i2 }, which is a contradiction because f is surjective. So the assertion
follows. 

3. By the hypothesis and the Möbius Inversion Formula, we have


X n X µ(d)
f (n) = µ(d)( )2 = n2 = n2 g(n),
d d2
d|n d|n
P µ(d)
for all n ∈ N, where g(n) = d|n d2 . The arithmetic function g is multiplicative
µ(d)
because d2 is a multiplicative function of d. If p is prime and α is a positive
integer, we can write
X µ(d) µ(1) µ(p) 1
g(pα ) = 2
= + 2 = 1 − 2.
d 1 p p
d|pα

Thus, if n = p|n pαp , then


Q

Y Y Y 1

αp αp
g(n) = g( p ) = g(p ) = 1− 2
p
p|n p|n p|n
     
Y 1 Y 1 φ(n) Y 1
= 1− 1+ = 1+ .
p p n p
p|n p|n p|n
2.28. TWENTY EIGHTH COMPETITION 211

Consequently,
n2 g(n) n2 φ(n) Y
  Y 
f (n) 1 1
= = 1+ =n 1+ ,
φ(n) φ(n) φ(n) n p p
p|n p|n

for all n ∈ N with n > 1, as desired. 

4. Let 1 ≤ i, k ≤ n be arbitrary and τ ∈ Gk ∩ Z(G). By the hypothesis, there exists


a σ ∈ G such that σ(k) = i. We can write

λ ∈ Gi ⇐⇒ λ(i) = i ⇐⇒ λ σ(k) = σ(k)
⇐⇒ σ −1 λσ(k) = k ⇐⇒ σ −1 λσ ∈ Gk
⇐⇒ λ ∈ σGk σ −1 .
Thus, Gi = σGk σ −1 , and hence στ σ −1 ∈ Gi because τ ∈ Gk . On the other hand,
τ ∈ Z(G) implies τ = στ σ −1 ∈ Gi . It follows that τ ∈ ∩ni=1 Gi = {e} because i was
arbitrary. This proves Gk ∩ Z(G) = {e}, which is what we want. 

5. We prove that the maximum number of such vectors is n. To this end, let
the vectors v1 , . . . , vk ∈ A be such that any two of which share an even number
of one entries. View the elements of A as vectors in the n-dimensional vector
space Zn2 , where Z2 = {0, 1} is the field of integers modulo 2. We claim that
the set {v1 , . . . , vk } is a linearly independent subset of Zn2 . To see this, suppose
c1 v1 + · · · + ck vk = 0 for some c1 , . . . , ck ∈ Z2 . If “.” denotes the usual inner
product on Zn2 , for all 1 ≤ i ≤ n, we have
ci = vi .(c1 v1 + · · · + ck vk ) = 0,
proving the claim. Thus, k ≤ n. On the other hand, if ei is the vector with 1 in
the ith place and zero elsewhere, then any two elements of the set {e1 , . . . , en }, the
standard basis of Zn2 , share an even number of one entries. Therefore, the maximum
number of such vectors is n, as desired. 

6. Let P = {p1 , . . . , pn }. As is usual, use Sym(P ) to denote the set of all per-
mutations on (the poset) P . Let’s view Sym(P ) as a probability space. Choose a
permutation σ on {1, . . . , n} at random which, in turn, gives rise to a permutation
sσ ∈ Sym(P ) at random so that sσ (pi ) = pσi for all 1 ≤ i ≤ n. For any 1 ≤ i ≤ n,
there exists a 1 ≤ j ≤ n such that σj = i, which is equivalent to pi = sσ (pj ) = pσj .
With all that in mind, for an element pi ∈ P for which pi = sσ (pj ) = pσj for some
1 ≤ j ≤ n, let Fpi be the event that the permutation sσ ∈ Sym(P ) satisfies the
following
∀p ∈ P \ Upi : p = sσ (pk ) = pσk =⇒ σj ≤ σk.
In other words, if  
p1 p2 · · · pn
sσ = ,
pσ1 pσ2 · · · pσn
then sσ ∈ Fpi if and only if the element pi precedes all of the elementsTofTP \Upi but
pi in the second row of the above representation of sσ . Set Epi := Fpi ( p∈Lp Fpc ),
i
where Fpc := Sym(P ) \ Fp .
We note that if pi and pi0 are distinct, then the events Epi and Epi0 cannot occur
simultaneously, and hence they are disjoint. Let’s prove this. First, assume that
for pi , pi0 ∈ P , Epi , and hence Fpi , occurs, and that pi and pi0 are not comparable.
Then, in particular pi0 6> pi yielding pi0 ∈ P \ Upi , which, in turn, implies σj < σj 0 ,
212 2. SOLUTIONS

where i = σj and i0 = σj 0 . Consequently, Fpi0 , and hence Epi0 , cannot occur, for
otherwise, we must have σj 0 < σj because pi 6> pi0 , which is a contradiction. That
is, Fpi and Fpi0 , and hence Epi and Epi0 , cannot occur simultaneously whenever
pi and pi0 are not comparable. Next, if the elements pi , pi0 ∈ P are comparable,
then it is plain that the events Epi and Epi0 cannot occur simultaneously. Thus,
the events Epi and Epi0 are disjoint whenever pi , pi0 ∈ P are distinct.
We now use induction on |Lpi | to prove Xpi = P (Epi ). If |Lpi | = 0, i.e., if pi
is a minimal element of P , then Epi = Fpi and sσ ∈ Fpi with sσ (pj ) = pi if and
only if σj ≤ σk whenever
p = s(pk ) = pσk ∈ P \ Upi for some 1 ≤ k ≤ n. As
P \ Up = n − Up , in this case, we can write
i i

n

(|Upi |)! n−|U pi |
(n − |Upi | − 1)! 1
P (Epi ) = P (Fpi ) = = .
n! n − |Upi |
Now, assuming that Xp = P (Ep ) for all p ∈ Lpi , we prove Xpi = P (Epi ). We claim
that \ \
Fpc = Epc ,
p∈Lpi p∈Lpi

for all pi ∈ P . To see this, first, let sσ ∈ p∈Lp Fpc be arbitrary. We show
T
i
sσ ∈ p∈Lp Epc . Suppose the contrary, implying that there is a q ∈ Lpi such that
T
i
sσ 6∈ Eqc . TIt follows that sσT∈ Eq , and hence sσ ∈ Fq . This is in contradiction
with sσ ∈ p∈Lp Fpc . Thus, p∈Lp Fpc ⊆ p∈Lp Epc . Next, letting sσ ∈ p∈Lp Epc
T T
i i i i
be arbitrary, we show sσ ∈ p∈Lp Fpc . Again suppose the contrary, implying that
T
i
there is a minimal q ∈ Lpi so that sσ 6∈ Fqc but sσ ∈ Fpc for all p ∈ T Lq .TIt follows
that sσ ∈ Fq but sσ 6∈ Fp for all p ∈ Lq . Consequently, sσ ∈ Eq = Fq ( p∈Lq Fpc ),
which is a contradiction. Therefore, p∈Lp Epc ⊆ p∈Lp Fpc , proving the claim.
T T
i i
Now, as Ep ’s (p ∈ Lpi ) are disjoint, in view of the claim, we can write
\ \ [ X
Fpc = P Epc = 1 − P
  
P Ep = 1 − Xp .
p∈Lpi p∈Lpi p∈Lpi p∈Lpi

Fpc
T
On the other hand, the events Fpi and p∈Lp are independent because the
i
position of pi in the permutation has no affect on Fp ’s (p ∈ Lpi ). Thus,
P
\ \ \ 1 − p∈Lp Xp
c c
  i
P (Epi ) = P ( Fp ) Fpi = P Fp P (Fpi ) = .
n − |Upi |
p∈Lpi p∈Lpi

Therefore, Xpi = P (Epi ) for all pi ∈ P , and hence 0 ≤ Xpi ≤ 1 for all pi ∈ P ,
which is what we want. 

2.28.2. Second Day. 1. Let g = f −1 . It follows from the hypothesis that


for all x, y with y = 2x2 , we have
Z x √
1 2 3
A = (2t2 − t2 )dt = x3 = y2 =
0 3 12
Z y r √ Z y
t  2 3
B = − g(t) dt = y −
2 g(t)dt,
0 2 3 0
2.28. TWENTY EIGHTH COMPETITION 213

P
B
A

1.5

Figure 13

implying √
Z y
2 3
g(t)dt = y2.
0 4
Taking derivative of both sides yields

3 2 1 9 32 2
x = g(y) = y 2 =⇒ x2 = y =⇒ y = x .
8 32 9
Thus, y = f (x) = 32 2
9 x , which is what we want. 

2. Set h(x) = f (x) − x. We have h(a) > 0 and h(b) < 0. Since h is continuous,
there exists a δ > 0 such that h(a + x) > 0 and h(b − x) < 0 whenever x ∈ (0, δ).
δ
Set α = n+1 and
g(x) = h(x) + h(x + α) + · · · + h(x + nα)
 
= f (x) + · · · + f (x + nα) − x + · · · + (x + nα) .
It is readily seen that g(a) > 0 and g(b − nα) < 0. By the Intermediate Value
Theorem, there exists a c ∈ (a, b − nα) such that g(c) = 0. This implies
f (c) + f (c + α) + · · · + f (c + nα) = c + (c + α) + · · · + (c + nα)
n(n + 1) n
= (n + 1)c + α = (n + 1)(c + α),
2 2
which is what we want. 

3. The answer is yes. To see this, noting that det(A + B) = det(At + B t ), we can
write
det(A) det(A + B) = det(A) det(At + B t ) = det(I + AB t )
= det(I + BAt ) = det(B) det(At + B t )
= det(B) det(A + B).
214 2. SOLUTIONS

Consequently, in view of det(A) + det(B) = 0, we have


det(A) det(A + B) = det(B) det(A + B) = − det(A) det(A + B),
yielding 2 det(A) det(A + B) = 0. But det(A) = ±1 because A is orthogonal. Thus,
det(A + B) = 0, as desired. 

4. Let x, y ∈ R be arbitrary. We can write


xn+2 y n+2 = (xy)n+2 = (xy)n+1 xy = xn+1 y n+1 xy,
yielding
xn+1 (xy n+1 − y n+1 x)y = 0.
As the above equality holds for all x, y ∈ R, changing x to x + 1 in the equality, we
obtain
(1 + x)n+1 (xy n+1 − y n+1 x)y = 0,
or
      
n+1 n+1 n + 1 n+1
+ x + ··· + x (xy n+1 − y n+1 x)y = 0.
0 1 n+1
Multiplying both sides of the equality by xn and using the aforementioned equality,
we get
xn (xy n+1 − y n+1 x)y = 0.
Analogously, changing x to x + 1 in this equality, we see that
xn−1 (xy n+1 − y n+1 x)y = 0.
Continuing in this way, we finally obtain
(xy n+1 − y n+1 x)y = 0. (∗)
On the other hand, by the hypothesis, we have
xn+1 y n+1 = (xy)n+1 = (xy)n xy = xn y n xy,
yielding
xn (xy n − y n x)y = 0.
Likewise, from this, we obtain
(xy n − y n x)y = 0.
Multiplying both sides of the above by y from the left, we get
yxy n+1 − y n+1 xy = 0,
from which, in view of (∗), we see that
(yx − xy)y n+1 = 0.
Once again, preforming an argument involving changing y to y + 1 and multiplying
both sides of the obtained equality by an appropriate power of y, we eventually
conclude that yx − xy = 0. That is, xy = yx for all x, y ∈ R, which is what we
want. 

5. Suppose that a person is chosen at random. Define the following events.


A (resp. B) := the event that the person is guilty (resp. innocent).
+ (resp. −) := the event that the person responds positively (resp. negatively).
E := the event that the person responds positively to four questions and neg-
atively to one question.
2.29. TWENTY NINTH COMPETITION 215

It follows from the hypothesis that P (A) = 31 , P (B) = 23 , P (+|A) = 0.8,


P (+|B) = 0.4, P (−|A) = 0.2, and P (−|B) = 0.6. Using Bayes’ Theorem, we see
that the desired probability is equal to
P (A ∩ E) P (A)P (E|A)
P (A|E) = =
P (E) P (A)P (E|A) + P (B)P (E|B)
1 5
 4
3 4 (0.8) (0.2) 8
= 1 5 2 5
 = .
4 4
3 4 (0.8) (0.2) + 3 4 (0.4) (0.6)
16
8
Thus, P (A|E) = 16 is the desired probability. 

6. We claim that for each j = 1, . . . , k, there are subsets A1 , . . . , Aj satisfying the


following inequalities.
1 ≤ |A1 ∩ · · · ∩ Aj | ≤ r − (j − 1)(I(F ) − 1) (∗).
Obviously, the assertion follows from (∗) by letting j = k. We prove (∗) by induction
on j. If j = 1, the assertion is trivial; in fact, any element A1 of F satisfies (∗) when
j = 1. Suppose that (∗) holds for j, where j ≤ k − 1. We prove the assertion for
j + 1. To this end, it follows from the hypothesis that A1 ∩ · · · ∩ Aj has nonempty
intersection with any element of F . This implies |A1 ∩ · · · ∩ Aj | ≥ I(F ). Let S be
a subset of A1 ∩ · · · ∩ Aj with I(F ) − 1 elements. Thus, there exists an Aj+1 ∈ F
such that S ∩ Aj+1 = ∅. So we can write
1 ≤ |A1 ∩ · · · ∩ Aj ∩ Aj+1 | ≤ |(A1 ∩ · · · ∩ Aj ) ∩ S c |

= |(A1 ∩ · · · ∩ Aj ) \ S| = |A1 ∩ · · · ∩ Aj | − I(F ) − 1
 
≤ r − (j − 1) I(F ) − 1 − (I(F ) − 1) = r − j I(F ) − 1 ,
proving the induction assertion, finishing the proof. 

2.29. Twenty Ninth Competition

2.29.1. First Day. 1. First solution: We can write


Z a Z a
dx
I := f (x)dx
0 f (x)
Z0 a Z a Z aZ a
1 1
= f (x) dxdy = f (y) dxdy,
0 0 f (y) 0 0 f (x)
from which, we obtain
Z aZ a 
f (x) f (y)
2I = I + I = + dxdy
0 0 f (y) f (x)
Z aZ a
f (x)2 + f (y)2
= dxdy.
0 0 f (x)f (y)
f (x)2 +f (y)2
On the other hand, obviously, we have f (x)f (y) ≥ 2 for all 0 ≤ x, y ≤ a. Thus,
Z aZ a
2I ≥ 2dxdy = 2a2 ,
0 0
yielding
I ≥ a2 ,
as desired.
216 2. SOLUTIONS

Second solution: By the Cauchy-Schwarz Inequality, we can write


Z 2 Z
ap 1 a  Z a
dx

2
a = f (x) × p dx ≤ f (x)dx ,

0 f (x) 0 0 f (x)

which is what we want. 

2. The “if part” is trivial. We prove the “only if part” of the assertion by con-
tradiction. To this end, suppose x = ab for some a, b ∈ N. It follows from the
contradiction hypothesis that there exists a k ∈ N such that nk+1 − 1 > b. Set
k
X 1
xk = .
i=1
n1 · · · ni
We can write
a 1 1
− xk = x − xk = + + ···
b n1 · · · nk+1 n1 · · · nk+1 nk+2
 
1 1 1 1 1 1
< + 2 + 3 + ··· = .
n1 · · · nk nk+1 nk+1 nk+1 n1 · · · nk nk+1 − 1
Consequently,
a 1
0 < n1 · · · nk − xk n1 · · · nk < ,
b nk+1 − 1
whence
b
0 < an1 · · · nk − bxk n1 · · · nk < < 1,
nk+1 − 1
which is a contradiction because an1 · · · nk − bxk n1 · · · nk ∈ N. Thus, the assertion
follows by contradiction. 

3. Note that 10φ(m) ≡ 1 (mod m) for all positive integers m satisfying


gcd(m, 10) =
1, where φ denotes Euler’s totient function. It follows that m 10kφ(m) − 1 when-
ever k, m ∈ N with gcd(m, 10) = 1. Moreover, if gcd(m, 3) = 1, then
kφ(m)
10 −1
m = 1 + 10 + · · · + 10kφ(m)−1 .
10 − 1
kφ(m) times
z }| {
Thus, if gcd(m, 30) = 1, then m 11 . . . 1 for all k ∈ N, proving the assertion. 

4. Proceed by way of contradiction. Suppose I is a nonzero right ideal in R whose


elements all have square zero. It follows that (x + y)2 = 0, and hence xy = −yx
for all x, y ∈ R. Thus, (xI)(xI) = 0, from which, we see that
(RxI)(RxI) ⊆ (Rx)I(xI) = R(xI)(xI) = 0,
for all x ∈ R. In other words, the two-sided ideal RxI is nilpotent, which, in view
of the hypothesis, yields RxI = 0. This shows that xI is contained in the left
annihilator of R in R, denoted by l.ann(R). But l.ann(R) is nilpotent. Thus,
xI ⊆ l.ann(R) = {0}, yielding xI = {0} for all x ∈ R. This easily implies I 2 =
{0}, from which, we obtain (RI)(RI) ⊆ RI 2 = {0}. But RI is a two-sided ideal
of R. So it follows from the hypothesis that RI = {0}. This implies that I is
contained in the right annihilator of R in R, denoted by r.ann(R). But r.ann(R) is
2.29. TWENTY NINTH COMPETITION 217

obviously a nilpotent two-sided ideal. Thus, I ⊆ l.ann(R) = {0}, yielding I = {0},


a contradiction. This proves the assertion. 

5. Define the function f : X −→ Y by



f (A) = k ∈ Z : 2k ∈ A .
It suffices to show that for any B ∈ Y , there exists a A ∈ X such that f (A) = B.
To this end, for given B ∈ Y , set

A := 2k : k ∈ B .
It is obvious that A ∈ X and f (A) = B, as desired. 

6. Note first that since S ≤ Zn2 is a vector space over Z2 , d is equal to the minimum
of the number of ones occurring in the nonzero elements of S. It is plain that S has
2k elements. Use a matrix A ∈ M(2k −1)×n (Z2 ) to denote the nonzero elements of
S, so that every nonzero element of S corresponds to a row of A. We may assume
that the first k rows of A form a basis for S. Denote this k × n submatrix of A by
B. It is plain that the number of one entries of the matrix A is at least d(2k − 1).
By showing that this number is at most n2k−1 , we conclude that d(2k −1) ≤ n2k−1 ,
proving the assertion. We claim that the number of ones in any column of A is
either 0 or 2k−1 . Since every row of A is a linear combination of the rows of B, the
number of ones in any column of A is computed as follows. First, if there exists no
“one” in column i (1 ≤ i ≤ n) of B, then neither does exist in the same column of
A. Next, let t be the number of ones in column i (1 ≤ i ≤ n) of B. Since any linear
combination of the rows of B gives rise to a row of A, and since the coefficients
come from Z2 , it follows that the number of ones in column i of A, denoted by 1i ,
is equal to the number of all subsets of {1, 2, . . . , n} with k elements each of which
contains an odd number of one entries from column i of B. Thus, 1i is equal to the
number of all odd numbered subsets of a set with t elements times the number of
all subsets of a set with k − t elements. In other words,
1i = 2t−1 2k−t = 2k−1 .
Now, as A has k columns, the number of all ones is at most n2k−1 , finishing the
proof. 

P+∞
2.29.2. Second Day. 1. First solution: Let f (z) = n=0 an z n . Set g(z) =
f (z̄). It is plain that
+∞
X
g(z) = ān z n ,
n=0
and that g is analytic on D. It follows that f − g is analytic on D and moreover
(f − g)( n1 ) = f ( n1 ) − g( n1 ) = 0 for all n ≥ 2. Hence, the set of zeros of the analytic
function f − g has a limit point in D. It thus follows from the Uniqueness Theorem
for analytic functions that f (z) = g(z) for all z ∈ D, whence an = ān for all n ∈ N.
f (n) (0)
Therefore, an ∈ R for all n ∈ N. This, in view of an = n! , yields f (n) (0) ∈ R
for all n ∈ N, as desired.
P+∞ (n)
Second solution: Let f (z) = n=0 an z n , where an = f n!(0) for all n ∈ N ∪ {0}.
It suffices to show that an ∈ R for all n ∈ Z with n ≥ 0. We prove this by induction
218 2. SOLUTIONS

on n. If n = 0, the assertion is trivial because a0 = f (0) = limn f ( n1 ). Assuming


that a0 , . . . , an−1 ∈ R, to prove an ∈ R, set
+∞  1 Pn−1 
X
i n f (z) − i=0 ai z i z 6= 0,
g(z) = ai+n z = z
an z = 0.
i=0

It is readily verified that g is an analytic function satisfying g( n1 ) ∈ R for all natural


numbers n ≥ 2. Thus, an = g(0) = limn g( n1 ) ∈ R, which is what we want. 

2. (i) The assertion can be stated as follows. For a given ε > 0, every point of the
space X can be reached from any other point of the space within a finite number of
steps each of which having a length less than ε. With this in mind, for given ε > 0,
fix an x ∈ X and set

Cε (x) := y ∈ X : ∃x1 = x, x2 , . . . , xn = y ∈ X  ∀i < n : d(xi , xi+1 ) < ε .
To prove the assertion, it suffices to show that Cε (x) = X. This, in view of the
hypothesis that X is connected, follows as soon as we show that Cε (x) is a clopen
subset, i.e., both a closed and open subset, of X. First, suppose z ∈ Cε (x). It
follows that there exists a sequence (yn )+∞
n=1 in Cε (x) such that limn yn = z. Thus,
there exists an N ∈ N such that d(yn , z) < ε whenever n ≥ N . On the other hand,
since yn ∈ Cε (x), we have Bε (yn ) := {t ∈ X : d(t, yn ) < ε} ⊆ Cε (x). Consequently,
z ∈ Cε (x) because z ∈ Bε (yn ). Thus, Cε (x) is a closed subset of X. Next, suppose
y ∈ Cε (x) is arbitrary. It follows that Bε (y) ⊆ Cε (x). This implies that Cε (x) is
an open subset of X. Therefore, Cε (x) = X because X is connected.

(ii) Plainly, X = [0, 1) ∪ (1, 2] is an example showing the converse of (i) does
not necessarily hold in general.

(iii) To prove the assertion by contradiction, suppose X is disconnected. It


follows that there is a clopen subset M of X. Since X is compact and both M and
M c = X \ M are closed subsets of X, they both are compact subsets of X. But
M ∩ M c = ∅. Hence, d(M, M c ) := inf{d(x, y) : x ∈ M, y ∈ M c } > 0. Pick x ∈ M
and y ∈ M c . It is now obvious that x cannot be reached from y within a finite
number of steps each of which having a length less than ε = d(M, M c ). Because,
otherwise ε = d(M, M c ) ≤ d(xi , xi+1 ) < ε, where xi ∈ M and xi+1 ∈ M c are the
consecutive foot steps in a walk from x to y with a finite number of steps each of
which having a length less than ε. This is a contradiction. Thus, the assertion
follows. 

3. (i) For given g ∈ G, define ϕg : K −→ K by ϕg (x) = g −1 xg. Since g ∈ NG (K),


the map ϕg is well-defined because g −1 xg ∈ K for all x ∈ K. This implies ϕg ∈
Aut(K). In other words, the map ϕ : NG (K) −→ Aut(K) defined by ϕ(g) = ϕg is
well-defined. It is easily verified that ϕ is a homomorphism of groups and that

ker ϕ = g ∈ NG (K)| ϕ(g) = id
g ∈ NG (K)| ∀x ∈ K : g −1 xg = x

=

= g ∈ NG (K)| ∀x ∈ K : gx = xg = CG (K).
Now, the assertion is a quick consequence of the First Isomorphism Theorem for
groups.
2.29. TWENTY NINTH COMPETITION 219

(ii) By (i), in view of the hypothesis that K E G, the group CGG(K) is isomorphic
to a subgroup of Aut(K). Since K is cyclic, Aut(K) is abelian, and hence so is
G 0 0
CG (K) , whence G ≤ CG (K). It follows that G ≤ CG (K) ≤ G because G = G.
Thus, G = CG (K), implying K ≤ Z(G), which is what we want. 

4. To prove the “if part”, it suffices to show that any two invariant subspaces of
any nilpotent transformation N on F n with N n−1 6= 0 are comparable with respect
to inclusion. Note first that N n = 0 and that there exists a vector α ∈ F n such
that N n−1 α 6= 0 because N n−1 6= 0. It follows that the set {N n−1 α, . . . , N α, α} is
a basis for F n . To see this, suppose c1 N n−1 α + · · · + cn−1 N α + cn α = 0 for some
ci ∈ F (1 ≤ i ≤ n). Taking N n−1 of both sides of this equality yields cn N n−1 α = 0,
implying cn = 0. Thus, c1 N n−1 α + · · · + cn−1 N α = 0. Now, taking N n−2 of both
sides of the equality yields cn−1 = 0. Continuing in this manner, we obtain ci = 0
for all 1 ≤ i ≤ n. Thus, the set {N n−1 α, . . . , N α, α} is a basis for F n because
dim F n = n. Set
M0 = 0 = {0}, Mi = N n−1 α, . . . , N n−i α .


We have
M0 = {0} < M1 = N n−1 α < · · · < Mn = F n .

It is plain that Mi is an invariant subspace of N for all 1 ≤ i ≤ n. This shows that


any nilpotent transformation with N n−1 6= 0 is triangularizable. Now, suppose
that M is an invariant subspace for N . We prove the assertion by showing that
M = Mj for some 1 ≤ j ≤ n. To this end, let 1 ≤ j ≤ n be the greatest natural
number for which there exists a nonzero vector x ∈ M such that
x = c1 N n−1 α + · · · + cj N n−j α, (∗)
j−1
where cj 6= 0. We show that M = Mj . Taking N of both sides of (∗), we
get cj N n−1 α = N j−1 x ∈ M, whence N n−1 α ∈ M because cj 6= 0. Now, using
N n−1 α ∈ M and taking N j−2 of both sides of (∗), we see that N n−2 α ∈ M. Con-
tinuing this way, we conclude that N n−j α ∈ M. Thus, Mj = hN n−1 α, . . . , N n−j αi ⊆
M. On the other hand, M ⊆ Mj because N n−j α ∈ M. Therefore, M = Mj , as
desired.
To prove the “only if part”, note first that the matrix A can have at most
one eigenvalue. Because, otherwise the corresponding eigenspaces of two distinct
eigenvalues are not comparable with respect to inclusion, which is in contradiction
with the hypothesis. So, with no loss of generality, we may assume that zero is
the only eigenvalue of A, for A − λI and A share the same lattice of invariant
subspaces. In other words, we may assume that A is nilpotent. We claim that
there exists a nonzero vector α ∈ F n such that the set {An−1 α, . . . , Aα, α} is a
basis for F n . Suppose to the contrary that the set {An−1 α, . . . , Aα, α} is not a
basis for F n . Let M := hAn−1 α, . . . , Aα, αi. It follows that the subspace M
is a nontrivial invariant subspace of A. Pick β ∈ F n \ M. It follows from the
contradiction hypothesis that the subspace M0 := hAn−1 β, . . . , Aβ, βi is also a
nontrivial invariant subspace of A. Obviously, the subspaces M and M0 are not
comparable, a contradiction. Therefore, there exists a nonzero vector α ∈ F n
such that the set {An−1 α, . . . , Aα, α} is a basis for F n . Since A is assumed to be
nilpotent, we see that An = 0 but An−1 6= 0 because An−1 α 6= 0. This completes
the proof. 
220 2. SOLUTIONS

5. Use A(m, n) to denote the average number of times that the players should play
till one of them runs out of the coins. If the game is played once, A wins with the
probability of 12 changing the number of the coins of A and B to m + 1 and n − 1,
respectively; and, likewise, B wins with the probability of 21 changing the number
of the coins of A and B to m − 1 and n + 1, respectively. This yields the following
recurrence relation on m, n ∈ N.
1 1
A(m, n) = 1 + A(m − 1, n + 1) + A(m + 1, n − 1)
2 2
The boundary conditions are A(0, n) = A(m, 0) = 0. To solve this boundary
recurrence equation, view A as a function of one variable, say n. It follows from
the recurrence equation that on the line m + n = c, where c is a constant, we have
A(m − 1, n + 1) + A(m + 1, n − 1) − 2A(m, n) = −2. (∗)
Thus, A(m, n) must be a quadratic function. As A(0, n) = A(m, 0) = 0, we obtain
A(m, n) = cmn. Substituting this into (∗), we obtain
−2c = c(m − 1)(n + 1) + c(m + 1)(n − 1) − 2cmn = −2,
implying c = 1. Consequently, A(m, n) = mn. Therefore, on average, the game is
played as many as the product of the coins owned by A and B altogether. 

6. It suffices to show that [0, 1] ⊆ C − C. To this end, let y ∈ [0, 1] be arbitrary. If


1−y
= 0.y1 y2 . . .
2
1−y
denotes the ternary expansion of 2 , where yi ∈ {0, 1, 2}, then we can write
1−y
= 0.a1 a2 . . . + 0.b1 b2 . . . ,
2
where ai , bi ∈ {0, 1} and yi = ai + bi for all i ∈ N. So we have
1−y = 2(0.a1 a2 . . .) + 2(0.b1 b2 . . .),
implying
y = 0.a01 a02 · · · + 0.b01 b02 · · · := a + b,
where a0i = 2 − 2ai and b0i = 2bi , and hence a0i , b0i ∈ {0, 2} for all i ∈ N. Therefore,
a, b ∈ C because there is no one in their ternary expansions. This finishes the proof.


2.30. Thirtieth Competition

2.30.1. First Day. 1. First solution: It suffices to evaluate the following


limit Z 2006
lim f (tx)dx.
t→+∞ 1385
Setting tx = u, we have
Z 2006 Z 2006t
1
lim f (tx)dx = lim f (u)du.
t→+∞ 1385 t→+∞ t 1385t
2.30. THIRTIETH COMPETITION 221

As limt→+∞ t = +∞, using L’Hopital’s rule, we can write


Z 2006  
lim f (tx)dx = lim 2006f (2006t) − 1385f (1385t) = 621.
t→+∞ 1385 t→+∞

Thus,
Z 2006
lim f (nx)dx = 621,
n→+∞ 1385
as desired.

Second solution: We can write


Z 2006 Z 2006 

lim f (nx)dx = lim f (nx) − 1 dx + 621 = 621,
n→+∞ 1385 n→+∞ 1385
R 2006 
provided that limn→+∞ 1385 f (nx) − 1 dx = 0. It follows from the hypothesis
ε
that for given ε > 0, there is an N > 0 such that |f (x) − 1| < 621 whenever x > N .
N
Thus, for all n > 1385 and x ∈ [1385, 2006], we have

f (nx) − 1 < ε.
This yields
Z 2006 
Z 2006

f (nx) − 1 dx ≤ f (nx) − 1 dx ≤ ε,


1385 1385
N
R 2006 
for all n > 1385 . That is, limn→+∞ 1385 f (nx) − 1 dx = 0, settling the assertion.


2. We need the following lemma.

Lemma. Let α π 6∈ R \ Q, where π = 3.14159 . . .. Then, the set {e


inα
: n ∈ N}
is dense in the unit circle.
Proof. Use T to denote the unit circle. Set S = {einα : n ∈ N}. Plainly, the
set S consists of distinct points, for απ 6∈ R \ Q, and it is a multiplicative semigroup.
As T is compact, we see that S has a limit point in T. Thus, there is a strictly
increasing sequence (nk )+∞ k=1 of natural numbers such that limk e
ink α
exists. This
imk α
implies that limk e = 1, where mk = nk+1 − nk . Now, let {eit : t ∈ (a, b)} be
an arbitrary open arc of the unit circle, where a, b ∈ R and a < b. It follows that
there is an ` ∈ N such that eimk α ∈ {eit : t ∈ (−δ, δ)} for all k ≥ `, where δ = b−a 2 .
Consequently, {einm` α : n ∈ N} ∩ {eit : t ∈ (a, b)} 6= ∅. This shows that S is dense
in T, which is what we want. 
Remark. A proof similar to that of the above lemma shows that every additive
subgroup of R is either dense or isolated, i.e., it has no limit points. A quick
consequence
of this is the following. Let α ∈ R \ Q. Then, the set m + nα : m, n ∈
Z is dense in R.

First, we claim that there is a sequence (kp )+∞


p=1 of natural numbers such that
k
limp→+∞ aj p = 1 for all 1 ≤ j ≤ m. Proceed by induction on m. If m = 1, then
limn→+∞ an1 = c. Firstly, c 6= 0 because |c| = limn→+∞ |a1 |n = 1. Secondly, by
showing that a1 = 1, we prove the claim in this case. From c = limn→+∞ an1 =
limn→+∞ an+1
1 = ca1 , we obtain c(1 − a1 ) = 0, which yields a1 = 1 because c 6= 0.
Assume that the assertion holds for m − 1. It follows from the induction hypothesis
k`
that there is a sequence (k` )+∞
`=1 such that lim`→+∞ aj = 1 for all 1 ≤ j ≤ m − 1.
222 2. SOLUTIONS

Without loss of generality, if necessary by passing to a subsequence of (k` )+∞


`=1 , we
can assume that lim`→+∞ akj ` = 1 for all 1 ≤ j ≤ m − 1 and lim`→+∞ akm` = bm ,
where bm ∈ C and |bm | = 1. Set
A = (ak1 , . . . , akm−1 , akm ) : k ∈ N .


Let A0 be the set of limit points of A. It is obvious that (1, . . . , 1, bkm ) ∈ A0 for all
k ∈ N. As |bm | = 1, it is easily seen from the lemma above that there exists a sub-
sequence (kr )+∞ kr
r=1 such that limr→+∞ bm = 1. Consequently, (1, . . . , 1, 1) ∈ (A ) ⊆
0 0
k k k
A0 . Thus, there is a subsequence (kp )+∞ p p p
p=1 such that limp→+∞ (a1 , . . . , am−1 , am ) =
k
(1, . . . , 1, 1), and hence limp→+∞ aj p = 1 for all 1 ≤ j ≤ m, proving the claim. It
thus follows that
m m m
k +1 k
X X X
lim aj p = c = lim aj p = 1 = m,
p→+∞ p→+∞
j=1 j=1 j=1
Pm k
yielding j=1 aj = m because limp→+∞ aj p = 1 for all 1 ≤ j ≤ m. Now, we can
write
m
X m
X 
Re(aj ) = Re aj = m,
j=1 j=1
Pm
yielding j=1 Re(aj ) = m. This implies Re(aj ) = 1, for Re(aj ) ≤ 1 for all 1 ≤ j ≤
m. As the aj ’s are on the unit circle, we see that aj = 1 for all 1 ≤ j ≤ m, which
is what we want. 

3. It is worth noting that the hypothesis that R is commutative is redundant.


Consider the following five elements of R/J
J, 1 + J, a + J, a2 + J, a3 + J.
It follows from the hypothesis that
 at least two of the above elements are equal.
Thus, one of the following 10 = 52 cases will occur
J = 1 + J, J = a + J, J = a2 + J, J = a3 + J,
1 + J = a + J, 1 + J = a2 + J, 1 + J = a3 + J,
a + J = a2 + J, a + J = a3 + J, a2 + J = a3 + J.
Consequently, at least one of the following elements of R belongs to J
1, a, a2 , a3 , a − 1, a2 − 1, a3 − 1 = a, a2 − a, a3 − a, a3 − a2 .
In view of the identity a(a − 1)(a + 1) = 1, it is readily verified that the above
elements are invertible elements of R, and hence J = R, as desired. 

4. Since p is an odd prime, by Fermat’s Little Theorem, we have 2p−1 ≡ 1 (mod p).
s
s be the smallest natural number for which 2 ≡ 1 (mod p). It follows that
Let
s p − 1 = 2q, and hence s ∈ {1, 2, q, 2q}, for q is prime.

To prove the assertion by
contradiction, assume that s < 2q. If s = 1, then p 1, which is a contradiction.
If s = 2, we obtain p = 3, yielding q = 1, which is a contradiction. Finally, if
q
s = q, then the equation x2 ≡ 2p (mod p) is solvable for x in Z. Thus 2p = 1,
where ap , with a, p ∈ Z, denotes the Legendre symbol, which is defined to be ±1

2.30. THIRTIETH COMPETITION 223

depending on whether or not the equation x2 ≡ a (mod p) is solvable for x in Z.


On the other hand,
2q  2 q  q(p28−1)  q2 (q+1)
= = −1 = −1 2
= −1,
p p
which is a contradiction. Note that the last equality in the above is implied by
q ≡ 1 (mod 4). Therefore, we obtain a contradiction in any event. So the assertion
follows. 

5. Let X = X1 + · · · + X2m , where Xi ’s (1 ≤ i ≤ 2m) are independent random


variables assuming the values zero and one with the probability of 21 . It is plain
that
2 m
µ = µX = m, σ = σX = .
2

Using the Chebyshev Inequality with λ = 2, we have
√  1
P |X − m| ≥ m ≤ ,
2
implying
√  1
P |X − m| < m ≥ .
2
On the other hand,
1 √  X  2m  1 2m
≤ P |X − m| < m = ,
2 √ m+k 2
|k|< m

yielding
X  2m 
≥ 22m−1 ,
√ m+k
|k|< m

which is what we want. 

6. Use p1 , . . . , pn to denote the members of the financial group. Suppose that the
number of coins owned by person pi after doing business for j times is aij , where
i = 1, . . . , n and j ∈ N. Without loss of generality, assume that the (j + 1)st
business is done among p1 , p2 , p3 , and p4 . It follows that
a3j + a4j − a1j − a2j + 2k > 0.
Multiplying both sides of the above inequality by 2k, we easily conclude that
(a3j + k)2 + (a4j + k)2 + (a1j − k)2 + (a2j − k)2 > a21j + a22j + a23j + a24j .
Consequently,
n
X n
X
a2i(j+1) > a2ij .
i=1 i=1
On the other hand, the sum of the number of coins owned by p1 , . . . , pn is a fixed
number. Therefore, a finite number of business can be done among these people,
proving the assertion. 
224 2. SOLUTIONS

2.30.2. Second Day. 1. First solution: As X is separable, X has a count-


able base, say B = {Bi }+∞ i=1 , e.g., the family of all open balls with centers at the
points of a countable dense subset of X and with positive rational radii is easily
seen to be a countable base for X. Let D1 = {a ∈ X : limx→a f (x) < f (a)}
and D2 = {a ∈ X : limx→a f (x) > f (a)}. It suffices to show that D1 ∪ D2 is
at most countable. To this end, for a ∈ D1 (resp. a ∈ D2 ) choose an ra ∈ Q
with limx→a f (x) < ra < f (a) (resp. limx→a f (x) > ra > f (a)) and Bia ∈ B with
a ∈ Bia such that f (x) < ra (resp. f (x) > ra ) for all x ∈ Bia \ {a}. Define
a function ι1 : D1 −→ Q × B (resp. ι2 : D2 −→ Q × B) by ι1 (a) = (ra , Bia )
(resp. ι2 (a) = (ra , Bia )). If (ra , Bia ) = (rb , Bib ) for some a, b ∈ X, then a = b,
for otherwise f (x) < ra = rb (resp. f (x) > ra = rb ) whenever x ∈ Bia \ {a}. But
Bia = Bib . Thus, f (b) < ra = rb < f (b) (resp. f (b) > ra = rb > f (b)), which is a
contradiction. This proves that ι1 (resp. ι2 ) is one-to-one. Therefore, D1 and D2
are countable, and hence so is D1 ∪ D2 , which is what we want.

Second solution: For a given f : X −→ R, use D and L to denote the the set of
points at which f is discontinuous and has a limit, respectively. By showing that
D ∩ L is at most countable, we prove the assertion. Define ω : X −→ [0, +∞) by
  
ω(x) = lim+ sup f Bδ (x) − inf f Bδ (x) ,
δ→0

where Bδ (x) = {y ∈ X : d(y, x) < δ} denotes the open ball with center at x
S+∞
and radius δ > 0. Note that D = n=1 Dn , where Dn = {x ∈ X : ω(x) > n1 }.
S+∞
Consequently, D ∩ L = n=1 (Dn ∩ L). It thus suffices to prove that Dn ∩ L is
at most countable for all n ∈ N. To this end, for a given a ∈ Dn ∩ L, we have
limx→a f (x) = la for some la ∈ R. It follows that there exists a δna > 0 such that
1
|f (x) − la | < 2n whenever x ∈ Bδna (a) \ {a}. This implies |f (x) − f (y)| < n1 for all
x, y ∈ Bδna (a) \ {a}. Hence, ω(x) ≤ n1 for all x ∈ Bδna (a) \ {a}. In other words,
x∈ / Dn for all x ∈ Bδna (a) \ {a}. This implies that Dn ∩ L is at most countable
for all n ∈ N, for otherwise Dn ∩ L would have (uncountably many) limit points
because X is separable. This obviously is in contradiction with x ∈ / Dn for all
x ∈ Bδna (a) \ {a} and a ∈ Dn ∩ L, finishing the proof. 

2. First solution: If M = +∞, the assertion is trivial. So, we may, without loss
of generality, assume that M < +∞. Let D = {z ∈ C : |z| < 1} denote the unit disk
around zero. Define g : D −→ C by g(z) = fM (Rz)−a0
+|a0 | . Plainly, g is analytic on D and
g(0) = 0. By the Maximum Modulus Principle, |f (Rz)| ≤ sup|z|=R |f (z)| = M for
all z ∈ D. It follows that

g(z) ≤ f (Rz) + a0 ≤ M + a0 = 1,

M + a0 M + a0
for all z ∈ D. Thus, by the Schwarz Lemma, |g(z)| ≤ |z| for all z ∈ D. Now,
if f (z0 ) = 0 for some z0 ∈ C, then zR0 ∈ D because |z0 | < R. Consequently,
|g( zR0 )| ≤ | zR0 |. But
z0  f R zR0 − a0

a0 z
g = = ≤ 0 ,
R M + a0 M + a0 R
R|a0 |
yielding |z0 | ≥ M +|a0 | , finishing the proof.
2.30. THIRTIETH COMPETITION 225

Second solution: Again, we may, without loss of generality, assume that M <
+∞. Let 0 < r < R be arbitrary. By the Cauchy Inequality, |an | ≤ Mr(r)
n for all
n ∈ N. In view of the Maximum Modulus Principle, we have M (r) ≤ M for all
r < R, and hence we can write
+∞ +∞
X
f (z) − a0 ≤
n
an z ≤ M (r)
X z n M z
= ,
n=1
r n=1
r − z
for all z ∈ C with |z| < R. On the other hand,

M z r|a0 |
< a0 ⇐⇒ |z| < .
r− z M + |a0 |
r|a0 |
Consequently, f (z) 6= 0 whenever |z| < M +|a0 | . Since r < R is arbitrary, we see
R|a0 |
that f (z) 6= 0 whenever |z| < M +|a0 | . This proves the assertion. 

3. Let H be the set of all elements of G whose orders are finite. Obviously, H is
not empty because 1 ∈ H. Let x, y ∈ H be arbitrary. It follows that there is an
n ∈ N such that xn = y n = 1. Consequently,
n
(xy −1 )n G0 = (xy −1 G0 )n = (xG0 )(y −1 G0 )
= (xG0 )n (y −1 G0 )n = (xn G0 )(y −n G0 ) = G0 ,
n
where the equality (xG0 )(y −1 G0 ) = (xG0 )n (y −1 G0 )n follows from the hypothesis
that GG0 is abelian. Thus, (xy −1 )n ∈ G0 , from which, we obtain an m ∈ N such that
(xy −1 )nm = 1. That is, xy −1 ∈ H, and hence H ≤ G, as desired. 

4. We prove the following, of which the assertion is a quick consequence. Let D


be a division ring and F a subfield of its center. If A ∈ Mn (D) is algebraic over
the subfield F and rank(A) = rank(A2 ), then there is an idempotent E in the F -
algebra generated by A which projects Dn onto the range of A along ker(A), i.e.,
Dn = im(A) ⊕ ker(A) and E(x + y) = x, where x ∈ im(A) and y ∈ ker(A).
As is usual, the members of Mn (D) can be viewed as linear transformations
acting on the left of Dn via the usual matrix multiplication. To show that Dn =
im(A) ⊕ ker(A),
 note that by the Rank-Nullity Theorem, we have dim im(A) +
dim ker(A) = dim(Dn ) = n. On the other hand,
 
dim im(A) + ker(A) = dim(im(A)) + dim(ker(A)) − dim im(A) ∩ ker(A) .
Thus, to prove Dn = im(A) ⊕ ker(A), it suffices to show that im(A) ∩ ker(A) = {0}.
To this end, let B := A|im(A) : im(A) −→ im(A). We can write
dim im(A2 ) = rank(A2 ) = rank(A)

  
= dim im(A) = dim ker(B) + dim im(B)
= dim im(A) ∩ ker(A) + dim im(A2 ) ,
 

yielding dim im(A) ∩ ker(A) = 0, whence im(A) ∩ ker(A) = {0}.
For the rest, let f (A) = Am + am−1 Am−1 + · · · + a1 A + a0 I = 0, where m is
minimal and a0 , a1 , ..., am−1 ∈ F . If a0 6= 0, then we have nothing to prove. If
a0 = 0, then (Am−1 + am−1 Am−2 + · · · + a1 I)A = 0. We claim that a1 6= 0. If
not, then (Am−2 + am−1 Am−3 + · · · + a2 I)A2 = 0. Now, since im(A) = im(A2 )
because rank(A) = rank(A2 ), it follows that (Am−2 + am−1 Am−3 + · · · + a2 I)A = 0,
226 2. SOLUTIONS

a contradiction. So a1 6= 0. Set G = a−1


1 A
m−1
+ a−1
1 am−1 A
m−2
+ · · · + a−1
1 a2 A + I
n
and note that D = ker(A) ⊕ im(A). Then E = I − G is the desired idempotent.
Because for each x = Ax0 ∈ im(A) and y ∈ ker(A), we can write
E(x + y) = Ex + Ey
Ax0 − a−1 0 −1 m−1
+ a−1 m−2
+ · · · + a−1
 
= 1 f (A)x − a1 A 1 am−1 A 1 a2 A y
= (Ax0 − 0) − 0 = Ax0 = x.
So the proof is complete. 

5. It is plain that the sets of odd and even numbers, denoted by O and E, respec-
tively, is a partition of the natural numbers such that neither O ⊕ O nor E ⊕ E
contains any prime. To prove the uniqueness, suppose that N = A ∪ B and that
neither A ⊕ A nor B ⊕ B contains any prime. Without loss of generality, assume
that 1 ∈ A. By showing that A = O and B = E, we finish the proof. Proceed by
induction. As 1 ∈ A and 1 + 2 = 3, 2 ∈ B. Assume that

1, 3, . . . , 2k − 1 ⊆ A,
and 
2, 4, . . . , 2k ⊆ B.
It follows from Bertrand’s conjecture that for n = 2k + 1, there is a prime p such
that
2k + 1 < p ≤ 4k + 2.
We have 2k + 1 < p ≤ 4k + 2 because k ≥ 1. But p − (2k + 1) is an even number
and 0 < p − (2k + 1) ≤ 2k. Hence, p − (2k + 1) ∈ B, whence 2k + 1 ∈ A. Likewise,
from 0 < p − (2k + 2) ≤ 2k + 1, we see that 2k + 2 ∈ B. So the proof is complete
by way of induction. 

6. Let T denote the closure of T . It is plain that the distance between any two
points of T is less than or equal to α. So, we may, with no loss of generality,
assume that T is closed. Let x, y ∈ T be such that the distance between x and
y is maximal. Use − → to denote the shortest arc joining x and y in the clockwise
xy
direction. Construct T 0 from T as follows. If z ∈ T ∩ − → then z ∈ T 0 . If z ∈ T
xy,
and z ∈ −
→ 0 0 0
/ xy, then z ∈ T , where z denotes the antipodal point of z. In this case,
we have z 0 ∈/ T , for otherwise the distance between z and z 0 is equal to one half
whereas 0 < α < 21 . Also, z 0 ∈ −
→ because otherwise the distance between z and x
xy,
or z and y will be greater than the distance between x and y, which is impossible.
Thus, T 0 ⊆ −→ On the other hand, the sum of the lengths of the arcs whose union
xy.
is T is equal to the sum of the lengths of the arcs whose union is T 0 . This obviously
proves the assertion. 

2.31. Thirty First Competition

2.31.1. First Day. 1. (a) and (b) The answer is no. Color the circle by two
colors, say black (b) and white (w), as shown in the figure. That is, (b) and (w) are,
respectively, assigned to the upper and lower half-open semicircles. It is obvious
that any equilateral triangle inscribed in the circle has a white vertex and two black
2.31. THIRTY FIRST COMPETITION 227

Figure 14

Figure 15

vertices or vise versa. Also, the hypotenuse of any right triangle inscribed in the
circle is a diameter of the circle whose vertices are of the two colors.
(c) The answer is yes. Proceed by contradiction. Assume that there is a
coloring of the circle for which there is no monocolored isosceles triangle inscribed
in the circle. First choose a monocolored chord, say with white ends, which is not
a diameter. Use w1 w2 to denote the chord. The points on the circle that together
with this chord form an isosceles triangle lie on the perpendicular bisector of w1 w2 ,
which is a diameter of the circle. By the contradiction hypothesis, the two ends
of this diameter are both black. Use b1 b2 to denote this diameter. Likewise, the
two ends of the diameter perpendicular to b1 b2 must be both white. Use w3 w4 to
denote this diameter. Now consider the two chords w1 w3 and w2 w4 . It follows that
the ends of the diameters perpendicular to w1 w3 and w2 w4 , respectively, must be
all black. Use b3 b4 and b5 b6 to denote these diameters. It thus follows that the
black-colored triangles b2 b4 b5 , b2 b3 b6 , b1 b4 b5 , and b1 b3 b6 are all isosceles. This is a
contradiction, finishing the proof. 

2. Using the Sequence Lemma, it is readily seen that


A0 + B ⊆ (A + B)0 , A + B 0 ⊆ (A + B)0 ,
implying that (A0 +B)∪(A+B 0 ) ⊆ (A+B)0 . To see (A+B)0 ⊆ (A0 +B)∪(A+B 0 ),
let x ∈ (A + B)0 be arbitrary. By the Sequence Lemma, there exists a sequence
(an + bn )+∞
n=1 with distinct terms such that x = limn (an + bn ), where the sequences
228 2. SOLUTIONS

(an )+∞ +∞
n=1 and (bn )n=1 are in A and B, respectively. As A is bounded, if necessary,
by passing to a subsequence, we may assume that there is an a ∈ A := A ∪ A0
such that a = limn an . It follows that limn bn = limn (an + bn ) − an = x − a,
and hence b := x − a ∈ B because B is closed. There are two cases to consider.
If a ∈ A0 , then x = a + b ∈ (A0 + B) ⊆ (A0 + B) ∪ (A + B 0 ), which is what we
want. If a ∈ A \ A0 , then, if necessary by discarding a finite number of indices,
we may assume that an = a for all n ∈ N. This implies that bn ’s (n ∈ N) are all
distinct. Moreover, limn bn = x − a ∈ B 0 . Thus, x = a + (x − a) ∈ (A + B 0 ) ⊆
(A0 + B) ∪ (A + B 0 ), as desired. So, in any case, x ∈ (A0 + B) ∪ (A + B 0 ). Therefore,
(A + B)0 ⊆ (A0 + B) ∪ (A + B 0 ). This completes the proof. 

3. Let H1 , . . . , Hn be all of the subgroups each of which has index 2 in G. It thus


follows that each Hi (i = 1, . . . , n) is normal in G, and hence so is H1 ∩ · · · ∩ Hn in
G. Also, H1 ∩ · · · ∩ Hn is of finite index in G because so is every Hi (i = 1, . . . , n)
G
in G. Set A := H1 ∩···∩H n
. We claim that A is the desired group. Suppose that B is
K
a subgroup of A of index 2. It follows that B := H1 ∩···∩H n
, where H1 ∩ · · · ∩ Hn ⊆
K E G. By the Third Isomorphism Theorem for groups , we have
G
A H1 ∩···∩Hn ∼ G
= K = .
B H1 ∩···∩Hn
K
Thus, the index of K in G is 2. Therefore, K = Hi for some 1 ≤ i ≤ n. Conse-
quently, A has exactly n subgroup of index 2 which are
H1 H2 Hn
, ,..., .
H1 ∩ · · · ∩ Hn H1 ∩ · · · ∩ Hn H1 ∩ · · · ∩ Hn
(Note that as Hi ’s are mutually disjoint, so are the aforementioned groups.) It
remains to show that A is abelian. For each i = 1, . . . , n, the group HGi is abelian
because Hi has index 2 in G. Thus, G0 ⊆ Hi for all 1 ≤ i ≤ n, from which we
obtain G0 ⊆ H1 ∩ · · · ∩ Hn . Therefore, A is abelian, as desired. 

4. No, one cannot find such two dice. To see this, proceed by contradiction.
Consider two such dice A and B. For each i = 1, . . . , 6, let Ai and Bi , re-
spectively, denote the probability of getting i for the two dice A and B. For
each j = 2, . . . , 12, denote by Pj , the probability of getting a sum of j. Con-
Pj
sequently, Pj = k=1 Ak Bj−k . We have P2 = A1 B1 , P12 = A6 B6 , and P7 =
A1 B6 + A2 B5 + A3 B4 + A4 B3 + A5 B2 + A6 B1 . So, we can write
p p
P7 ≥ A1 B6 + A6 B1 ≥ 2 A1 B6 A6 B1 = 2 A1 B1 A6 B6
r
p 2 2 4
= 2 P2 P12 ≥ 2 . = ,
33 33 33
2 4

which is impossible because Pi ∈ 33 , 33 for all 2 ≤ i ≤ 12. 

5. Note that the set of all balls whose centers are rational points of R2 and whose
radii are positive rationals is countable. Use {B1 , B2 , B3 , . . .} to denote this set.
Choose a set {xi }+∞ +∞
i=1 such that xi ∈ Bi for all i ∈ N. As the set D := {xi }i=1 is
evidently dense in the plane, it suffices to show that D can be so chosen that no
three points of D are collinear. To this end, use induction on n to choose xn in such
a way that for all i < j < n, the points xi , xj , xn are not collinear. This obviously
can be done because no ball is a union of a finite number of lines. 
2.31. THIRTY FIRST COMPETITION 229

6. Let p(x) = xn + a1 xn−1 + · · · + an be the characteristic polynomial of A and let


λ1 , . . . , λn be the eigenvalues of A, which are the roots of p(x) = 0. As, for each
i = 1, . . . , n, the polynomial p(x) is divisible by x − λi , we can write
p(x)
= xn−1 + (λi + a1 )xn−2 + · · · + (λn−1
i + a1 λn−2
i + · · · + an−2 λi + an−1 ).
x − λi
p(x) p(x)
We note that p0 (x) = x−λ1 + ··· + x−λn . It thus follows that
 
p0 (x) = nxn−1 + tr(A) + na1 xn−2 + · · ·
 
+ tr(An−1 ) + a1 tr(An−2 ) + · · · + an−2 tr(A) + nan−1 .

On the other hand, p0 (x) = nxn−1 + (n − 1)a1 xn−2 + · · · + an−1 . Thus,


a1 = −tr(A)
a1 tr(A) + 2a2 = −tr(A2 )
a1 tr(A2 ) + a2 tr(A) + 3a3 = −tr(A3 )
.. .
. = ..
a1 tr(An−2 ) + · · · + an−2 tr(A) + (n − 1)an−1 = −tr(An−1 )
a1 tr(An−1 ) + · · · + an−1 tr(A) + nan = −tr(An )
Note that the last equality in the above is obtained from p(λ1 ) + · · · + p(λn ) = 0.
It now follows from Cramer’s Rule that
 
1 0 0 ··· ··· −tr(A)
 tr(A) 2 0 ··· ··· −tr(A2 )

 tr(A2 ) −tr(A3 )
 
tr(A) 3 0 ··· 
det 
 
.. .. . . . .. .. .. 

 . . . . . 

n−2 n−3
 tr(A ) tr(A ) · · · tr(A) n − 1 −tr(An−1 ) 
tr(An−1 ) tr(An−2 ) · · · ··· tr(A) −tr(An )
an =   .
1 0 0 ··· ··· 0
 tr(A) 2 0 ··· ··· 0 
 tr(A2 )
 
tr(A) 3 0 ··· 0 
det 
 
.. .. . .. .. .. .. 

 . . . . . 

 tr(An−2 ) tr(An−3 ) ··· tr(A) n − 1 0 
tr(An−1 ) tr(An−2 ) tr(An−3 ) ··· tr(A) n
Note that the denominator of the fraction above is equal to n!. Thus, in view of
an = (−1)n det A, the assertion follows by substitution. 

2.31.2. Second Day. 1. For the first part, just note that
n−1 n−1 n−1
+ n = n2 .
  
+1 + + 2 + ··· +
2 2 2
For the rest, suppose that m, k are positive integers such that (m + 1) + (m + 2) +
· · · + (m + 12) = k 2 . It follows that 12m + 78 = k 2 . Consequently, n is even, and
hence k = 2t for some t ∈ N. By substitution, we obtain 2t2 − 6m = 39, which is
impossible because the left hand side is an even integer. 
230 2. SOLUTIONS

2. Performing the substitution u = xα and y = y β and depending on the signs of


α and β, we need to investigate the limit of
u2 v 2
u3 + v 3
as u and v tend to 0 or +∞. There are four cases to consider.
(a) α > 0, β > 0.
2 2
In this case, we need to investigate limu,v→0+ uu3 +v v
3. But by the Sandwich
Theorem, this limit exists and equals zero because

u2 v 2  u3 v  32 −1 √ uv
0≤ 3 = 2
+ uv ≤ ,
u + v3 v u 2
for all u, v > 0.
(b) α > 0, β < 0.
2 2
In this case, we need to investigate limu→0+ ,v→+∞ uu3 +v v
3 . Again, by the Sand-

wich Theorem, the limit exists in this case and equals zero because
u2 v 2 u2 u2
0≤ = u3
≤ ,
u3 + v 3 v2 +v v
for all u, v > 0.
(c) α < 0, β > 0.
2 2
Just as in (b), one can readily see that limu→+∞,v→0+ uu3 +v
v
3 = 0.

(d) α < 0, β < 0.


By showing that there is a path on which the limit is infinity, we conclude that
the limit does not exist in this case. To this end, assuming that u = v, we have
u2 v 2 u
lim 3 3
= lim = +∞,
u,v→+∞,u=v u +v u→+∞ 2

as desired.
Alternatively, the cases (a), (b), and (c) above could be proven using the fol-
lowing inequalities
u2 v 2 u2 v 2 
0≤ 3 ≤ min , ≤ max(u, v),
u + v3 v u
for all u, v > 0. 

3. Let x = (x1 , . . . , xn ) and y = (y1 , . . . , yn ). Define the function f : An → An by


f (z) = z 0 , where z = (z1 , . . . , zn ), z 0 = (z10 , . . . , zn0 ), and

 xj zj = xj 6= yj ,
zj0 = yj zj = yj 6= xj ,
zj otherwise,

for all 1 ≤ j ≤ n. It is readily seen that


d(z, x) = d(f (z), y) = d(z 0 , y), d(z, y) = d(f (z), x) = d(z 0 , x).
Therefore, f is a one-to-one correspondence between the sets C and D, which is
what we want. 

4. First, we prove that “if F is a field, then the ring F [x] has infinitely many
maximal ideals.” To see this, noting that the ideal generated by an irreducible
monic polynomial is a maximal ideal and that such ideals are distinct whenever
2.31. THIRTY FIRST COMPETITION 231

the generating polynomials are, it suffices to show that there are infinitely many
irreducible monic polynomials in F [x]. To this end, motivated by Euclid’s proof of
the infinitude of primes, suppose by contradiction that for some n ∈ N, there are
only n irreducible monic polynomials in F [x], say f1 , . . . , fn . But the polynomial
f1 · · · fn + 1 must have an irreducible monic divisor. This implies that some fi
(1 ≤ i ≤ n) divides f1 · · · fn + 1. Consequently, fi must divide 1, which is a
contradiction. Thus, there are infinitely many irreducible monic polynomials in
F [x].
Now, to prove the assertion, let M be a maximal ideal of R. By the preceding
R
paragraph, the ring M [x] has infinitely many maximal ideals. Since the mapping
R
φ : R[x] → M [x], defined by f (an xn + · · · + a1 x + a0 ) = (an + M )xn + · · · +
(a1 + M )x + (a0 + M ) is an epimorphism, i.e., a surjective homomorphism, of rings,
the ring R[x] has has infinitely many maximal ideals as well. This is because for
every maximal ideal I of M R
[x], φ−1 (I) is also a maximal ideal of R[x] and that
−1 −1
φ (I) 6= φ (J) whenever I 6= J. This completes the proof. 

5. First solution: The hypothesis can be rephrased as follows. We have AX = 0,


where
     
0 ±1 · · · ±1 x1 0
. .. . .. .
..  . ..
 ..
     
 ±1   . 
A=  .
 , X = 
 .
 ,0 =  ..
 .
 .. .. .. 
 ..
  
. . ±1    . 
±1 · · · ±1 0 2n×2n
x2n 2n×1
0 2n×1

That is, the diagonal entries of A are all zero and the off diagonal entries of A are
all either 1 or −1. It suffices to prove that A is invertible. Since the determinant
2
of A is an integer, we prove the assertion by showing that det A ≡ 1. We can write
   
0 ±1 · · · ±1 0 1 ··· 1
..  . 
 ±1 . . . . . .  1 . . . . . . .. 
 
. 
≡2
det A = det   . det 
 . .

 .. .. .. 
 .. .. ... 1 

. . ±1 
±1 · · · ±1 0 1 ··· 1 0
= the number of derangements of 2n elements
1 1 1 1 
= 2n! − + − · · · + (−1)2n
0! 1! 2! (2n)!
2
≡ 1.
Recall that a derangement on n elements is a permutation on n elements hav-
ing no fixed points. Also, a standard argument employing the inclusion-exclusion
principle shows that the number of derangements on n elements is equal to
n
X (−1)k
n! .
k!
k=0

Second solution: We prove the following, of which the assertion is a quick conse-
quence. Let G be an abelian torsion free group, i.e., the identity is the only element
of the group having finite order. Let n, k ∈ N with n > k ≥ 2 and xi ’s (1 ≤ i ≤ n)
232 2. SOLUTIONS

be elements of G such that removing any of them the remaining ones can be par-
titioned into k subsets with equal sums. Prove that xi ’s are all equal. Therefore,
either xi ’s are all zero or they are all equal to a nonzero element, in which case
k
n ≡ 1.
Since the subgroup generated by xi ’s (1 ≤ i ≤ n) is finitely generated, in
view of the Fundamental Theorem of Finitely Generated Abelian Groups, we may,
without loss of generality, assume that G = Z. Also, if necessary, by adding an
appropriate positive integer to all xi ’s (1 ≤ i ≤ n), we may assume that xi ∈ N
k Pn
for each i = 1, . . . , n. First from the hypothesis, we easily obtain xj ≡ i=1 xi for
each j = 1, . . . , n. Next, write xi ’s in the basis k. Plainly, there is an N ∈ N such
that for all 1 ≤ j ≤ n, we can write
N
X
xj = aij k i ,
i=0
k Pn
where aij ∈ N with 0 ≤ aij < k and 0 ≤ i ≤ N . Note that xj ≡ i=1 xi for each
j = 1, . . . , n obtains a0j = a01 for each j = 1, . . . , n. We use induction on N to
prove that that xi ’s are all equal. If N = 0, the assertion is easy because in this
case xj = a0j = a01 = x1 for all 1 ≤ j ≤ n, as desired. Assuming that the assertion
holds for N , we prove it for N + 1. To this end, noting that a0j = a01 for each
j = 1, . . . , n, define
N +1
xj − a01 X
yj := = aij k i−1 ,
k i=1
where 1 ≤ j ≤ n. It readily follows from the hypothesis that removing any of the
yi ’s (1 ≤ i ≤ n) the remaining ones can be partitioned into k subsets with equal
sums. Thus, the induction hypothesis applies to yi ’s (1 ≤ i ≤ n), and hence yi ’s
are all equal. Therefore, so are xi ’s (1 ≤ i ≤ n), which is what we want. 

6. (a) Let A = (t1 q1 , 1 − t1 ) and B = (t2 q2 , 1 − t2 ), where t1 , t2 ∈ (0, 1] and


q1 , q2 ∈ Q are distinct. Let r be an irrational number between q1 , q2 ∈ Q. It
follows that the line ` joining M and (r, 0) partitions the plane into the line ` and
two open half-planes, say P1 and P2 . It is obvious that T \ {M } ⊂ P1 ∪ P2 and
that {A, B} 6⊂ P1 whereas {A, B} ⊂ T \ {M } ⊂ P1 ∪ P2 . It thus follows that any
continuous path from A to B in the set T must intersect the line ` at the point M
because every other point of the line ` does not belong to the set T . This proves
the assertion.

(b) Let f : T → T be a continuous function and f (M ) = M 0 = (t0 q0 , 1 − t0 )


for some t0 ∈ [0, 1] and q0 ∈ Q. If t0 = 0, there is nothing to prove, for the
point M would be a fixed point of f . If not, define the function g : [0, 1] → T by
g(t) = f (tt0 q0 , 1 − tt0 ). Plainly, g is continuous. Define the set D as follows
n o n o
D = (tt0 q0 , 1 − tt0 ) ∈ T : t ∈ [0, 1] = t(t0 q0 , 1 − t0 ) + (1 − t)(0, 1) : t ∈ [0, 1] .
It is plain that D is the line segment joining the points M = (0, 1) and M 0 =
(t0 q0 , 1 − t0 ). Note that D is homeomorphic to the compact interval [0, 1]. We
prove the assertion by showing that f has a fixed point in D. If f takes no point
of D to the point M , the assertion is evident. That is because it would then follow
from (a) that f (D) is contained in D, and hence f has a fixed point in D, for D
2.31. THIRTY FIRST COMPETITION 233

is homeomorphic to the compact interval [0, 1]. So assume that there is a t ∈ [0, 1]
such that g(t) = M . Define
n o
tm = inf t ∈ [0, 1] : g(t) = M .
Since g is continuous and g(0) = f (M ) 6= M , we see that tm > 0 and moreover
g(tm ) = M . Also, it follows from (a) that the image of [0, tm ] under g is contained
in D. Use π2 : R2 → R to denote the projection on the y-axis. We can write
 
π2 g(0) = 1 − t0 < 1 − 0t0 = 1
and  
π2 g(tm ) = π2 (M ) = 1 > 1 − tm t0 .
 
Consequently, there is a t1 ∈ (0, tm ) such that π2 g(t1 ) = 1−t1 t0 . Since g(t1 ) ∈ D,
we obtain
f (t1 t0 q0 , 1 − t1 t0 ) = (t1 t0 q0 , 1 − t1 t0 ),
which means f has a fixed point, as desired. 
CHAPTER 3

Problem Index

First Competition
Analysis.
Problem 1. (routine) Real analysis. Derivatives. Baire functions.
Problem 2. Matrix analysis.
Problem 3. Real analysis. Limits. Continuity.
Algebra.
Problem 1. (routine) Ring theory. Commutative rings with identity. Prime
ideals. Maximal ideals.
Problem 2. (routine) Group theory. Abelian groups.
Problem 3. Baby set theory. Isomorphic sets.
General.
Problem 1. (routine) Triangles.
Problem 2. (routine) Decimal expansion.
Problem 3. (routine) Complex numbers.
Problem 4. (routine) Real numbers. Inequalities.
Differential Equations.
Problem 1. (routine) Second order differential equations.
Probability and Statistics.
Problem 1. (routine) Independent random variables.
Topology.
Problem 1. (routine) Point-set topology of real numbers. (Ir)rational numbers.

Second Competition
Analysis.
Problem 1. Real analysis. Continuous functions. Limit of sequences.
Problem 2. (routine) Real analysis. Integral norm of continuous functions.
Problem 3. Point-set topology of real numbers. Cantor set.
Algebra.
Problem 1. (routine) Finite fields.
Problem 2. (routine) Ring theory. (Right/Left) Quasi-regular elements.
Problem 3. (routine) Homomorphism of rings. Polynomial rings in one variable.
General.
Problem 1. (routine) Arithmetic. Congruences.
Problem 2. (routine) Real numbers. Inequalities.
235
236 3. PROBLEM INDEX

Problem 3. Real analysis. Second order derivatives. Mechanics. Particle accel-


eration.
Problem 4. (routine) Counting. The product rule of combinatorics.
Probability and Statistics.
Problem 1. (routine) Covariance of random variables.
Topology.
Problem 1. (routine) Point-set topology. Connectedness.
Differential Equations.
Problem 1. (routine) First order differential equations.

Third Competition
Analysis.
Problem 1. Real analysis. Derivatives.
Problem 2. (routine) Real analysis. Continuous functions shifting forward se-
quences of real numbers.
Problem 3(a). Roots of complex polynomials.
Problem 3(b). Continuity of the roots of complex polynomials.
Problem 3(c). Continuous algebraic functions. Holder functions.
Algebra.
Problem 1. (routine) Modules. Dimension of vector spaces.
Problem 2. (routine) Group theory. Homomorphisms characterizing abelian
groups.
Problem 3. (routine) Semigroups. Groups.
General.
Problem 1. (routine) Real functions of two variables.
Problem 2. (routine) Binomial coefficients.
Problem 3. Plane geometry.

Fourth Competition
Analysis.
Problem 1. Real analysis. Continuity. Injectivity.
Problem 2. Sequences of nonnegative real numbers.
Algebra.
Problem 1. (routine) Ring theory. Commutative rings with identity. Radicals
of ideal. Prime ideals.
Problem 2. (routine) Linear algebra. Center of the algebra of all linear trans-
formations.
Problem 3. Ring theory. Artinian integral domains are fields.
General.
Problem 1. Integral calculus.
Problem 2. Conditional probability.

Fifth Competition
Analysis.
EIGHTH COMPETITION 237

Problem 1. Real analysis. Composition of Riemann integrable functions.


Problem 2. Real analysis. Continuous functions not assuming any value more
than twice.
Problem 3. Point-set topology of real numbers. Congestions points.
Algebra.
Problem 1. (routine) Linear algebra. Eigenvalues of certain linear transforma-
tions.
Problem 2. Group theory. Groups having two elements satisfying certain rela-
tions.
Problem 3. Ring theory. Rings whose elements satisfy certain polynomial equa-
tions.
General.
Problem 1. Number theory. Divisibility.
Problem 2. Roots of complex polynomials and those of their derivatives. Closed
convex hulls.
Problem 3. Plane geometry. Conic sections. Optics.

Sixth Competition
Analysis.
Problem 1. (routine) Analytic geometry. Rational points.
Problem 2. (routine) Metric spaces. Uniform convergence of sequences of con-
tinuous functions.
Problem 3. Real analysis. Limits of improper integrals.
Algebra.
Problem 1. Group theory. Subgroups.
Problem 2. Matrices.
Problem 3. Ring theory. Rings.
General.
Problem 1. Integer numbers.
Problem 2. Integer matrices.
Problem 3. (routine) Plane geometry.

Seventh Competition
Analysis.
Problem 1. Point set topology of real numbers.
Problem 2. Real analysis. Continuity.
Problem 3. Limits of sequences of of real numbers.
Algebra.
Problem 1. Ring theory. Nilpotent elements. Idempotent elements.
Problem 2. Group theory. Normal subgroups.
Problem 3. Linear algebra. Linear transformations.

Eighth Competition
Analysis.
Problem 1. (routine) Real analysis. Continuous extension.
238 3. PROBLEM INDEX

Problem 2. Convergence of series of real numbers.


Problem 3. (routine) Riemann-Stieltjes integral. Convergence of sequences of
Riemann-Stieltjes integrals.
Algebra.
Problem 1. (routine) Real matrices. The general linear group of order 2.
Problem 2. Ring Theory. Infinite integral domains.
Problem 3. (routine) Real matrices.
General.
Problem 1. (routine) Minimum without using derivative.
Problem 2. (routine) Arithmetic.
Problem 3. Probability. Average function value.

Ninth Competition
Analysis.
Problem 1. (routine) Point-set topology of real numbers. Equivalence relation.
Problem 2. Real analysis. Continuity of inverse functions.
Problem 3. (routine) Real analysis. Increasing continuous function.
Problem 4. Real analysis. Riemann integrable function.
Problem 5. Real analysis. Continuity. Injectivity. Surjectivity. Square filling
curves.
Algebra.
Problem 1. Group theory. Finite groups. Subgroups of prime indexes. Normal
subgroups.
Problem 2. Field of integers mod 3, Z3 . Polynomial ring with coefficients in
Z3 . Finite fields.
Problem 3. (routine) Polynomial ring with coefficients in Z7 . Greatest common
divisor.
Problem 4. Field theory. Quadratic extensions.
Problem 5. Linear algebra. Linear transformations.
General.
Problem 1. Determinants.
Problem 2. (routine) Algebraic equations.
Problem 3. Real analysis. Continuous functions. Integrals.
Problem 4. Diophantine equations. Number of solutions.
Problem 5. Puzzles.
Problem 6. Probability.
Problem 7. Determinants.

Tenth Competition
Analysis.
Problem 1. Real analysis. Derivatives. Limits.
Problem 2. Real analysis. Higher derivatives. Definite integrals.
Problem 3. (routine) Real analysis. Limits of definite integrals. Convergence
of function sequences.
Algebra.
TWELFTH COMPETITION 239

Problem 1. Group theory. Additive group of real numbers. Maximal sub-


groups.
Problem 2. (routine) Ring theory. Ideals.
Problem 3. Field theory. Field extensions.
Problem 4. (routine) Matrix theory over general fields. Nilpotent matrices.

General.
Problem 1. Probability. Random variables.
Problem 2. Point-set topology of the plane.
Problem 3. (routine) Differential and integral calculus. Differential equations.
Integral equations.
Problem 4. (routine) Arithmetic. Divisibility.

Eleventh Competition
Analysis.
Problem 1. Real analysis. Derivatives.
Problem 2. Real analysis. Convergence of function sequences. Functional equa-
tions. Uniform convergence.
Problem 3. Real analysis. Higher order derivatives. Convergence of series.

Algebra.
Problem 1. Ring theory. Ideals. (Right) identity elements.
Problem 2. Field theory. Algebraic numbers. Finite field extensions.
Problem 3. Group theory. Finite groups.

General.
Problem 1. Probability. Determinants.
Problem 2. Real numbers. Algebraic equations.
Problem 3. Baby set theory. Group theory.

Twelfth Competition
Analysis.
Problem 1. Real analysis. Continuity. Injectivity. Surjectivity.
Problem 2. Real analysis. Uniform convergence of series.
Problem 3. (routine) Integral calculus. Definite integrals.

Algebra.
Problem 1. Group theory. Finite groups. Sylow theory.
Problem 2. Group theory. Finite abelian groups. Homomorphisms.
Problem 3. Ring theory.
Problem 4. Ring theory. Principal ideals.
Problem 5. Matrix theory over general fields. Determinant. Trace.
Problem 6. Algebraic numbers.

General.
Problem 1. Plane Geometry.
Problem 2. Combinatorics. Pigeonhole principal.
Problem 3. Binomial coefficients.
240 3. PROBLEM INDEX

Thirteenth Competition
Analysis.
Problem 1. Real analysis. Riemann integrable functions. Additive functions.
Problem 2. Convergence of series of real numbers.
Problem 3. Real analysis. Convergence of function sequences. Definite inte-
grals.

Algebra.
Problem 1. Ring theory. Integral domains.
Problem 2. Ring theory. Right (Left) ideals.
Problem 3. Group theory.
Problem 4. Group theory. Permutations.
Problem 5. (routine) Matrix theory. Eigenvectors. Eigenvalues.

General.
Problem 1. (routine) Plane geometry.
Problem 2. Rational polynomials.
Problem 3. Space geometry.

Fourteenth Competition
Analysis.
Problem 1. Real analysis. Differentiability.
Problem 2. Real analysis. Uniform continuity. Limits.
Problem 3. Real analysis. Continuous functions.

Algebra.
Problem 1. Group theory. Finite p-groups. Derived subgroups.
Problem 2. Group theory. Finite groups. Normal subgroups. Center of groups.
Problem 3. Ring theory. Greatest commons divisors. Least common multiples.
Problem 4. (routine) Ring theory. Polynomial rings of two variables. Irre-
ducibility.
Problem 5. Finite-dimensional vector spaces. Direct sums.

General.
Problem 1. Combinatorics.
Problem 2. Combinatorics. Counting.
Problem 3. Real numbers.

Fifteenth Competition
Analysis.
Problem 1. Real analysis. Convexity.
Problem 2. (routine) Real analysis. Continuous functions. Uniform conver-
gence of function sequences.
Problem 3. Real analysis. Real functions.

Algebra.
Problem 1. Group theory. Finite groups. Sylow p-subgroups.
Problem 2. Ring theory. (Right/Left) ideals. Prime ideals. Nilpotent elements.
Problem 3. (routine, as stated, but its nontrivial counterpart is nonroutine!)
NINETEENTH COMPETITION 241

General.
Problem 1. Elementary number theory.
Problem 2. Integral calculus.
Problem 3. Combinatorics. Probability. Binomial coefficients.

Sixteenth Competition
Analysis.
Problem 1. (routine) Real analysis. Uniform convergence.
Problem 2. Real analysis.
Problem 3. Real analysis. Uniform continuity.
Algebra.
Problem 1. Group theory. Finite abelian groups. Inner automorphisms.
Problem 2. Ring theory. Nilpotent elements. Idempotent elements.
Problem 3. Matrix theory. Number theory.

Seventeenth Competition
Analysis.
Problem 1. Real analysis. Derivatives.
Problem 2. (routine) Differential and integral calculus. Real polynomials. Di-
visibility.
Problem 3. Real continuous functions on metric spaces.
Problem 4. Real analysis. Continuity.
Algebra.
Problem 1. Group theory. Finite groups.
Problem 2. Ring theory. Polynomial rings in two variables with coefficients
from matrix rings over fields.
Problem 3. Linear algebra. Linear transformations. Rank. Nullity.

Eighteenth Competition
Analysis.
Problem 1. Real analysis. Power series.
Problem 2. Real analysis. Periodic functions. Limits of definite integrals.
Problem 3. Real analysis. Open maps.
Algebra.
Problem 1. Group theory. Finite groups. Normal subgroups.
Problem 2. Ring theory. Ascending chains of right ideals.
Problem 3. Vector spaces. Direct sums.

Nineteenth Competition
Analysis.
Problem 1. (routine) Real analysis. Differential and integral calculus. Zeros of
continuous functions.
Problem 2. Real analysis. Limits.
Problem 3. Real analysis. Continuous nowhere differentiable functions.
Algebra.
Problem 1. Group theory. Normal subgroups. Posets.
242 3. PROBLEM INDEX

Problem 2. Ring theory. Minimal left ideals. Ideals.


Problem 3. (routine) Matrix theory. Rank.
Problem 4. Matrix theory. Rank. Determinant. Trace.

Twentieth Competition
Problem 1. Vector spaces. Complex numbers. Linear independence.
Problem 2. Real analysis. Sequences of real numbers. Limits.
Problem 3. Group theory. Automorphism groups. Solvability.
Problem 4. Real analysis. Functional equations. Uniform continuity.
Problem 5. Ring theory. Commutative rings with identity. Zero divisors.
Problem 6. Real analysis. Periodic functions. Convergence of improper inte-
grals.

Twenty First Competition


Analysis.
Problem 1. Real analysis. Limits.
Problem 2. Real analysis. Continuous functions. Limits. Integrals.
Problem 3. Real analysis. One-sided limits. Riemann integrable functions.
Algebra.
Problem 1. Ring theory. Small ideals.
Problem 2. Group theory. Center of groups. Derived subgroups.
Problem 3. Matrix theory. Trace. Symmetric matrices. Characteristic polyno-
mials.

Twenty Second Competition


Analysis.
Problem 1. Real analysis. Derivatives.
Problem 2. Real continuous functions on metric spaces. Series. Uniform con-
vergence.
Problem 3. Complex analysis. Sequences of analytic functions. Integrals.
Algebra.
Problem 1. Group theory. Group index.
Problem 2. Ring theory. Commutative rings. Maximal ideals.
Problem 3. Linear algebra. Characteristic polynomials. Rank.
General.
This section has forty routine multiple choice problems.

Twenty Third Competition


Analysis.
Problem 1. Real analysis. Continuous bounded functions. Uniform norm.
Compactness. Uniform continuity.
Problem 2. Real analysis. Trigonometric series.
Problem 3. Real analysis. Convergence of function sequences. Differentiability.
Algebra.
Problem 1. Group theory. Inner automorphism groups. Nonabelian groups.
Problem 2. Ring theory. (Right) Ideals. Division rings.
TWENTY SIXTH COMPETITION 243

Problem 3. (routine) Matrix theory. Zero trace.

Twenty Fourth Competition


First Day.
Problem 1. Real analysis. Derivatives.
Problem 2. Real analysis. Real valued harmonic functions.
Problem 3. Number theory. Primes. Divisibility. Congruences.
Problem 4. (routine) Group theory. Infinite abelian groups.
Problem 5. Combinatorics. Counting.
Problem 6. Probability. Conditional probability.
Second Day.
Problem 1. Euclidean spaces. Metric spaces. Open balls.
Problem 2. (routine) Real analysis. Differential and integral calculus. Contin-
uous functions.
Problem 3. Matrix theory. Rank inequalities.
Problem 4. (routine) Ring theory. (Left) ideals.
Problem 5. Combinatorics.
Problem 6. Combinatorics. Game theory.

Twenty Fifth Competition


First Day.
Problem 1. Group theory. Finite groups. Group centers.
Problem 2. Linear algebra. Idempotents. Image. Kernel.
Problem 3. Real analysis. Higher derivatives.
Problem 4. Real analysis. Derivatives.
Problem 5. Probability.
Problem 6. Combinatorics. Triangles with integer sides. Partitions of numbers.
Second Day.
Problem 1. Ring theory. Integral domains. Multiplicative group of units.
Problem 2. Number theory. Prime numbers. Divisibility. Congruences.
Problem 3. Continuous functions on metric spaces.
Problem 4. Functions of one complex variable.
Problem 5. Combinatorics. Matrices with {−1, 1} entries.
Problem 6. Finite sets. Relations.

Twenty Sixth Competition


First Day.
Problem 1. Real analysis. Continuous functions. Derivatives.
Problem 2. Complex analysis. Analytic functions.
Problem 3. (routine) Number theory. Primes. Congruences.
Problem 4. Ring theory. Commutative rings with identity. Maximal ideals.
Problem 5. Combinatorics.
Problem 6. Combinatorics. Finite sets.
Second Day.
Problem 1. Real analysis. Continuous functions.
Problem 2. Real analysis. Continuous functions.
244 3. PROBLEM INDEX

Problem 3. Group theory. Maximal subgroups.


Problem 4. Matrix theory. Eigenvalues. Eigenvectors.
Problem 5. Combinatorics. Probability.
Problem 6. Discrete mathematics.

Twenty Seventh Competition


First Day.
Problem 1. Matrix analysis.
Problem 2. Complex analysis. Analytic functions.
Problem 3. Sequences of real numbers. Recurrence relations.
Problem 4. Set theory.
Problem 5. Matrices. Product of matrices.
Problem 6. Group theory. Normal subgroups.
Second Day.
Problem 1. Real analysis. Derivatives.
Problem 2. Point set topology of real numbers.
Problem 3. Analytic geometry.
Problem 4. Combinatorics. Finite sets.
Problem 5. Number theory. Congruences. Divisibility.
Problem 6. Ring theory. Boolean rings. Isomorphisms.

Twenty Eighth Competition


First Day.
Problem 1. Complex analysis.
Problem 2. Euclidean spaces. Continuous functions.
Problem 3. Number theory. Arithmetic functions. Euler’s totient function.
Problem 4. Group theory. Symmetric groups. Group center.
Problem 5. Vector spaces over Z2 .
Problem 6. Posets.
Second Day.
Problem 1. (routine) Integral calculus.
Problem 2. Real analysis. Continuous functions.
Problem 3. Complex matrices. Orthogonal matrices. Determinants.
Problem 4. Ring theory. Unital rings whose elements satisfy certain relations.
Problem 5. Probability. Conditional probability.
Problem 6. Combinatorics. Set theory.

Twenty Ninth Competition


First Day.
Problem 1. (routine) Integral calculus.
Problem 2. Rational numbers. Series of rational numbers.
Problem 3. (routine) Number theory. Divisibility.
Problem 4. Ring theory. Nilpotent ideals. Right ideals.
Problem 5. (routine) Baby set theory.
Problem 6. Combinatorics.
Second Day.
THIRTY FIRST COMPETITION 245

Problem 1. Complex analysis. Analytic functions.


Problem 2. Connected metric spaces. Compact metric spaces.
Problem 3. Group theory. Normalizer subgroup. Centralizer subgroup. Auto-
morphism group. Cyclic groups. Derived subgroups. Group center.
Problem 4. Matrix theory. Triangular matrices. Invariant subspaces. Nilpotent
matrices.
Problem 5. Probability. Expectation values.
Problem 6. Point-set topology of real numbers. Cantor set.

Thirtieth Competition
First Day.
Problem 1. (routine) Integral calculus.
Problem 2. Limits of sequences of complex numbers.
Problem 3. Ring theory.
Problem 4. Number theory. Primes. Congruences. Divisibility.
Problem 5. Binomial coefficients.
Problem 6. Natural numbers.
Second Day.
Problem 1. Real functions on separable metric spaces. Points of discontinuity.
Problem 2. Complex analysis. Analytic functions. Power series.
Problem 3. Group theory. Derived subgroups. Elements of finite order.
Problem 4. Matrix theory. Rank. Image. Kernel. Direct sum. Idempotents.
Problem 5. Number theory. Primes.
Problem 6. Point-set topology. Circles. Arc lengths.

Thirty First Competition


First Day.
Problem 1. Plane geometry. Triangles. Coloring circles by two colors.
Problem 2. Point-set topology of Rn . Minkowski sums. Limit points.
Problem 3. Group theory. Subgroups of index two. Finite abelian groups.
Problem 4. Probability. Dice throwing.
Problem 5. Point-set topology of R2 . Collinearity.
Problem 6. Matrix theory. Determinant. Trace.
Second Day.
Problem 1. Number theory. Complete squares.
Problem 2. Calculus of two real variables. Limits.
Problem 3. Set theory. One-to-one correspondence.
Problem 4. Ring theory. Polynomial rings over commutative rings with iden-
tity. Maximal ideals.
Problem 5. Real numbers. Partitions of sets with equal sums.
Problem 6. Metric spaces. Fixed point theory.
246 3. PROBLEM INDEX

Classification of problems
• Algebra,
Ring theory,
Index

Abel’s Continuity Theorem, 180, 182 Connected, 5, 34, 37, 46


Acceleration, 5 Connected component, 6
Algorithm, 35 Convergent, 13, 17, 20, 28, 35
AM-GM Inequality, 55 pointwise, 17
Algebraic, 16, 47 uniformly, 10, 23, 29
Algebraic numbers, 18
Archimedean property of real numbers, 52 Darboux’s property, 52
Area, 19, 32, 43 Darboux’s Theorem, 51, 52, 169, 176, 184,
Arzela’s Theorem, 129 207
Automorphism, 18 Dedekind’s Extension of Abel’s Theorem,
126, 127
Baire Category Theorem, 163, 208 Dense, 14, 47, 48
Baire function, 1, 52 Derangement, 231
Banach space, 162, 191 Derivative, 1, 10, 13, 16, 17, 32, 37, 46
Bayes’ Theorem, 74, 188-190, 215 Derived subgroup, 22, 29, 47, 90, 167
Bertrand’s conjecture, 48, 226 Determinant, 15, 18, 105, 107, 118, 231
Bertrand’s principle, 48 Dihedral group, 183
Binomial Theorem, 70, 72, 79, 85, 94 Dini’s Theorem, 116
Bisection method, 34 Dirichlet’s Theorem, 181
Boolean ring, 78, 122, 209 Disconnected, 1, 41
Division algorithm, 111
Cantor set, 3, 46 Division ring, 37, 59, 85, 111, 146, 154, 193,
Cauchy Inequality, 225 225
Cauchy product of two polynomials, 70 Domain, 37
Cauchy-Riemann Equations, 185
Cauchy-Schwarz Inequality, 216 Eigenvalue, 10, 21, 41
Cauchy’s criterion for the convergence of Eigenvector, 21, 41
improper integrals, 168 Eisenstein’s Criterion, 117, 123
Cauchy’s Integral Formula, 177, 184 Element
Cauchy’s Theorem, 117 idempotent, 12, 25, 42
Cayley-Hamilton Theorem, 123 identity, 4, 12
Centralizer, 117, 151, 196, 204 invertible, 4
Chebyshev Inequality, 47, 223 left identity, 12
Clopen subset, 218 left quasi-regular, 4
Chain maximal, 27
ascending, 26, 90, 122, 122, 159 nilpotent, 12
descending, 9, 73 right quasi-regular, 4
Characteristic, 14, 19 right identity, 7
Circle, 10, 11, 19, 21, 48 quasi-regular, 4
Class equation, 117 Ellipse, 10, 81
Closed convex hull, 80 Endomorphism, 4, 8
Commutator subgroup, 22, 167 Equation
Compact, 6, 28, 36 differential, 2, 5, 17, 30, 34
Comparison Test, 127, 155 functional, 17
Complete metric space, 38 integral, 17
Complete square, 49 Euclidean
Congestion point, 9, 77 algorithm, 103

247
248 INDEX

metric, 43, 192 Fundamental Theorem of finite(ly


norm, 1 generated) abelian groups, 58, 118,
plane, 8, 42, 130 151, 175, 232
ring, 60, 103
Euclid’s proof of the infinitude of primes, Gamma function, 34
231 Gauss-Lucas Theorem, 79
Euler’s totient function, 43, 59, 152, 216 Greatest common divisor, 14, 22
Expansion, Group
binary, 26, 35 abelian, 2, 19, 37, 42, 48
decimal, 24, 35 commutative, 2, 19, 37
hexadecimal, 35 cyclic, 4, 19, 58, 59, 111, 121
octal, 35 finite, 14, 18, 22, 26, 26, 39
ternary, 3, 46 finite abelian, 48
Extension, 13, 14, 16, 18 finite nonabelian, 25
finite p-, 22
infinite, 37
Fermat’s Little Theorem, 138, 185, 202, 222 nonabelian, 25, 37
Field, 4, 7, 9, 13, 14, 16, 18, 19, 22, 23, 26, of inner automorphisms, 25, 37, 150, 167
27, 28, 30, 37, 41, 45, 46, 47 of permutations, 20, 102, 178, 211
First Isomorphism Theorem of units, 198
for groups, 102, 132, 145, 150, 167, 178, p, 22, 117, 137, 150
218 symmetric, 52, 102, 131, 145, 178
for modules, 69, 168 quaternionic, 196
for rings, 178 simple, 26
Flux, 33 simple abelian, 111
Frobenius Inequality, 193 solvable, 28, 137, 167
Function
analytic, 29, 40, 41, 46, 47 Hamel basis, 72
arithmetic, 43 Hyperbola, 81
auxiliary, 16
bounded, 21, 29, 32 Ideal
choice, 36 left, 20, 23, 38
maximal, 1, 13, 30, 40, 49
complex valued, 6
minimal left, 27
continuous, 1, 3, 6, 6, 8, 9, 10, 12, 13, 14,
nilpotent, 45
15, 16, 17, 17, 18, 20, 21, 24, 25, 26, 27,
prime, 1, 8, 23
28, 29, 32, 37, 39, 40, 41, 43, 45, 46,49
principal, 19, 40
continuous bounded, 36
right, 20, 23, 26, 37, 45
continuously differentiable, 13, 27, 39
small, 29
convex, 23
two-sided, 18, 27, 37, 38, 45
decreasing, 28, 32
Inclusion, 27, 46
differentiable, 17, 20, 21, 25, 29, 32, 39,
Inclusion-exclusion principle, 148, 231
40, 42
Instantaneous
entire, 43
speed, 61
harmonic, 37
velocity (vector), 62
increasing, 14, 23, 32 Integral, 9, 24, 33, 36
integral part, 24 Integral domain, 9, 13, 20, 39
nonnegative, 3, 24 Integration by parts, 64, 93, 116, 120, 149
nowhere differentiable, 27 Intermediate value property, 51, 207
one-to-one, 8, 14, 17, 18, 42, 43 Intermediate Value Theorem, 51, 62, 65,
periodic, 24, 26, 28 77, 97, 119, 128, 133, 137, 137, 153,
real, 1, 2, 6, 7, 16, 23 161, 170, 193, 202, 205, 207, 213
surjective, 18, 39, 43, 45 Invariant subspace(s), 46
uniformly continuous, 21, 24, 28, 32, 36 Inverse Probability Theorem, 74
Fundamental Theorem of Algebra, 210 Isomorphic sets, 2
Fundamental Theorems of Calculus, Iteration method, 35
The First, 92, 113, 160, 184
The Second, 92, 109 Jacobian, 33
INDEX 249

Labeled graph, 139, 187 hyperbolas, 81


Lagrange’s Theorem, 85, 111, 118 Orthogonal matrix, 44
Largest prime power dividing a factorial,
114 Partition, 39, 42, 47
Least common multiple, 22 Perfect subset, 207
Lebesgue number, 98, 98 Permutation, 11, 20
Lebesgue’s Integrability Criterion for Persian alphabet, 5
Riemann integrals, 75, 98, 174 Persian calendar, 4
Lebesgue’s Number Lemma, 98 Picard’s Great Theorem, 210
Left annihilator, 216 Plane, 7, 21, 35
Legendre symbol, 222 Poisson random variable, 55, 56
Leibniz’s Theorem, 181 Polynomial, 10, 14, 21, 25, 26, 47
L’Hopital’s rule, 161, 221 characteristic, 29, 30, 108, 122, 178, 206,
Limit, 1, 29, 46, 49 229
Limit Comparison Test, 117, 127 minimal, 123, 179, 207
Limit point, 28, 48 elementary symmetric, 142
Line, 4, 11, 21, 35, 41, 42 Poset, 43
Linearly dependent, 8, 28 Prime ring, 198
Linear transformation, 4, 10, 12, 14, 26, 30, Primitive root, 47
39 Probability, 3, 9, 13, 15, 17, 18, 24, 31, 38,
39, 41, 44, 48
Matrix, 1, 11, 13, 15, 15, 18, 19, 21, 23, 27,
29, 37, 38, 40, 41, 42, 44, 46, 47, 48 Radical of an ideal, 8
Maximal subgroup, 16 Radius of convergence, 26, 47
Maximum Modulus Principle, 202, 224 Random variables, 3, 5, 17, 47
Maximum Modulus Theorem for harmonic Rank, 23, 26, 27, 30, 38, 47
functions, 184, 185 Rank-Nullity Theorem, 85, 90, 104, 108,
Mean Value Property, 184 154, 165, 225
Mean Value Theorem, 51, 64, 116, 128, Rational Root Theorem, 133
176, 183, 197, 202 Regular n-gon, 21
Mean Value Theorem for integrals, 173 Riemann integrable, 9, 19, 20, 21, 29
Mean Value Theorem for second order Riemann’s criterion for integrability, 75, 98,
derivatives, 62, 62 156
Metric space Right annihilator, 216
compact, 39, 46, Ring
complete, 38 commutative, 1, 8, 10, 19, 20, 28, 30, 40,
connected, 46 44, 47, 49
separable, 47 division, 37, 59, 85, 111, 146, 154, 193,
225
Minkowski sum, 48 noetherian, 154
Module, 7 uncountable,
Morera’s Theorem, 177 unital, 16, 19, 20, 23, 40, 44
Möbius with identity, 1, 9, 10, 11, 12, 18, 27, 28,
function, 151 47, 49
Inversion Formula, 210 Rolle’s Theorem, 109, 207
Root, 6, 10, 18, 19, 21, 27, 34, 35, 42
Nilpotent matrix, 16, 46 Rouché’s Theorem, 67
Noetherian ring, 154
Normalizer, 145 Sandwich Theorem, 230
Normal subgroup, 12, 14, 22, 26, 42 Schroder-Bernstein Theorem, 53, 101
Normed linear space, 149 Schwarz Lemma, 224
Nowhere dense subset, 163, 207 Second Isomorphism Theorem for groups,
90, 150
Odd prime, 19, 23, 39, 40 Second Mean Value Theorem for integrals,
One-to-one correspondence, 2, 23, 45, 49 168
Open map, 26 Semigroup, 7
Operator norm, 205 Sequence, 6, 8, 10, 12, 17, 20, 23, 24, 26,
Optical property of 28, 29, 32, 37, 42, 45
ellipses, 81 decreasing, 6, 13, 18
250 INDEX

increasing, 6, 17, 45
Series, 18, 20, 31, 35, 36, 45
power, 26, 33, 47
Set
of measure zero, 75
partially ordered, a.k.a. poset, 43
power, 2
totally ordered, 76
Simple left module, 168
Speed, 4
Square, 19
Squeeze Lemma, 110
Stolz’s Theorems on limits
the first theorem, 87
the second theorem, 87, 173
Sylow p-subgroup(s), 23, 121, 145
Sylow’s Theorems
the first theorem, 137
the second theorem, 145
the third theorem, 121
Sylvester Inequality, 193
Sylvester’s problem, 133
Symmetric difference of sets, 118

Tangential acceleration, 62
Taylor’s Formula, 79
Third Isomorphism Theorem for groups,
163, 228
Tree, 139
Triangle, 2, 39, 48
Topological space, 5
Torsion free group, 231
Turán’s Theorem, 187, 187

Uniqueness Theorem for analytic functions,


205, 217

Vandermonde’s determinant formula, 142


Variation of parameters, 55
Vector field, 33, 34
Vector space, 3, 4, 8, 10, 12, 14, 16, 22, 26,
30, 39, 45

Well-ordering principle of natural numbers,


52, 54, 158

Zero divisor(s), 28
Zorn’s Lemma, 36, 72, 94, 146

Das könnte Ihnen auch gefallen